You are on page 1of 646

• ENGINEERING MECHANICS

A N INTRODUCTION TO

• DYNAMICS
• ENGINEERING MECHANICS
Fourth Edition • A N INTRODUCTION TO

• DYNAMICS

DAVID J . M C G I L L AND WILTON W . KING


Professors Emeritus, Georgia Institute of Technology
Copyright © 2003 by David J. McGill and Wilton W. King

All rights reserved. No part of this book may be reproduced, stored in a retrieval
system, or transmitted, in any form or by any means —electronic, mechanical,
photocopying, recording, or otherwise—without prior written permission of the
authors.

For more information, contact:

Tichenor
Publishing Tichenor Publishing
5005 North State Road 37 Business
Bloomington, IN 47404-1626
Division of T.I.S., Inc.
To OUR WIVES, C A R O L Y N AND KAY
CONTENTS

PREFACE xi

1 KINEMATICS OF MATERIAL POINTS OR PARTICLES 1


1.1 Introduction, 2
1.2 Reference Frames and Vector Derivatives, 3
Position, Velocity, and Acceleration, 6
1.4 Kinematics of a Point in Rectilinear Motion, 8
1.5 Rectangular Cartesian Coordinates, 24
1.6 Cylindrical Coordinates, 31
1.7 Tangential and Normal Components, 43
Summary, 53
Review Questions, 54
2 KINETICS OF PARTICLES AND OF MASS CENTERS
OF BODIES 55
2.1 Introduction, 56
2.2 Newton's Laws and Euler's First Law, 56
2.3 Motions of Particles and of Mass Centers of Bodies, 62
2.4 Work and Kinetic Energy for Particles, 87
2.5 Momentum Form of Euler's First Law, 101
2.6 Euler's Second Law (The Moment Equation), 117
Summary, 125
Review Questions, 128
3 KINEMATICS OF PLANE MOTION OF A RIGID BODY 129
3.1 Introduction, 130
3.2 Velocity and Angular Velocity Relationship for Two Points of the Same
Rigid Body, 134
3.3 Translation, 147
3.4 Instantaneous Center of Zero Velocity, 149
3.5 Acceleration and Angular Acceleration Relationship for Two Points of
the Same Rigid Body, 163
vii
Page viii

3.6 Rolling, 170


3.7 Relationship Between the Velocities of a Point with Respect to Two
Different Frames of Reference, 198
3.8 Relationship Between the Accelerations of a Point with Respect to
Two Different Frames of Reference, 207
Summary, 215
Review Questions, 216

KINETICS OF A RIGID B O D Y IN PLANE M O T I O N /


DEVELOPMENT AND SOLUTION OF THE DIFFERENTIAL
EQUATIONS GOVERNING THE M O T I O N 217
4.1 Introduction, 218
4.2 Rigid Bodies in Translation, 219
Moment of Momentum (Angular Momentum), 227
Moments and Products of Inertia/The Parallel-Axis Theorems, 229
4.5 The Mass-Center Form of the Moment Equation of Motion, 246
4.6 Other Useful Forms of the Moment Equation, 272
4.7 Rotation of Unbalanced Bodies, 291
Summary, 301
Review Questions, 303

5 SPECIAL INTEGRALS OF THE EQUATIONS OF PLANE


M O T I O N OF RIGID BODIES: W O R K - E N E R G Y AND
IMPULSE-MOMENTUM METHODS 304
5.1 Introduction, 305
5.2 The Principle(s) of Work and Kinetic Energy, 305
5.3 The Principles of Impulse and Momentum, 343
Summary, 375
Review Questions, 377

6 KINEMATICS OF A RIGID B O D Y IN
THREE-DIMENSIONAL M O T I O N 379
Introduction, 380
Relation Between Derivatives/The Angular Velocity Vector, 380
6.3 Properties of Angular Velocity, 384
6.4 The Angular Acceleration Vector, 398
6.5 Velocity and Acceleration in Moving Frames of Reference, 401
The Earth as a Moving Frame, 410
Velocity and Acceleration Equations for Two Points of the Same Rigid
Body, 414
6.8 Describing the Orientation of a Rigid Body, 428
6.9 Rotation Matrices, 434
Summary, 441
Review Questions, 442
iPage x

7 KINETICS OF A RIGID BODY IN GENERAL MOTION 444


7.1 Introduction, 445
7.2 Moment of Momentum {Angular Momentum) in Three Dimensions, 446
7.3 Transformations of Inertia Properties, 448
7.4 Principal Axes and Principal Moments of Inertia, 455
7.5 The Moment Equation Governing Rotational Motion, 472
7.6 Gyroscopes, 492
7.7 Impulse and Momentum, 499
7.8 Work and Kinetic Energy, 504
Summary, 513
Review Questions, 514
8 SPECIAL TOPICS 516
8.1 Introduction, 517
8.2 Introduction to Vibrations, 517
8.3 Euler's Law for a Control Volume, 535
8.4 Central Force Motion, 543
Review Questions, 554
APPENDICES CONTENTS 557
Appendix A UNITS, 558
Appendix B EXAMPLES OF NUMERICAL ANALYSIS/THE NEWTON-
RAPHSON METHOD, 564
Appendix C MOMENTS OF INERTIA OF MASSES, 567
Appendix D ANSWERS TO ODD-NUMBERED PROBLEMS, 573
INDEX, 583

SUPPLEMENTARY PROBLEMS 589


PREFACE

An Introduction to Dynamics is the second of two volumes covering basic


topics of mechanics. The first two-thirds of the book contains most of the
topics traditionally taught in a first course in dynamics at most colleges of
engineering.
In the writing of this text we have followed one basic guideline — to
write the book the same way we teach the course. To this end, we have
written many explanatory footnotes and included frequent questions
interspersed throughout the chapters. These questions are the same kind
as the ones we ask in class; to make the most of them, treat them as
serious homework as you read, and look at the answers only after you
have your own answer in mind. The questions are intended to encourage
thinking about tricky points and to emphasize the basic principles of the
subject.
In addition to the text questions, a set of approximately one dozen
review questions and answers are included at the end of each chapter.
These true-false questions are designed for both classroom discussion
and for student review. Homework problems of varying degrees of diffi­
culty appear at the end of every major section. There are over 1,100 of
these exercises, and the answers to the odd-numbered ones constitute
Appendix D in the back of the book.
There are a number of reasons (besides carelessness) why it may be
difficult to get the correct answer to a homework problem on the first try.
The problem may require an unusual amount of thinking and insight; it
may contain tedious calculations; or it may challenge the student's ad­
vanced mathematics skills. We have placed an asterisk beside especially
difficult problems falling into one or more of these categories.
Some examples and problems are presented in SI (Systeme Interna­
tional) metric units, whereas others use traditional United States engi­
neering system units. Whereas the United States is slowly and painfully
converting to SI units, our consulting activities make it clear that much
engineering work is still being performed using traditional units. Most
United States engineers still tend to think in pounds instead of newtons
and in feet instead of meters. We believe students will become much
Page xi
Page xii

better engineers, scientists, and scholars if they are thoroughly familiar


with both systems.
Dynamics is a subject rich in its varied applications; therefore, it is
important that the student develop a feel for realistically modeling an
engineering situation. Consequently, we have included a large number
of actual engineering problems among the examples and exercises. Being
aware of the assumptions and accompanying limitations of the model
and the solution method is a valuable skill that can only be developed by
sweating over many problems outside the classroom. Only in this way
can a student develop the insight and creativity that must be brought to
bear on engineering problems.
Kinematics of the particle, or of a material point of a body, is covered
in Chapter 1. The associated kinetics of particles and mass centers of
bodies follows logically in Chapter 2. Here it will be seen that we have not
dwelt at length upon the "point-mass" model of a body. Since the engi­
neering student will be dealing with bodies of finite dimensions, we
believe that it is important to present equations of motion valid for such
bodies as quickly as possible. Thus Euler's laws have been introduced
relatively early; this provides for a compact presentation of general prin­
ciples without, in our opinion and experience, any loss of understanding
on the student's part. This is not meant to deprecate the point-mass
model, which surely plays an important role in classical physics and can
be utilized in a number of engineering problems. As we shall see in
Chapter 2, however, these problems may be attacked directly through
the equation of motion of the mass center of a body without detracting
from the view that the body has finite dimensions. Trajectory problems,
sometimes placed with particle kinematics, will be found in this kinetics
chapter also, since a law of motion is essential to their formulation.
The rigid body in plane motion is treated in detail in the center of the
book — the kinematics in Chapter 3 and the kinetics in Chapters 4 and 5.
In Chapter 3 the topic of rolling is discussed only after both the velocity
and acceleration equations relating two points of arigidbody have been
covered. Further, we treat the equations of velocity and acceleration of a
point moving relative to two frames of reference ("moving frames") in
Chapters 3 (plane kinematics) and 6 (three-dimensional, or general, kine­
matics), after the student has been properly introduced to the angular
velocity vector in these chapters.
Chapter 4 approaches plane kinetics from the equations of motion,
written with the aid of a free-body diagram of the body being studied —
that is, a sketch of the body depicting all the external forces and couples
but excluding any vectors expressing acceleration. Thus the free-body
diagram means the same thing in dynamics as it does in statics, facilitat­
ing the student's transition to the more difficult subject. Moments and
products of inertia are covered right where they appear in the develop­
ment of kinetics. This presentation gives students an appreciation of
these concepts, as well as a sense of history, as they encounter them along
the same paths that were traveled by the old masters.
Chapter 5 is dedicated to solving plane kinetics problems of rigid
Page xiii

bodies with certain special yet general solutions (or integrals) of the
equations of Chapters 2 and 4. These are known as the principles of
work and kinetic energy, impulse and momentum, and angular
impulse and angular momentum.
Chapters 6 and 7 deal comprehensively with the kinematics and
kinetics, respectively, of rigid bodies in three dimensions. There is no
natural linear extension from plane to general motion, and the culprit
is the angular velocity vector which depends in a much more com­
plicated way than on the angles used to orient the body in three
dimensions. We have found that if students understand the angular
velocity vector they will have little trouble with the general motion
of rigid bodies. Thus we begin Chapter 6 with a study of and its
properties. In three dimensions, the definition of angular velocity is
motivationally developed through the relationship between deriva­
tives of a vector in two different frames of reference. While this point
of view is often associated with more advanced texts, we have found
that college students at the junior level are quite capable of appreciat­
ing and exploiting the power of this approach. In particular, it allows
the student to attack, in an orderly way, intimidating problems such
as motions of gear systems and those of universal joints connecting
noncollinear shafts.
Chapter 8 is an introduction to three special topics in the area of
dynamics: vibrations, mass redistribution problems, and central force
motion.
We have received a number of helpful suggestions from those
who taught from earlier editions of the text, and we are especially
grateful to Lawrence Malvern of the University of Florida, Don Carlson
of the University of Illinois and to our colleagues at Georgia Institute of
Technology, in particular Erian Armanios, Olivier Bauchau, Don
Berghaus, Al Ferri, Janet Hampikian, Satya Hanagud, Dewey Hodges,
Larry Jacobs, Steve Johnson, Manohar Kamat, George Kardomateas,
Harvey Lipkin, Chris Lynch, Richard Neu, John Papastavridis, Mimi
Philobos, Jianmin Qu, Nader Sadegh, Marilyn Smith, Virgil Smith, Jeff
Streator, Ray Vito, Wan-lee Yin, and Minami Yoda.
We thank Meghan Root for her cheerful assistance with prior edi­
tions. And for their many useful suggestions, we are grateful to our
third-edition reviewers, who were:
William Bickford Vincent WoSang Lee
Arizona State University University of Southern California
Donald E. Carlson Joseph Longuski
University of Illinois at Urbana Purdue University
Robert L. Collins Robert G. Oakberg
University of Louisville Montana State University
John Dickerson Joseph E. Parnarelli
University of South Carolina University of Nebraska
Page xiv

John F. Ely Mario P. Rivera


Northern Carolina State University Union College
Laurence Jacobs Wallace S. Venable
Georgia Institute of Technology West Virginia University
Seymour Lampert Carl Vilmann
University of Southern California Michigan Technological University

We are grateful to the following professors, who each responded


to a questionnaire we personally sent out in 1991: Don Carlson,
University of Illinois; Patrick MacDonald and John Ely, North Carolina
State University; Vincent Lee, University of Southern California; Charles
Krousgrill, Purdue University; Samuel Sutcliffe, Tufts University; Larry
Malvern and Martin Eisenberg, University of Florida; John Dickerson,
University of South Carolina; Bill Bickford, Arizona State University;
James Wilson, Duke University; Mario Rivera, Union College; and Larry
Jacobs, Georgia Institute of Technology. Their comments were also
invaluable.
Last but not least, we are indebted to Kevin Theile, Manager of
Tichenor Publishing, for seeing this new edition through to comple­
tion.
David J. McGill
Wilton W. King
•ENGINEERING MECHANICS
• A N INTRODUCTION TO

• DYNAMICS
1 KINEMATICS OF MATERIAL
POINTS OR PARTICLES

1.1 Introduction
1.2 Reference Frames and Vector Derivatives
1.3 Position, Velocity, and Acceleration
1.4 Kinematics of a Point in Rectilinear Motion
The v-t Diagram
1.5 Rectangular Cartesian Coordinates
1.6 Cylindrical Coordinates
1.7 Tangential and Normal Components
SUMMARY
REVIEW QUESTIONS

Page 1
Page 2

1.1 Introduction

Dynamics is the general n a m e given to t h e study of the motions of bodies


a n d the forces that accompany or cause those motions. The b r a n c h of the
subject that deals only with considerations of space a n d time is called
kinematics. T h e b r a n c h that deals w i t h the relationships b e t w e e n forces
a n d motions is called kinetics, b u t since the force-motion relationships
involve kinematic considerations, it is necessary to study kinematics first.
In this chapter w e present some fundamentals of the kinematics of a
material point or, equivalently, a n infinitesimal element of material. We
shall use the term particle for such a n element, b u t w e shall also use this
term in a broader sense to d e n o t e a piece of material sufficiently small
that the locations of its different material points n e e d not b e distin­
guished. The vagueness of this definition correctly suggests that, for some
purposes, a truck or a space vehicle or even a planet might b e modeled
adequately as a particle.
The key elements of the kinematics of a point (or particle) are its
position, velocity a n d acceleration. Velocity is rate-of-change of position,
a n d acceleration is rate-of-change of velocity. It is the acceleration in
N e w t o n ' s Second Law a n d the position-velocity-acceleration relation­
ships that allow u s to d e d u c e t w o things: the forces that m u s t act for a
particle to achieve a certain motion; or the evolution of a particle's posi­
tion u n d e r t h e action of a set of prescribed forces.
Position, velocity, a n d acceleration vectors are d e n n e d in Section 1.3.
These definitions are i n d e p e n d e n t of the choice of a n y particular coordi­
n a t e system. However, solution of a practical problem almost always
involves the use of some specific coordinate system, the most c o m m o n
being rectangular (Cartesian), cylindrical a n d spherical. The rectangular
a n d the cylindrical coordinate systems are treated in great detail in this
chapter (Sections 1.4-1.6) because w e judge t h e m to b e of greatest prac­
tical use, particularly for problems of motion confined to a plane. These
developments are sufficient for establishing the procedures to follow
should the reader later find it desirable to develop counterpart relation­
ships using some other system.
If w e focus our attention o n the p a t h being traversed by a point (or
particle), w e find that t h e velocity of the point is tangent to the p a t h a n d
that the acceleration h a s parts n o r m a l a n d tangent to the p a t h w i t h
special significances. These characteristics are developed a n d exploited
in Section 1.7.
We n o w begin our study of particle kinematics with a preliminary
section devoted to the calculus of vectors that d e p e n d u p o n scalars. In
particular, w e n e e d to u n d e r s t a n d h o w to take derivatives of vectors with
respect to time a n d to acknowledge the crucial role of the frame of
reference.
Page 3

1.2 Reference Frames and Vector Derivatives

In the next section a n d t h r o u g h o u t the book, w e are going to b e differen­


tiating vectors; the derivative of the position vector of a point will b e its
velocity, for example. T h u s in this preliminary section it seems wise to
examine the concept of the derivative of a vector A, w h i c h is a function of
time t. The definition of dA/dt, w h i c h is also c o m m o n l y written as A, is
deceptively simple:

This definition closely parallels the definition of the derivative of a scalar,


such as dy/dx, as found in a n y calculus text. But w h a t w e m u s t realize
about a vector is that it can change with time to two w a y s — i n direction as
well as in m a g n i t u d e . This m e a n s that A is intrinsically tied to the frame
of reference in w h i c h the derivative is taken.
To illustrate this idea, consider the t w o points P a n d Q o n the surface
of the p h o n o g r a p h record in Figure 1.1. The record rests o n a turntable
that revolves in the indicated direction at, say, rpm.
Suppose w e call R the vector that is the directed line segment from P
to Q, a n d inquire about the rate at w h i c h R changes with time as the
turntable rotates. Even t h o u g h w e perceive the record a n d turntable to
b e h a v e as a rigid b o d y so that the distance b e t w e e n P a n d Q (that is, the
m a g n i t u d e of R) is constant, most of us w o u l d judge dR / dt to be nonzero,
owing to the varying direction of R. This conclusion follows from our
automatically having a d o p t e d the building (or earth) as our frame of
Figure 1.1 reference. If w e w e r e to ride o n the turntable a n d blind ourselves to the
surroundings, however, our perception w o u l d b e that R is a constant
vector a n d consequently h a s a vanishing derivative. Thus R, relative to
the turntable, is a constant vector a n d , relative to the building, is a
constant-magnitude b u t varying-direction vector. It is therefore seen that
dR/dt cannot b e evaluated except b y specific association w i t h a f r a m e of
reference, w h i c h is n o t h i n g more n o r less t h a n a rigid body. We shall
discuss the frame of reference concept further in the next section a n d
again in C h a p t e r 3.
We shall b e needing several vector derivative relationships that h a v e
analogs in the calculus of scalars; these relationships follow directly from
the definition (Equation 1.1). If a is a scalar a n d A a n d B are vector
functions of t, t h e n
page 4

The first a n d second of these equations allow us to be m o r e specific


about t h e m a n n e r in w h i c h differentiation is linked to a frame of refer­
ence. Suppose that are mutually perpendicular unit vectors* a n d
A j , A , A are t h e corresponding scalar c o m p o n e n t s of a vector A so that
2 3

If is t h e frame of reference** a n d w e denote t h e derivative of A


relative to b y , then

N o w if w e chooseooseto h a v e fixed directions in they are each


constant there a n d

which is t h e most straightforward w a y to express t h e derivative of a


vector a n d its intrinsic association with a frame of reference. We n o w give
o n e example of the use of Equation (1.8) and, assuming the reader to be
familiar with Equations (1.1) to (1.5), t h e n m o v e o n to Section 1.3 a n d the
task of describing t h e motion of a point (particle) P.

EXAMPLE 1.1

If the distance from P to Q on the rpm record in Figure El. 1 a is 3 in. and if A is
the vector from P to Q, find where the frame is the cabinet of the stereo in
which the axes (x, y, z) are embedded. It is also given that the line PQ is in the
indicated position (parallel to y) when f = 0.
Solution
Figure El.la At a later time t (in seconds), the vector A is seen in Figure El. l b to make an angle
with y of

* Note that we could use any set of base vectors (that is, linearly independent reference
vectors) here, in which case A,, A , A are not necessarily orthogonal components of A.
2 3

Equation (1.6) simply illustrates the most common choice of scalars and base vectors.
When this is the case, the magnitude of A, written | A | or sometimes A, is
**Throughout the book, frames (rigid bodies) are denoted by capital script letters. These
are intended simply to be the capital cursive letters we use in writing; thus we write the
names of bodies and print the names of points. We do this because, as we shall see in
Chapter 3, points and rigid bodies have very different motion properties.
Note that the derivatives of the scalar components of A, such as dA,/d(, need not be
Figure E1.1b "tagged" since they are the same in any frame.
Page 5

The vector A, expressed in terms of the unit vectors i and j in the respective
directions of x and y, then has the following form:

Noting that the unit vectors do not change in direction in 3, we obtain, using
Equation (1.8),

We see from this result, for example, that:


1. s in the x direction.
2. is in the —y direction.
3. is in the — x direction.
4. is in the y direction.
In all four cases, and at all intermediate angles as well, the derivative of A in
is seen to be that of the cross product:

The bracketed vector represents what will come to be called the angular velocity of
the record in the reference frame (stereo cabinet) In later chapters we shall
see that it is precisely this cross product that must be added to to obtain
Here, of course, A is constant relative to the turntable 5 0 that its derivative in
(that is, vanishes.

PROBLEMS • Section 1.2

In Problems 1.1-1.8, are mutually perpendicular


unit vectors having directions fixed in the frame of refer­
ence. In each case t is time measured in seconds ("s" in SI
units). Determine at t = 3 s the rate of change (with re­
spect to time) of vector L. 1.9-1.16 If the vectors enumerated in Problems 1.1-1.8
represent various forces F, find the integral of each force
over the time interval from t = 2 through 5 sec. Let the
metric units become newtons and the U.S. units become
pounds.
Page 6

1.3 Position, Velocity, and Acceleration

In this short b u t important section, w e present the definitions of the


position, velocity, a n d acceleration vectors of a material point P as it
moves relative to a frame of reference It is important to mention that
while a frame of reference is usually identified b y the material constitut­
ing the reference body (for example, the earth, the m o o n , or the b o d y of a
truck), the frame is actually composed of all those material points plus the
points generated by a rigid extension of the body to all of space. Thus, for
example, w e refer to a point on the centerline of a straight pipe as a point
in (or of) the pipe.
W e n o w consider a point P as it m o v e s along a p a t h as s h o w n in
Figure 1.2. The p a t h is the locus of points of that P occupies as time
passes. If we select a point O of to be our reference point (or origin), t h e n
the depicted vector from O to P is called a p o s i t i o n vector for P in a n d is
written r . O P

The first a n d second derivatives (with respect to time) of the position


vector are respectively called the velocity (v ) a n d acceleration (a ) of
P P

point P in
Figure 1.2 Position vector for P in
(The magnitude of v is called the s p e e d of P.)
P

The derivatives in Equations (1.9) a n d (1.10) are calculated in frame


the only frame u n d e r consideration here. Later, however, w e shall
sometimes find it necessary to specify the frame in which derivatives,
velocities, a n d accelerations are to b e computed; w e shall t h e n tag the
derivatives as in Equation (1.8) a n d write

W h e n e v e r there is just one frame involved, w e shall omit the on both


sides a n d write an equation such as (1.11) in the form of (1.9).
T h r o u g h o u t the text w e h a v e inserted questions for the reader to
think about. (The answer is always on the same page as the question.)
H e r e is the first question:

Question 1.1 Do the velocity and acceleration of a point P depend


upon: (a) the choice of reference frame? (b) the origin selected for the
position vector?

Answer 1.1 (a) Yes; w e could simply define a frame in w h i c h P is fixed, and it w o u l d
then have v = 0 = a . (b) N o ; letting O' be a second origin in and differentiating the
P P

relationship r = i , + r . in 3 s h o w s that: v (with origin O) = v (with origin O').


o p 00 0 P P P

The derivative of r - in is, of course, zero! See Problem 1.17.


QO
Page 7

At this point it is reasonable to wonder w h y w e have not chosen to


introduce time derivatives of the position vector higher than the second.
The reason is that the relationships between forces and motions do not
involve those higher derivatives. As w e shall see later w h e n w e study
kinetics, if w e know the accelerations of the particles making up a body,
the force-motion laws will yield the external forces; conversely, for rigid
bodies, if w e know the external forces, w e can calculate the accelerations
and then, by integrating twice, the position vectors. The force-motion
laws turn out to be valid only in certain frames of reference; for that
reason writers sometimes refer to motion relative to such a frame as
absolute motion. We have not used the word absolute here because w e
wish to emphasize that kinematics inherently expresses relationships of
geometry and time, independent of any laws linking forces and motions.
Thus in kinematics all frames of reference are of the same importance.
Finally, w e note that positions (or locations) of points are normally
established through the use of a coordinate system. The ways in which
positions, velocities, and accelerations are expressed in two of the most
common systems, rectangular and cylindrical, are presented in the next
three sections.

PROBLEMS • Section 1.3

1.17 Show that the velocity (and therefore the accelera­ ence; v is the velocity of a point P moving in the frame; t
P

tion also) of a point P in a frame does not depend on the is time measured in seconds. Determine at t = 2 s the
choice of the origin. Hint: Differentiate the following re­ acceleration of the point for the velocity given.
lationship in (see Figure P 1 . 1 7 ) :

1.23-1.27 The displacement of a point over a time inter­


Figure PI.17 val f ] to t is denned to be the difference of the position
2

vectors—that is, r(f ) ~ r(f i). For the cases enumerated in


2

Problems 1 . 1 8 - 1 . 2 2 , find the displacement and the mag­


In Problems 1 . 1 8 - 1 . 2 2 , are mutually perpendicular nitude of the displacement over the interval t = 4 s to
unit vectors having directions fixed in the frame of refer­ t = 6 s.
Page 8

1.4 Kinematics of a Point in Rectilinear Motion

In this section w e study problems in w h i c h point P m o v e s along a straight


line in t h e reference frame this situation is called rectilinear motion,
a n d the position of P m a y b e expressed with a single coordinate x mea­
sured a l o n e t h e fixed line o n w h i c h P m o v e s (see Figure 1.3).
A position vector for P is simply

in w h i c h the unit vector is parallel to t h e line as s h o w n in Figure 1.3 a n d


hence does not change in either m a g n i t u d e or direction in Therefore P
has t h e following very simple velocity a n d acceleration expressions:

Figure 1.3
In rectilinear motion, there are three interesting cases w o r t h y of
special note:

1. Acceleration is a k n o w n function of time, /(t).


2. Acceleration is a known function of velocity, g(v),where
3. Acceleration is a known function of position, h(x).

In each case, w e can go far with general integrations. W e shall consider


each case in turn a n d give a n example.

in w h i c h Q a n d C are to b e determined b y t h e initial conditions on


2

velocity a n d position, respectively, once t h e proble m (and t h u s /(f)) is


stated a n d t h e mdefinite integrals are performed. Alternatively, w e might
k n o w t h e values of x at t w o times, instead of o n e position a n d one
velocity. In a n y case, w e n e e d t w o constants.

EXAMPLE 1.2

The acceleration of a point P in rectilinear motion is given by the equatior


2
m / s , with initial conditions i(0) = 2 m / s and x(0) = — 7 m. Find x(t).
Solution
We note that this is the problem of a point moving with a quadratically varying
acceleration magnitude and with the initial conditions being the position and
velocity of P at t = 0 as shown in Figure El.2. Integrating as above,

And integrating once more,


Figure El.2
Page 9

The constants are found from the initial conditions to be C = 2 m / s and C


x 2

= — 7 m, as follows:

Thus the motion of the point P is given by the integrated function of time:

Suppose v(t) can be inverted to give t(v); then

If the integral can be found as a function pip), then w e may be able to


and
solve the equation p(v) = t + C for the velocity:
3

v = q(t)

If so, then

so that

EXAMPLE 1,3 x = / q(t) dt + C 4 (1.16)


This procedure
Suppose should become
that the acceleration clearer
of a point P inwith the following
one-dimensional example.
motion is propor­
tional to velocity according to with the same initial conditions as in
the previous example. Solve for the motion x(t).
Solution
Page 10

so that, integrating,* we get

Since v = 2 when t = 0, then C = ( - I n 2 ) / 2 and


3

When acceleration is a function of position,


Therefore w e may
combine and to obtain the useful relation
But x = — 7 m when t = 0 s gives C = — 6 m, and so we obtain our solution:
4

Then, if a function r\x) exists such that we obtain, from

Equation (1.17), the following:

and integrating with respect to time,

EXAMPLE 1 . 4
Thus the square of the speed is
2 2
Let x = h(x) = —4x m / s . Find v (x) if the initial conditions are the same as in
Examples 1.2 and 1.3.
Equation (1.19) will be called an energy integral in Chapters 2 and 5.

* This problem could also be solved b y first integrating the linear differential equation
2t
v + 2v = 0, observing that A e ~ is the general solution.
Page 11

Solution
We are dealing with the equation

Actually we know that the solution to this equation, by the theory of differen­
tial equations, is x = A sin 2t + B cos 2t—which, with x(0) = — 7 m and i(0)
= 2 m / s , becomes x = sin It — 7 cos 2t meters. But let us obtain the desired re­
sult by using the procedure described above, which applies even when h(x) is
2
not linear. Here h(x) = — Ax, so with r{x) = — 2x , Equation (1.19) gives

or

where C has been found by using v = 2 m / s and x = — 7 m at t = 0.


5

The v-t Diagram


In problems of rectilinear kinematics in w h i c h t h e acceleration is a k n o w n
function of time (Case 1), w e sometimes use w h a t is called t h e v-t dia­
gram. W e shall give just o n e example of its use because it is a m e t h o d
s o m e w h a t limited in application. (We discuss this shortcoming at t h e e n d
of the example.)

EXAMPLE 1.5

A point P moves on a line, starting from rest at the origin with constant accelera­
2
tion of 0.8 m / s to the right. After 10 s, the acceleration of P is suddenly reversed
2
to 0.2 m / s to the left. Determine the total time elapsed when P is again passing
through the origin.
Solution
If we graph the velocity versus time, the acceleration (dv/ dt) will of course be the
slope of the curve at every point. The v-t diagram for this problem is shown in
Figure E1.5a.

Figure El.5a
Page 12

We note not only that

but also that

Hence the change in the position x between any two times is nothing more
than the area beneath the v-t diagram between those points. Thus four points, or
times, are important in the diagram for this problem:
t j = starting time (in this case zero)
t = time when acceleration changes (given to be 10 s)
2

t = time when velocity has been reduced to zero (deceleration causes P to stop
3

before moving in opposite direction)


t = required total time elapsed before point P is again at origin
4

Figure El.5b 2
The velocity at time f is seen to be 0.8 m / s X 10 s = 8 m / s . To find the
2

time interval t — t , we use the similar triangles shown in Figure El.5b.


3 2

The total distance traveled before the point (momentarily) stops is thus

This is the distance traveled by the point in the positive direction (to the right).
The point will be back at x = 0 when the absolute value of the negative area
beneath the t axis (the distance traveled back to the left) equals the 200 m traveled
to the right (represented by the area above the axis):

which can be rewritten as

The only root of this equation larger than 50 s is f = 94.7 s, and this is the answer
4

to the problem.

A n alternative approach to the preceding v-t diagram solution is as


follows. Integrating the acceleration during the interval
with x during this interval called x , 1

Integrating again (over the same interval), w e get


Page 13

Thus at t = 10 s, b y substitution,

Next, after the deceleration starts, using x in this interval,


2

a n d since Therefore

Integrating again, we get

A n d with x = 40 m w h e n t = 10 s, t h e n C = — 50 m:
2 4

2
x = - 0 . l t + lOf - 50 m
2

W h e n x = 0, w e can solve for t h e time; the equation is the s a m e as in t h e


2

v-t diagram solution:

Of t h e roots, t = 5.28 a n d 94.7 s, only the latter is valid since 5.28 s


4

occurs prior to the change of acceleration expressions.


Even t h o u g h b o t h approaches yield t h e correct a n s w e r of 94.7 s in
the preceding example, w e m u s t r e c o m m e n d t h e latter a p p r o a c h of inte­
grating t h e accelerations a n d matching velocities a n d positions b e t w e e n
intervals. The reason is that w h e n w e are faced with nonconstant acceler­
ations, the v-t diagram a p p r o a c h requires us to find areas u n d e r curves,
the formulas for w h i c h are not ordinarily memorized.
It is interesting, in using t h e equations, to start a n e w time measure­
m e n t t at t h e beginning of t h e second interval:
2

Integrating again, w e get

T h e n x = 0 yields t h e equation
2

which h a s the positive root t = 84.7—which, a d d e d to the 10-s duration


2

EXAMPLE 1.6
of the first interval, gives again 94.7 s of total time elapsed. It is slightly
easier to calculate
A point B starts from rest t h e integration
at the originconstants
at t = 0 and with this a p p rato aacconstant
accelerates h of "start­
rate
ing
k m /time
s ino vrectilinear
2
e r " t h a n motion.
to use t hAfter
e s a m6es,t tthe
h r oacceleration
u g h o u t . T h echanges
only price w etime-
to the pay
for this convenience
dependent function is that
m /ws e in
2
m uthe
s t opposite
a d d the direction,
times at wheret h e end.
2 t = 0 when
Page 14

t= 6 s. If the point stops at t = 26 s (from the starting time) and reverses direc­
tion, find the acceleration k during the first interval and the distance traveled by B
before it reverses direction. Then find the total time elapsed before B passes back
through the origin.
Solution
We begin the solution by deterrruning the motion (*i(f)) during the first time
interval; we integrate the acceleration to obtain the velocity and then again to get
the position:

At t! = 6 s, the acceleration changes to a negative value and point B "decel­


erates." At the beginning of this second interval, the speed and position of B are
given by the "ending" values during the first interval. These values are and x t

at fj = 6:

Note that we start time t at the beginning of the second interval, during which
2

we have

where we note that the minus sign is needed to express the deceleration. Integrat­
ing, we get

where c was computed by using the initial condition that x = 18k meters when
4 2

t = 0.
2

Now we use the fact that x is zero at time t = 26 — 6 = 20 s; this strategy


2 2

will allow us to determine k:


3
0 = -0.002(20 ) + 6fc
2
k = 2.67 m / s
Substituting k into the x expression at f = 20 s gives us the position of B at the
2 2

"turnaround":
4
*2STOP = -0.0005(20 ) + 6(2.67)(20) + 18(2.67)
= 288 m
Finally, to obtain the time f 2END when B is passing back through the origin we set
Page 15

Rewriting, we get

On a calculator, the only positive root to this equation* is found in a matter of


minutes to be (to three significant figures):

The total time is f 2END plus the duration of the first interval, or 38.7 s.

Before w e leave this example w e wish to note that during the first
time interval, while the acceleration is constant,

where

Letting v = x, w e note further that

and eliminating t w e obtain

This expression gives us the magnitude of the velocity in terms of dis­


placement. Most students have used this relationship in high school or
perhaps elementary college physics. There is sometimes a tendency,
however, to forget the conditions under which it is valid; it holds only for
rectilinear motion with constant acceleration. Thus it could not be used
EXAMPLE
during 1.7 interval of the preceding example, nor could the equa­
the second
tions
Two carsfor in
x and v from which
a demolition it was
derby are derived.a common point (the origin in
approaching
Figure El.7), each at 55 mph in a straight line as indicated. Car C does not speed
x

up or slow down; the driver of C applies the brakes. Find the smallest rate of
2

deceleration of C that will allow C to precede it through the intersection, if:


2 x

a. d = 200 ft
2

b. d = 1 0 0 ft
2

* Descartes' rule of signs tells us that the maximum number of positive real roots to
Equation ( 1 ) is o n e (the number of changes in sign o n the left-hand side). A n d there will
be exactly o n e because the left side is negative at and positive for large values
of
Page 16

Figure El.7

Solution
Placing the origin at the point of intersection of the two cars' paths, we have the
following for

Using the initial condition that x = — 300 ft when t = 0, we get


t

The back of car C will be at the origin (point of possible collision) when x = 0:
x

Now let us study the motion of car C . We use the coordinate q as shown for
2

this car. Calling the unknown deceleration K, we obtain

so that

and

But C = 0, since q = 0 at t = 0.
3

Next we see that at 3.72 sec the position of is

Finally, car just passes the rear of if q is d at this time:


2

(f = -6.92K + 300ft
2
Page 17

Hence:
i . If d = 200 ft then K = 14.5 ft/sec .
2
2

I . If d = 100 ft, then K - 28.9 ft/sec .


2
2

Note also that if d = 300 ft, then K = 0; this is because no deceleration is needed
2

for the same distances at the same speeds. Further, if d > 300, then K is negative,
2

meaning that car would have to accelerate to arrive at the intersection at the
same time as car

Sometimes there are special conditions in a problem that require


ingenuity in expressing the kinematics. If there is an inextensible rope,
string, cable, or cord present, for example, w e may have to express the
constancy of length mathematically. This is the case in the following
example.

EXAMPLE 1.8

Solution
The length L of the rope that passes around both small pulleys is a constant. This
is a constraint equation that must be used in the solution. The procedure is as
follows (see Figure El.8):

Differentiating and noting that L, n, r , r , and K are constants, we get


c a

The velocities of C and B are equal since both points move on the same path with
Figure El.8 a constant length separating them. Hence

(Note that C moves upward since j is downward!)

Therefore

EXAMPLE 1.9
The ends A and B of the rigid bar in Figure El .9 are to move along the horizontal
and vertical guides as shown. End A moves to the right at a constant speed of
8 m / s . Find the velocity and acceleration of B at the instant when A is 3 m from
Figure El.9 the comer.
Page 18

Solution
In terms of the parameters and unit vectors shown in Figure El.9,

Using the fact that the distance from A to B is a constant 5 m,


2 2
x + y = 25

so that

or

Thus when x = 3 m, y = 4 m and

or

so that

Differentiating again,
Our last example illustrates a different kind of constraint, that of a
Therefore
point on aatbody
the instant of interest
mamtaining contact with a surface (or line) on another
moving body.
or

and

EXAMPLE 1 . 1 0

The curve AB on block B (see Figure E1.10a) is a parabola whose vertex is at A. Its
2
equation is x = (64/3)y. The block B is pushed to the left with a constant
velocity of 10 ft/sec. The rod slides on the parabola so that the plate p is forced
upward. Find the acceleration of the plate.
Solution
We first note that plate p and rod IS together constitute a single body, each of
whose points has one-dimensional (y) motion. The velocities and accelerations of
Page 19

Figure El. 10a

all these points are therefore the same. We shall then focus on point D, the lowest
point of which is in contact with
Defining the ground to be the reference frame we establish its origin at O
as shown in Figure El. 10b.

2
But because D always rests on the parabolic surface of B, y = (3/64)x so that

To get the acceleration of D, we first find its velocity:

To obtain x, we differentiate i OA from Equation (1):

since it is given that all points of the body Bhetve the constant velocity 10 ft/sec
Figure E1.10b to the left.
Substitution of into Equation (3) then gives

and we see that the velocity of D depends upon x. Differentiating v will give usD

the acceleration of D:

Equation (6) gives the acceleration of all the points of the plate. Note that the
acceleration of D is a constant.

Question 1.2 Would a be a constant if instead of being quadratically


D

shaped, the inclined surface were (a) flat or (b) cubic?

Answer 1.2 If the surface is flat, then a vanishes. If it is cubic, then a is linear in x.
D D
Page 20

PROBLEMS • Section 1.4

1.28 A slider block moves rectilinearly in a slot (see 1.35 x = - 1 1 2 5 m


0

Figure PI.28) with an acceleration given by Time interval: 0 < t < 20 s (See Figure P1.35.)

Figure PI.28

Figure PI.35

Find the motion x(t) of the slider block if at t = 0:


a. It is passing through the origin, and
1.36 x = 10 m
0

b. It has velocity Time interval: 0 < t £ 30 s (See Figure P1.36.)


1.29 Suppose an airplane touches down smoothly on a
runway at 60 mph. If it then decelerates to a stop at the
2
constant deceleration rate of 10 ft/sec , find the required
length of runway.
1.30 A train is traveling at 60 k m / h r . If its brakes give
2
the train a constant deceleration of 0.5 m / s , find the
distance from the station where the brakes should be ap­
plied so that the train will come to a stop at the station,
How long will it take the train to stop?
Figure PI.36
1.31 A point P starts from rest and accelerates uniformly
(meaning x = constant) to a speed of 88 ft/sec after trav­
eling 120 ft. Find the acceleration of P.
1.32 If in the preceding problem a braking deceleration 1.37 x = 1 0 m 0

2
of 2 ft/sec is experienced beginning when P is at 120 ft, Time interval: 2 s ( < 5 s (See Figure PI.37.)
determine the time and distance required for stopping.
1.33 A car is traveling at 55 mph on a straight road. The
driver applies her brakes for 6 sec, producing a constant
2
deceleration of 5 ft/sec , and then immediately acceler­
2
ates at 2 ft/sec . How long does it take for the car to
return to its original velocity?
1.34 In the preceding problem, suppose the acceleration
following the braking is not constant but is instead given
2
byx = 0.6r ft/sec . Now how long does it take to return to Figure PI.37
55 mph?

In Problems 1.35-1.37, the graph depicts the velocity of a


point P in rectilinear motion. Draw curves showing the 1.38 Ahotrod enthusiast accelerates his dragster along a
position x(t) and acceleration a{t) of P if the point is at the straight drag strip at a constant rate of acceleration from
indicated position x at t = 0.
0 zero to 120 mph. Then he immediately decelerates at a
Page 21

constant rate to a stop. He finds that he has traveled a b. Determine the distance traveled by the particle
total distance of \ mi from start to stop. How much time over the same time interval.
passes from the instant he starts to the time he stops?
1.46 A p o i n t P m o v e s o n a line. The acceleration of P is
Hint: Sketch a v-t diagram.
gi ven by The velocity
1.39 Ben Johnson set a world record of 9.83 seconds in of P at t = 0 is — 60i m / s , with the point at x = 7 m at
P

the 100-meter dash on August 31, 1987. He had also set that time. Find the distance traveled by P in the time
the record for the 60-meter dash of 6.40 seconds that interval t = 0 to t = 13 s.
same year. Assuming that in each race Johnson acceler­
• 1.47 The position of a point P on a line is given by
ated uniformly up to a certain speed v and then held that
0
the equation The point starts moving at
same maximum speed to the end of both races, find
f = 0. Find the total distance traveled by P when it passes
(a) the time t to reach v„; (b) the value of v ; and (c) the
0 0
through the origin (counting the start as the first pass) for
distance traveled while accelerating.
the third time.
1.40 A train travels from one city to another which is 1.48 A particle moving on a straight line is subject to an
134 miles away. It accelerates from rest to a maximum acceleration directly proportional to its distance from a
speed of 100 mph in 4 min, averaging 65 mph during fixed point P on the line and directed toward P. Initially
this time interval. It maintains maximum velocity un­ the particle is 5 ft to the left of P and moving to the right
til just before arrival, when it decelerates to rest at with a velocity of 24 ft/sec. If the particle momentarily
an average speed during the deceleration of 40 mph. If
comes to rest 10 ft to the right of P, find its velocity as it
the total travel time was 110 min, find the deceleration
passes through P.
interval.
1.49 A particle moving on the x axis has an acceleration
1.41 A point Q in rectilinear motion passes through the always directed to the origin. The magnitude of the accel­
origin at t = 0, and from then until 5 seconds have eration is nine times the distance from the origin. When
2
passed, the acceleration of Q is 6 ft/sec to the right. the particle is 6 m to the left of the origin, it has a velocity
Beginning at t = 5 seconds, the acceleration of Q is of 3 m / s to the right. Find the time for the particle to get
2
12t ft/sec to the left. If after 2 more seconds point Q is from this position to the origin.
13 feet to the right of the origin, what was the velocity of
Q at t = 0? 1.50 A point P has an acceleration that is position-
dependent according to the equation
1.42 A point begins at rest at x = 0 and experiences con­ Determine the velocity of P as a function of its position x if
stant acceleration to the right for 10 s. It then continues at P is at 0.3 m with
constant velocity for 8 more seconds. In the third phase of
2
its motion, it decelerates at 5 m / s and is observed to be 1.51 Suppose initial conditions are the same as in the
passing again through the origin when the total time of preceding problem but Find as a function of
travel equals 28 s. Determine the acceleration in the first time.
10 s. 1.52 The velocity of a particle moving along a horizontal
1.43 An automobile passes a point P at a speed of path is proportional to its distance from a fixed point
80 mph. At P it begins to decelerate at a rate proportional on the path. When t = 0, the particle is 1 ft to the right
to time. If after 5 sec the car has slowed to 50 mph, what of the fixed point. When v = 20 ft/sec to the right,
2

distance has it traveled? a = 5 ft/sec to the right. Determine the position of


the particle when t = 4 sec. (See Figure PI.52.)
1.44 Work the preceding problem, but suppose the de­
celeration is proportional to the square of time. The other
information is the same.
1.45 A particle has a linearly varying rectilinear accel­
Figure PI.52
eration of Two observations of
the nartirlp's motion are made: Its velocity at t = 1 s is
and its position at t = 2 s is given by
meters.
a. Find the displacement of the particle at t = 5 s
relative to where it was at f = 0. * Asterisks identify the more difficult problems.
Page 22

1.53 A speeder zooms past a parked police car at a the driver of the car reacts by slamming on her brakes,
constant speed of 70 mph (Figure PI.53). Then, 3 sec giving her car a deceleration a . Find the minimum value
c

later, the policewoman starts accelerating from rest at of a for which the car will not collide with the truck. Hint:
c
2
10 ft/sec until her velocity is 85 mph. How long does Enforce for all time t before the vehicles are
it take her to overtake the speeding car if it neither stopped.
slows down nor speeds up?
1.57 Point B of block B has a constant acceleration of
2
10 m / s upward. At the instant shown in Figure PI .57, it
is 30 m below the level of point A of At this time, v A

and v are zero. Determine the velocities of A and B as


a

they pass each other.


1.58 The accelerations of the translating blocks
are respectively. (See Fig­
ure P1.58.)

Figure P1.53

1.54 In the preceding problem, suppose the speeder sees


the policewoman 10 sec after she begins to move, and
2
decelerates at 3 ft/sec . How long does it take the police­
woman to pass the car if she is actually chasing a faster
speeder ahead of it?
1.55 Two cars start from rest at the same location and at
the same instant and race along a straight track. Car
2
accelerates at 6.6 ft/sec to a speed of 90 mph and then
2 Figure P1.57
runs at a constant speed. Car accelerates at 4.4 ft/sec to
a speed of 96 mph and then runs at a constant speed.
a. Which car will win the 3-mi race, and by what
distance?
b. What will be the maximum lead of 3ver ?
c. How far will the cars have traveled when
passes '
* 1.56 A car is 40 ft behind a truck; both are moving at
55 mph. (See Figure P1.56.) Suddenly the truck driver
slams on his brakes after seeing an obstruction in the road
2
ahead, and he decelerates at 10 ft/sec . Then, 2 sec later,

Figure P1.56 Figure PI.58


Page 23

Figure PI.59

Figure PI.60

Figure P1.61

The entire system is at rest at the given instant. Find how


long it will take for block to hit the ground. (Do not
assume that pulleys and remain at the same level!)
Figure P1.62
• 1 . 5 9 Block has v = 10 m / s to the right at f = Oanda
A
2
constant acceleration of 2 m / s to the left. Find the dis­
tance traveled by block ^during the interval t = 0 to 8 s. * 1.61 The cord shown in Figure PI .61, attached to the
(See Figure P1.59.) wall at D, passes around a small pulley fixed to at B; it
1.60 A man and his daughter have figured out an inge­ then passes around another small pulley and ends at
nious way to hoist 8000 lb of shingles onto their roof, point A of body. The cord is 44 m long, and the system
several bundles at a time. They have rigged a pulley onto is being held at rest in the given position. Suddenly point
a frame around the chimney (Figure PI.60) and will use B is forced to move to the right with constant acceleration
2

the car to raise the weights. When the bumper of the car is a = 2 m / s . Determine the velocity of A just before it
B

at x = 0 (neglect d), the pallet of shingles is on the ground reaches the pulley.
with no slack in the rope. While the car is traveling to the 1.62 The ends of the rigid bar in Figure PI.62 move
left at a constant speed of v = 2 mph, find the velocity
A while maintaining contact with floor and wall. End A
and acceleration of the shingles as a function of x. Do this moves toward the wall at the constant rate of 2 ft/sec.
by using the triangle to the left of the figure to express y as What is the acceleration of B at the instant when A is 6 feet
a function of x; then differentiate the result. from the wall?
Page 24

1.63 The velocity of point A in Figure PI.63 is a constant


2 m / s to the right. Find the velocity of B w h e n * = 10 m.

Small wheels

Figure PI.63

1.64 The collars in Figure PI .64 are attached at Q and C 2

to the rod by ball and socket joints. Point C has a velocity


2

of • m / s and no acceleration at the instant shown. Figure PI.64


Find the velocity and acceleration of Q at this instant.
1.65 The wedge-shaped cam in Figure PI.65 is moving
to the left with constant acceleration a . Find the acceler­
0

ation of the follower


1.66 In Example 1.10, let the equation of the incline
3
be given by x = (512/3)y. If the motion starts when
x = y = 0, find the acceleration of the plate when
y = 2 ft. Figure P1.65

1.5 Rectangular Cartesian Coordinates

In this section w e merely a d d the y a n d / o r z c o m p o n e n t s of position to


the rectilinear (x) c o m p o n e n t studied in the preceding section. This step
allows the point P to m o v e o n a curve in t w o - or three-dimensional space
instead of being constrained to m o v e m e n t along a straight line in the
reference frame
Suppose that P is in a state of general (three-dimensional) motion in
frame W e m a y study this motion b y e m b e d d i n g a set of orthogonal
axes in as s h o w n in Figure 1.4 on the next page. The position vector of
point P m a y t h e n be expressed as

in w h i c h (x, y, z) are rectangular Cartesian coordinates of P measured


along the e m b e d d e d axes a n d are unit vectors respectively paral­
lel to these axes (Figure 1.4). Using t h e basic definitions (Equations 1.9
a n d 1.10), w e m a y differentiate i a n d obtain expressions for velocity
O P

a n d acceleration in rectangular Cartesian coordinates:


Page 25

Figure 1.4 Rectangular Cartesian coordinates of P.

We shall n o w consider examples in w h i c h points m o v e in t w o a n d three


dimensions.

EXAMPLE 1 . 1 1

The position vector of a point P is given as

Find the velocity and acceleration of P at t = 1 sec.


Solution

Differentiating the position vector, we obtain the velocity vector of P:

Another derivative yields the acceleration of P:

Therefore, at t = 1 sec, the velocity and acceleration of P are

Note that the speed (magnitude of velocity) of. /sec and


the magnitude of the acceleration at We shall return to this
example in Section 1.7.

We see from t h e previous example that if t h e position vector of P is


k n o w n as a function of time, it is a very simple matter to obtain t h e
velocity a n d acceleration of the point. In the following example w e are
given t h e acceleration of P a n d asked for its position. Since this problem
requires integration instead of differentiation, initial conditions enter the
picture. These conditions allow us to compute t h e constants of integra­
tion, just as they did for rectilinear motion in t h e preceding section.
Page 26

EXAMPLE 1 . 1 2

A point Q has the acceleration vector

Att = 0, the point Q is located at (x, y, z) = (1, 3, —5) m and has a velocity vector
of When ( = 3 s , find the speed of Q and its distance from
the starting point.
Solution
Integrating, we get

in which c is a vector constant. Using the initial condition for velocity at t = 0, we


obtain

so that

Therefore

Integrating again, we get

where c' is another vector constant, evaluated below from the initial condition for
the position of Q at t = 0:

so that

and thus

Substituting the required time, t = 3 s, into the expressions for will


give the answers:

Thus the speed of Q is given by

Continuing, we have
Page 27

The distance d between Q and its starting point is therefore given by

EXAMPLE 1 . 1 3

The point P in Figure E1.13 travels on the parabola (with focal distance m)
at the constant speed of 0.2 m / s . Determine the acceleration of P: (a) as a func­
4f> m X
tion of x and (b) at x = 2 m.
Solution

x (ml
We may obtain the velocity components by differentiating:

Figure El.13

Thus

Similarly the acceleration of P is

Since | v |, or v , is constant, we have


P P

We also see from (2) that we need x; differentiating (3), we get

Substituting
or
When x = 2 m,
(3) and (4) into (2), we get
Page 28

In closing this section, w e remark that the simple forms of Equations


(1.24) a n d (1.25) are d u e to t h e fact that t h e unit vectors remain
constant in b o t h m a g n i t u d e a n d direction w h e n t h e axes are fixed in the
frame of reference. For planar applications (Chapter 3), w e shall set t h e z
c o m p o n e n t of velocity identically to zero, obtaining t h e following for a
point in p l a n e motion (moving only in a p l a n e parallel to t h e xy plane):

PROBLEMS • Section 1.5

1.67 The moving pin P of a rotating crankhasalocation


defined by

Find the velocity of P when and 2 s.


1.68 A bar of length 2L moves with its ends in contact
with the guides shown in Figure P1.68. Find the velocity
and acceleration of point C in terms of and its deriva­
tives.
Figure P1.69

Figure PI.71

1.72 Point P is constrained to move in the two slots


Figure P1.68 shown: one cut in the body the other cut in the refer­
ence frame The constant acceleration of. is given to be
2
4 c m / s to the left. If point P reaches the bottom of the
1.69 A point P moves on a circle in the direction shown slot (in. ) 2 sec after the instant shown in Figure PI.72,
in Figure PI.69. Express r in (x, y, z) coordinates and
0 P

differentiate to obtain v and a . (Angle is in radians.)


P P

1.70 Repeat the preceding problem. In this case, how­

ever, the angle . increases quadrationlly, instead of lin­


early,
1.71 A with timePaccording
point starts at to
the origin rad.
and moves along
the parabola shown in Figure PI.71 with a constant
x-component of velocity, sec. Find the veloc­
ity and acceleration of P at the point (x, y) = (1, 1). Figure PI.72
Page 29

when is at rest, a. Find the velocity Vp(t) and acceleration a (t) of P.


P

a. Through what height h did the marble move? b. Find the position, velocity, and acceleration of P
b. What distance did the marble travel? when t = 4 s.
c. Eliminate the time t from the x and y expres­
1.73 The pin P shown in Figure PI.73 moves in a para­ sions and obtain the equation of the path of P.
bolic slot cut in the reference frame and is guided by the
vertical slot in body For body m locates 1.76 The motion of a particle P is given by x = C cosh kt
the centerline of its slot. and y = C sinh kt, where C and k are constants. Find the
equation of the path of P by eliminating time t.
a. Find the acceleration of P at t = 5 s.
1.77 In the preceding problem, find the speed of P as a
b. Find the time(s) when the x and y components
functionofthedistana from the origin to P.
of a are equal.
P

1.78 Describe precisely the path of a particle's motion if


1.74 A pin P moves in a slot that is cut in the shape of a 2 2
its xy coordinates are given by (2.5f + 7, 6t + 9) meters
hyperbolic sine as shown in Figure PI.74. It is guided when t is in seconds.
along by the vertical slotted body all the points of
which have velocity 0.08 m / s to the right. Find v and a
P P
1.79 A particle P moves in the xy plane. The motion of P
when x = 0.2 m. is given by
x = 30f + 6 ft
y = 20r - 7 ft
Find the equation of the path of P in the form y = f{x).
1.80 Repeat Problem 1.79 if:
x = 5t m
3
y = -250t m
1.81 Repeat Problem 1.79 if:

Figure P1.73 x = 2 + 3 sin t ft


y = 4 cos t ft
1.82 A cycloid is the curve traced out by a point (such as
P) on the rim of a rolling wheel. In terms of the parameter
(the angle in Figure PI.82), the equations of the cycloid
are:

Noting that changes with time, find the speed of P at


radians, in terms of a and

Figure P1.74

1.75 A point P travels on a path and has the following


coordinates as functions of time t (in seconds):

Figure P1.82
Page 30

In Problems 1.83-1.86 (see Figures P1.83-P1.86), a 1.91 Two points P and Q have position vectors in a ref­
point P travels on the curve with a constant x component erence frame that are given by r = 50fl meters and
O P

of velocity, x = 3 in./sec. Each starts on the curve at r = 40i — 20fj meters. Find the minimum distance
O Q

x = 1 when t = 0. Find the velocity vector of P when between P and Q and the time at which this occurs.
f = 10 sec in each case.
1.92 Describe the path of a point P that has the following
1.83 Logarithmic curve rectangular Cartesian coordinates as functions of time:
x = a cos cot, y = a sin cot, and z = bt, where a, b, and co
1.84 Exponential curve
are constants. Identify the meanings of the three con­
1.85 First-quadrant branch of rectangular hyperbola stants.
1.86 First-quadrant branch of semicubical parabola 1.93 For the following values of the constants, find the
1.87-1.90 Find the respective acceleration vectors at velocity of P at t = 5 s in the preceding problem: a = 2 m,
f = 10 s of the points whose motions are described in b = 0.5 m / s , and co = 1.2 r a d / s .
Problems 1.83-1.86. 1.94 The acceleration of a point is given by

At t = 0. the initial conditions are that m / s and


meters. Find the position vector of P at
f = 3 s, and determine how far P then is from its position
at t = 0.
1.95 A point moves on a path, with a position vector as a
function of time given by in
Figure PI.83 units of meters when t is in seconds. Find:
a. The speed of the point at t = 0.
b. Its acceleration at t =
c. The component of the velocity vector, at t = 0,
which is parallel to the line tin the xy plane
given by shown in Figure P1.95.

Figure P1.84

Figure PI.95

Figure PI.85 • 1.96 A car travels on a section of highway that approxi­


mates the cosine curve in Figure P1.96. If the driver

Figure PI.86 Figure P1.96


Page 31

maintains a constant speed of 55 mph, determine his x 1.98 Determine the minimum magnitude of acceleration
and y components of velocity when x = 2500 ft. of the car in Problem 1.96. Where on the curve is this
• 1 . 9 7 A car travels along the highway of the preceding acceleration experienced?
problem with a constant x-component of velocity of 1.99 Find the maximum magnitude of acceleration of
54.9 mph. Over what sections of the highway does the the car in Problem 1.97. Where does it occur on the curve?
driver exceed the speed limit of 55 mph?

1.6 Cylindrical Coordinates

If a point P is moving in such a w a y that its projection into the xy plane is


more easily described with polar (r a n d i coordinates t h a n with x a n d y,
t h e n w e m a y use cylindrical coordinates to a d v a n t a g e . These coordi­
nates are nothing more t h a n the polar coordinates r a n d together with
a n "axial" coordinate z. T h u s r a n d locate the projection point of P in a
plane, while z gives the distance of P from the plane.
Embedding the same set of rectangular axes (x, y, z) in the reference
frame as w e did in the preceding section, we n o w s h o w the coordinates
r and as well (Figure 1.5). N o t e that is the projection of P into the
plane xy. From Figure 1.5 w e see that the unit vectors are d r a w n
in the xy plane a n d that:

1. The direction of is that of


2. is normal to in the direction of increasing
It will b e helpful later in the section to note carefully at this point that
zhange (in direction) with changes in but not with r or z. Thus
if the point P m o v e s along either a radial line or a vertical line, the two
unit vectors remain the same. But if P moves in such a way as to alter
Figure 1.5 Cylindrical coordinates of P. then the directions of and will vary.
The rectangular a n d cylindrical coordinates (both having z in com­
m o n ) are related t h r o u g h

which can b e differentiated to produce, by Equations (1.24) a n d (1.25),


formulas for velocity a n d acceleration in terms of the cylindrical coordi­
nates (and their derivatives) a n d the unit vectors and It is usually
more desirable, however, to express the velocity a n d acceleration in
terms of the set of unit vectors which are naturally associated
with cylindrical coordinates. Thus it is useful to express a position vector
T p as
0

Question 1.3 Why is there no term in Equation (1.30)?

Answer 1.3 From Figure 1.5, w e see that is perpendicular to r . Note, however, that
o p

implicit in the writing and use of Equation (1.30) is the polar angle
Page 32

Differentiating Equation (1.30), w e obtain the velocity of P:

To evaluate w e note from Figure 1.6 that


Hence
Figure 1.6

and thus the velocity in cylindrical coordinates takes the form

Differentiating again, w e get

Using Equations (1.32), w e find that

Thus the acceleration expression in cylindrical coordinates is

In the special case for which the motion is in a plane defined by


z = constant, w e have In this case w e need only the polar
coordinates r and and the directions of are said to be radial
and transverse, respectively.
Question 1.4 If a point P moves with
W h y then isn't I v I = r?
P

Before turning to the examples in this section, w e return briefly to


calculation of the derivatives of the unit vectors We note that
each derivative turns out to be perpendicular to the vector being differ­
entiated. To understand w h y this is the case, w e consider an alternative
derivation
Answer of one
1.4 For the thing,
formular canfoi . The But
be negative. timeI vdependence
P| ^ | r | either, of is due
because to the
the magni­
tude ofdependence
time the derivative of
of the
a vector is not equalon
coordinate to the absolutedepends
which value .of the derivativethus
explicitly; of
the magnitude of the vector. Differentiating t produces a term (r8e ) in v in addition
w e can
tore,.
write OP e P
Page 3 3

Let us study the derivative By definition,

With the aid of Figure 1.7 w e can see that:

1. The direction of approaches that of


approaches zero.
2. The magnitude of which ap­
proaches unity as approaches zero.

Figure 1.7 Change in e, as 6 changes.

Thus and w e obtain in agreement with Equa­


tion (1.33). The reader may wish to sketch a similar geometric proof of
Equation (1.36).
This mutual orthogonality of a vector and its derivative, incidentally,
is not just restricted to unit vectors. It is in fact a property of all vectors of
constant magnitude. We can show that this is the case by noting that if A
is such a vector, then

and thus

or

Hence,
vector
shall
some
or make
examples
and
provided
the
use derivative
of
ofthat
frequently
velocity
neither
are
and
mutually
throughout
the
acceleration
vector
perpendicular.
nor
thein
its
book.
cylindrical
derivative
WeThis
n ocoordinates.
vanishes,
w
is aproceed
result the
wtoe
Page 34

EXAMPLE 1 . 1 4

The pin P in Figure El. 14a moves outward with respect to a horizontal circular
2
disk, and its radial coordinate r is given as a function of time by r = 3 f / 2 meters.
2
The disk 2> turns with the time-dependent angle 6 = 4 t / 3 rad. Find the velocity
and acceleration of P at f = 1 s.
Solution

Figure E 1.14a

From Equation (1.34) we have

Velocity v P

Figure El. 14b

Thus
2
dp = 21.4 m / s

and we note that the speed o f P a t f = l s i s 5 m / s .


SinceContinuing,
at t = 1 wefrom
haveEquation
r = 3 / 2 (1.37)
m andwe8 get
= 4/3 rad, we show the preceding
results pictorially in Figures El. 14b and c.
ThusNote that there is a time, f = when the parts of the
radial component of a cancel each u m u , making this component zero at that
P
Acceleration a P
instant of time. The reader is urged to compute and sketch v and a at another
P P

Figure El. 14c time, say t = 2 s.

EXAMPLE 1 . 1 5

In the preceding example, discard the given Suppose instead that


= constant = 0.3 r a d / s and that the pin slides not only in the slot of disk
(see Figure El. 15a), but also in the spiral slot cut in the reference frame and de­
fined by meters, with n radians. Find the velocity and acceleration of
the pin when rad.
Solution
From
Figure El. 15a Therefore
Page 35

Now for the acceleration:

We shall leam in the next section that the velocity is always tangent to the
path of the point. Thus the angle cf> between the path and the —x axis can be
found from the velocity components, as shown in Figure El.15b, as follows:

Figure El. 15b

In our next examples, there is motion in the z direction as well as the


radial (r) a n d transverse (6) of t h e previous examples.

EXAMPLE 1 . 1 6
A point Q moves on a helix as shown in Figure El. 16a. The pitch, p, of the helix is
0.2 m, and the point travels at constant speed 20 m / s . Find the velocity of Q in
terms of its cylindrical components.
Solution
The meaning of the pitch of a helix is the (constant) advance of Q in the z direction
for each revolution in 8. Therefore

so that

Figure El. 16a

or, for this problem,

Noting that r = 0 since Q travels on a cylinder (with r therefore constant),


Equation (1.34) then gives the following for the point's velocity:

or, using (3),

The speed of Q is constant at 20 m / s ; thus


Page 36

From Equation (3), we then get

Hence the velocity vector of is (substituting (9) and (8) into (5))

Note that | v | = 20.0 m / s , as it must be. Note also that a larger pitch will spread
c

out the helix (see Figure El. 16b). The equations of this example then show that
Small p Larger p the component will become larger in comparison to the component for
larger p.
Figure El. 16b

EXAMPLE 1 . 1 7

Find the acceleration of in the preceding example.


Solution
From Equation (1.37) we get

Because r is constant on the cylinder, this equation reduces to

Furthermore, since | v | is constant, Equations (8) and (3) of the previous


Q

example show that are constants. Therefore there is only one non-
vanishing acceleration component here:

Note that even though point never has a radial component of velocity (see
Figure E1.17 Figure El. 17), it has only a radial component of acceleration!

EXAMPLE 1 . 1 8
Find the velocity and acceleration vectors of point in Example 1.16 if, instead of
the speed of being constant, we have its vertical position given as the function
of time:
3
z = 0.08f m
Solution

Referring to Example 1.16 (see Figure El.18), we find

Figure El. 18

Therefore
Page 37

This time, however, the velocity is seen to depend on the time; for example,
at t = 10 s,

For the acceleration, we note that is still zero, so that

This time, all three terms are nonzero. We have

Thus

In t h e final example of this section, w e consider t h e case in which, in


addition to t h e changing a n d z of t h e preceding three examples, the
radius varies.

EXAMPLE 1 . 1 9

A point P moves on a spiraling path that winds around the paraboloid of revolu­
tion shown in Figure El.19. The focal distance m, and the point P advances
4.0 m vertically with each revolution. If the speed of P is 0.7 m / s , a constant,
determine the vertical component of the velocity vector of P as a function of r.
Solution

2
From z = r , we obtain

Figure E1.19

And from the pitch relationship we get

Therefore the speed of P may be expressed as

Thus the answer is


Page 38

Let u s extend t h e preceding example slightly. W e can see that varies


with t h e radius r (distance from t h e z axis to P) a n d that it is zero initially
a n d approaches zero again for large r. Its m a x i m u m m a y b e determined
from calculus:

or

This yields the equation a n d result:

at w h i c h

Note from Equation (2) in the example that at this value of z,

FromQuestion
Equation1.5(1),By
w einspection
see that at t h e little
(with s a m eortime
no writing), what is the
maximum magnitude of the radial component of v ? P

Answer 1.5 W h e n r = 0, w e have z = 0 = Thus r, the radial component of v , is


P

maximum there at the value 0.7, w h i c h is the constant speed. (Note that decreases
continuously toward zero from there.)

PROBLEMS • Section 1.6 a n d therefore, w h e n z is m a x i m u m , the speed is

1.100 The airplane in Figure PI.100 travels at constant 1.101 A ball bearing is moving radially outward in a slot­
speed at a constant altitude. The radar tracks the plane ted horizontal disk that is rotating about the vertical z axis.
and computes the distance D, the angle and the rate of At the instant shown in Figure P1.101, the ball bearing is
change of at all times. In terms of and D, find the 3 in. from the center of the disk. It is travehng radially
speed of the airplane. outward at a velocity of 4 in. /sec relative to the disk

as it should be, since it does not change with time.


Page 39

1.103 A particle moves on a curve called the "Lemniscate


of Bernoulli,'' defined by It moves along
the branch shown in Figure PI. 103 with a"ows, and
passes through point P at f = 0. The angle increases
with time according to rad, with t measured
in seconds. At the point P, find the velocity and accelera­
tion of the particle.

Figure PI .100

Figure PI. 103

* A point P moves on the "Spiral of Archimedes" at


constant speed 2 m / s . (See Figure PI.104.) The equation
of the spiral is Find the acceleration of P when

Figure PI .101

and has a radial acceleration with respect to the disk of


2
5 in./sec outward. What would and have to be
at the instant shown for the ball bearing to have a total
acceleration of zero?
1.102 The disk shown in Figure PI. 102 is horizontal
and turns so that about the vertical. Forces cause Figure PI. 104
a marble to move in a slot such that its radial distance
2
from the center equals kt . Note that c and k are constants.
a. Find the acceleration of the marble.
b. At what time does the radial acceleration vanish?

Figure P1.105

1.105 The cardioid in Figure PI. 105 has the equation


Point P travels around this curve, in the
direction indicated, in such a way that
In terms of K and the length a, find the velocity of P at the
four points where its path intersects the coordinate axes.
Express the result in terms of radial and transverse com­
ponents, and then convert to rectangular components by
Figure P1.102 expressing in terms of at each position.
Page 40

1.106 In the preceding problem, find the acceleration of P • 1.110 The point Pin Figure PI.110 moves on the limacon
at the same four points. Again, do the problem first in defined in polar coordinates by
components and then convert the results to
components.
A point P starts at the origin and moves along the If the polar angle is quadratic in time according to
parabola shown in Figure P1.10 7 with a constant x-com- rad, find the velocity of P when it is at its highest
ponent of velocity, Using the following ap­ point.
proach, find the radial and transverse components of the 1.111 In the preceding problem, determine the accelera­
velocity and acceleration of P at the point (x, y) = (1, 1): tion of P at (a) the same highest point and (b] rad.
Find v and a in rectangular components (see Problem
P P
• 1.112 A point P moves on the figure eight in the indicated
1.71); then resolve these vectors along to obtain
direction (Figure PI.112) at constant speed 2 m / s . Find
their radial and transverse components.
the acceleration vector of P the next time its velocity is
horizontal.

Figure P1.107

1.108 Solve the preceding problem by a different ap­ Figure P1.112


proach: Recall the polar coordinate relations
and and differentiate to obtain t, f, 6, and ' 1.113 An insect is asleep on i rpm record, 6 in. from
for entry into Equations (1.34) and (1.37). the spindle. When the record is turned on, the insect
1.109 The four-leaf rose in Figure PI.109 has the Equa­ wakes up and dizzily heads toward the center, in a
tion r = 3 sin A particle P starts at the origin and straight line relative to the disk, at 1 in./sec (Fig­
travels on the indicated path with rad/sec ure PI.113). If the bug can withstand a maximum accel­
2
= constant. When P is at the highest point in the first eration magnitude of 100 in./sec , does it make it to the
quadrant, find: spindle (a) if it starts after the record is up to speed? (b) if
it starts as soon as the record is turned on? Assume
a. the speed of P that the turntable accelerates linearly (with time) up to
b. the acceleration of P speed in one revolution, and that until
in./sec.

Figure P1.109

Figure PI. 113

1.114 David throws a rock at Goliath with a sling. He


whirls it around one revolution plus 135° more and re­
Figure P1.110 leases it there, as shown in Figure PI.114, at 50 ft/sec.
Page 41

1.116 Two people moving at 2 ft/sec to the right are


using a rope to drag the box along the ground at the
lower level (Figure P1.116). Determine the speed of as a
function of the angle between the rope and the vertical.
•1.117 The rigid rod in Figure PI. 117 moves so that its
ends, A and B, remain in contact with the surfaces. If, at
the instant shown, the velocity of A is 0.5 ft/sec to the
right, find the velocity of B.
1.118 In Problem 1.117, find the acceleration of B at the
2
instant in question if the acceleration of A is 2 ft/sec to
the left at that time.
1.119 An ant travels up the banister of a spiral staircase
(Figure PI. 119) according to

Find the position and velocity of the ant when t = 30 s.

Figure PI .114

As he whirls the sling, the speed of the rock increases


linearly with the time f; that is, 0 = kt, where A: is a con­
stant. Find the acceleration of the rock just prior to re­
lease. Figure P1.117
In Problem 1.60 show that the velocity of the shin­
gles may also be obtained by simply taking the compo­
nent of the velocity of the bumper attachment point A
along the rope. Using the cylindrical coordinate expression
for velocity, explain why this procedure works.

Figure PI .116 Figure P1.119


Page 42

Figure PI .121

Figure PI.123

• 1.123 The mountain shown in Figure PI.123 is in the


2
shape of the paraboloid of revolution H — z = kr , where
H = height = 5000 ft, r is the radius at z, and k is a con­
stant. The base radius is also 5000 ft. A car travels up the
mountain on a spiraling path. Each time around, the car's
altitude is 1000 ft higher. The car travels at the constant
speed of 50 mph. Find the largest and smallest absolute
Figure PI. 122 values of the radial component of velocity on the journey,
and tell where the car is at these two times.
1.120 Find the acceleration of the ant (again at t = 30 s) in • 1.124 In the preceding problem, find the locations of the
the preceding problem. car for which the following velocity components
1.121 A point P starts at t = 0 at the origin and proceeds are equal.
along a path on the paraboloid of revolution shown in a. Radial and transverse
Figure PI. 121. The path is described (with time as a pa­
rameter) by b. Radial and vertical
c. Transverse and vertical
1.125 Show that the velocity of a point P in spherical
coordinates is given by
Find the position, velocity, and acceleration vectors of the
point when it reaches the top edge of the paraboloid.
(H, R, k-i, and k are constants.)
2

1.122 A bead B slides down and around a cylindrical sur­


face on a helical wire (Figure PI .122). The vertical drop of
the bead as changes by In is called the pitch p of the
helix; R is the radius of the helix.
a. Noting that (and therefore also z) is a function
of time, write the equations for r , v , and a
OB B B

in cylindrical coordinates.
b. For the values R = 0.3 m, p = 0.2 m, and
rad/s, find and sketch the velocity
and acceleration vectors of B when f = 10 s. Figure PI.125
Page 43

See Figure PI .125. Hint: As intermediate steps, obtain the


results

Then differentiate the simple position vector


1.126 Show by differentiating v in the preceding prob­
P

lem that the corresponding expression for the accelera­


tion in spherical coordinates is

Reference frame
Figure PI. 127

• 1.127 The velocity of a point P moving in a plane is the


resultant of one part, , along the radius from a fixed
point O to the point P, and another part, which is
always parallel to a fixed line. (See Figure PI. 127.) Prove
that the acceleration of P may be written as where r is the length of the radius vector from O to P and
where is the angle it makes with the fixed direction.

1.7 Tangential and Normal Components


In this section w e examine yet a n o t h e r m e a n s of expressing the velocity
a n d acceleration of a point P. Instead of focusing o n a specific coordinate
system, this time w e shall study the w a y in w h i c h the motion of P is
related to its path. Consequently, the c o m p o n e n t s of velocity a n d accel­
eration that result are sometimes called intrinsic or natural.
The p a t h of point P, as m e n t i o n e d in Section 1.3, is the locus of points
of the reference frame successively occupied b y P as it moves. We
begin, then, b y defining some reference point o n the p a t h . From this
arclength origin w e t h e n m e a s u r e the arclength s along the p a t h to
the point P. Clearly, the arclength coordinate d e p e n d s on the time;
that is, s = s(t).
In Figure 1.8 w e see a position vector, r , for point P. This vector w a s
0 P

seen in preceding sections to define the location of P, a n d t h u s it m a y b e


considered a function of the arclength s:

Forming the velocity of P differentiation (the definition is the same,


regardless of h o w w e choose to represent the vectors), w e get

Figure 1.8 Arclength measurement of point


P on its path.
Page 44

and, b y the chain rule,

Figure 1.9 s h o w s the quantities

Figure 1.9 Changes in \ 0P as s changes. Figure 1.10 Shrinking As toward zero.

We suggest that the reader sketch a n arc o n a large sheet of p a p e r a n d


t h e n use a straightedge to d r a w the triangle OPP' (Figure 1.10). T h e n the
limit in Equation (1.40) can be taken by, let us say, dividing As in half each
time. After just a few m o r e divisions of As o n t h e large sheet, it will
become clear that as As approaches zero — that is, as P' backs u p t o w a r d
P — t w o interesting things h a p p e n :

1. Ar OP becomes tangent to the p a t h of P at arclength s.


2. The m a g n i t u d e of approaches

These t w o results, taken together, prove that dt /ds is always a unit


OP

vector that is tangent to the p a t h a n d pointing in the direction of increas­


ing s. It is for these reasons that this vector is called e , the unit tangent.
(

Equation (1.40) m a y t h e n b e rewritten as

From Equation (1.41) w e see that the velocity vector of point P is


always tangent to its path. The absolute value | s | of the scalar part —
which is the same as the m a g n i t u d e | v | of the velocity vector v — is
P P

called the speed of P in 3, as w e m e n t i o n e d in Section 1.3.


45

Next w e shall differentiate again in order to obtain the acceleration of


P. Using Equation (1.41), w e get

Since is a unit vector, is perpendicular to and hence perpen­


dicular, or normal, to the path. Equation (1.42) shows an important
separation of the acceleration into two parts, one tangent and the other
normal to the path of P. The component tangent to the path, is (for
i the rate of change of the velocity magnitude, or speed, of P. The
component normal to the path reflects the rate of change of the direction
of the velocity vector.
Further examination o f i n Equation (1.42) is facilitated if w e
first restrict our attention to tne case of a two-dimensional (plane) curve.
To that end, let be the inclination of a tangent to the plane curve as
shown in Figure 1.11. We can visualize that as s increases, turns in such
a way that points toward the inside of the curve — that is, in the
direction of shown in the figure. We can obtain this result analytically
if w e write

Noting from Figure 1.11 that

Figure 1.11 Tangent and normal to a plane


curve.

which is a unit vector normal to the curve. If dd/ds is positive, as is


w e may differentiate
illustrated to obtain
in Figure 1.11, then is negative (curve
concave downward), then points toward the outside of the curve,
as the reader may wish to confirm with a sketch, and again points
toward the inside of the curve. Thus, in either case

where is understood to point toward the inside of the curve.


From studies in calculus the reader probably recognizes as
the curvature of a plane curve. The reciprocal of the curvature is the
radius of curvature p. The radius of curvature is the radius of the circle
that provides the best local approximation to an infinitesimal segment of
the curve. Equation (1.43) may thus be written
Page 46

In three dimensions t h e situation is m o r e difficult to visualize. We


cannot use t h e preceding d e v e l o p m e n t because cannot b e expressed as
a function of a single angle such as Consequently, in the general case
w e adopt a definition of curvature that, in t w o dimensions, reduces to
w h a t w e h a v e just established. That is, w e simply define the curvature
11p to b e t h e m a g n i t u d e of t h e vector T h e n the unit vector e„ as
defined b y

is called t h e principal unit normal to t h e curve. U p o n substituting into


Equation (1.42), w e t h e n obtain

A n alternative form in w h i c h t h e arclength p a r a m e t e r s is not explic­


itly involved follows if w e choose t h e m e a s u r e m e n t of s so that at the
instant of interest and

This expression m o r e vividly depicts t h e natural decomposition of accel­


eration into parts related to rate of change of m a g n i t u d e of velocity a n d
rate of change of direction of velocity. W e n o w consider some examples
of the use of tangential a n d n o r m a l c o m p o n e n t s .

EXAMPLE 1 . 2 0
2
A car starts at rest at A and increases its speed around the track at 6 ft/sec ,
traveling counterclockwise (see Figure El.20). Determine the position and the
2
time at which the car's acceleration magnitude reaches 20 ft/sec .
Solution

The accelerationmagnitudeofQis
Figure El.20

2
When a = 20 ft/sec , we obtain the equation
Q
Page 47

At t = 10.3 sec, s = 318 ft, which represents 318/(2jrr) = 0.253 of a revolution,


or 91.1° counterclockwise from the x axis.

EXAMPLE 1 . 2 1

Verify the results of Example 1.13, at x = 2 m, by using e, and e„ components.


Solution
We are given that s = | v | = 0.2 m / s . Since v is tangent to the path of P, we can
P P

calculate e :
t

2 2
x x

y = -^ = y f o r / = i (see Figure E1.21a)

Figure E 1.2la dy

tan 6 = = x= 2 (at the given point)

1
0 = tan- (2) = 63.4°

e, = cos 8i + sin 6) (see Figure El.21b)


= 0.448i + 0.894J
The radius of curvature comes from calculus:

Figure El.21b

EXAMPLE 1 . 2 2

In Example 1.11 find the following for point P at t = 1 sec: tangential and normal
components of acceleration, radius of curvature, and the principal unit normal.
Solution
We obtained
3 2
r O P = 2fi + f j + 3f k ft
2
v = 2i + 3f j + 6fk ft/sec
P

2
a = 6tj + 6k ft/sec
P
Page 48

If we write the velocity v as a magnitude times a unit vector, we can determine


p

and for P:

where we note that since we are choosing the direction of increasing s to be


that of the velocity.
Let us find the tangential and normal components of the acceleration of P at
f = 1 sec:

Now that we have 8„ we can use it to split the acceleration


HgiraEI.22 (at f = 1) into its tangential and normal components (see Figure E1.22). The
tangential component of a (that is, the component parallel to e ) is seen from the
P (

figure to be the dot product of with

Next we obtain the normal acceleration component by vectorially subtract­


ing the component from the total acceleration a . That is, since
P

we obtain

And since we obtain the radius of curvature:

The unit vector • follows from

as
It isa instructive
function oftotime:
make a direct calculation of since we here know
Page 49

Thus

which is, of course, the result we have already obtained by investigating the
components of the acceleration vector.

Question 1.6 How would you find the position vector from the origin
O to the center of curvature at t = 1 sec?

In closing this section, w e remark that tangential a n d n o r m a l c o m p o ­


nents of velocity a n d acceleration will b e very useful to us later w h e n w e
h a p p e n to k n o w t h e p a t h of a point (the center C of a w h e e l rolling o n a
curved track, for instance). We can t h e n use Equations (1.41) a n d (1.47) to
express v a n d a .
P P

Answer 1.6 If w e call the center of curvature C, then with


everything evaluated at the time of interest (in this case f = 1 sec).

PROBLEMS • Section 1.7

1.128 Particle P moves on a circle (Figure P1.12 8) with an 1.130 In Problem 1.67 determine the expression for
arclength given as a function of time as shown. Find the Integrate, for a motion beginning at t = 0 at (x, y)
time(s) and the angle(s) when the tangential and normal = (20, 0) m, and obtain s(f). Evaluate the arclength at
acceleration components are equal. t = 2 s and show that the result, as it should be, is
the circumference of the circle on which P travels.

3
1.131 A point P moves on a path with s = ct where
3
c = constant = 1 ft/sec . At t = 2 sec, the magnitude
2
of the acceleration is 15 ft/sec . At that time, find the rad­
ius of curvature of the path of P.

Figure PI. 128 1.132 A point D moves along a curve in space with a
speed given by where f is measured from
zero when D is at the arclength origin s = 0. If at a certain
2
1.129 In Problem 1.78 find the arclength s as a function of time t' the acceleration magnitude of D is 12 m / s and the
time. radius of curvature is 3 m, determine f'.
Page 50

1.133 At a certain instant the velocity and acceleration of 1.138 Find the radius of curvature of the "Witch of Ag-
a point are as shown in Figure PI.133. At this instant find nesi" curve at x = 0. (See Figure PI.138.)
a.

b. the radius of curvature of the path

Figure PI. 138

1.139 A point P moves from left to right along the curve


defined in the preceding problem with a constant x com­
ponent of velocity. Find the acceleration of P when it
Figure PI. 133 reaches the point (x, y) = (0, 2a).
1.140 In Problem 1.105, at the same four points express
1.134 At a certain instant, the velocity and acceleration of v in terms of tangential and normal components.
P

a point are 1.141 In Problem 1.105, for the position express


a in terms of tangential and normal components, and
P

find the radius of curvature of the path of P at that point.


1.142 A point P starts at the origin and moves along
the parabola shown in Figure PI.142 with a constant
At this instant find (a) (b) the radius of curvature of x-component of velocity, ft/sec. Find the tangen­
the path, (c) the principal unit normal. tial and normal components of the velocity and accelera­
tion of P at the point (x, y) = (1, 1).
1.135 At an instant the velocity and acceleration of a
point are

At this instant find:


a.

b. the radius of curvature of the path.


1.136 At a certain instant, the velocity and acceleration of
Figure PI. 142
a point are

* 1.143 In Problem 1.104, find the center of curvature of


the path of P when
1.144 At a particular instant a point has a velocity
Find:
At
a. this instant find: (a) the principal unit normal, (b) the cur­
b. vature of the path, and (c) the time rate of change of the
1.137 In Problem 1.103, find the radius of curvature of point's speed.
the path of P at the instant given. Note that 1.145 A point P has position vector
and meters. Find the vector from the origin to the center of
Page 51

curvature of the path of P at t = 1 s. Find at the 2


1.151 A particle moves on the curve (x — a) + y = a ,2 2

same instant. where a is a constant distance in meters. The first and


1.146 The position vector of a point is given as a function second time derivatives of the arclength s are related by
of time by Find the tangential and
normal components of acceleration at t = 1 sec and de­
termine the radius of curvature at that time. in which the constant K has the value 1 second/meter.
The distance s is measured counterclockwise on the curve
1.147 A particle P has the x, y, and z coordinates from the point (2a, 0) meters. When t = 0, the speed of
(3f, 0, 4 In f) meters as functions of time. What is the vec­ the particle is = 1 meter /second and s = a. Find the
tor from the origin to the center of curvature of the path of normal and tangential components of the acceleration at
P at t = 1 s? time t = 0. Show these components on a diagram.
1.148 Show by expressing the velocity and acceleration in
1.152 Use Example 1.21 to show that the center of curva­
tangential and normal components that
ture does not have to be on the y axis for a curve symmet­
ric abouty. Hint: Use/ = 1 and* = 2, find/?, and compare
with the distance from (2, 1) to the y axis along the normal
to the tangent at this point.
so that
* 1.153 In Problem 1.96 find the tangential and normal
components of the car's acceleration when x = 2500 ft.
Check your result by also computing x and there and
showing that
1.149 There is another formula for the radius of curvature
p from the calculus; this one is in terms of a parameter • 1.154 A particle P starts from rest at the origin and moves
such as time t, and for a plane curve: along the parabola shown in Figure PI. 154. Its speed is
given by where s is in meters per second when
s is in meters. Determine the velocity of P when its x
coordinate is 5 m. Also determine the elaosed time. Hint:
Derive this from the result of the preceding problem
and use it to find the radius of curvature at 8 sec if Substitute y' and use a table of integrals to get s(x).

1.150 Find the difference between the velocities (and also


the accelerations) of cars A and B in Figure PI. 150 if, at
the instant shown,

Figure PI. 154

1.155 There is a third unit vector associated with the mo­


tion of a point on its path. It is called the binormal and
forms an orthogonal moving trihedral with
defined by
a. Differentiate with respect to s. Then,
using prove that
= 0 and therefore that is parallel to
b. Using part (a), let „ . (T is called the
tnrsinn of the path or curve.) Then differentiate
with respect to s and prove that

Figure PI. 150 The three equations marked with daggers give the deriva-
Page 52

rives of the three unit vectors associated with a space


2.
curve and are called the Serret-Frenet formulas.
1.156 The derivative of acceleration is called the jerk and 3- V = dy /dt.
0 M

is studied in the dynamics of vehicle impact and in the 4. From dividing and rearranging steps 2 and 3, we get
kinematics of mechanisms involving cams and followers.
Show that the jerk of a point has the following form in
5. From step 1, we have
terms of its intrinsic components:
6. From steps 4 and 5, we have
7. L e t t i n g f r o m step 6 we get
= dx/2x.
8. By integrating step 7 with a table of integrals, we get
" 1.157 The following "pursuit" problem is very difficult,
yet it illustrates exceptionally well the idea that the veloc­
ity vector is tangent to the path. Thus we include it along
with a set of steps for the courageous student who wishes 9. From step 8, we get
to "pursue" it. A dog begins at the point (x, y) = (D, O) 10. From step 9, we get
and runs toward his master at constant speed 2 V . (See0
11. From step 10, squaring both sides and solving for p,
Figure PI.157.) The dog's velocity direction is always
toward his master, who starts at the same time at the
origin and moves along the positive y direction at speed
V . Find the man's position when his dog overtakes him,
0

and deterrnine how much time has elapsed. Hints: The 12. when x = D (initial condition).
man's y coordinate is y (which of course is V*„f). Show
M 13. From steps 11 and 12 we have Q = D, so that
that:
14. Integrating step 13, we get
1. where (x, y ) represent the dog's
D

coordinates at any time. 15. y = 0 when x = D (initial condition).


D

16. From steps 14 and 15 we have C = 2 D / 3 , so


2

that

17. y =V t.M 0

18. Finally, write the conditions relating to y , y ,


M D

and x when the dog overtakes his master, and


wrap it up!

Figure PI. 157


Page 53

COMPUTER PROBLEM • Chapter 1

" 1.158 A particle moves in the xy plane according to the


equation where k is a constant, and has the con­
stant speed v . The particle passes through the origin with
0

(See Figure PI.158.)


a. Show that
b. With the trigonometric substitution 6 = tan </>,
and then consulting integral tables, integrate
the equation and obtain:

Figure PI. 158

c. For the case v /k=


0 1, use the computer to plot
versus time until 0 has increased from 0 to 2n
radians.

S U M M A R Y • Chapter 1
In this chapter w e h a v e studied t h e position, velocity a n d acceleration of
a point (or particle). With O being a point fixed in the frame of reference
a n d P denoting the m o v i n g point, t h e n r is a position vector for P a n d
O P

w e defined

Velocity:

Acceleration:
With rectangular coordinates a n d associated unit vectors, a n d with 0
chosen as the origin of t h e coordinate system,

In similar fashion for cylindrical coordinates,

With a p a t h - l e n g t h parameter s(f), describing t h e motion of P o n a given


curve (path) a n d w i t h and being unit tangent a n d principal unit
normal, respectively,
Page 54

where p is the radius of curvature of the path at the point occupied by P at


time t. Sometimes it is convenient to choose to measure s(f) so that
in an interval of time of interest. In this case, w e have expressions that
don't involve s explicitly:

REVIEW QUESTIONS • Chapter 1


True or False?

1. The velocity v of a point P is always tangent to its path.


P

2. v depends on the reference frame chosen to express the position


P

of P.
3. v depends on the origin chosen in the reference frame.
P

4. The magnitude and direction in space of v depend on the choice of


P

coordinates used to locate the point relative to the reference frame.


5. a always has a nonvanishing component normal to the path.
P

6. For any point


7. A point can have f = 0 but still have a nonvanishing radial compo­
nent of acceleration.
B. If a ball on a string is being whirled around in a horizontal circle at
constant speed, the center of the ball has zero acceleration.
9. Studying the kinematics of a particle results in the same equations as
studying the kinematics of a point.
10. In our study of the kinematics of a point, the following terms have
not appeared in any of the equations: mass, force, moments, gravity,
momentum, moment of momentum, inertia, or Newtonian (inertial)
frames.
11. The acceleration vector of P, at the indicated point on the path shown
in the figure, can lie in any of the four quadrants.
12. A particle moving in a plane, with constant values of will at
all times have zero acceleration.

Answflrs: 1. T 2. T 3. F 4. F 5. F 6. T 7. T 8. F 9. T 10. T 11. F 12. F


2 • KINETICS OF PARTICLES AND
OF M A S S CENTERS OF BODIES

2.1 Introduction
2.2 N e w t o n ' s L a w s and Euler's First L a w
Motion of the Mass Center
2.3 Motions o f Particles and o f M a s s Centers o f Bodies
The Free-Body Diagram
2.4 W o r k and Kinetic Energy for Particles
Work and Kinetic Energy for a Particle
Work Done by a Constant Force
Work Done by a Central Force
Work Done by a Linear Spring
Work Done by Gravity
Conservative Forces
Conservation of Energy
Work and Kinetic Energy for a System of Particles
M o m e n t u m Form of Euler's First L a w
Impulse and Momentum; Conservation of Momentum
Impact
Coefficient of Restitution
2.6 Euler's Second L a w (The M o m e n t Equation)
Moment of Momentum
Momentum Forms of Euler's Second Law
Conservation of Moment of Momentum

SUMMARY
REVIEW QUESTIONS

Page 55
Page 56

2.1 Introduction
In this chapter w e begin to consider t h e m a n n e r in w h i c h the m o t i o n o f a
b o d y is related to external m e c h a n i c a l actions (forces a n d couples). O u r
kinematics notions o f space a n d time m u s t n o w b e a u g m e n t e d b y those o f
m a s s a n d force, w h i c h , like space a n d time, are primitives o f the subject
o f m e c h a n i c s . W e simply h a v e to agree in a d v a n c e that s o m e m e a s u r e s of
quantity of matter (mass) a n d mechanical action (force) are basic ingre­
dients in a n y attempt to analyze t h e m o t i o n o f a b o d y . W e a s s u m e that
the reader h a s a working k n o w l e d g e , p r o b a b l y from a study o f statics, of
the characteristics o f forces a n d m o m e n t s a n d their vector descriptions.
W e use t h e term body to d e n o t e s o m e material o f fixed identity; w e could
think o f a specific set o f a t o m s , a l t h o u g h t h e m o d e l w e shall e m p l o y is
b a s e d u p o n viewing material on a spatial scale such that m a s s is per­
ceived to b e distributed continuously. A b o d y n e e d n o t b e rigid or e v e n a
solid, but, since o u r subject is classical d y n a m i c s ( n o relativistic effects), a
b o d y necessarily h a s c o n s t a n t m a s s .
In S e c t i o n 2 . 2 w e u s e N e w t o n ' s laws for a particle a n d for interacting
particles to deduce that t h e s u m o f t h e external forces o n a b o d y o f a n y
size is e q u a l t o t h e s u m o f t h e m a ' s o f t h e b o d y , alternatively expressed as
the total m a s s multiplied b y t h e acceleration o f t h e m a s s center. This
result is usually called Euler's first law. Applications o f this are developed
in S e c t i o n 2 . 3 along w i t h a review o f t h e critically important c o n c e p t o f
the free-body diagram.
T h e Principle o f W o r k a n d Kinetic E n e r g y for a particle is developed
in S e c t i o n 2 . 4 , w h e r e i n are found expressions for t h e w o r k d o n e b y
several special types o f forces. T h e c o n c e p t o f a conservative force is
introduced, a n d t h e condition for w h i c h W o r k a n d Kinetic E n e r g y b e ­
c o m e s C o n s e r v a t i o n o f M e c h a n i c a l E n e r g y is established. Finally, the
implications o f W o r k a n d Kinetic E n e r g y for a s y s t e m o f particles are
explored.
In S e c t i o n 2 . 5 the i m p u l s e - m o m e n t u m form o f Euler's first l a w is
developed, a n d conditions for conservation o f m o m e n t u m are d e m o n ­
strated. Applications are m a d e to p r o b l e m s o f impact.
Euler's s e c o n d l a w is t h e subject o f S e c t i o n 2 . 6 . W e return to N e w ­
t o n ' s laws so as to derive this important result w h i c h states that t h e s u m
of t h e m o m e n t s o f the external forces o n a b o d y (or system o f particles)
equals t h e s u m o f the m o m e n t s o f t h e b o d y ' s ma's. M o m e n t u m forms o f
this are developed, primarily for later applications in C h a p t e r s 4 , 5 , a n d 7.

2.2 Newton's Laws and Euler's First Law


T h e usual starting point for relating t h e external forces o n a b o d y to its
m o t i o n is N e w t o n ' s laws. T h e s e were proposed in t h e E n g l i s h m a n Isaac
N e w t o n ' s f a m o u s w o r k t h e Principia, p u b l i s h e d in 1 6 8 7 , a n d are c o m ­
m o n l y expressed today a s :
Page 57

1. If t h e resultant force F o n a particle is zero, t h e n the particle h a s


c o n s t a n t velocity.
2. I f F # 0, t h e n F is proportional to t h e time derivative o f the particle's
m o m e n t u m mv (product o f m a s s a n d velocity).
3. T h e interaction o f t w o particles is t h r o u g h a pair o f self-equilibrating
forces. T h a t is, t h e y h a v e t h e s a m e m a g n i t u d e , opposite directions,
a n d a c o m m o n line o f action.

Clearly the first l a w m a y b e regarded a special c a s e o f t h e s e c o n d , a n d o n e


must add a n assumption about t h e frame o f reference, since a point m a y
h a v e its velocity c o n s t a n t in o n e frame o f reference a n d varying in an­
other. F r a m e s of reference in w h i c h t h e s e l a w s are valid a r e variously
called N e w t o n i a n , G a l i l e a n , or i n e r t i a l . F u r t h e r m o r e , t h e c o n s t a n t o f
proportionality in t h e s e c o n d l a w c a n b e m a d e unity b y appropriate
choices o f units so that t h e l a w b e c o m e s

w h e r e a is the acceleration o f t h e particle.


A s w e m e n t i o n e d briefly in C h a p t e r 1, a particle is a piece o f material
sufficiently small that w e n e e d n o t m a k e distinctions a m o n g its material
points w i t h respect to locations (or to velocities or accelerations). W e also
n o t e d that this definition allows, for some purposes, a truck or a s p a c e
vehicle or e v e n a p l a n e t to b e a d e q u a t e l y m o d e l e d as a particle. In the
Principia, N e w t o n u s e d h e a v e n l y b o d i e s as t h e particles in his e x a m p l e s
a n d treated t h e m as m o v i n g points s u b j e c t o n l y to universal gravitation
a n d their o w n inertia.* N e w t o n did n o t e x t e n d his w o r k to p r o b l e m s for
w h i c h it is n e c e s s a r y to a c c o u n t for t h e actual sizes o f t h e b o d i e s a n d h o w
their m a s s e s are distributed. It w a s to b e over 5 0 years before t h e S w i s s
m a t h e m a t i c i a n L e o n h a r d E u l e r p r e s e n t e d the first o f t h e t w o principles
that h a v e c o m e to b e called E u l e r ' s l a w s .
For a b o d y c o m p o s e d o f a set o f N particles, w e m a y d e d u c e Euler's
laws from N e w t o n ' s laws. A s suggested b y Figure 2 . 1 , w e s e p a r a t e the
TH
forces acting on the I particle into t w o groups: there are the N - l forces
exerted by o t h e r particles o f the system, b e i n g that exerted by the
T H
particle; then there is F , the net force exerted o n t h e I particle b y things
s

T H
external to t h e system. Applying N e w t o n ' s s e c o n d law to t h e I particle
w h o s e m a s s is m, a n d w h o s e acceleration is a,,
Figure 2.1

in w h i c h w e u n d e r s t a n d

* See C. Truesdell, Essays in the History of Mechanics (Berlin: Springer-Verlag, 1968).


Page 58

N o w w e s u m the N s u c h equations to obtain

But N e w t o n ' s third l a w tells us that

so that

Figure 2.2
T h u s w e c o n c l u d e that

w h i c h is the particle-system form o f Euler's first l a w a n d states that the


s u m o f the external forces o n the s y s t e m equals the s u m o f the ma's o f the
particles m a k i n g u p the system.
For a b o d y w h o s e m a s s is continuously distributed, as depicted in
Motion of the M a s s Center
Figure 2 . 2 , the counterpart to equation (2.3) is
W e close this section b y developing the relationship b e t w e e n the external
w h e r e acting
forces dm* is ao ndifferential
a b o d y a n edl ethe
m em
n toot ifomn aos sf ,its
a is
mits
a s sacceleration, a nthis
center. T o do d wise
the sum of the external forces acting on the body.
first construct position vectors for the particles o f a system as s h o w n in
th
(Fixed in inertial frame) Figure 2 . 3 a . T h u s the acceleration o f the i particle m a y b e written
Figure 2.3a

Applying this to E q u a t i o n ( 2 . 3 ) ,

T h e location o f the m a s s center, C, o f a system o f particles is defined b y

* dm = pdV where p is mass density and dV is an infinitesimal element of volume.


When using rectangular coordinates x, y, z, then dV = dx dy dz.
Page 59

w h e r e m is t h e m a s s o f t h e s y s t e m . T h u s E u l e r ' s first l a w b e ­
comes

or

Question 2.1 W h a t h a p p e n e d to t h e and terms in going from

(Fixed in
inertial frame) For a c o n t i n u o u s b o d y (Figure 2 . 3 b ) , the counterparts to E q u a ­
Figure 2.3b tions ( 2 . 5 - 8 ) are

and

resulting again in E q u a t i o n ( 2 . 8 ) . T h u s w e see that t h e resultant external


force o n t h e b o d y is t h e product o f t h e c o n s t a n t m a s s m o f t h e b o d y a n d
t h e acceleration a o f its m a s s center. H e n c e t h e m o t i o n o f t h e m a s s center
c

of a b o d y is g o v e r n e d b y a n equation identical in form to N e w t o n ' s


s e c o n d l a w for a particle. It is v e r y i m p o r t a n t to realize that for a rigid
b o d y t h e m a s s c e n t e r C coincides at e v e r y instant w i t h a specific material
point o f t h e b o d y or o f its rigid e x t e n s i o n (for e x a m p l e , t h e c e n t e r o f a
h o l l o w s p h e r e ) . T h i s is n o t t h e case for a d e f o r m a b l e b o d y .
S o m e t i m e s it is useful to subdivide a b o d y into t w o parts, say o f
m a s s e s m a n d m with m a s s - c e n t e r locations C a n d C . Recalling a
1 2 1 2

property o f m a s s centers,

so that after differentiating twice w i t h respect to time,

Answer 2.1 They are zero since our definition of a body requires that its mass be constant.
Page 60

W e see that Equation ( 2 . 8 ) c a n also b e written

T h e principal purpose o f this section h a s b e e n the derivation of


Equation ( 2 . 8 ) , a n d the n e x t section is devoted w h o l l y to applications o f
it. But the natural o c c u r r e n c e o f the m a s s center b e t w e e n Equations
(2.6) a n d (2.8) motivates a brief review o f the calculation of its location.
T h e e x a m p l e a n d p r o b l e m s that follow are designed to provide that
review in instances for w h i c h the b o d y comprises several parts, the m a s s
centers o f w h i c h are k n o w n .

EXAMPLE 2 . 1

A uniform prismatic rod of density p and length 2L is deformed in such a way that
the right half is uniformly compressed to length L/2 with no change in cross-
sectional area A. (See Figure E2.1.) The left half of the rod is not altered. Letting
the x axis be the locus of cross-sectional centroids, find the coordinates of the
mass center in the deformed configuration.
Solution
In the first configuration the center-of-mass coordinates are (L, 0, 0); that is, the
center of mass is at the interface of the two segments. In the second configuration,
however,
Figure E2.1

Thus the mass center no longer lies in the interface. This example illustrates that
the mass center of a deformable body does not in general coincide with the same
material point in the body at different times.

PROBLEMS • Section 2.2

2.1 Show that the mass center C of a body is unique.


Hint: Consider the two mass centers C and C , respec­
1 2

tively:

and relate R to R . (See Figure P2.1.) Using this relation,


1 2 Figure P2.1
Page 61

show that r = ro1 c 2 , which means that C and C are


1 2 2.6 Consider a body that is a composite of a uniform
the same point! sphere and a uniform cylinder, each of density p. Find the
mass center of the body. (See Figure P2.6.)
2.2 Find the mass center of the composite body shown
in Figure P2.2. Note that the three parts are composed of 2.7 Find the mass center of the body in Figure P2.7
different materials. which is a hemisphere glued to a solid cylinder of the
same density, if L = 2R.
2.8 In the preceding problem, for what ratio of L to R is
Hollow wood cylinder: the mass center in the interface between the sphere and
21 slugs the cylinder?
2.9 In Figure P2.9 find the height H of the cone of
uniform density (in terms of R) so that the mass center of
the cone plus hemisphere is at the interface of the two
shapes (i.e., z = 0 ) .

Steel bar, 1 s l u g
- Aluminum sphere.
11 slugs

Figure P2.2

Figure P2.6
2.3 Find the center of mass of the body composed of
two uniform slender bars and a uniform sphere in Fig­
ure 2.3.
2.4 Find the center of mass of the bent bar, each leg of
which is parallel to a coordinate axis and as uniform den­ Figure P2.7
sity and mass m. (See Figure P2.4.)
2.5 Repeat Problem 2.4 if the four legs have uniform,
2.10 A thin wire is bent into the shape of an isosceles
but different, densities, so that the masses of and
triangle (Figure P2.10). Find the mass center of the object,
are, respectively, m, 2m, 3m, and 4m.
and show that it is at the same point as the mass center of
a triangular plate of equal dimensions only if the triangle
is equilateral. (Area of cross section = A and mass
density = p, both constant.)

Figure P2.3

Figure P2.4 Figure P2.9 Figure P2.10


Page 62

2.3 Motions of Particles and of Mass Centers of Bodies


A l t h o u g h the m a s s center o f a b o d y does n o t always coincide w i t h a
specific material point o f the b o d y , the m a s s center is n o n e t h e l e s s clearly
an important point reflecting the distribution o f the b o d y ' s m a s s . Fur­
thermore, there are a n u m b e r o f situations in w h i c h our objectives are
satisfied if w e can determine the m o t i o n o f a n y material or characteristic
point o f the b o d y . Clearly this is the case w h e n w e attempt to describe the
orbits in w h i c h t h e planets m o v e a r o u n d the sun. Closer to h o m e , a
football c o a c h is overjoyed i f h e finds a punter w h o can consistently kick
t h e ball 6 0 yards in the air, regardless o f w h e t h e r the ball gets there e n d
over end, spiraling, or floating like a " k n u c k l e b a l l . " In such cases the
material point u p o n w h i c h w e focus our attention is unimportant. H o w ­
ever, there is a strong computational advantage in focusing on the m a s s
center: It is that the m o t i o n o f that point is directly related to the external
forces acting o n the body.
W e are m o r e likely to think o f the football as particle-like w h e n
exhibiting the knuckleball b e h a v i o r t h a n w h e n it is rapidly spinning.
N o n e t h e l e s s , the mass center's m o t i o n in e a c h case is g o v e r n e d b y Euler's
first law, although those m o t i o n s might b e quite different b e c a u s e o f the
different sets o f external force i n d u c e d b y the differing interactions o f the
ball with the air.
If the external forces acting o n the b o d y are k n o w n functions o f time,
the m a s s center's m o t i o n can b e calculated from Euler's first law:

or, alternatively,

w h e r e r is a position vector for the m a s s center. It is easily seen that two


o c

integrations o f ( 2 . 1 0 ) with respect to time yield r ( f ) provided that initial


o c

values o f TQC a n d are k n o w n .

T h e Free-Body Diagram

O n l y o n e thing remains to b e d o n e prior to studying several e x a m p l e s


that m a k e use o f Euler's first l a w to analyze the m o t i o n s o f the mass
centers o f bodies. It is to r e v i e w the c o n c e p t o f the free-body diagram
w h i c h the reader should already h a v e m a s t e r e d in the study o f statics.
W i t h o u t the ability to identify the external forces ( a n d later the m o m e n t s
also), the student will n o t b e able to write a correct set o f equations o f
motion.
A free-body diagram is a sketch o f a b o d y in w h i c h all the external
forces a n d couples acting u p o n it are carefully d r a w n with respect to
location, direction, a n d magnitude. T h e s e forces might result from
pushes or pulls, as the b o y a n d girl are exerting on the crate and rope in
Figure 2 . 4 a . O r the forces m i g h t result from gravity, such as the weight of
Page 63

Figure 2.4a

Resultant gravity
G r a v i t y a c t s on force on crate
each elemental and rope
particle

Figure 2.4b

Friction force
Resultant of
distribution
friction forces
beneath crate

Resultant of normal
N o r m a l force d i s t r i b u t i o n forces beneath cr.ite
Figure 2.4c

the crate in Figure 2 . 4 b . ( N o t e that t h e forces n e e d n o t touch t h e b o d y to


be i n c l u d e d in the free-body diagram; a n o t h e r s u c h e x a m p l e is electro­
m a g n e t i c forces.) O r the forces m i g h t result from supports, such as the
floor b e n e a t h the crate in Figure 2 . 4 c . I f the c r a t e / r o p e b o d y is acted u p o n
simultaneously b y all t h e forces in t h e s e figures, its c o m p l e t e free-body
diagram is as s h o w n in Figure 2 . 5 .
It is i m p o r t a n t to r e c o g n i z e that t h e free-body diagram:

1. Clearly identifies the b o d y w h o s e m o t i o n is to b e analyzed.


2. P r o v i d e s a catalog o f all the external forces ( a n d couples) on t h e b o d y .
3. A l l o w s us to express, in a c o m p a c t w a y , w h a t w e k n o w or c a n easily
c o n c l u d e a b o u t the lines o f action o f k n o w n a n d u n k n o w n forces. For
e x a m p l e , w e k n o w that t h e pressure (distributed n o r m a l force) e x ­
erted b y t h e floor o n t h e b o t t o m o f t h e b o x h a s a resultant that is a
Figure 2.5 force with a vertical line o f action. T h e s y m b o l N a l o n g with the
arrow is a c o d e for c o m m u n i c a t i n g the fact that w e h a v e decided to
express that u n k n o w n (vector) force as T h e fact that w e do not
k n o w t h e location o f t h e line o f action o f that force is displayed b y the
p r e s e n c e o f the u n k n o w n l e n g t h d.

In d y n a m i c s , as in statics, the o n l y characteristics o f a force that are


manifest in the equations o f m o t i o n are the vector describing the force
a n d the location o f its line o f action; that is, w e m u s t s u m u p all the
Page 64

external forces, a n d w e m u s t also s u m their m o m e n t s about s o m e point.


C o n s e q u e n t l y , everything w e n e e d t o k n o w about the external forces is
displayed o n the free-body diagram, a n d w e m a y readily c h e c k our work
by glancing b a c k a n d forth b e t w e e n our diagram a n d the equations w e
are writing.
W h e n w e focus individually o n t w o or m o r e interacting bodies, the
free-body diagrams provide an e c o n o m i c a l w a y to satisfy—and s h o w
that w e h a v e satisfied—the principle o f action a n d reaction. T h e free-
b o d y diagram o f the girl in our e x a m p l e is s h o w n in Figure 2 . 6 . S i n c e w e
h a v e already established b y Figure 2.5 that the force exerted b y the girl o n
Figure 2.6 the rope will b e then, b y the action-reaction principle, the force
exerted b y the rope o n the girl must b e as s h o w n in Figure 2.6. In
other words, consistent forces of interaction are expressed through the
single scalar a n d the arrow code.

EXAMPLE 2 . 2

Ignoring air resistance, find the trajectory of a golf ball hit off a tee at speed v and0

angle with the horizontal.


Solution
It is convenient here to set up a rectangular coordinate system as shown in Fig­
ure E2.2 and let time t = 0 be the instant at which the ball leaves the club. With x,
y, and z as the coordinates of the mass center of the ball and since the only
external force on the ball is its weight, we have from Equation (2.10):

Figure E2.2

Thus, collecting the coefficients of we obtain

Integrating, we get

Because of the way we have aligned the x and z axes,

Therefore

x(0)
center
Integrating
Our =location
of
y(0)
the
=again,
ball
of
z(0)
the
is=
wegiven
origin
0,get
so that
by
of the
C coordinate
4 = C = C =system
5 0 and atthe
6 thetrajectory
"launch"ofsite
theyields
mass
65

which describes a parabola in the xy plane—that is, in the vertical plane defined
by the launch point and the direction of the launch velocity.
Letting the time of maximum elevation be t , we find that
1 yields

so that t = (v /g)
x 0 sin 6 and the maximum elevation is

If t is the time the ball strikes the fairway (assumed level), then
2

which is, not surprisingly, twice the time (t ) to reach maximum elevation. The
2

length of the drive is

which, with v fixed, is maximized by


0 That is, for a given launch speed
we get maximum range when the launch angle is 45 °.
The results of this analysis apply to the unpowered flight of any projectile as
long as the path is sufficiently limited that the gravitational force is constant
(magnitude and direction) and we can ignore the medium (air) through which the
body moves. Interaction with the air is responsible not only for the drag (retard­
ing of motion) on a golf ball but also for the fact that its path is usually not planar
(slice or hook!). On one of the Apollo moon landings in the early 1970s, astronaut
Alan Shepard drove a golf ball a "country mile" on the moon because of the
absence of air resistance and, more important, because the gravitational accelera­
tion at the moon's surface is only about one-sixth that at the surface of the earth.

EXAMPLE 2 . 3

If the 20-kg block shown in Figure E2.3a is released from rest, find its Speed after
it has descended a distance d = 5 m down the plane. The angle and the
Figure E2.3a (Coulomb) coefficients of friction are
Page 66

Solution
In the statement of the problem we are using some loose but common terminol­
ogy in referring to the speed of the block. In fact we may only speak of the speed
of a point, but here we are tacitly assuming that the block is rigid and translating so
that every point in the block has the same velocity and the same acceleration. In
contrast to the preceding example, note that here we do not know all the external
forces on the body before we carry out the analysis, because the surface touching
the block constrains its motion. That constraint is acknowledged by expressing
the velocity of (the mass center of) the block by xi and its acceleration by
Referring to the free-body diagram shown in Figure E2.3b,
Figure E2.3b

or

First we must determine if in fact the block will move. For equilibrium, x = 0 and f
is limited by Hence
and

or

Thus the block moves (and, as it does, is acted on by N up the plane as shown in
Figure E2.3c). We note that tan is sometimes called the angle of friction. Here
Figure E2.3c But
tan is 16.7°, and this of course is the angle for which tan it means
that any angle 16.7° (like our 60°) will result in sliding, or a loss of equilib­
rium.
Having checked the statics and briefly reviewed friction, we now solve the
equation of motion for

or

Thus
Page 67

and C = 0 since
1 if t = 0 is the instant at which the block is released.
Hence

and C = 0 if we choose the measurement of x so that x(0) = 0.


2

If we let t be the time at which x = d, then


1

For we get

from which t = 1.17 s.


1

Since the velocity is given by the speed at t is merely the magnitude (or
1

absolute value) of and

Finally we should note that the plausibility of our numerical results can be
verified from the fact that, owing to the steep angle and moderate coefficient of
friction, they should be of the same orders of magnitude as those arising from a
free vertical drop (acceleration g) for which

and

EXAMPLE 2 . 4

A ball of mass m (see Figure E2.4) is released from rest with the cord taut and
Find the tension in the cord during the ensuing motion.
Cord
Solution
In this problem we make two basic assumptions:

1. The cord is inextensible.


2. The cord is attached to the ball at its mass center (or equivalently the ball is
small enough to be treated as a particle). Either way the point whose motion
Figure E2.4 is to be described has a path that is a circle. Thus the problem is similar to
Example 2.3 in that the path of the mass center is known in advance (a circle
here and a straight line there) and consequently among the external forces
are unknowns caused by constraints (the tension in the cord here and the
surface reaction in the preceding problem).
Page 68

Using polar coordinates (Section 1.6), we may express the acceleration as

Since the polar coordinate r is the constant I here, referring to the free-body
diagram in Figure E2.4 we have

so that

and

The first of these component equations (Equation (1)) yields the tension T if
we know the second (2) is the differential equation that we must integrate to
obtain . In Example 2.3 the counterpart of Equation , which
of course was easily integrated.
Here not only do we have a nontrivial differential equation in that is a
function of but we have the substantial complication that Equation (2) is
nonlinear because cos is a nonlinear function of However, a partial integra­
tion of Equation (2) can be accomplished; to this end we write the equation in the
standard form

and then multiply by to obtain

which we recognize to be

or

Equation (3) is called an energy integral of Equation (2) and is closely related to
the "work and kinetic energy" principle that is introduced in the next section.
For the problem at hand the constant C, may be obtained from the fact that
when then thus

or
Thus from Equation (3) we get
69

which we may substitute in Equation ( 1 ) to obtain

or

Even though we have not obtained the time dependence of the tension,* the
energy integral has enabled us to find the way in which the tension depends on
the position of the ball. As we would anticipate intuitively, the maximum tension
occurs when , at which time T = [3(1) — 1] mg = 2 mg.

EXAMPLE 2 . 5

A planet P of mass m moves in a circular orbit around a star of mass M, far away
P(planet) from any other gravitational or other forces (see Figure E2.5). If the planet com­
pletes one orbit in T units of time, find the orbit radius, using the fact that in a
circular orbit the speed is constant.

Solution
Writing the component of the equation of motion for the planet P in the radial
direction.

Figure E2.5 The only external force on P is gravity, in the direction (towards the star).
Letting G be the universal gravitational constant, we substitute and obtain:

where from Equation (1.37), the radial acceleration component is Since


r = R = constant, we have, with = orbital rate,

But so that

or

* This would require solving the differential equation (4) for and substituting into
Equation (5).
Page 70

EXAMPLE 2 . 6

A car accelerates from rest, increasing its speed at the constant rate of K = 6
2
ft / sec . (See Figure E2.6a.) It travels on a circular path starting at point A. Find
the time and the position of the car when it first leaves the surface due to
Sun
excessive speed.
Solution
Before the car (treated as a particle) leaves the surface, the free-body diagram is as
shown in Figure E2.6b. We shall work this problem in general (without substitut­
ing numbers until the end). The purpose is to illustrate the concept of nondimen-
sional parameters. The equation of motion in the tangential i direction is

Figure E2.6a

Equation (1) shows that the friction exerted on the tires by the road is the
external force which moves the car up the path. Note that after it passes the top of
the circular hill, we have sin and then the gravity force adds to the friction in
accelerating the car on the way down.
The following equation of motion is the one that will help us in this problem;
it equates and m a in the normal direction.
c

Question 2.2 Are the components of and m a equal in all direc­


c

tions or just in coordinate directions?


Figure E2.6b

where
We note that the car will lose contact with the road when N becomes zero.
(The ground cannot pull down on the car for further increases of t, which would
require N < 0!) Therefore, at the point of leaving the ground,

Question 2.3 What is the meaning of the fact that m cancels in Equa­
tion (3)?

Now 6 is related to s according to

Answer 2.2 The components of the two sides of a vector equation are equal in any direction.
Answer 2.3 It means the answer does not depend on the mass of the car.
Page 71

And from we get another expression for s:

Hence from equations (4) and (5) we get

Substituting for from (6) into (3), we have

or
Equation (7) allows us to solve for the dimensionless parameter q = 2
(Kt /2R),
once we have selected a value of the car's dimensionless acceleration K/g. In this
problem, for example,

The following table shows how (with a calculator)* we can quickly arrive at the
value of q that solves Equation (8):

q
0.373q

01 0.7742 0.0373
0.5 0.9595 0.1865
07 8 5 4
1 0.2930
(at the top)
1.0 0.9771 0 3730
1.3 08705 0.4849
1.6 0.6862 0 5968
1.7 0.6101 0.6341
1.69 0.6180 0.6304
1.68 0 6258 0.6266

Thus at the car leaves the circular track due to excessive


2
speed. Therefore for K = 6 ft / s e c and R = 1000 ft,

* See Appendix B for a numerical solution to this problem using the Newton-Raphson
method.
Page 72

The angle at loss of contact is given by Equation (6):

EXAMPLE 2 . 7

In the system shown in Figure E2.7a, each of the blocks weighs 10 lb and the
pulleys are very much lighter. Find the accelerations of the blocks, assuming the
belt (or rope) to be inextensible and of negligible mass.
Solution
If there is negligible friction in the bearings of the pulley, and the pulley is much
lighter than other elements of the system, then the belt tension won't change
from one side of the pulley to the other. So, referring to Figure E2.7b,

T =T
1 2
For the block in Figure E2.7c:

Figure E2.7a
And for the block and pulley in Figure E2.7d:

We also have a kinematic constraint relationship between y and y because


x 2

the belt is inextensible. For this problem it is that (see Example 1.8) and
Figure E2.7b consequently

Figure E2.7c
Solving Equations (1), (2), and (3) simultaneously,

2
so the left block accelerates upward at 6.44 ft / sec and the right block accelerates
10 2
downward at 12.9 f t / s e c .
Figure E2.7d
Page 73

EXAMPLE 2 . 8

Find the accelerations of the blocks shown in Figure E2.8a when released from
rest. Then repeat the problem with the friction coefficients reversed.

Figure E2.8a

Solution
We know from statics that if the two blocks move as a unit, their motion will occur
when

that is, when

which is the case here. But before our solution is complete we must determine
whether either block moves without the other. We consider the free-body dia­
grams of each translating block (see Figure E2.8b) and write the equations of
motion:

We mention that the sum of Equations (1) and (3) gives the "x equation" of the
overall system; the sum of (2) and (4) yields the "y equation" (C is the mass center
of the combined blocks):

Figure E2.8b

Note that f and N , disappear in (5) and (6), as they become internal forces on the
1

combined system.
Page 74

Equation (6) tells us that N = 1870 N, regardless of which motion takes


2

place. The equation for the x motion (Equation 5) shows again that if f 2

< mg(sin 25°), then one or both of the blocks must slide:

and so f cannot be zero. Assuming first that the blocks both move, then f is at its
c 2

maximum:

If they move together as one body, then Equation (5) gives us

Substituting this acceleration into Equation (1), we can check to see if body
additionally slides relative to

/ j = - 1 0 0 ( 0 . 5 8 6 ) + 415 = 356 N

But the maximum value that f can have is given by


x

Hence block slides on and the blocks do not move together; our assumption
was incorrect. We then substitute into Equation (1) and proceed:

This is then the acceleration of the top block. Substituting f into Equation (3)
x

gives

For no morion of the bottom block, f clearly needs to be at least 723 N. Since it
2max

is in fact 748 N, the bottom block does not move for this combination of parame­
ters, and
If the friction coefficients are now swapped, nothing changes until we begin
to analyze the six equations. We have

(as before)

Again, then, cannot be zero. Assuming again that the blocks both move, f is its 2

maximum and Equation (5) gives

Substituting this acceleration into Equation (1), we get

This time we have more friction than we need in order to prevent from slipping
2
on .Thus both and are 1.48 m / s .
Page 75

PROBLEMS Section 2.3

Figure P2.12

In Problem 1.114 what is the acceleration of the vertically upward, the ball passes the level of the observer
rock just after release? 1.5 sec after leaving the bat, and it passes this level again
on its away down 4 sec after leaving the bat. Disregarding
2.12 A cannonball is fired as shown in Figure P 2 . 1 2 .
air friction, find the maximum height reached by the
Neglecting air resistance, find the angle a that will result
in the cannonball landing in the box. baseball and determine the ball's initial velocity as it
leaves the bat (which is 3 ft above the ground at impact).
2.13 A baseball slugger connects with a pitch 4 ft above
the ground. The ball heads toward the 10-ft-high center- 2.15 A soccer ball (Figure P2.15) is kicked toward the
field fence, 455 ft away. The ball leaves the bat with a goal from 60 ft. It strikes the top of the goal at the highest
velocity of 125 f t / s e c and a slope of 3 vertical to 4 hori­ point of its trajectory. Find the velocity and angle at
zontal. Neglecting air resistance, determine whether the which the ball was kicked, and determine the time of
ball hits the fence (if it does, how high above the ground?) traveL..
or whether it is a home run (if it is, by how much does it 2.16 The motorcycle in Figure P2.16 is to be driven by a
clear the fence?). stunt man. Find the minimum takeoff velocity at A for
2,14 From a high vantage point in Yankee Stadium, a which the motorcycle can clear the gap, and determine
baseball fan observes a high-flying foul ball. Traveling the corresponding angle 6 for which the landing will be
tangent to the road at B and hence smooth.

Figure P2.15

Figure P2.16
Page 76

Figure P2.17

2.17 A baseball pitcher releases a 90-mph fastball 5 ft off


the ground (Figure P2.17). If in the absence of gravity the
ball would arrive at home plate 4 ft off the ground, find
the drop in the actual path caused by gravity. Neglect air
resistance.
2.18 In the preceding problem, find the radius of curva­
ture of the path of the baseball's center at the instant it
arrives at the plate.
2.19 In the preceding problem, the batter hits a pop-up
that leaves the bat at a 4 5 ° angle with the ground. The
shortstop loses the ball in the sun and it lands on second
base, ft from home plate. What was the velocity of
the baseball when it left the bat?
2.20 The pilot of an airplane flying at 300 km / h r wishes
to release a package of mail at the right position so that it
hits spot A. (See Figure P2.20.) What angle 9 should his Figure P2.21
line of sight to the target make at the instant of release?
2.21 A darts player releases a dart at the position indi­
cated in Figure P.2.21 with the initial velocity vector mak­ 2.22 In the preceding problem, suppose the initial speed
ing a 10° angle with the horizontal. What must the dart's of the dart is 20 f t / s e c . What must the angle a be if a
initial speed be if it scores a bull's-eye? bull's-eye is scored?

Figure P2.20
Page 77

Figure P2.23

2.23 Find the angle , firing velocity v and time t of


u f

intercept so that the ballistic missile shown in Fig­


ure P2.23 will intercept the bomber when x = d. The
bomber, at x = D when the missile is launched, travels
horizontally at constant speed v and altitude H. What
0

has been neglected in your solution?


2.24 The garden hose shown in Figure P2.24 expels
water at 13 m / s from a height of 1 m. Determine the
maximum height H and horizontal distance D reached by
the water.
Figure P2.26
* 2.25 In the preceding problem, use calculus to find the
angle that will give maximum range D to the water.
* 2.26 Find the range R for a projectile fired onto the in­
2.28 A child drops a rock into a well and hears it splash
clined plane shown in Figure P2.26. Determine the maxi­
into the water at the bottom exactly 2 sec later. (See Fig­
mum value of R for a given muzzle velocity u. (Angle
ure P2.28.) If she is at a location where the speed of sound
a = constant.)
is v = 1100 ft / s e c , determine the depth of the well with
s

2.27 If a baseball player can throw a ball 90 m on the fly and without considering v . Compare the two results.
s

on earth, how far can he throw it on the moon where the


gravitational acceleration is about one-sixth that on
earth? Neglect the height of the player and the air resist­
ance on earth.

Figure P2.24 Figure P2.28


Page 7 8

2.29 At liftoff the space shuttle is powered upward by 2.36 The 200-lb block is at rest on the floor ( = 0.2)
6
two solid rocket boosters of 12.9 X 1 0 N each and by the before the 50-lb force is applied as shown in Figure P2.36.
three Orbiter main liquid-rocket engines with thrusts What is the acceleration of the block immediately after
6
of 1.67 X 1 0 N each. At liftoff, the total weight of the application of the force? Assume the block is wide enough
shuttle (orbiter, tanks, payload, boosters) is about that it cannot tip over.
6
19.8 X 1 0 N. Determine the acceleration experienced by
the crew members at liftoff. (This differs from the initial
acceleration on earlier manned flights; demonstrate this
by comparing with the Apollo moon rocket, which
6
weighed 6.26 X 1 0 lb at liftoff and was powered by five
6
engines each with a thrust of 1.5 X 1 0 lb.) Neglect the Figure P2.36
change in mass between ignition and liftoff.

2.30 What is the apparent weight, as perceived through


2.37 Repeat Problem 2.36 with
pressure on the feet, of a 200-lb passenger in an elevator
2
accelerating at the rate of 10 f t / s e c upward (a) or down­
ward (b)?
2.31 When a man stands on a scale at one of the poles of
the earth, the scale indicates weight W. Assuming the
earth to be spherical (4000-mile radius) and assuming the
earth to be an inertial frame, what will the scale read
when the man stands on it at the equator?

2.32 Assuming the earth's orbit around the sun to be


Figure P2.38
circular and supposing that a frame containing the earth's
center and poles and the center of the sun is inertial,
repeat Problem 2.31. Neglect the earth's tilt.
2.39 The two blocks in Figure P2.39 are at rest before the
2.33 In an emergency the driver of an automobile ap­ 100-Newton force is applied. If friction between and the
plies the brakes and locks all four wheels. Find the time floor is negligible and if = 0 . 4 between and find the
and distance required to bring the car to rest in terms of magnitude and direction of the subsequent friction force
the coefficient of sliding friction the initial speed v, and exerted on by
the gravitational acceleration g.
2.34 A box is placed in the rear of a pickup truck. Find
the maximum acceleration of the truck for which the
block does not slide on the truck bed. The coefficient of
friction between the box and truck bed is .
2.35 The truck in Figure P2.35 is traveling at 45 mph.
Find the minimum stopping distance such that the Figure P2.39
250-Ib crate will not slide. Assume the crate cannot tip
over.
2.40 Find the largest force P for which in Figure P2.40
will not slide on

Figure P2.40

2.41 Work the preceding problem if P is applied to


Figure P2.35 instead of
Page 79

2.42 The blocks in Figure P2.42 are in contact as they


slide down the inclined plane. The masses of the blocks
are kg and kg, and the friction coeffi­
cients between the blocks and the plane are 0.5 for and
0.1 for Determine the force between the blocks and find
their common acceleration.

Figure P2.47

Figure P2.42
Figure P2.48

2.43 In the preceding problem, let be the coefficient of


friction between and the plane. Using the two motion 2.48 In Figure P2.48 the masses of ire 1 0 , 6 0 ,
equations of the blocks, find the range of values of for and 50 kg, respectively. The coefficient of friction be­
which the blocks will separate when released from rest. tween and the plane is , and the pulleys have
2.44 If all surfaces are smooth for the setup of blocks and negligible mass and friction. Find the tensions in each
planes in Figure P2.44, find the force P that will give block cord, and the acceleration of B, upon release from rest.
2
an acceleration of 4 f t / s e c up the incline. 2.49 If the system in Figure P2.49 is released from rest,
how long does it take the 5-lb block to drop 2 ft? Neglect
friction in the light pulley and assume the cord connecting
the blocks to be inextensible.
2.50 The coefficient of friction is the same be­
tween as it is between and the plane. (See Fig­
Figure P2.44 ure P2.50.) Find the tension in the cord at the instant the
system is released from rest. Neglect friction in the light
pulley.
2.45 Work the preceding problem if the planes are still
smooth but the friction coefficient between and is

2.46 Work the preceding problem if the coefficient of


friction is 0.3 for all contacting surfaces.
* 2.47 Generalizing Example 2.8, let the blocks, friction
coefficients, and angle of the plane be as shown in Fig­
ure P2.47. Show that: Figure P2.49
a. If tan motion will occur, and if so:
b. If the blocks move together
c. If , then slides on . In this case, the
lower block does not move if

d. If tan , then the lower block will not


move. In this case, the upper block slides on it
if and only if tan Figure P2.50
Page 80

2.51 The system in Figure P2.51 is released from rest. 2.57 In Figure P2.57 the masses of blocks are
50, 20, and 30 kg, respectively. Find the accelerations of
a. How far does block A move in 2 sec?
each if the table is removed. Which block will hit the floor
b. How would the solution be changed if the coef­ first? How long will it take?
ficient of friction between the floor and A were

Light pulley

Smooth

Block A: 10 kg
Block B: 20 kg
Pulleys: massless

Figure P2.57
Figure P2.51

2.58 Body in Figure P2.58 weighs 223 N and body


weighs 133 N. Neglect the weight of the rigid member
2.52 A child notices that sometimes the ball m does not
connecting and The coefficient of friction is 0.3 be­
slide down the inclined surface of toy when she pushes
tween all surfaces. Determine the accelerations of and
it along the floor. (See Figure P2.52.) What is the mini­
just after the cord is cut.
mum acceleration of to prevent this motion? As­
sume all surfaces are smooth.

Solid
sphere m

Figure P2.52

Figure P2.58
2.53 In the preceding problem, suppose the acceleration
of is 2a . What is the normal force between the vertical
min

surface of and the ball? The ball's weight is 0.06 lb.


2.54 Let the mass of in Problem 1.57 be 20 kg. What
then must be the mass of to produce the prescribed
motion? Neglect the masses of the pulleys.
• 2 . 5 5 Find the tension in the cord in Problem 1.61 at the
Figure P2.59
onset of the motion if the mass of is 10 kg.
2.56 For the cam-follower system of Problem 1.65 find
the force that must be applied to the cam to produce the 2.59 A particle P moves along a curved surface S as
motion. Let the masses of cam and follower be m and m
1 2
shown in Figure P 2.5 9. Show that P will remain in contact
and neglect friction. with S provided that, at all times,
Page 81

2.60 Find the condition for retention of contact if P and is the radius of the circle on which the ball
moves along the outside of a surface defined by the same moves, find the conical speed in terms of and the ac­
curve as in the preceding problem. (See Figure P2.60.) celeration of gravity.
2.63 For an object at rest on the earth's surface, we can
2
write mg = (GMm) / R , so that the unwieldly constant
2
GM may be replaced by gR , which for the earth is ap­
2 3 2
proximately 32.2[3960(5280)] ft / sec . Use this, plus the
result of Example 2.5, to solve for the distance above the
earth of a satellite in a circular, 90-minute orbit. (Let the
satellite replace the planet, and the earth replace the star,
in the example.)
Figure P2.60
2.64 Communications satellites are placed in geo­
synchronous orbit, an orbit in which the satellites are
2.61 A ball of mass m on a string is swung at constant always located in the same position in the sky (Fig­
speed v in a horizontal circle of radius R by a child. (See
0 ure P2.64).
Figure P2.61.)
a. Give an argument why this orbit must lie in the
a. What holds up the ball? equatorial plane. Why must it be circular?
b. What is the tension in the string? b. If the satellites are to remain in orbit without
c. If the child increases the speed of the ball, what expending energy, find the important ratio of
provides the force in the forward direction the orbit radius r to the earth's radius t . Hint:
3 e

needed to produce the ? Explain. Use Newton's law of universal gravitation

together with the law of motion in the radial


direction, and note that if the satellite were sit­
ting on the earth's surface, the force would be

Figure P2.61 so that the product Gm may be rewritten as gr ,


e
2
e

as in Problem 2.63. Use r = 3960 mi. t

Figure P2.64
Figure P2.62

2.62 There is a speed, called the conical speed, at which 2.65 Using the result of the preceding problem, show
a ball on a string, in the absence of all friction, moves that a minimum of three satellites in geosynchronous
on a specific horizontal circle (with the string sweeping orbit are required for continuous communications cover­
out a conical surface) with no radial or vertical component age over the whole earth except for small regions near the
of velocity (Figure P2.62). If is the length of the string poles.
Page 8 2

remain against the wall at the same level. Use the equa­
tion to explain the phenomenon.
Noting that each person is "in equilibrium vertically,"
solve for the minimum to prevent people from slip­
ping if R = 2 m and the expected friction coefficient be­
tween the rough wall and the clothing is

2.68 In preparation for Problem 2.69, for the ellipse


shown in Figure P2.68, the equation is

Show that the radius of curvature p of the ellipse, as a


function of x, is

Coefficient of friction
Hint: Recall from calculus that if y = y(x), then

Figure P2.66

2.66 In terms of the parameters and g defined in


Figure P2.66, find the rninimum speed for which the mo­
torcycle will not slip down the inside wall of the cylinder.
2.67 In the "spindle top" ride in an amusement park,
people stand against a cylindrical wall and the cylinder
is then spun up to a certain angular velocity . (See
Figure P2.67.) The floor is then lowered, but the people

Figure P2.68

2.69 In a certain amusement park, the tallest loop in a


somersaulting ride (Figure P2.69) is 100 ft high and
shaped approximately like an ellipse with a width of
95 ft. The ride advertises "five times the earth's pull at
over 50 mph." Use the result of the preceding exercise to
compute the radius of curvature at the bottom of the
loop. Assuming that the normal force resultant is
5 mg, determine whether or not the maximum speed is
over 50 mph. Treat the cars as a single particle.

2.70 If bar shown in Figure P2.70 were raised slowly,


block would start to slide at the angle
which was seen in statics to be one way of determining
the friction coefficient Suppose now that the bar is
suddenly rotated, starting from the position at
constant angular velocity For and
= O.lg, compute the angle at which slips downward
- 1
Figure P2.67 on and compare the result with t a n
Page 83

Figure P2.72

Figure P2.69

Figure P2.73
Figure P2.70

2.71 In the preceding problem, let remain at 0.5 but 2.74 A bead slides down a smooth circular hoop that, at
2
consider increasing the parameter . At what value a certain instant, has r a d / s e c and rad/sec
of this parameter will slide outward on ? At what in the direction shown in Figure P2.74. The angular speed
angle will this occur? of line OP at this time is r a d / s e c and
Find the value of and the force exerted on the bead
2.72 A horizontal wheel is rotating about its fixed axis
by the hoop at the given instant, if the mass of the bead
at a rate of 10 rad/sec, and this angular speed is increas­
is 0.1 kg and the radius of the hoop is 20 cm. Hint: Use
ing at the given time at r a d / s e c . (See Figure 2.72.)
2

spherical coordinates.
At this same instant, a bead is sliding inward relative to
the spoke on which it moves at 5 ft/sec; this speed is
slowing down at this time at 2 f t / s e c . If the bead weighs
2

0.02 lb and is 1 ft from the center in the given configura­


tion, find the external force exerted on the bead. Is it
possible that this force can be exerted solely by the spoke
and not in part by other external sources?

2.73 A ball bearing is moving radially outward in a


slotted horizontal disk that is rotating about the vertical Hoop
z axis. At the instant shown in Figure P2.73, the ball bear­
ing is 3 in. from the center of the disk. It is traveling
P (mass = 1 kg]
radially outward at a velocity of 4 in. / sec relative to the
disk. If r a d / s e c and is constant, find and the force
exerted on the ball by the disk at this instant. Assume no
friction and take the weight of the ball to be 0.05 lb. Figure P2.74
Page 84

• The four light rods are pinned at the origin and at 2.78 A wintertime fisherman of mass 70 kg is in trou­
each mass in such a way that as these seven bodies are ble—he is being reeled in by Jaws on a lake of frozen ice.
spun up about the vertical, the masses m move outward At the instant shown in Figure P2.78, the man has a ve­
and the mass M slides smoothly up along the vertical rod locity component, perpendicular to the radius r, of
Oy. There is a relationship between g, m, and M = 0.3 m / s at an instant when r = R, = 5 m. If Jaws pulls
such that at the particular spin-speed i , the bodies be­ in the line with a force of 100 N, find the value of
have as one rigid body (meaning remains constant). when the radius is R = 1 m. Hint:
2

Find the relationship. Hint: Use separate free-body dia­


grams of m and M, and write equations of motion for
each. The unknowns are F (force in each top rod) and F
T B

(force in each bottom rod), and . There will be three


useful equations. See Figure P2.75.

Figure P2.75

0
Figure P2.78

Figure P2.76 2.79 In the preceding problem, show that the differen­
tial equation of the man's radial motion is
to integrate this, and if
2.76 A particle P of mass m moves on a smooth, hori­ when r = 5 m, show that the radial component
zontal table and is attached to a light, inextensible cord of the man's velocity when r = 1 m is 3.04 m / s .
that is being pulled downward by a force F(t) as shown in 2.80 Particle P of mass m travels in a circle of radius a on
Figure P2.76. Show that the differential equations of mo­ the smooth table shown in Figure P2.80. Particle P is
tion of P are connected by an inextensible string to the stationary par­
ticle of mass M. Find the period of one revolution of P.
• 2.81 A weight of 100 lb hangs freely from a light rope
(Figure P2.81). It is pulled up by a force that is 150 lb at
Then show that Equation (2) implies that t = 0 but diminishes uniformly in magnitude at 1 lb per
2.77 In the preceding problem, let the particle be at foot pulled up. Find the time required to pull the weight
r = r„ at t = 0, and let the part of the cord beneath the up to the platform from rest, and determine its velocity
table be descending at constant speed v . If the transverse upon reaching the top.
c

component of velocity of P is find the • 2.82 Rework Problem 2.81, but this time assume that the
tension in the cord as a function of time t. force increases by 1 lb per foot pulled up.
Page 85

Figure P2.8D
Figure P2.83

PLATFORM

Figure P 2 . 8 4

2.85 Using the result of the preceding problem and ex­


pressing v as dx/dt, solve for x(f) if x = 0 when f = 0.
2.86 The identical plastic scottie dogs shown in Fig­
ure P2.86 are glued onto magnets and attract each other
2
with a force F = K / (2x:) , where K is a constant related
to the strength of the magnets. Find the speeds at
which the dogs collide if the magnets are initially sepa­
rated by the distance S.
Figure P2.81 Initially

2.83 The acceleration of gravity varies with distance z Start from


rest
above the earth's surface as

where g is the acceleration of gravity on the surface and R


is the earth's radius. Find the minimum firing velocity i>j
Smooth plane
that a projectile must have in order to escape the earth if
Figure P2.86
fired straight up (Figure P2.83). Hint: Not to return to
earth requires the condition that as z gets large for
the minimum possible v . t
2.87 A ball is dropped from the top of a tall building. The
2.84 The mass m shown in Figure P2.84 is given an ini­ motion is resisted by the air, which exerts a drag force
tial velocity of v in the x direction. It moves in a medium
0
2
given by Dv ; D is a constant and v is the speed of the ball.
that resists its motion with force proportional to its veloc­ Find the terminal speed (the limiting speed of fall) if there
ity, with proportionality constant K. By solving for v(x), is no limit on the drop height. What is the drop height for
determine how far the mass travels before stopping. Then which the ball will strike the ground at 95 percent of the
solve in a different manner for v(t) if v = v„ when f = 0. terminal speed?
Page 86

in 12 sec after exiting a stationary blimp. Assuming


velocity-squared air resistance, solve the differential
equation of motion

and determine the constant k.

• 2.91 In the preceding problem, suppose the parachutist


opens his chute at a height of 1000 ft. If the value of k then
2
becomes 0.63 lb / ( f t / s e c ) , find the velocity at which the
parachutist strikes the ground, if ft/sec.
2.92 The drag car of mass m shown in Figure P2.92,
traveling at speed v , is to be initially slowed primarily by
0

the deployment of a parachute. The parachute exerts a


Figure P2.88 force F proportional to the square of the velocity of the
d

car, Neglecting friction and the inertia of the


wheels, determine the distance traveled by the car before
its velocity is 40 percent of v . If the car and driver weigh
0

2 2
1000 lb and C = 0.182 l b - s e c / f t , find the distance in
feet.

Figure P2.92

2.93 In the preceding problem, suppose the drag car's


Figure P2.90
speed at parachute release is 237 mph. Find the time it
takes to reach 40 percent speed.
• 2.88 Over a certain range of velocities, the effect of air
* A 50-lb shell is fired from the cannon shown in
resistance on a projectile is proportional to the square of
Figure P2.94. The pressure of the expanding gases is in­
the object's speed. If the object can be regarded as a parti­
versely proportional to the volume behind the shell. Ini­
cle, the drag force is expressible as , in which
tially this pressure is 10 tons per square inch; just before
p is the density of the air, A is the projected area of the
exit, it is one-tenth this value. Find the exit velocity of the
object onto a plane normal to the velocity vector, and C D
shell.
is a coefficient that depends on the object's shape. If
2 2
pAC = 0.0004 l b - s e c / f t for the 76-lb cannonball of
D

Figure P2.88, find the maximum height it reaches.


Compare your result with the answer neglecting air
resistance.

• 2 . 8 9 In the preceding problem, find the velocity of the


cannonball just before it hits the ground; again compare
with the case of no air resistance.
• 2.90 A 160-lb parachutist in the "free-fall spread-stable
position" (Figure P2.90) reaches a velocity of 174 f t / s e c Figure P2.94
Page 87

2.95 The block of mass m shown in Figure P2.95 is


brought slowly down to the point of contact with the end
of the spring, and then (at t = 0) the block is released.
Write the differential equation governing the subsequent
motion, clearly defining your choice of displacement pa­ 2.98 Continue the preceding exercise and show that the
rameter. What are the initial conditions? Find the maxi­ particle's position is given by the equations
mum force induced in the spring and the first time at
which it occurs.

SVinw further that both x and y approach asymptotes as


(The limiting value of is known as the terminal
velocity of P, after which the weight is balanced by the
viscous resistance so that the acceleration goes to zero.)
* 2.99 A particle moves on the inside of a fixed, smooth
FIGURE P2.95 vertical hoop of radius a. It is projected from the lowest
point A with velocity Show that it will leave the
hoop at a height 3a/2 above A and meet the hoop again
2.96 Show that for a particle P moving in a viscous me­ at A.
dium in which the air resistance is proportional to velocity
* 2.100 The two particles in Figure P2.100 are at rest on a
(Figure P2.96), the differential equations of motion are
smooth horizontal table and connected by an inextensible
string that passes through a small, smooth ring fixed
to the table. The lighter particle (mass m) is then pro­
jected at right angles to the string with velocity v . 0

Prove that the other particle will strike the ring with
velocity Hint: Use polar coordinates
and note that is constant for each particle.

Figure P2.96

Figure P2.100
2.97 In the preceding problem, show by integration that
the components of velocity of P are given by

2.4 Work and Kinetic Energy for Particles


In E x a m p l e 2 . 4 w e w e r e a b l e to get useful i n f o r m a t i o n from a n energy
integral o f t h e governing differential equation. T h e s a m e result m a y b e
o b t a i n e d in general b y a n integration o f

F o r m i n g t h e dot product o f e a c h side with t h e velocity v c of the mass


center, w e h a v e
Page 88

Integrating,* w e get

W o r k and Kinetic Energy for a Particle

For a particle, • v dt is called the w o r k d o n e o n the particle b y


or, for
the a particle,
resultant o f external forces. W e n o t e that if there are N forces acting
on the particle, t h e n the resultant is given b y F ! + F + ... + F a n d
2 N

E a c h term o f this equation represents the rate o f w o r k o f o n e o f the forces.


T h u s the left side o f E q u a t i o n ( 2 . 1 2 ) m a y b e read as the s u m o f the works
of the individual forces acting on the particle. T h e s e statements are all
consistent with the presentation to c o m e in C h a p t e r 5 in w h i c h w e define
the rate of w o r k done b y a force F to b e F • v, w h e r e v is t h e velocity o f the
point o f the b o d y at w h i c h the force is applied. T h e left side o f Equation
( 2 . 1 1 ) m a y t h e n b e interpreted as the w o r k that would b e d o n e b y the
external forces were e a c h to h a v e a line o f action t h r o u g h the m a s s center.
For a particle, is called the kinetic energy, usually written
T. T h u s for the particle, Equation ( 2 . 1 2 ) is the w o r k a n d kinetic energy
principle:

W o r k d o n e o n the particle = C h a n g e in the particle's kinetic energy

* Sometimes (t , t ), referring to "initial" and "final," are used instead of (t


f f u t ).
2

Thus the appropriate unit of work and of energy in SI is the joule (J), the joule being
i N • m; in U.S. units the ft-lb is the unit of work and energy. The N • m and lb-ft are
usually reserved for the moment of a force. Note, that work, energy, and moment of
force all have the same dimension.
Page 89

or

For a body, the kinetic energy is defined to b e the s u m o f the kinetic


energies o f the particles constituting the b o d y . I f all the points in a b o d y
h a v e the s a m e velocity ( w h i c h is t h e n v ) , t h e n
c is the total
kinetic energy o f In general, h o w e v e r , the b o d y is turning or deforming
(or b o t h ) a n d this is n o t the case; the b o d y t h e n h a s additional kinetic
energy due to its c h a n g e s in orientation (that is, due to its angular m o t i o n )
or due to the deformation. For a b o d y w e shall also see in C h a p t e r 5
that, in general, the left side o f Equation ( 2 . 1 1 ) does n o t constitute the
total w o r k d o n e on b y the external forces a n d couples. This is b e c a u s e ,
for a b o d y , the forces do n o t h a v e to b e concurrent as t h e y are for a
particle. Equation ( 2 . 1 3 ) still turns out to b e true for a rigid b o d y , h o w ­
ever, with the two sides o f Equation ( 2 . 1 1 ) representing parts o f W
and
Finally, with n o restrictions o n the size o f the body, the energy
integral (Equation 2 . 1 1 ) states that the work that w o u l d b e d o n e if the
external forces acted at the m a s s center equals the c h a n g e in w h a t w o u l d
b e the kinetic energy if every point in the b o d y h a d the velocity o f the
m a s s center. W e could call this result the " m a s s center w o r k a n d kinetic
e n e r g y principle."

Work Done by a Constant Force


Before attempting to apply the w o r k - e n e r g y principle to a specific p r o b ­
lem, it is helpful to determine the w o r k d o n e b y t w o classes o f forces.
First, suppose F is a constant force a n d suppose w e let r b e a position
vector for the particle. T h e n

which states that the work done is the dot product of the force with the
displacement o f the particle. W e recall that this dot product c a n b e e x ­
pressed as the product o f the force m a g n i t u d e a n d the c o m p o n e n t of
displacement in t h e direction of the force or as t h e product o f t h e dis­
p l a c e m e n t magnitude a n d the c o m p o n e n t o f force in the direction o f the
displacement.

Work Done by a Central Force


T h e s e c o n d case to w h i c h w e give special attention is that o f a central
force. S u c h a force is defined to h a v e a line o f action always passing
through the s a m e fixed p o i n t in t h e frame o f r e f e r e n c e a n d a magnitude
Page 90

that d e p e n d s o n l y u p o n the distance r o f the particle from that fixed


point, as s h o w n in Figure 2.7.
T h e velocity o f the particle m a y b e expressed as

Figure 2.7

since b y Equation ( 1 . 3 8 ) w e k n o w that T h u s the w o r k d o n e by


is

If is a function o f r so that then the w o r k m a y b e written as

W o r k Done b y a Linear Spring

A special central force is that exerted b y a spring o n a particle w h e n the


other e n d o f the spring is fixed. In the case o f a linear spring o f instanta­
n e o u s l e n g t h r, w e n o t e that f=k(r — L„), w h e r e L is its natural, or
0

unstretched, length a n d k is called the spring m o d u l u s or stiffness. In this


case, or, m o r e simply, where is the
spring stretch. T h u s b y equation ( 2 . 1 6 ) ,

Question 2.4 What assumption about the mass of the spring is to be


understood in the force-stretch relationship?

Answer 2.4 (k) . (stretch) gives the equal-in-magnitude but opposite-in-direction force
acting at the ends of a spring in equilibrium. If particles in the spring are accelerating,
as is generally the case in dynamics problems, there is no simple force-stretch law. If the
spring is very light, however, so that its mass may be neglected (compared to the masses
of other bodies in the problem), and the forces on the spring are instan­
taneously related just as if the spring were in equilibrium. The hidden assumption is
that the mass of the spring may be neglected.
Page 91

Work Done by Gravity

A s e c o n d special case o f a central force is the gravitational force exerted


on a b o d y b y the earth. B y N e w t o n ' s l a w o f universal gravitation,

w h e r e r is the radius o f the earth, m is the m a s s o f the attracted b o d y , and


e

g is the gravitational strength (or acceleration) at t h e surface o f the earth.*


By Equation (2.15) we have

W e n o t e t h a t for this case the function is given b y

If the m o t i o n is sufficiently n e a r t h e surface o f the earth,

a n d so a c o n s t a n t . In this case,

= (weight) X (decrease in altitude o f m a s s center o f b o d y )


(2.20)

a n d the function b e c o m e s (if z is positive u p w a r d )


c

* The force of gravity in fact results in infinitely many differential forces, each tugging
on one of the body's particles. For nearly all applications on the planet earth, these
forces may be thought of as equivalent to a single force through the mass center of the
body. For applications in astronomy or in space vehicle dynamics, however, the gravity
moment that accompanies
Conservative Forces the force at the mass center becomes important. In Skylab,
for example, three huge control-moment gyros were present to "take out" the angular
momentum builtwup
In e a c h case e byh a av egravity moment ofthe
considered, only
w ao rfew
k hlb-ft.
a s dAnd
epenthe
d egravity
d o n l ymoment
o n the
exerted on the earth by the sun and moon's gravitation causes the earth's axis to precess
initial a n d final positions o f the p o i n t w h e r e the force is applied. S u c h a
in the heavens once every 25,800 yr. The gravity moment vanishes if the body is a uni­
forcesphere
form w h o s(which
e w o rthe
k isearthi n dise pnot,
e n dbeing
e n t bulged
o f the atp the
a t hequator
traveled
and bhaving
y the varying
point on
density). A further discussion of this luni-solar precession is presented in Chapter 7.
Page 92

w h i c h it acts is called conservative. Furthermore, the w o r k m a y b e ex­


pressed as the c h a n g e in a scalar function o f position; w e s a w this to b e
the case for the central force, and w e m a y m a k e the s a m e s t a t e m e n t for
the constant force b y d e n n i n g to b e —F • r.

Question 2.5 Why the minus sign in front of F • r?

Conservation of Energy

If all forces acting are conservative a n d if is the s u m o f all their


functions, t h e n the w o r k a n d kinetic e n e r g y equation ( 2 . 1 3 ) b e c o m e s

or

or

W e call the potential energy a n d the total (mechanical) energy.


T h u s Equation ( 2 . 2 2 ) is a s t a t e m e n t o f conservation o f m e c h a n i c a l energy
w h e n all the forces are conservative ( p a t h - i n d e p e n d e n t ) .

Question 2.6 How would Equation (2.22) read if instead of we had


chosen to construct the scalar function so that the work done by a
force is the increase in its ?

In closing it is important to realize that n o t all forces are conservative.


A n e x a m p l e is the force o f friction acting o n a b l o c k sliding on a fixed
surface. T h a t force does negative w o r k regardless o f the direction o f the
motion, a n d a potential function c a n n o t b e found for it.

Answer 2.5 It is needed so that the work equals the decrease in that is
North
Answer 2.6 we use the simply so that we may say that mechanical
energy is the sum of its two parts.

EXAMPLE 2 . 9

In an accident reconstruction, the following facts are known:

1. Identical cars 1 and 2, respectively headed west and south as indicated in


Figure E2.9, collided at point A in an intersection.
2. With locked brakes indicated by skid marks, the cars skidded to the final
positions B a n d B shown in the figure.
1 2

Assuming the cars are particles, determine their velocities immediately following
Figure E 2 . 9 the collision if the friction coefficient between the tires and road is 0.5.
Page 9 3

Solution
After separation, each car is brought to rest by the friction force acting on its tires.
For Car 1, we have

in which T = 0 since the cars end up at rest, and v is the speed of Car 1 just after
f t

the cars separate. Thus

Similarly for Car 2,

In the next section, we will return to this example and use the principle of impulse
and momentum to approximate the speeds of the cars before the impact.

EXAMPLE 2 . 1 0

We repeat Example 2.4 (see Figure E2.10): For a ball of mass m released from rest
with the cord taut and we wish to find the tension in the cord as a
function of
Solution
If, as before, we write the force-acceleration component equation in the radial
direction, we have

Now we apply Equation (2.13) by letting t be the initial time at which


t ,
and letting t be the time at which we are applying (1). We note that
2

Figure E2.10

and the work done by the cord tension T is zero since that force is always
perpendicular to the velocity of (the center of mass of) the ball. By Equation (2.20)
the work of the weight is Thus Equation (2.13) yields

or

Substituting in Equation (1) above, we get

which is precisely the result obtained previously.


Page 94

EXAMPLE 2 . 1 1

The block shown in Figure E2.11a slides on an inclined surface for which the
coefficient of friction is Find the maximum force induced in the spring if
the motion begins under the conditions shown.
Solution
We assume that the block can be treated as rigid; thus the end of the spring, once
it contacts the block, will undergo the same displacements as the mass center (or
for that matter any other point) of the block. To apply the mass-center work and
kinetic energy principle we let t be the initial time shown above and let t be the
1 2

time of maximum compression of the spring. To catalog the external forces that
do work, we consider a free-body diagram at some arbitrary instant between
and t . (See Figure E2.11b.) Since the mass center has a path parallel to the
2

inclined plane, there is no component of acceleration perpendicular to it and

or

so that the friction force is 0.3(20) = 6 lb.


Denoting the left side of Equation (2.11) by Work (fj, t ) we have
2

Figure E2.1 l a

where

and the various works are

1. For N, work t , t ) = 0 since the force is perpendicular to v at each instant.


1 2 c

2. For friction, work


3. For the weight, work
4. For the spring,

Work (t , t ) = work(t , contact) + work(contact, t )


1 2 1 2

Figure E2.1 lb

Thus,

or

From the quadratic formula,


Page 95

from which only the positive root is meaningful:

The corresponding force is 100(1.64) = 164 lb.


EXAMPLE 2 . 1 2

In the preceding example, find the next position at which the block comes to rest.
Solution
At time t the spring force (164 lb) exceeds the sum of the component of weight
2

along the plane (15 lb) and the maximum frictional resistance (6 lb), so we know
that the block is not in equilibrium and must then begin to move back up the
plane, with the friction force now acting down the plane as shown in Fig­
ure E2.12. Suppose we let r be the time at which the block next comes to rest and
3

let d represent the corresponding compression of the spring. Then, since


we have

For the spring, the work is for the friction force, the work
is for the weight, the work is Thus

or
Figure E2.12

The negative sign here tells us that the spring must be stretched 1.22 in. when the
block again comes to rest. If, as intended here, the spring does not become
permanently attached to the block on first contact (that is, contact is maintained
only in compression), our analysis only tells us that contact is broken before the
block comes to rest. We therefore need to modify the expression for the work
done by the spring, which we now see should have been It is
convenient to let e be the distance from the end of the spring to the block
(measured up the plane). Then

It is instructive to obtain this result by using the work and kinetic energy principle
over the interval t to f , for which the net work done by the spring is zero. Noting
1 3

then that the mass center of the block drops in. vertically and that the
distance traveled by the block on the plane is in., we have
Page 9 6

which is the same result we obtained before except for the third significant
figure — a consequence of rounding off at an intermediate step.

EXAMPLE 2 . 1 3

A particle P of mass m rests atop a smooth spherical surface. (See Figure E2.13a.)
A slight nudge starts it sliding downward in a vertical plane. Find the angle 6 at
L

which the particle leaves the surface.


Solution
Mechanical energy is conserved here because (1) there is no friction, (2) the
normal force does not work since it is always normal to the velocity of P, and (3)
the only other force is gravity. (See Figure E2.13b.) Therefore, using Equations
(2.21) and (2.22),

Figure E2.13a

+ r direction

Equation (1) contains two unknowns; to eliminate the velocity v , we use the
2

equation of motion in the radial direction:

But N has just become zero when P is at the point of leaving. Therefore
Figure E2.13b

Equating the right sides of Equations (1) and (3), we get


Page 97

Work and Kinetic Energy for a System of Particles


As h a s b e e n m e n t i o n e d b e f o r e , the kinetic e n e r g y o f a s y s t e m o f particles
is given b y

or

N o w if w e let W, b e t h e w o r k o f all t h e forces acting o n t h e i'th particle,

and, s u m m i n g o v e r all t h e particles,

or

for the system. T h e work, W, h o w e v e r , is the n e t w o r k o f all t h e forces


external and internal that act on t h e particles. S o m e t i m e s E q u a t i o n ( 2 . 2 3 )
can b e u s e d effectively b e c a u s e t h e n e t w o r k o f internal forces c a n b e
evaluated. For e x a m p l e , i f t w o m o v i n g particles are j o i n e d b y a linear
spring, n o simple formula c a n b e written for t h e w o r k o f t h e spring force
on one o f t h e particles. H o w e v e r , t h e n e t w o r k o f t h e e q u a l a n d opposite
spring forces on t h e t w o particles is given b y E q u a t i o n ( 2 . 1 7 ) (see P r o b ­
lem 2 . 1 2 8 ) . Particles that are rigidly c o n n e c t e d interact t h r o u g h forces
for w h i c h n o n e t w o r k is d o n e . T h u s , w e shall find in C h a p t e r 5 that
w h e n w e use for a rigid b o d y , t h e w o r k o n l y involves external
forces.
PROBLEMS • Section 2.4

2.101 A truck body weighing 4000 lb is carried by four


light wheels that roll on the sloping surface. (See Fig­
ure P2.101 ) The truck has a velocity of 5 f t / s e c in the
position shown. Determine the modulus of the spring if
the truck is brought to rest by compressing the spring 6 in.
Note: Light wheels with good bearings imply negligible
friction. Figure P2.101
2.102 The block shown in Figure P2.102 weighs 100 lb
and the spring's modulus is 10 lb/ft. The spring is un-
stretched when the block is released from rest. Find the
minimum coefficient of friction such that the block will
not start back up the plane after it stops. Figure P2.102
Page 98

2.103 The block shown in Figure P2.103 is released from


rest. What is its velocity when it first hits the spring?
2.104 How far does the block in the preceding problem
rebound back up the plane after compressing the spring?
2.105 The 6-lb block shown in Figure P2.105 is released
from rest when it just contacts the end of the unstretched
spring. For the subsequent motion, find: (a) the maximum
force in the spring; (b) the maximum speed of the block.

2.106 Block weighs 16.1 lb and translates along a


smooth horizontal plane with a speed of 36 ft / sec. (See
Figure P2.106.) The coefficient of friction between and
the inclined surface is and the spring constant is Figure P2.107
100 l b / f t . Determine the distance that moves up the
incline before coming to rest.
2.107 The weight shown in Figure P2.107 is prevented 2.108 At the instant shown in Figure P2.108, the block is
from sliding down the inclined plane by a cable. An engi­ traveling to the left at 7 m / s and the spring is un­
neer wishes to lower the weight to the dashed position by stretched. Find the velocity of the block when it has
inserting a spring and then cutting the cable. Find the moved 4 m to the left.
modulus of a spring that will accomplish this task without 2.109 Repeat the preceding problem with You
allowing the block to move back up the incline after it
will need
stops. Hint: You are free to specify the initial stretch — try
zero!

where a and c are constants.


2.110 The Bernoulli brothers posed and then solved the
"brachistochrone problem." (See Figure P2.110.) The
problem was to determine on which single-valued, con­
tinuous, smooth path a particle would arrive at B in mini­
mum time under uniform gravity, after beginning at rest
at a higher point A. Their solution, beyond the scope of
Figure P2.103 this book, was that this path of "quickest descent" is a
cycloid. Show that, regardless of the path, the speed on
arrival is as if the particle had been dropped freely
through the same height H.

Figure P2.1D5

Figure P2.108

Smooth
Figure P2.10G Figure P2.110
Page 99

2.111 A small box (see Figure P2.111) slides from rest


down a rough inclined plane from A to B and then falls
onto the loading dock. The coefficient of sliding friction
between the box and plane is Find the distance D
to the point C where the box strikes the dock.
2.112 A particle is released at rest at A and slides on the
smooth parabolic surface to B, where it flies off. (See
Figure P2.112.) Find the total horizontal distance D
that it travels before hitting the ground at C.
2.113 A particle of mass m slides down a frictionless chute
and enters a circular loop of diameter d. (See Fig­
ure P2.113.) Find the minimum starting height h in order
Figure P2.111
that the particle will make a complete circuit of the loop
and exit normally (without having lost contact with the
loop).
2.114 An 80-lb child rides a 10-lb wagon down an incline
(Figure P2.114). Neglecting all frictional losses, find the
"weight" of the child at A as indicated by a scale upon
which she is sitting.
A skier descends the smooth slope, which may be
approximated by the parabola (see Fig­
ure P2.115). If she starts from rest at "A" and has a mass
Figure P2.112
of 52 kg, determine the normal force she exerts on the
ground the instant she arrives at "B", and her acceleration
there. Note: treat the skier as a particle, and neglect fric­
tion.

Figure P2.113

Starts at rest here

Figure P2.115

Figure P2.114
Page 100

2.116 Find the speed sought in Problem 2.81 using work 2.126 The system in Figure P2.126 consists of the 12-lb
and kinetic energy. body the light pulley the 8-lb "rider" and the
10-lb body Everything is released from rest in the
2.117 Use work and kinetic energy to solve Problem 2.83.
given position. Body then falls through a hole in
2.118 Use work and kinetic energy to solve Problem 2.94. bracket which stops body Find how far descends
2.119 Use work and kinetic energy to solve Problem 2.86. from its original position.

2.12 0 Show that the equation can 2.127 At the instant shown in Figure P2.127, block is
be obtained by conservation of mechanical energy 30 m below the level of block At this time, v and v
A B

in Example 2.10. Why can this princi­ are zero. Determine the velocities of as they pass
ple not be used in Examples 2.11 and 2.12? each other. have masses of 15 kg and 5 kg, re­
spectively. The pulleys are light.
Check the solutions to Problems 2.78 and 2.79 by
using the principle of work and kinetic energy.
2.122 Show that for central gravitational force
as distinct from the uniform grav-
in the text, the potential is given by

where G is the universal gravitational constant and M and


m are the masses of the two attracting bodies. Note that in
view of Equation (2.19), one simply needs to show that

2.123 Using the result of the preceding problem, calculate


the work done by the earth's gravity on a satellite be­
tween the times of launch and insertion into a geo­ Figure P2.126
synchronous orbit with radius 6.61 times the radius of
earth. (See Problem 2.64.)
2.124 For Problem 2.49, use to find the speeds of
the blocks when the 5-lb block has droppped 2 ft.
Figure P2.127
2.125 Block in Figure P2.125 is moving downward at
5 ft / sec at a certain time when the spring is compressed
6 in. The coefficient of friction between block and the 2.128 Suppose the ends of a spring are attached to "parti­
plane is 0.2, the pulley is light, and the weights of cles" of mass m and m . Show that the sum of the works
x 2

are 161 and 193 lb, respectively. of the spring forces on the particles is given by Equation
a. Find the distance that falls from its initial po­ (2.17).
sition before coming to zero speed. • The blocks in Figure P2.129 are released from rest.
b. Determine whether or not body will start to Determine where they are when they stop permanently.
move back upward. What is the spring force then? Hint: Write the work-
energy equation for each block, add the two equations,
and use the result of Problem 2.125. Also, think about the
motion of the mass center.

Figure P2.125 Figure P2.129


Page 101

• 2.130 Show that if the surface in Example 2.13 is the


parabola shown in Figure P2.130, the particle will never
leave the surface. Hint: Show that

and use this in our equation:

Figure P2.130

Earth
• 2.131 Find the least velocity with which a particle could
Moon
be projected from the moon and reach the earth. (See
Figure P2.131.) For this problem assume that the centers
together
of withand earth are both fixed in an inertial frame.
the moon

Figure P2.131

2.5 Momentum Form of Euler's First Law


T h e momentum o f a particle is defined to b e t h e product o f its m a s s a n d
its velocity. F o r a s y s t e m o f particles (Figure 2 . 8 a ) t h e m o m e n t u m is
defined to b e t h e s u m o f t h e m o m e n t a o f t h e particles in t h e s y s t e m . T h u s ,
if w e d e n o t e t h e m o m e n t u m o f a s y s t e m (or b o d y ) b y L,* t h e n

or, for a b o d y o f c o n t i n u o u s l y distributed m a s s (Figure 2 . 8 b ) ,


(Fixed in inertial frame)
Figure 2.8a
t h
If R is a position v e c t o r for the 1 particle, t h e n
i

and Equation (2.24) becomes

or
(Fixed in
inertial frame)
Figure 2.8b

* Momentum is sometimes called linear momentum.


Page 102

Recalling [see Equation ( 2 . 7 ) a n d Figure 2 . 3 a ] that

then

or

L = mv c (2.26)

w h e r e v is the velocity o f the m a s s center o f the s y s t e m or body.


c

T h e c o n n e c t i o n b e t w e e n external forces a n d m o m e n t u m n o w can b e


m a d e easily b y differentiating Equation ( 2 . 2 6 ) to obtain

But

so that

w h i c h is the m o m e n t u m form o f Euler's first law.

Question 2.7 Should we expect Equation (2.27) to be valid for a sys­


tem for which the mass is changing with time, such as a rocket with its
varying-mass contents?

Impulse and M o m e n t u m ; Conservation of M o m e n t u m

A straightforward integration o f the first l a w o f m o t i o n (Equation 2 . 2 7 )


yields

Answer 2.7 No, at several places in the development we have needed to require that
the mass be constant.
* The impulse is sometimes called the linear impulse.
Page 103

a specific time interval. If, during s o m e time interval, t h e s u m o f the


external forces vanishes, t h e n a n d h e n c e the m o m e n t u m is a c o n ­
stant, or is conserved, during that interval.
S i n c e Equation ( 2 . 2 8 ) is a vector equation (unlike the scalar w o r k a n d
kinetic e n e r g y equation), w e m a y u s e a n y or all o f its c o m p o n e n t e q u a ­
tions. For e x a m p l e :

and similarly for y a n d z. W e n o t e that w e m a y h a v e a planar situation, for


example, in w h i c h over an interval. I f this is t h e case,
m o m e n t u m is c o n s e r v e d in the x direction but not in the y direction.*

Impact

S o m e t i m e s it is possible, b y conservation o f m o m e n t u m , to obtain limited


quantitative information a b o u t the m o t i o n s o f colliding bodies. A s a rule
this can b e d o n e w h e n the b o d i e s interact for a relatively brief interval
— before a n d after w h i c h it is r e a s o n a b l e to treat their m o t i o n s as rigid.
W h i l e the analysis is b e s t discussed w i t h examples, w e m a k e the obser­
vation h e r e that generally it m a k e s little sense to treat the bodies as rigid
during the collision. I f w e wish to describe the m o t i o n that e n s u e s w h e n a
bullet is fired into a w o o d e n b l o c k , for e x a m p l e , t h e b l o c k clearly cannot
be regarded as rigid during the penetration process. O n the other h a n d , it
m a y b e quite plausible to a s s u m e that rigid m o t i o n o f the b l o c k a n d
e m b e d d e d bullet occurs subsequent to p e r m a n e n t reorientation o f m a t e ­
rial.
A k e y feature in the analysis o f collision (or impact) p r o b l e m s is t h e
fact that the m o m e n t u m o f a b o d y m a d e u p o f t w o parts is the s u m o f the
m o m e n t a o f the individual parts. T h i s feature follows directly from the
definition o f a b o d y ' s m o m e n t u m as the integral

w h e r e the subscripts (1 a n d 2 ) identify the t w o constituent parts o f the


body.

EXAMPLE 2 . 1 4

A wooden block of mass m is at rest on a smooth horizontal surface when it is


x

struck by a bullet of mass m traveling at a speed v as shown in Figure E 2 . 1 4 a .


2

After the bullet becomes embedded in the block, the block slides to the right at
speed V. Find the relationship between v and V.

Smooth

Figure E2.14a * Ballistics problems are of this type if air resistance is neglected.
Page 104

Solution
Let t be the time at which the bullet first contacts the block and let f be the time
1 2

after which the bullet/block composite behaves as a rigid body in translation.


For t < t < t , a complex process of deformation and redistribution of mass is
x 2

ocairring within the block. If we isolate the block/bullet system during this
interval (see Figure E2.14b), Equation (2.28) yields

Figure E2.14b
But

and

since v is the speed of the mass center of the bullet and the block has no momen­
tum at t . Therefore, equating the coefficients, we get
1

or

We note that, in the absence of an external force with a horizontal component,


the horizontal component of momentum is conserved.
While we cannot calculate the reaction N during the collision, we can calcu­
late its impulse:

or

Similarly, the impulse of the force F exerted on the bullet by the block can be
calculated if we apply Equation (2.28) to the bullet:

The reader should note that with a high-speed collision occurring in a short
period of time, the impulses can be accurately estimated by neglecting the
impulses of the weights of the bodies. In this example we would have
Other examples of impact problems are treated in Chapter 5.
Page 105

EXAMPLE 2 . 1 5

A block is at rest on a smooth horizontal surface before being struck by an


identical block sliding at speed v. (See Figure E2.15a.) Find the velocities of the
two blocks after the collision assuming (1) that they stick together or (2) that the
system experiences no loss in kinetic energy.

Figure E2.15a Solution


Let v and v be the speeds of the mass centers of the left and right blocks at the
L R

end of the collision; that is, is the velocity of the left block. The free-body
diagram of the system of two blocks during the collision (see Figure E2.15b)
shows that there is no external force with a horizontal component. Thus the
horizontal component (the only component not zero here) of momentum is
conserved and

Figure E2.15b
or

If the blocks remain attached after the collision is completed and they are
behaving as rigid bodies, we have

so that

Question 2.8 What would be the common velocity if the right block
had 100 times as much mass as the left block? If it had 10,000 times as
much mass?

If, however, the blocks do not stick together, the conservation of momentum
statement alone is not adequate to deterrnine their subsequent velocities. What
we need is some measure of their tendency to bounce off each other — or, to put
it another way, a measure of how much energy is expended in permanent defor­
mations or vibrations (or both) of the blocks. The parameter used to describe
these effects (the coefficient of restitution) is discussed in the text that follows this
example. At this point we simply note that when the blocks stick together the
kinetic energy of the system is less after the collision than before. That loss is

which is to say that one-half of the mechanical energy was dissipated in the
collision in this case.
The other extreme case is that in which no mechanical energy is expended
during the collision process. In this case

Answer 2.8 v / 1 0 1 ; v / 1 0 , 0 0 1 .
Page 106

But since v = v — v from Equation (2), we have


R L

or

Therefore either v = 0 and u = v, or v = v and v = 0. The latter case must be


L R L R

rejected as physically meaningless since it would require the left block to pass
through the stationary right block.
An extension of this result to the case of three blocks is shown in
Figures E2.15c and E2.15d:

Rest Rest Rest Rest

Before After
Figure E2.15c Figure E2.15d

If we let the spacing between the blocks initially at rest approach zero and add
more of them, then we have the mechanism for a popular adult toy (see
Figures E2.15e,f):

Before After
Figure E2.15e Figure E2.15f

Coefficient of Restitution
In t h e p r e c e d i n g e x a m p l e w e n o t e d t h e n e e d for s o m e m e a s u r e o f the
capacity o f colliding b o d i e s t o r e b o u n d off e a c h other. T h e introduction
of a p a r a m e t e r called t h e coefficient o f restitution w h i c h provides this
information is m o s t easily a c c o m p l i s h e d t h r o u g h a s i m p l e e x a m p l e .
S u p p o s e that, as depicted in Figure 2 . 9 , t w o disks are sliding a l o n g a
s m o o t h floor. T h e p a t h s are t h e s a m e straight line a n d disk is just a b o u t
to o v e r t a k e a n d c o n t a c t disk at time t . T h e centers o f m a s s o f t h e disks
1

will a p p r o a c h e a c h o t h e r until, at time t , t h e y h a v e t h e c o m m o n velocity


2

v . T h e n t h e y will r e c e d e from e a c h o t h e r until at time t c o n t a c t is


c 3

b r o k e n . T h e e q u a l a n d opposite forces o f interaction F(t) are s h o w n on


t h e disks in Figure 2 . 9 . A p p l y i n g t h e i m p u l s e - m o m e n t u m principle dur-
Page 107

ing the intervals o f approach and t h e n separation o f the centers o f m a s s


of the disks,

and

Defining the coefficient of restitution, e, b y

Figure 2 . 9 w e obtain, after using the i m p u l s e - m o m e n t u m equations above,

Eliminating v , there results


c

w h i c h is s e e n to b e t h e quotient o f the "relative velocity o f separation"


a n d the "relative velocity o f a p p r o a c h . " T h e coefficient o f restitution is
inherently nonnegative, a n d the case e = 0 yields v = v , w h i c h m e a n s
Af Bf

that t h e disks stick together. In E x a m p l e 2 . 1 5 , for the case o f n o energy


dissipation, w e h a d

Exercise P r o b l e m 2 . 1 4 7 provides an outline o f p r o o f that, u n d e r the


conditions o f our discussion here,
T h e impact just described is called central, b e c a u s e the line o f action
of the equal a n d opposite forces o f interaction is the line joining the m a s s
centers o f the bodies. It is also called direct b e c a u s e the preimpact veloci­
ties are parallel to that line of action. Generalization to the case o f indi­
rect, but still central, impact is easily accomplished, assuming the disks
are s m o o t h a n d t h a t t h e time o f contact is so small that there are n o
significant c h a n g e s in their positions during the collision. T h e n , the v e ­
locity c o m p o n e n t s perpendicular to t h e line o f action o f t h e impulsive
force (called the line o f impact) are u n c h a n g e d b y the collision. T h e
Page 108

e q u a t i o n s a b o v e n o w refer t o v e l o c i t y c o m p o n e n t s a l o n g t h e l i n e o f i m ­
pact. T h u s , t h e coefficient o f restitution i s t h e r a t i o o f r e l a t i v e v e l o c i t y
c o m p o n e n t s a l o n g t h e line o f i m p a c t .
Experiments* indicate that the coefficient of restitution depends
upon just about everything involved in an impact: materials, geometry,
and initial velocities. Therefore, numerical values must be used with care.
Nonetheless, the fact that the coefficient must have a value between zero
and unity is valuable information in bounding the behavior of colliding
bodies. The use of the coefficient of restitution for other than central
impact is discussed in Chapter 5.

EXAMPLE 2 . 1 6

Two identical hockey pucks collide, coming into contact in the positions shown.
Their velocities before the collision are also shown in Figure E2.16a.

Line of impact

Figure E2.16a

If the coefficient of restitution is 0.8, find the velocities of the pucks after the
collision. Then find the impulse of the force of interaction.
Solution
Neglecting friction and assuming insignificant deformation, the forces of interac­
tion will act along the line of impact shown in Figure E2.16a. It is convenient to
choose and parallel and perpendicular to this line as shown. Let m be the mass
of each puck and let v and Af be the final and initial velocities of puck. and
similarly for puck

Thus

where v is the unknown component along the line of impact. Also


x

and

* See W. Goldsmith, Impact (London: Edward Arnold Publishers, Ltd., 1960).


Page 109

Since there are no external forces on the system of two pucks, momentum is
conserved:

The component equation for the directions perpendicular to the line of impact is
automatically satisfied, and for the -direction

or

By the definition of the coefficient of restitution

or

Solving (1) and (2),

so that

and

Figure E2.16b Thus the paths of the pucks are as shown in Figure E2.16b.
To compute the impulse of the force of interaction we apply the impulse-
momentum principle to puck noting that a hockey puck weighs about
6 ounces. Thus with m = ( 6 / 1 6 ) / 3 2 . 2 = 0.0116 slug,

EXAMPLE 2 . 1 7

In Example 2.9 we found the velocities of the identical Cars 1 and 2, just after
they collided in the given position, to be 40.1 and 25.4 ft / s e c , respectively, as
shown in Figure E2.17. Now, using the principle of impulse and momentum,
find the velocities of the cars prior to impact. Remember that Cars 1 and 2 were
heading west and south, respectively. Assume the collision is instantaneous.
Solution
If the impact occurs over a vanishingly small time then the impulse from the
road (due to the friction force on the tires) during the impact is negligible, so that
the linear momentum of the system of two cars may be assumed to be conserved
Page 110

North

Figure E2.17

during In expressing this conservation, we shall use "i" for initial (before
impact) and "f " for final (after impact):

or

and

Using the results of Example 2.9 and the angles in the figure above, we obtain the
following post-collision velocity components:

Using Equations (1,2), we find

We remark that the energy lost during the collision may now be calculated:

Note that the work done by the road friction was (see Example 2.9) equal to
0.5(32.2)(50 + 20)m = 1130m, and that this energy change plus the 260m lost in
the collision (to deformation, sound, vibration, etc.) gives the total original kinetic
energy, 1390 m.
Page 111

PROBLEMS • Section 2.5

2.132 Figure P2.132 presents data pertaining to a system 2.133 The astronaut in Figure P2.133 is finding it difficult
of two particles. At the instant shown find the: to stop his forward momentum while jogging on the
moon. Using a friction coefficient of and a gravi­
a. Position of the mass center
tational acceleration one-sixth that of earth's, illustrate
b. Kinetic energy of the system the difficulty of stopping a forward momentum of mv
c. Linear momentum of the system = (5 slugs)(12 f t / s e c ) . Specifically, use the principle of
d. Velocity of the mass center impulse and momentum to find the time it takes to stop
e. Acceleration of the mass center on earth versus on the moon.
2.134 A horizontal force F(f) is applied for 0.2 sec to a cue
ball (weighing 0.55 lb) by a cue stick; the form of the force
is as shown in Figure P2.134. If the velocity of the center
of the ball is 8 ft / sec after contact with the stick is broken,
find the peak magnitude F of the force. Neglect friction.
0

Force F is measured in pounds.


2.135 The 50-lb box shown in Figure P2.135 is at rest
before the force F(f) = 5 + 2f pounds is applied at t = 0.
Assume the box to be wide enough not to tip over and
suppose the coefficient of friction between box and floor
to be 0.2. Find the velocity of (the mass center of) the box
Fiqure P2.132 at f = 10 sec.
2.136 Repeat the preceding problem for the case in which
the force F(f) has a vertical component as shown in
Figure P2.136.
2.137 A force P applied to at t = 0 varies with time
according to lb, where t is in seconds.
(See Figure P2.137.) How long will it take for to begin
sliding? What will be its velocity at t = 30 sec?

Figure P2.133
Figure P2.135

Figure P2.136

Figure P2.134 Figure P2.137


Page 112

2.138 An unattached 2.2-lb roofing shingle slides down­


ward and strikes a gutter. (See Figure P2.138.) The angle
at which the shingle would be just on the verge of slipping
is 20°. Determine the impulse imparted to the shingle by
the gutter if there is no rebound. If the interval of impact is
0.1 sec, find the average force imparted to the gutter by
the shingle.

2.139 Two railroad cars are coupled by a collision occur­


ring just after the instant shown in Figure P2.139. Ne­
glecting the impulse caused by friction from the tracks, Figure P2.142
determine the final velocity of the two cars as they move
together.
2.140 In the preceding problem, find the average impul­ 2.142 A man of mass m and a boat of mass M are at rest as
sive force between the cars if the coupling requires 0.6 sec shown in Figure P2.142. If the man walks to the front of
of contact. the boat, show that his distance from the pier is then
is the ratio of the masses
2.141 In a rail yard a freight car moving at speed v strikes of man and boat. Explain the answer in the limiting cases
two identical cars at rest. (See Figure P2.141.) Neglecting in which m Neglect the resistance of
any resistance to rolling, find the common velocity of the the water to the boat's motion.
three-car system after the coupling has been completed
and any associated vibrations have died out. '2.143 Two men each of mass m stand on the ends of a
flatcar of mass M. The car is free to move on frictionless
level tracks. All is at rest initially. One man runs to the
right end of the car and jumps off horizontally, parallel to
the tracks with a velocity U relative to the car. Then the
other man runs to the left end of the car and jumps off
horizontally, parallel to the tracks also with a velocity U
relative to the car. Find the final velocity of the car and
indicate clearly the direction of its motion.

* 2.144 In Figure P2.144 the man of mass m stands at end A


of a 20-ft plank of mass 3m that is held at rest on the
smooth inclined plane by the cord. The man cuts the cord
and runs down to end B of the plank. When he gets there,
Figure P2.138 end B is in the same position on the plane as it was origi­
nally. Find the time it takes the man to run down from A
toB.
I ft/sec
At rest

20 tons 30 tons
Figure P2.139

Figure P2.141
Page 113

Horizontal

Line of centers

Smooth

Goal line
Figure P2.144 Figure P2.149

2.145 A ball is dropped from a height Hand bounces. (See 2.148 Use the result of the preceding problem to show
Figure P2.145.) If the coefficient of restitution is e, find that for a head-on collision at equal speeds v and equal
the height to which the ball rises after the second bounce. masses m,
2.146 Two identical elastic balls and move toward
each other. Find the approach velocity ratio that
will result in coming to rest following the collision. The
coefficient of restitution is e. (See Figure P2.146.) so that if e = 0 then all of the initial T is lost and if e = 1
then none of T is lost. Is this true for differing speeds and
2.147 Use the two equations
masses?
2.149 In soccer, a goal is scored only when the entire
and ball is over the entirety of the 4-in.-wide goal line. (See
Figure P2.149.) Neglecting friction between ball and
post, determine the maximum coefficient of restitution
( = coefficient of restitution)
for which a goal will be scored before the ball hits the
ground. The velocity of the ball's center C makes an angle
to prove that the loss in kinetic energy as the bodies and
with the horizontal of 15°. Neglect the deviation caused
collide (Figure P2.147) is
by gravity on the trajectory between post and ground.
2.1 SO Repeat Example 2.16 for the line of impact shown
in Figure P2.150.

Deduce from this result that

Figure P2.146

Figure P2.145 Figure P2.147 Figure P2.150


Page 1 1 4

2.151 Let disk in Problem 2.150 weigh 8 oz (and 2.155 Using the angle a that will land the cannonball of
weigh 6 oz as before), and then repeat the problem. Problem 2.12 in the cart, find the maximum deflection of
2.152 Repeat Example 2.16 for the case where the line of the spring. (See Figure P2.155.)
impact is parallel to the before-collision velocity of 2.156 Find the total time after firing for the cannonball
2.153 Let disk in Problem 2.152 weigh 9 oz (and and box to either stop or strike the wall, whichever comes
weigh 6 oz as before), and then repeat the problem. first. (See Figure P2.156.)

2.154 A 10-kg block swings down as shown in Fig­ 2.157 A cannonball is fired as shown in Figure P2.157
ure P2.154 and strikes an identical block. Assume that the with an initial speed of 1600 f t / s e c at 60°. Just after the
6 m rope breaks during impact and the blocks stick to­ cannon fires, it begins to recoil, and strikes a plate at­
gether after colliding. How long will it be before they tached to a spring. Find the maximum spring deflection if
come to rest? How far will they have traveled? the plane is smooth and the spring modulus is 5 0 0 lb / ft.

Figure P2.157
Figure P2.154

Figure P2.155

Figure P2.156
Page 115

bullet is fired with a speed of 1800 f t / s e c 2.160 Block in Figure P2.160 weighs 16.1 lb and is
into a 10-lb block. (See Figure P2.158.) If the coefficient traveling to the right on the smooth plane at 50 f t / s e c .
of friction between block and plane is 0.3, find, neglecting Block weighs 8.05 lb and is in equilibrium with the
the impulse of friction during the collision: spring barely preventing it from sliding down the rough
section of plane. Body impacts the coefficient of
a. The distance through which the block will slide
restitution Find the maximum spring deflection.
b. The percentage of the bullet's loss of initial ki­
netic energy caused by sliding friction, and the 2.161 The 16-kg body and the 32-kg body shown in
percentage caused by the collision Figure P2.161 are connected by a light spring of modulus
12,000 N / m . The unstretched length of the spring is
c. How long it takes block and bullet to come to
0.15 m. The blocks are pulled apart on the smooth hori­
rest after the impact.
zontal plane until the distance between them is 0.3 m and
2.159 Weight W falls from rest through a distance H; it
t then released from rest. Determine the velocity of each
lands on another weight W , which was in equilibrium
2 block when the distance between them has decreased to
atop a spring of modulus k. (See Figure P2.159.) If the 0.22 m. Hint: As in Problem 2.129, form the sum of the
coefficient of restitution is zero, find the spring compres­ work-energy equations for the two blocks.
sion when the weights are at their lowest point.
2,162 The cart and block in Figure P2.162 are initially at
rest, when the bullet slams into the block at speed
and sticks inside it. The combined body then starts sliding
on the cart.
Find:

a. the speed of the block just after impact;


Figure P2.158
b. the energy lost during impact;
• c. the time when the block leaves the cart.

Figure P2.161

Figure P2.159

No friction in
small light wheels
Figure P2.162

Figure P2.160
Page 116

2.163 A chain of length L and mass per unit length is at rest for t < 0 and is subjected to the uniform pres­
held vertically above the platform scale shown in Fig­ sure (over the end of area A) shown in Figure P2.165b.
ure P2.163 and is released from rest with its lower end If the disturbance has not reached the right end, that is if
just touching the platform. Assume that the links quickly t<L/c, then for t > t0 the particle velocities ui and
come to rest as they stack up on the platform and that they accelerations which vary only with x and t, are as
do not interfere with the links still in free fall above the shown in Figures P2.165c and d, where p is the density of
platform. Draw a free-body diagram of the entire chain the bar.
and express the momentum as a function of the distance The first part of this problem is to evaluate the
through which the upper end has fallen. Then determine integral
the force read on the scale in terms of this distance.

The value that should be obtained is and, since this


equals the external force on the bar, Equation (2.4) is thus
confirmed for this case. It is important to recognize that
only the interval from x = ct — ct to x = ct contributes to
o

the value of the integral; that is, only the particles in that
region are accelerating.

Figure P2.165i

Figure P2.163

• 2.164 A block of mass ml, which can move on a smooth


horizontal table, is attached to one end of a uniform chain
of mass m per unit length. Initially the block and the chain RgireP2.165b
are at rest, and the chain is completely coiled on the table.
A constant horizontal force mLf is then applied to the
block so that the chain begins to uncoil. Show that the
length x uncoiled after time t is given by

until the chain is completely uncoiled. If the length of the


chain is very large compared with L, show that the veloc­
l / 2
ity of the block is approximately equal to ( L / ) at the
moment when the chain is completely uncoiled. Figure P2.165e

• 2.165 An important problem in the dynamics of deform-


able solids is that of describing the motion which ensues
when pressure is rapidly applied to the end of a slender,
uniform, elastic bar. A useful approximate theory yields
the one-dimensional wave equation as the governing
equation of motion. This theory predicts that a pressure
applied at one end of the bar creates a disturbance (wave)
that propagates into the bar at a constant speed c. To be
specific, suppose the bar shown in Figure P2.165a is Fi§ureP2.165d
Page 117

The second element of this problem is to evaluate the In the second,


momentum

For the case at hand there is no reason to express a prefer­


The result will be ence for the order of differentiating with respect to time
and integrating over the body. If the pressure were sud­
denly applied at full strength (t = 0), however, there
0

would be a discontinuity in particle velocity (shock wave)


and a consequent undefined acceleration at the wave-
The second term in the brackets, a constant, is the contri­ front x = ct. Because of this undefined (or infinite) accel­
bution from integrating over the interval ct where the
0
eration, becomes meaningless and no longer pro­
particles are accelerating. The time dependence of L ap­ vides L. There is no difficulty involved in evaluating L,
pears through the increasing number of particles having however, since the particle velocities are
velocity As expected, we see that and zero for x > ct. Thus
In effect we have confirmed Euler's law, in
two forms. In the first,

and

2.6 Euler's Second Law (The Moment Equation)


A s e c o n d relationship b e t w e e n t h e external forces o n a particle s y s t e m or
a b o d y is o b t a i n e d if, referring to Figure 2 . 1 0 , w e take the cross p r o d u c t o f
r, with b o t h sides o f E q u a t i o n ( 2 . 1 ) :

T h e first term o n the left is r e c o g n i z e d as t h e m o m e n t a b o u t point P o f t h e


external force F,. T h e cross product r, X fy is t h e m o m e n t w i t h respect to
(

T H
P o f t h e force exerted o n t h e I particle b y t h e particle. A s b e f o r e , w e
n o w s u m t h e N e q u a t i o n s typified b y E q u a t i o n ( 2 . 3 1 ) to obtain
(Fixed in
inertial frame)
Figure 2.1 G
T e r m s in t h e d o u b l e s u m o c c u r in pairs, s u c h as

But r X f 2 2I = r X f
1 2 1 s i n c e r a n d r b o t h terminate o n t h e line o f action
2 1

of f . M o r e o v e r , f
21 2 1 = —f 1 2 so that

a n d similarly for o t h e r s u c h pairs. T h a t is, t h e m o m e n t s o f t h e internal


forces o f interaction s u m to zero. T h u s
Page 118

OR

w h i c h is the particle-system form o f Euler's s e c o n d l a w and states that


the s u m o f the m o m e n t s o f the external forces a b o u t a point equals the
sum o f the m o m e n t s o f the ma's a b o u t that point.
For a b o d y w h o s e m a s s is continuously distributed, the counterpart
to Equation ( 2 . 3 3 ) is

Question 2.9 In Equation 2.33 (or 2.34) must point P be fixed in the
inertial frame of reference?

Equations (2.4) a n d ( 2 . 3 4 ) play the s a m e roles in dynamics as do the


equations o f equilibrium in statics. A n d in fact w e obtain those equations,
from (2.4) a n d ( 2 . 3 4 ) , if w e set to zero the accelera­
tions o f all points o f a body.

M o m e n t of M o m e n t u m

Just as Euler's first l a w can b e expressed in terms o f the time derivative of


m o m e n t u m o f a b o d y , so Euler's s e c o n d l a w can b e expressed in terms of
the time derivative o f a quantity called m o m e n t o f m o m e n t u m , or a n g u ­
lar m o m e n t u m . * T h e m o m e n t o f m o m e n t u m w i t h respect to a point
P is designated H a n d is defined to b e the sum o f the m o m e n t s (with
P

Reference frame'7 respect to P ) o f the m o m e n t a o f the individual particles m a k i n g u p the


Figure 2.11 body. Referring to Figure 2 . 1 1 , w h e r e v,- is the velocity in reference frame
t h
of the 1 particle, w e h a v e

Before proceeding to the d e v e l o p m e n t o f several o f the forms of


Euler's s e c o n d law, w e shall develop a very useful relationship b e t w e e n
m o m e n t s o f m o m e n t u m . Noting from the definition, Equation ( 2 . 3 5 ) ,
and from Figure 2 . 1 1 that

and that

t h e n for a n y point P,

Answer 2.9 No. Nowhere in the development did we need to fix P.


* The term angular momentum stems from the fact that the moment of momentum of a
rigid body is related to the angular velocity of the body.
Pat4119

But from S e c t i o n 2 . 5 , is t h e m o m e n t u m L, also expressed as

L = mv c

so that

T h u s t h e m o m e n t o f t h e m o m e n t u m o f a b o d y a b o u t any point P (not


necessarily fixed in t h e reference frame) is t h e s u m o f t h e m o m e n t o f
m o m e n t u m about its m a s s center, C, a n d t h e m o m e n t o f its (linear)
m o m e n t u m L about P, w h e r e L is given a " l i n e o f a c t i o n " t h r o u g h C.
N o w w e can return to t h e definition o f m o m e n t o f m o m e n t u m ,
Equation ( 2 . 3 5 ) , a n d apply it for t h e case o f a point, O, fixed in t h e frame
o f reference. T h u s , using definition ( 2 . 3 5 ) for t h e third time,

N o w , differentiating w i t h respect to time in

But b e c a u s e O is fixed in

so that

Therefore

Momentum Forms of Euler's Second Law

N o w t h e fundamental form o f Euler's s e c o n d law, Equation ( 2 . 3 3 ) , tells


us that if the frame in w h i c h the a,< are calculated is an inertial frame,
then

so that, from the last two equations, w e see that

A n o t h e r similar form o f Euler's s e c o n d l a w can b e d e d u c e d i f w e first


use E q u a t i o n ( 2 . 3 6 ) in t h e case o f a fixed point O:

Differentiating with respect to time in t h e reference frame


Page 120

w h e r e w e h a v e u s e d t h e fact that

But w e again recall that

L = mv c (so that v c X L = 0)

and therefore

N o w w e k n o w from our study o f equipollent force systems in statics that


the external forces o n the b o d y must produce m o m e n t s a b o u t O a n d C
that are related b y

T h i s l a w o f resultants h a s nothing to do with w h e t h e r or n o t the b o d y is in


equilibrium. A n d since is an inertial frame, t h e n w e also k n o w

a n d thus w e m a y subtract the t w o Equations ( 2 . 4 1 ) a n d ( 2 . 4 2 ) to obtain

N e i t h e r o f the a b o v e equations is a n y m o r e
basic or special t h a n the other, as e a c h o n e c a n b e derived from the other.
T h e y are therefore equivalent forms. H o w e v e r , t h e equation does not
h o l d for a n y arbitrary point P, i.e., in general

Conservation of M o m e n t of M o m e n t u m

W e n e x t n o t e that — as w a s the case with linear m o m e n t u m — there are


situations in w h i c h a m o m e n t o f m o m e n t u m is conserved. In particular,
if for an interval o f time t h e n during that interval and
thus H is constant. For e x a m p l e , let the b o d y o f interest b e a single
G

spherical planet in its m o t i o n a r o u n d its star. T h e gravitational force


exerted o n the planet b y the star a l w a y s passes through the star's mass
center 0 , so a n d thus H o f the planet is a constant. This result is
D

s h o w n in Section 8.4 to lead to the elliptical orbit o f the earth a r o u n d the


sun.
For an arbitrary point P, there is a form o f Euler's s e c o n d l a w that is o f
particular value in analyzing the m o t i o n s o f rigid bodies, although it
remains valid for non-rigid bodies as well. T o derive it w e again use our
k n o w l e d g e a b o u t force s y s t e m s to write
Page 121

T h u s , using Euler's l a w s (Equations ( 2 . 2 7 a n d ( 2 . 4 3 ) )

or

Question 2.10 Must point P be fixed in the inertial frame of reference,


w h i c(a)
h wfore Equation
shall use(2.36)
later.to be true? (b) for Equation (2.45) to be true?

Finally, w e r e m i n d t h e reader that all o f the relationships o f this c h a p t e r


pertain o n l y to a specific collection o f material — that is, a s y s t e m (or
b o d y ) o f c o n s t a n t m a s s . H o w e v e r , t h e m o m e n t u m forms o f Euler's l a w s
provide t h e natural starting point for developing relationships appropri­
ate to " v a r i a b l e m a s s " systems s u c h as rockets. I f desired, t h e reader n o w
h a s t h e proper b a c k g r o u n d to study that special topic w h i c h is found in
Section 8.3.

Answer 2.10 (a) No; (b) No. Point P was unrestricted in both derivations.

EXAMPLE 2 . 1 8

Two gymnasts of equal weight (see Figure E2.18a) are hanging in equilibrium at
the ends of a rope passing over a relatively light pulley for which the bearing
friction can be neglected. Then the gymnast on the right begins to climb the rope,
while the gymnast on the left simply holds on. When the right gymnast has raised
himself through height h (relative to the floor), what has been the change in
position of the left gymnast?
Figure E2.18a
Solution
Constructing a free-body diagram (Figure E2.18b) of the pulley-rope-gymnasts
system in which we neglect the weights of the pulley and the rope, we see that

Therefore, H Q is constant during the motion, and since everything starts from
rest,

Treating the gymnasts as particles and neglecting the moment of momentum


of the pulley,

or
Figure E2.18b
Page 1 2 2

But

H = 0
o

so that

and the left gymnast, "going along for the ride," rises at the same rate as the right
one. Thus when the right gymnast has pulled himself up height h, the left one has
been pulled up the same height h. Note that if the rope is inextensible, the right
gymnast therefore would have climbed 2h relative to the rope!

EXAMPLE 2 . 1 9

Suppose the "counterweight" gymnast on the left in the preceding example were
to weigh twice that of the climbing gymnast, as suggested by Figure E2.19. What
then would be the relationship between their elevation changes?
Solution
From the free-body diagram

Since the moment of momentum is not conserved this time. Inte­


grating,

Figure E2.19
or

so that

If we define y and y so that y„ = y = 0 at t = 0, then


R L L

Thus we see that it's possible for the lighter gymnast to raise the heavier gymnast
by climbing rapidly enough.
For an inextensible rope, the right gymnast climbs, as before, at a rate of
relative to the rope.

PROBLEMS • Section 2.6

2
2.166 The uniform rigid bar in Figure P2.166 weighs a = 32.2 ft / s e c as shown, determine the tension T in
60 lb and is pinned at A (and fastened by the cable DB) the cable and the force exerted by the pin at A on the bar.
to the frame If the frame is given an acceleration
Page 123

Now note that (a) if the Q are a set of forces F then


i i

Equation (2.42) results with R being ; and (b) if the Q i

are a set of momenta m v of a group of particles, then


i i

Equation (2.36) results with R being (or L or mv ). c

Thus we may conclude that both equations, (2.36) and


(2.42), are practical examples of the very same law of
resultants!
Figure P2.16G
2.171 The angular momentum about point Q is defined as

2.167 A force F causes the carriage to move with rectilin­


ear horizontal motion defined by a constant acceleration Differentiate this expression in the inertial reference
2
of 20 f t / s e c (see Figure P2.167). A rigid, slender, homo­ frame and show by the result and that of the preceding
geneous rod of weight 32.2 lb and length 6 ft is welded to problem that, in general, (i.e., not just at an isolated in­
the carriage at B and projects vertically upward. Find, in stant of time):
magnitude and direction, the bending moment that the
carriage exerts on the rod at B.

2.168 A uniform slender bar of density p, cross-sectional only if (a) Q is fixed in or (b) Q is the mass center C; or (c)
area A, and length L undergoes small-amplitude, free v is parallel to v .
Q c

transverse vibrations according to 2.172 In Problem 2.132 find: (a) the angular momentum
sin where y is the displacement perpendicular to the of the system with respect to the origin; (b) the angular
axis (x) of the bar. (See Figure P2.168.) Neglecting other momentum of the system with respect to the mass center.
components of displacement (and hence acceleration),
calculate the maximum force generated at one of the sup­ 2.173 A massless rope hanging over a massless, friction-
ports during the motion. less pulley supports two monkeys (one of mass M, the
other of mass 2M). The system is released at rest at t = 0,
as shown in Figure P2.173. During the following 2 sec,
monkey B travels down 15 ft of rope to obtain a massless
peanut at end P. Monkey A holds tightly to the rope dur­
ing these 2 sec. Find the displacement of A during the time
interval.

Figure P2.167

Figure P2.168

2.169 Show that in Equation (2.36), point C need not be


the mass center, i.e., if Q is another arbitrary point like P,
then show that H = H + r X L (where L is of course
P Q P Q

still ).
2.170 L e t S b e a s e t o f vectors Q , Q , . . . , Q . . . , Q o f
1 2 i N

equal dimension. Define the resultant of S as ,


and place each Q at a point P . Define the moment of the
i i

set of vectors about a point A by

and show that

Figure P2.173
Page 124

2.174 A starving monkey of mass m spies a bunch of 2.176 Define the angular momentum of a particle about a
delicious bananas of the same mass. (See Figure P2.174.) fixed axis and state the conditions under which the angu­
He climbs at a varying speed relative to the (light) rope. lar momentum remains constant. A man (to be regarded
Determine whether the monkey reaches the bananas be­ as a particle) stands on a swing. His distance from the
fore they sail over the pulley if the pulley's mass is negli­ smooth horizontal axis of the swing is L when he crouches
gible and L — H when he stands. As the swing falls he
crouches; as it rises he stands — the changeover is as­
sumed instantaneous. If the swing falls through an angle
a and then rises through an angle show that

The relative angular momentum of a body with respect to


a point P is defined to be

(See Figure P2.177, where the velocity v of dm is the de­


rivative of r in inertial frame ) Note that what makes it
"relative" is that the velocity in the integral is the differ­
ence between v (of dm) and v . Now solve the following
P

problems.

Figure P2.174

Figure P2.177

Figure P2.175 2.177 Show that H = H + mrpc X v . (Thus H


P P r e l P c

= H always!)
C r e |

2.178 Show that is


2.175 Two gymnasts A and B, each of weight W, hold onto not generally equal to !)
the left side of a rope that passes over a light pulley to a
counterweight C of weight 2W. (See Figure P2.175.) Ini­ 2.179 Show that
tially the gymnast A is at depth d below B. He climbs the 2.180 Show that H = H P P r e ( if and only if v X v
P c =
rope to join gymnast B. Determine the displacement of TpC X cLp.
the counterweight C at the end of the climb.

COMPUTER PROBLEMS • Chapter 2

2.181 In the table on the next page are data of the mass column. Use the computer to integrate numerically the
center velocity versus time for a 30-lb crate that was lifted velocity from t = 0 to 4.4 sec, thereby obtaining and
approximately straight up by two people. The "velocity comparing the heights to which the crate was lifted by the
1" column represents a taller person than the "velocity 2 " two people.
Page 125

Time Velocity 1 Velocity 2 Time Velocity 1 Velocity 2


(sec) (in./sec) (in./sec) (sec) (in./sec) (in./sec)

00 00 0 0 2 3 2 5 . 0 (peak) 23.5
02 00 0 0 24 24.0 25.5
04 1.5 0.0 2.6 22.0 32 0
0.6 3 5 00 2.7 20.5 3 7 5 (peak)
08 5 5 00 2 8 190 35.0
1.0 8.5 2 0 3.0 16 5 25.0
1.2 12.0 40 3.2 14.5 21.5
1 4 165 6 5 34 13.0 170
1 6 190 10.0 3.6 12 0 15 0
1.8 21 0 140 3.8 10.0 13 0
20 22 5 17 0 40 8.5 9.5
2 2 24.5 21 0 4 2 7.5 85
4 4 6.5 8.0

SUMMARY • Chapter 2
In this chapter w e h a v e set out the fundamental relationships b e t w e e n
forces o n a b o d y a n d its motion, a n d w e h a v e illustrated their use for the
solution o f a variety o f p r o b l e m s , m a n y of w h i c h are closely associated
with our everyday experience.
T h e starting point h e r e w a s N e w t o n ' s s e c o n d l a w for a particle,

where is the s u m o f all the forces acting o n the particle, m is the m a s s


of the particle a n d a is its acceleration relative to an inertial frame o f
t h
reference. Extending to a system o f particles, the 1 h a v i n g m a s s m, a n d
acceleration a , i

where is the s u m o f the external forces on the system. A n o t h e r form of


particular value is

where is the m a s s o f the s y s t e m o f particles, or the b o d y c o m ­


prising t h e m , a n d a is the acceleration o f the m a s s center C. In addition, it
c

is sometimes useful to d e c o m p o s e a b o d y into t w o (or m o r e ) parts with


masses m, a n d m a n d m a s s centers C and C , a n d t h e n w e m a y use:
2 1 2

T h e preceding equations, for a b o d y o f finite size, are forms o f w h a t is


often called Euler's first l a w a n d are counterparts in dynamics to the
equilibrium equation, studied in statics. W e h a v e used t h e m to
solve a variety of problems such as finding accelerations and constraining
forces w h e n s o m e forces a n d p a t h s were prescribed a n d also integrating
to find the m o t i o n o f a particle (or m a s s center o f a b o d y ) w h e n external
forces w e r e prescribed. Central to the problem-solving process w a s the
free-body diagram, the i m p o r t a n c e o f w h i c h c a n n o t b e overstated.
Page 126

T h e Principle o f W o r k a n d Kinetic E n e r g y is very useful in solving


problems in w h i c h the speeds o f a particle at different locations in space
are o f interest. T h e Kinetic Energy, T, o f a particle is defined to b e

T h e principle states that the w o r k d o n e b y all the forces acting over an


interval o f time is equal to the c h a n g e in kinetic energy. Or, in s y m b o l s ,

W = T -T
2 1

This is a derived result following from integrating a n d assigning the term


" w o r k o f a force o n a p a r t i c l e " to

T w o special forces arise frequently e n o u g h in p r o b l e m s to evaluate


the w o r k a n d express it in symbols:

a. C o n s t a n t force: W = F • (r — r ), or in words: (magnitude o f


2 1

force) * (magnitude o f displacement) * (cosine o f angle b e ­


t w e e n force a n d displacement). For weight (force exerted b y
gravity n e a r the earth's surface) this m e a n s (weight) * (decrease
in altitude o f m a s s center).
b. Force exerted b y a linear spring:

where is spring stretch a n d k is the spring modulus.

A force w h o s e w o r k does n o t d e p e n d u p o n the p a t h o f the point o f


application is called conservative a n d a potential energy, , is associated
with it so that the w o r k d o n e is the negative o f the c h a n g e in that
potential energy,

C o m b i n i n g this with assuming all forces to b e conservative,

or

w h i c h m e a n s that in this case, kinetic plus potential e n e r g y is conserved.


A potential e n e r g y for a linear spring is

a n d for weight (with z b e i n g elevation)

For a s y s t e m o f particles,
Page 127

a n d the Principle o f W o r k a n d Kinetic E n e r g y applies so l o n g as o n e


considers the w o r k o f all internal forces as well as the w o r k o f external
forces; that is,

This is o f practical value o n l y in special situations w h e r e it's possible to


readily evaluate the w o r k o f internal forces. O n e e x a m p l e is a rigid b o d y
or system o f rigidly c o n n e c t e d particles, for t h e n W = 0. A n o t h e r
internal

case is that o f a pair o f particles j o i n e d b y a linear spring, for w h i c h the net


w o r k o f the equal a n d opposite internal forces is

T h e c o n c e p t o f m o m e n t u m is particularly useful in p r o b l e m s o f im­


pact or collision in w h i c h very intense forces o f interaction m a y act for a
very brief interval. M o m e n t u m is defined to b e

L = mv (particle)
(system o f particles)

from w h i c h , for a b o d y in general,

L = mv c

Euler's first l a w c a n b e written

w h i c h w h e n integrated yields the i m p u l s e - m o m e n t u m principle

S o if external forces do n o t act during the interval,

L(t ) = L(t ),
2 1

a n d so m o m e n t u m is conserved. This is quite often (approximately) the


case in p r o b l e m s o f collision.
Finally in this chapter w e h a v e d e v e l o p e d the counterpart in dy­
n a m i c s to the s e c o n d equilibrium equation, , in statics. In dy­
n a m i c s this is, for a s y s t e m o f particles,

when refers, as it did in statics, to the m o m e n t s o f external forces.


This is often called Euler's s e c o n d l a w . T h e r e are several different forms
in w h i c h this l a w c a n b e expressed, a m o n g t h e m expressions involving
the m o m e n t o f m o m e n t u m , defined as

for a s y s t e m o f particles. A useful relationship is

Hp — H + r c p c X L,
Page 128

but the k e y expressions are the forms that Euler's s e c o n d l a w c a n take,

1M C = H c

and

w h e r e "O" is a point fixed in the inertial frame o f reference.

REVIEW QUESTIONS • Chapter 2


True or False?

1. A t a given time, t h e m a s s c e n t e r o f a d e f o r m a b l e b o d y c a n b e s h o w n
to b e a unique point.
2. T h e m o m e n t u m o f a n y b o d y (or s y s t e m o f bodies) in a frame c a n b e
s h o w n to b e equal to the total m a s s times the velocity o f the mass
center in e v e n i f is n o t an inertial frame.
3. Euler's first l a w applies to deformable bodies w h e t h e r
solid, liquid, or gaseous, as well as to rigid b o d i e s a n d particles.
4. Neither the laws o f m o t i o n n o r the inertial frame is o f a n y value
without the other.
5. T h e m a s s center o f a b o d y h a s to b e a physical, or material, point
of
6. T h e w o r k d o n e b y a linear spring d e p e n d s o n the p a t h s traversed by
its endpoints b e t w e e n the initial a n d final positions.
7. T h e w o r k d o n e b y the friction force u p o n a b l o c k sliding o n a fixed
p l a n e d e p e n d s o n the p a t h taken b y the block.
8. T h e w o r k d o n e b y gravity o n a b o d y d e p e n d s o n the lateral as well
as the vertical displacement o f the m a s s center o f
9. S i n c e n o external w o r k w a s d o n e o n the two bodies o f E x a m p l e 2 . 1 4
during the impact, their total kinetic energy is the s a m e after the
collision as it w a s before.
10. F o r all b o d i e s o f c o n s t a n t density, t h e centroid o f v o l u m e a n d the
center o f m a s s coincide.
11. In studying the m o t i o n o f the earth a r o u n d the sun, it is acceptable to
treat the earth as a particle; in studying the daily rotation o f the earth
o n its axis, h o w e v e r , it w o u l d n o t m a k e s e n s e to consider the earth as
a particle.
12. T h e external forces acting on a b o d y w h i c h together form the
resultant F , must e a c h h a v e a line o f action passing through the m a s s
r

center o f in order for Euler's first l a w to apply.


13. Euler's s e c o n d l a w c a n take the form regardless o f the
m o t i o n o f point P in the inertial frame.

Answan:1.T 2. T 3. T 4. T 5. F 6. F 7. T B. F 9. F 10. T 11. T 12. F 13. F


3 KINEMATICS OF PLANE
MOTION OF A RIGID BODY

3.1 Introduction
3.2 Velocity and Angular Velocity Relationship for T w o Points of
the S a m e Rigid Body
Development of the Velocity and Angular Velocity Relationship
Important Things to Remember About Equation (3.8)
3.3 Translation
3.4 Instantaneous Center of Zero Velocity
Proof of the Existence of the Instantaneous Center
The Special Case in Which the Normals Do Not Intersect
The Special Case in Which the Normals are Coincident
3.5 Acceleration and Angular Acceleration Relationship for T w o
Points of the S a m e Rigid Body
Development of the Acceleration and Angular Acceleration
Relationship
3.6 Rolling
Rolling of a Wheel on a Fixed Straight Line
Rolling of a Wheel on a Fixed Plane Curve
Gears
3.7 Relationship Between the Velocities of a Point w i t h Respect
to T w o Different Frames of Reference
Relationship Between the Derivatives of a Vector in T w o Frames
Velocity Relationship in T w o Frames
3.8 Relationship B e t w e e n the Accelerations of a Point w i t h
Respect to T w o Different Frames of Reference
SUMMARY
REVIEW QUESTIONS

Page 1 2 9
Page 1 3 0

3.1 Introduction
In this chapter our goals are to develop the relationships b e t w e e n veloci­
ties, accelerations, angular velocity, a n d angular acceleration w h e n a
rigid b o d y m o v e s in p l a n e m o t i o n in a reference frame Before doing
so, h o w e v e r , w e shall first explain precisely w h a t w e m e a n b y such terms
as rigid body, plane motion, rigid extension, reference plane, a n d several
other c o n c e p t s w e shall b e needing in this chapter a n d those to follow.
A r i g i d b o d y is taken to b e a b o d y in w h i c h the distance b e t w e e n
each a n d every pair o f its points r e m a i n s the s a m e throughout the m o ­
tion.* T h e r e is, o f course, n o such thing as a truly rigid b o d y (since all
bodies d o some deforming); h o w e v e r , the deformations of m a n y bodies
are sufficiently small during their m o t i o n s to allow the bodies to be
treated as t h o u g h they were rigid with good results.
T h e significance o f the rigid-body m o d e l is that velocities o f different
points will b e found to differ b y s o m e t h i n g proportional to the rate at
w h i c h the b o d y turns, w h a t w e shall c o m e to call its angular velocity. A n d
accelerations will b e found to b e related through the angular velocity and
its rate o f c h a n g e w h i c h w e k n o w as the angular acceleration. T h u s a very
small a m o u n t o f information will characterize all the accelerations in the
body. T h e r e are a n u m b e r o f w a y s in w h i c h this is important, b u t fore­
most is the fact that the right-hand side o f our m o m e n t equation in
C h a p t e r 2 will in C h a p t e r 4 b e seen to take on a c o m p a c t form involving
the angular velocity a n d the angular acceleration.
P l a n e m o t i o n is treated in this b o o k as m o t i o n in the xy p l a n e (fixed
in ) or in planes parallel to it. Let a point P b e located originally at
coordinates (x , y ,z ).
p p p To say that P h a s p l a n e m o t i o n simply m e a n s that
it stays in the p l a n e z = z throughout its motion. Extending this defini­
p

tion, w e s a y that rigid b o d y h a s p l a n e m o t i o n w h e n e v e r all its points


remain in the s a m e planes (parallel to xy) in w h i c h t h e y started.

Question 3.1 How few points of a rigid body must be in plane motion
to ensure that they all are?

A third c o n c e p t w e n e e d to u n d e r s t a n d in rigid-body kinematics is


that o f the b o d y e x t e n d e d , also called a r i g i d e x t e n s i o n o f the b o d y . This
idea, briefly m e n t i o n e d in C h a p t e r 1, says that w e s o m e t i m e s n e e d to
i m a g i n e points ( w h i c h are n o t physical or material points o f ) m o v i n g
with as t h o u g h they w e r e in fact a t t a c h e d to it. A n e x a m p l e w o u l d be
the points o n the axis o f a pipe that are in the space inside it b u t o f course
m o v e rigidly with it. W e shall imagine a "rigid e x t e n s i o n " o f the b o d y to
pick up such points w h e n e v e r it is useful to do so. N o t e that any point Q

* We have already encountered this concept in Chapter 1, where it was seen to be syn­
onymous with the concept of frame.
Answer 3.1 Three noncollinear points are needed.
Page 1 3 1

m a y b e considered a point o f any b o d y extended, provided that Q


moves with as if it w e r e rigidly a t t a c h e d to it.
With these three concepts in mind, w e are n o w prepared to define the
r e f e r e n c e p l a n e . First w e must realize that w e are faced with the p r o b l e m
of determining w h e r e all the points o f a b o d y are as functions o f time t.
T h e s e points' locations (x(t), y(t)) w o u l d take forever to find if w e h a d to
do so for e a c h o f the infinitely m a n y points o f Fortunately, for a rigid
b o d y in p l a n e motion, if w e k n o w the locations o f all its points in a n y o n e
p l a n e o f ( w h i c h w e shall call the reference p l a n e ) , t h e n w e automati­
cally k n o w the locations o f all its other points in all other p l a n e s . T h e
reason for this is as follows. For e a c h point B o f that does n o t lie in the
reference plane, there is a " c o m p a n i o n p o i n t " o f in the reference plane
(suggested b y A in Figure 3 . 1 ) that h a s the s a m e (x, y) coordinates at the
b e g i n n i n g o f the motion. It t h e n follows that the (x, y) coordinates o f A
a n d B always m a t c h throughout the m o t i o n o f !

xy = reference plane

Axis of symmetry of

(Rigid body in plane motion


parallel to xy plane)

(Axes x, y, z arc
embedded in
frame of reference )

Figure 3.1

Question 3.2 Why is x = x and y = y as time passes?


A B A B

Body in Figure 3.1 is a c o n e pulley, w h i c h turns about its axis o f s y m ­


metry. N o t e from its varying cross-sectional diameter that a b o d y n e e d
n o t h a v e constant cross section to b e in p l a n e motion.
H a v i n g laid the n e c e s s a r y groundwork, w e n o w let xy b e our refer­
ence p l a n e . F r o m Figure 3.1 w e see that

Answer 3.2 If ever (or both), either the rigid body or the plane mo­
tion assumptions (or both) will have been violated.
Page 132

a n d w e m a y differentiate this equation to obtain

In these equations w e h a v e used t h e facts that x = x , y = y , a n d


B A B A

z = constant. E q u a t i o n s ( 3 . 2 ) a n d ( 3 . 3 ) s h o w clearly that if w e c o m ­


B

pletely describe t h e velocities a n d accelerations in o n e reference plane,


w e t h e n k n o w t h e m for all t h e points o f t h e b o d y . T h i s allows us to focus
on o n e p l a n e o f t h e b o d y throughout this chapter a n d m o s t o f t h e next
two as well.
The reference p l a n e is t h u s a v e r y important c o n c e p t , for it allows us
to study t h e m o t i o n o f a n entire b o d y b y c o n c e r n i n g ourselves o n l y with
t h o s e o f its points that lie in this p l a n e . W e say w e " k n o w the m o t i o n " of
w h e n w e k n o w w h e r e all its points are at all times. W e h a v e already
reduced this task to k n o w i n g t h e locations o f the points in the reference
plane. ( T h e rest o f t h e b o d y " g o e s along for the r i d e . " ) But in fact if w e
k n o w the location o f j u s t two points (say P a n d P ) o f the reference plane,
1 2

t h e n w e k n o w t h e w h e r e a b o u t s o f all points o f this p l a n e a n d t h u s o f the


w h o l e body! T h i s is b e c a u s e e a c h point o f t h e reference p l a n e must
maintain t h e s a m e position relative to t h e points P a n d P . T h i s idea is
1 2

illustrated in Figure 3 . 2 . N o t e in t h e figure that if P a n d P are correctly


1 2

located with respect to t h e reference frame all o t h e r points o f are


necessarily in their correct positions.

Position of E

Reference frame
Figure 3.2

Question 3.3 Is knowledge of the locations of two points sufficient for


us to know the motion of a body in general (three-dimensional) motion?

I n s t e a d o f k n o w i n g t h e locations o f t w o points o f the b o d y — ( x y v , 1

of Pi a n d (x , y ) o f P — w e m a y alternatively locate t h e b o d y if w e k n o w
2 2 2

w h e r e just one point, P, is l o c a t e d plus t h e value o f t h e orientation angle 8


(about a n axis t h r o u g h P a n d parallel to z); see Figure 3 . 3 .
l

Answer 3 . 3 No; the body could rotate around the line joining the two points. In three
dimensions it takes three points, not all on the same line!
Page 1 3 3

Line fixed in

Position of

Reference frame
Figure 3.3

Question 3.4 Knowing the locations of two points requires four vari­
ables (x , y ,x ,
x y ), whereas one point plus the angle takes but three
1 2 2

x ,y , ). Why do these numbers of variables differ?


1 1

T h e foregoing is i n t e n d e d to suggest quite correctly that the velocities of


different points in the reference p l a n e will b e linked together b e c a u s e o f
the rigidity o f the b o d y , a n d similarly for accelerations. In the n e x t section
w e will turn to the d e v e l o p m e n t o f t h e relationship b e t w e e n the veloci­
ties o f points s u c h as P a n d P in Figure 3 . 2 .
t 2

Answer 3.4 The 2 X 2 = 4 coordinates of P and P are not independent. The distance
1 2

between the points is a constant, so

can be used to find any one of x , y , x , y in terms of the other three.


1 1 2 2

PROBLEMS • Section 3.1

3.1 Which of the bodies shown in Figure P 3 . 1 ( a - f )


are in plane motion in frame _ ?

(a) A turkey being barbecued by (b) A cone rolling on a tabletop. (c) A spinning coin if the base is
slowly turning on a rotisserie. fixed.

Figure P3.1(a-c) (See next page for d-f)


Page 1 3 4

(d) A can rolling down an (e) T h e bevel gear which meshes (f) T h e (shaded) crosspiece of a
inclined plane with another bevel gear universal joint.

Figure P3.1(d-f)

3.2 Give three examples of plane motion besides those


in the previous problem. Then give three examples of
motion that is not planar.

3.2 Velocity and Angular Velocity Relationship for T w o


Points of the Same Rigid Body
In this section w e derive a v e r y useful relationship b e t w e e n t h e velocities
in o f a n y t w o points in t h e reference p l a n e o f a rigid b o d y in p l a n e
m o t i o n a n d t h e angular velocity v e c t o r o f in Let P a n d Q denote
t h e s e t w o points o f a n d let us e m b e d t h e a x e s (x, y, z) in reference
frame as s h o w n in Figure 3 . 4 .

Figure 3.4

W e are saying that e v e n t h o u g h m a y m o v e with respect to the


reference frame t h e xy p l a n e o f a l w a y s c o n t a i n s t h e points o f interest
P and Q of A g o o d e x a m p l e is f o u n d in t h e classroom; let t h e b o d y
b e a b l a c k b o a r d eraser. Letting t h e b l a c k b o a r d itself b e the reference
frame (so that x a n d y are fixed i n t h e p l a n e o f t h e b l a c k b o a r d ) , the
eraser u n d e r g o e s p l a n e m o t i o n w h e n e v e r the professor erases t h e b o a r d .
O u r points P a n d Q are a n y t w o points o f the erasing surface o f t h e eraser.
N o t e h o w e a c h point o f t h e eraser r e m a i n s t h e s a m e z distance from the
Page 135

blackboard (where z = 0 ) during the erasing. T h e eraser is n o longer in


p l a n e m o t i o n , h o w e v e r , o n c e it leaves the surface o f the b o a r d a n d its
points m o v e with z c o m p o n e n t s o f velocity.
N o t i c e from Figure 3 . 4 that is a unit vector a l w a y s directed from P
toward Q so that where rP Q is the distance PQ (that is, the
magnitude o f the vector r ) . N o t e further that the orientation (angular
P Q

rotation) o f the b o d y is described b y the angle , m e a s u r e d b e t w e e n any


line fixed in the reference frame (we shall h e r e use the x axis) and
a n y line fixed in the b o d y (for t h e m o m e n t , w e shall u s e t h e line s e g m e n t
from P to Q ) .

Development of the Velocity and Angular Velocity Relationship

W e are n o w ready to develop the velocity a n d angular velocity relation­


ship for rigid bodies. F r o m Figure 3.4 w e see that

so that w e h a v e , u p o n differentiation in frame

Recognizing the first t w o vectors as the definitions o f the velocities o f P


a n d Q (in w h e r e O is fixed), w e m a y write

In obtaining Equation ( 3 . 5 ) , all derivatives were taken in , so that, for


example, there is n o n e e d to write
In order to write as a vector w e can use, w e express as a
magnitude times a unit vector. W i t h the help o f Figure 3.4 w e get

Differentiating this expression as in Section 1.6, w e h a v e

Therefore w e h a v e derived a useful expression for :

Question 3.5 In the preceding development, why is ?

Substituting Equation ( 3 . 7 ) into ( 3 . 5 ) yields

* Throughout this book and constitute a right-handed system so that


Answer 3.5 Because is rigid, is the constant distance between points P and Q.
Page 1 3 6

w h i c h relates the velocities o f the points P a n d Q a n d introduces the


a n g u l a r v e l o c i t y o f in reference frame T h e G r e e k letter o m e g a is
usually u s e d to denote this vector:

W h e n there is n o confusion a b o u t the b o d y a n d reference frame


involved, w e m a y drop the subscripts a n d write as simply . Also,
s o m e prefer to write as rather t h a n ; w e shall use b o t h forms,
feeling that the latter is a nice reminder that in p l a n e motion, angular
velocity is proportional to the time rate o f c h a n g e o f an angle.*
The magnitude J o f the angular velocity is called the
a n g u l a r s p e e d o f in frame N o t e that itself can b e negative.
N o t e further that neither the angular velocity v e c t o r n o r Equation ( 3 . 8 )
depends o n w h i c h body-fixed line s e g m e n t (such as P Q a b o v e ) is c h o s e n
to m e a s u r e . T h e p r o o f o f the preceding s t a t e m e n t is n o t difficult and
will b e given later as a n exercise.
N o t e that if our angle o f orientation were c h o s e n as s h o w n in Fig­
ure 3 . 5 , t h e n the angular velocity w o u l d b e given by

The angular velocity vector is always in the direction given b y the right-
h a n d rule w h e n w e turn our fingers in the direction o f rotation o f the
Figure 3.5 body. Referring to b o t h Figures 3.4 a n d 3 . 5 , and

and is directed out o f the page if the b o d y is turning counterclockwise,


and into the page if it's turnine clockwise.

Important T h i n g s to Remember About Equation (3.8)

W e n o w h a v e the result that the angular velocity vector is a property of


the overall b o d y , a n d not a property o f its individual points. T h i s idea
cannot be overemphasized. Remember:

1. A point h a s position, velocity, a n d acceleration.


2. A body h a s orientation, angular velocity, a n d angular acceleration.

R e m e m b e r too that a point does n o t h a v e orientation, and a


finite-sized body does n o t h a v e a u n i q u e r, v, a n d a.**
W e shall e m p h a s i z e t h e s e property differences b e t w e e n points and
bodies in the following w a y : throughout this b o o k , points are d e n o t e d by
capital italic printed letters while bodies are d e n o t e d b y ordinary capital

* We remark that in general (three-dimensional) motion, such a simple relationship


as between angular velocity and body orientation does not exist.
Angular acceleration (a), the derivative of angular velocity, is discussed in Section 3.5.
** Of course the particle, being treated as small enough that we need not distinguish be­
tween the locations of its points, must be considered as having an r, v, and a — and not
an or a.
Page 137

cursive letters. H e n c e , for e x a m p l e , P, A, a n d B d e n o t e points, w h i l e


and d e n o t e bodies. Therefore, w e shall simply print t h e n a m e s o f
points, a n d write t h e n a m e s o f bodies in cursive script.
W e n o w m o v e toward a n u m b e r o f e x a m p l e s o f the use o f our n e w
Equation ( 3 . 8 ) ; in e a c h application o f this equation, t h e following three
rules m u s t b e followed without exception:
3. This vector extends from the point (P)
on this (right) side of the equation to
the point (Q) on the other (left) side.

1. These two 2. This is the angular


points are velocity vector of
on the
same rigid
body

Also helpful in using Equation ( 3 . 8 ) is t h e k i n e m a t i c diagram presented in


Figure 3.6 Figure 3 . 6 . T h e velocity o f Q, from Equation ( 3 . 8 ) , is the s u m o f the two
vectors in Figure 3 . 6 (see Figure 3 . 7 ) . N o t e that d e p e n d i n g o n t h e relative
sizes o f v a n d
P , t h e velocity v could lie o n either side, or e v e n along,
0

line PQ. N o t e further that the difference b e t w e e n t h e velocities o f Q a n d


P — that is, v — v — is simply
0 P . T h i s m e a n s that t h e o n l y w a y
in w h i c h t h e velocities o f two points o f a rigid b o d y in m o t i o n in frame
can differ is b y t h e term n o r m a l to the line joining t h e m . W e shall
return to this idea following t h e first three e x a m p l e s o f this section.
Incidentally, s o m e b o o k s describe v — v as " t h e velocity o f point Q
Q P

relative to point P." W e m e n t i o n this o n l y b y w a y o f explanation; our


definition o f v in S e c t i o n 1.3 s h o w s that points h a v e velocities relative to
P

frames, not relative to o t h e r points. If o n e uses the p h r a s e " t h e velocity of


Figure 3.7 point Q relative to point P," o n e m e a n s the velocity o f Q in a reference
frame in w h i c h P is fixed a n d w h i c h translates relative to .*
In e a c h o f the e x a m p l e s that follow, n o t e t h e i m p o r t a n c e o f selecting
and depicting t h e unit vectors to b e u s e d in t h e solution. Also, in e a c h of
the first three examples, p a y careful attention to t h e w a y t h e velocity o f a
point (say, B) is expressed if t h e t a n g e n t to t h e p a t h o f B is k n o w n ; using
w h a t w e l e a r n e d in S e c t i o n 1.7, v is expressed as a single u n k n o w n scalar
B

(whose absolute value is t h e s p e e d o f B) times a unit vector along the


k n o w n tangent.

EXAMPLE 3.1

A 30-ft ladder is slipping down in a warehouse with the upper contact point T
moving downward on the wall at a speed of 2 ft/sec in the position shown in
Figure E3.1. Find the velocity of point B, which is sliding on the floor.

* "Translates" means that the frame moves in without rotating. Translation is dis­
Figure E3.1 cussed in more detail in Section 3.3.
Page 138

Solution
We relate v and v by using Equation (3.8):
T B

Noting that v has no -component and that v has no -component, we write:


B T

Matching the coefficients, we have

Matching the coefficients, we have

Thus the velocity of B is


Note that a direction indicator must be attached to in order to specify
correctly the angular velocity vector of the ladder:

or, alternatively,

Note that the directions of make sense. Such visual checks on


solutions should be made whenever possible.

EXAMPLE 3.2

At the instant shown in Figure E3.2, the velocity of point A is 0.2 m / s to the right.
Find the angular velocity of rod and determine the velocity of its other end
(point B), which is constrained to move in the circular slot.
Solution
We shall use Equation (3.8), featuring the points A and B of the rod:

Noting that the velocity of B has a known direction (tangent to its path), we write
v as an unknown scalar times a unit vector in this direction:
B

The component equations are:

Figure E3.2
Page 139

Solving Equations (1) and (2) gives

and therefore the answers (vectors are what are asked for!) are

or, equivalently,

In t h e n e x t e x a m p l e , two bodies h a v e angular velocity; t h u s w e shall


h a v e to subscript t h e . W e shall simply denote b y
the angular velocity o f and t h e angular velocity o f

EXAMPLE 3.3

The crank arm shown in Figure E3.3a turns about a horizontal z axis, through
its pinned end O, with an angular velocity of 10 r a d / s e c clockwise at the given
instant. Find the velocity of the piston pin B.

Solution
We apply Equation (3.8) first to relate the velocities of A and O on body and
then to relate v to v on rod .Note that A is a "linking point" of both and ,
B A

since it belongs to both bodies. On body

On body

Figure E3.3b and using the Pythagorean theorem (see Figure E3.3b),

Now point B is constrained to move only horizontally. Therefore,


140

Equating the coefficients:

Equating the coefficients:

Therefore,

Substituting into (1), we have v = 30.3 in./sec and


B in./sec.

In all three o f the preceding e x a m p l e s , w e r e - e m p h a s i z e that it is


absolutely essential to correctly incorporate the kinematic constraints
i m p o s e d b y slots, walls, floors, a n d so forth.
It is often helpful in studying the kinematics o f rigid bodies to m a k e
u s e o f the following result, w h i c h is a corollary o f Equation (3.8):

Corollary: If P and Q are two points of a rigid body, their velocity


components along the line joining them must be equal.

Intuitively, w e see that the difference b e t w e e n these c o m p o n e n t s is the


rate o f stretching o f the line PQ, a n d this h a s to vanish. Also, w e h a v e
seen that v a n d v differ o n l y b y the term
Q P w h i c h is clearly
normal to the Line PQ joining the points. M a t h e m a t i c a l l y , w e c a n see this
immediately b y dotting Equation (3.8) with the unit vector parallel to r , P Q

w h i c h is r / r :
P Q P Q

component of component of zero


v along PQ v along PQ
Q r

T h u s , if w e k n o w the velocity o f o n e point o f the b o d y , w e can find


a n y other without involving the angular velocity b y using Equation
(3.11). F o r instance, in E x a m p l e 3.2, t h e unit v e c t o r r / r
A B A Bis simply
, a n d dotting this with the equation

from that e x a m p l e gives

or
Page 1 4 1

The algebra is seen to be simpler; w e h a v e w o r k e d with one equation in


o n e u n k n o w n rather than t w o in two.
W e n o w return to t h e vector formulation (Equation 3 . 8 ) for t w o final
examples in this section.

EXAMPLE 3.4

In the linkage shown in Figure E3.4, the velocities of A and C are given to be

at the instant given. Find the velocity of point B at the same instant.

Figure E3.4

Solution
On bodyi

On bar

Equating the two vector expressions for v , we get


B

Solving these two equations,

From Equation (1), it follows that

and the same result follows from (2), as a check.


Page 1 4 2

Question 3.6 If the velocities of A and C were given to be


and for an interval of time, and not just at the instant shown,
would the solution be any different at (a) the same instant? (b) some
other instant?

Answer 3.6 (a) No. (b) Yes, because the geometry would be different.

EXAMPLE 3.5

The end B of rod travels up the right half of the parabolic incline in Figure E3.5a
at the constant speed of 0.3 m / s . Find the angular velocity of, and the velocity of
point A, which is at the origin at the given instant.
Solution
We shall relate v to v using Equation (3.8):
B A

Next we use Equation (1.41) to express v :


Figure E3.5a B

To get the unit tangent for point B, we use Figure E3. 5b, noting that is tangent
to the parabola at all times:

Figure E3.5b Therefore, for point B,

And thus

Since point A likewise has a velocity tangent to its path, we may write

and so Equation (1) gives

Collecting the coefficients of and we have

j coefficients:

so that

coefficients:
Page 143

Substituting for and solving,

so that

Applications of Equation ( 3 . 8 ) t o rolling bodies are presented in


Section 3 . 6 after w e h a v e e x a m i n e d that topic in detail.

PROBLEMS • Section 3.2

3.3 The angular velocity of the bent bar is indicated in 3.6 At a certain instant, the coordinates of two points A
Figure P3.3. Find the velocity of the endpoint B in this and B of a rigid body in plane motion are given in
position. Figure P3.6. Point A has and the velocity of
B is vertical. Find v and the angular velocity of
3.4 The velocities of the two endpoints A and B of a g

rigid bar in plane motion are shown in Figure P3.4. Find 3.7-3.11 in the following five problems involving a
t h e velocity of the midpoint of the bar in the given posi­ "four-bar linkage" (the fourth bar in each case is the rigid
tion. ground length between fixed pins!), the angular velocity
of one of the bars is indicated. Find the angular velocities
3.5 If in./sec, find and . See Fig­
of the other two bars.
ure P3.5.

Figure P3.3

.7 (reference framel

Figure P3.4 Figure P3.6

Figure P3.5 Figure P3.7


Page 1 4 4

3.12 The equilateral triangular plate shown in Fig­


ure P3.12 has three sides of length 0.3 m each. The bar
has an angular velocity r a d / s counterclockwise
and is pinned to at A Body is also pinned to a block at
B, which moves in the indicated slot. At the given time,
find the angular velocity of .
3.13 Crank arm shown in Figure P3.13 turns counter­
clockwise at a constant rate of 1 rad / s. Rod is pinned to
at A and to a roller at B that slides in a circular slot.
Determine the velocity of B and the angular velocity of
at the given instant.
3.14 The wheel shown in Figure P3.14 turns and slips in
such a manner that its angular velocity is while
Figure P3.8 the velocity of the center C is 0.3 m / s to the left. Deter­
mine the velocity of point A.

Figure P3.9

Figure P3.12

Figure P3.13
Figure P3.10

Figure P3.11 Figure P3.14


Page 1 4 5

3.15 For the configuration shown in Figure P3.15, find 3.19 Wheel in Figure P3.19 has a counterclockwise
the velocity of point P of the disk angular velocity of 6 r a d / s . What is the velocity of point B
at the instant shown?
3.16 The speed of block in Figure P3.16 has the value
shown. Find the angular velocity of rod and determine 3.20 Block in Figure P3.20, which slides in a vertical
the velocity of pin A of block , when slot, is pinned to bars and at A. The other ends of
and are pinned to blocks that slide in horizontal slots.
3.17 Wheel , (Figure P3.17) turns and slips in such a
Block translates to the left at constant speed 0.2 m / s .
way that its angular velocity is r a d / s while the veloc­
Find the velocity of B: (a) at the given instant; (b) when C
ity of C is 0.4 m / s to the left. Determine the velocity
is at point D, (c) when C is at point E.
of point B, which slides on the plane. Bar is pinned to
at D. 3.21 The four links shown in Figure P3.21 each have
length 0.4 m, and two of their angular velocities are indi­
3.18 Point A of the rod slides along an inclined plane as
cated. Find the velocity of point C and detennine the
in Figure P3.18, while the other end, B, slides on the hori­
angular velocities of and at the indicated instant.
zontal plane. In the indicated position, rad/
sec. Find the velocity of the midpoint of the rod at this
instant.

Figure P3.18

Figure P3.15
Figure P3.19

Figure P3.16

Figure P3.20

Figure P3.17 Figure P3.21


Page 146

3.22 In the mechanism shown in Figure P3.22, the • 3 . 2 5 Block has a controlled position in the slot given
sleeve is connected to the pivoted bar by the 15-cm by in. for sec. (See Fig­
link . Over a certain range of motion of , the angle ure P3.25.) The time is t = 0 sec in the indicated
varies according to rad, starting at t = 0 with position. Find the angular velocities of the rod and the
and horizontal. Find the velocity of pin S and the angu­ wheel at (a) t = 0 sec and (b) t = 5 sec.
lar velocities of and when Time f is mea­
sured in seconds.

Figure P3.22

Figure P3.25

• 3 . 2 3 Find the velocity of point B of the rod if end A has 3.26 Crank of the slider-crank mechanism shown in
constant velocity 2 m / s to the right as shown in Fig­ Figure P3.26 has a constant angular speed Find the
ure P3.23. The rollers are small. Compare the use of equation for the angular velocity of the connecting rod
Equation (3.8) with the procedure used to solve Problem as a function of
1.63.

Figure P3.26

3.27 In the preceding problem, plot as a function of


from
3.28 Referring to Section 3.2, show that neither the an­
Figure P3.23 gular velocity vector nor Equation (3.8) depends on which
body-fixed line segment (such as PQ in the text) is chosen
to measure . Use two other points P' and Q' and their
angle as suggested in Figure P3.28 for your proof.
• 3 . 2 9 Rod begins moving at (see Figure P3.290
and is made to rum at the constant angular rate
3.24 Find the velocity of the guided block at the instant rad/s. The cord is attached to the end of and passes
shown in Figure P3.24.
around a pulley. The other end of the cord is tied to
weight at point B. Observe that moves downward
until , when it reverses direction. Write an equa­
tion that gives the velocity of point B as a function of
for Hint: Using trigonometry, write y as a
function of and the length L of the cord. Then differ­
Figure P3.24 entiate.
Page 1 4 7

Before (time t = t,) Later (time t = t )2

Figure P3.28

• 3 . 3 0 Repeat the preceding problem by using Equation


(3.8) to obtain v ; then resolve v into two components:
A A

(a) one along PA that equals the magnitude of v and B

(b) the other normal to PA, which does not affect B.


(These are sometimes called stretching and swinging com­
ponents, respectively.) Figure P3.29

3.3 Translation
W h e n a rigid b o d y m o v e s during a certain time interval in such a w a y
that its angular velocity vector r e m a i n s identically zero, t h e n t h e b o d y is
said to b e t r a n s l a t i n g , or to b e in a state o f t r a n s l a t i o n a l m o t i o n during
that interval. F r o m Equation (3.8) w e thus see that for translation

T h a t is, all points o f t h e b o d y h a v e the s a m e velocity vector. B y differen­


tiating Equation ( 3 . 1 2 ) , w e see that the accelerations o f all points o f are
also equal for translation. N o t e that if only at an instant (that is, at a
single value o f time rather t h a n over an interval), t h e n all points o f t h e
b o d y h a v e equal velocities at that instant but need not have equal accelera­
tions.

Question 3.7 Why is this the case?

Answer 3.7 The derivative of is not zero merely because happens to be zero
at one instant of time. To be able to differentiate v = v , this equation must be valid
e P

for all values of t and not just one!


Page 1 4 8

Translation c a n b e either:

1. Rectilinear: E a c h point o f m o v e s along a straight line in


2. Curvilinear: E a c h point o f m o v e s o n a c u r v e d p a t h in

E x a m p l e s o f translation are s h o w n in Figure 3 . 8 . Part (a) s h o w s an


e x a m p l e o f rectilinear translation: B o d y is c o n s t r a i n e d to m o v e in a
straight slot. P a r t ( b ) s h o w s a n e x a m p l e o f a i r v i l i n e a r translation:
B o d y is c o n s t r a i n e d b y t h e identical links.

(a) (h)
Figure 3.8 Examples of translation.

P e r h a p s a n e v e n b e t t e r pair o f e x a m p l e s is t h e b l a c k b o a r d eraser
(Figure 3 . 9 ) , w h i c h w e u s e d earlier to explain p l a n e m o t i o n in S e c t i o n 3.2.
In part (a), t h e professor m o v e s t h e eraser s o that e a c h o f its points stays
on a straight line; it is therefore in a state o f rectilinear translation. In part
(b), t h e professor m o v e s t h e eraser o n a curve; b u t if t h e w o r d eraser is
a l w a y s horizontal during t h e erasing, t h e n a n d the eraser is in a
state o f curvilinear translation. E v e n t h o u g h e a c h o f its points m o v e s o n a
curve, all t h e velocities ( a n d accelerations) are e q u a l at all times. T h e r e is
o n e n o t a b l e exception to our earlier s t a t e m e n t that "points, n o t bodies,
h a v e velocities a n d a c c e l e r a t i o n s . " In this present c a s e o f translation,
since all t h e points h a v e t h e same v ' s a n d a's, o n e c o u l d loosely refer to
" t h e velocity o f t h e e r a s e r " w i t h o u t ambiguity.

(a) (b)
Figure 3.9 Another example of translation.
Page 149

T h e r e are n o examples or p r o b l e m s in this section b e c a u s e translation


p r o b l e m s o f rigid bodies require n o n e w theory b e y o n d w h a t w a s devel­
oped in C h a p t e r 1.
S u m m a r i z i n g , w h e n a b o d y is translating (either rectUinearly or cur-
vilinearly), its angular velocity is identically zero, and all its points
h a v e equal velocities (and accelerations). I f o n l y at an instant, then
all the points o f t h e b o d y h a v e the s a m e velocity at that instant b u t n e e d
n o t h a v e equal accelerations.

3.4 Instantaneous Center of Zero Velocity


If P is a point in the reference p l a n e h a v i n g zero velocity at s o m e instant,
t h e n t h e velocity field o f is t h e s a m e as if the b o d y w e r e constrained at
that instant to rotate a b o u t an axis through P n o r m a l to the reference
plane. T h i s axis is called t h e i n s t a n t a n e o u s a x i s of rotation, a n d point P
is called t h e i n s t a n t a n e o u s c e n t e r (abbreviated ) o f z e r o velocity* of
T h u s if Q is a n y other point o f , t h e n w e h a v e

a n d since v is t h e n n o r m a l to b o t h of t h e vectors
Q and , w e see that
each point moves with its velocity perpendicular to the Urn- joining it to __.
Figure 3.10 Instantaneous center of a
rolling wheel. This c o n c e p t is illustrated in Figure 3 . 1 0 for a rolling* w h e e l , in w h i c h
is the contact point.
Proof of the Existence of the Instantaneous Center

W e can s h o w that i f a b o d y has at a given instant, then it


h a s an i n s t a n t a n e o u s center.

Question 3.8 Why can there be no point whenever is zero?

To d e m o n s t r a t e t h e existence o f , w e shall u s e Equation ( 3 . 1 3 ) in


conjunction with Figure 3 . 1 1 . A s w e h a v e n o t e d a b o v e , t h e vector v , Q

being equal to for t h e point having is n o r m a l t o b o t h


. H e n c e w e h a v e these results:

1. T h e vector lies in t h e reference p l a n e a n d is n o r m a l to v . It thus


Q

Figure 3.11 lies along the line in Figure 3 . 1 1 .

* The phrase is admittedly redundant, but it is in common usage. "Instantaneous center


of velocity" would perhaps be more concise, and "center of velocity" even more so.
"Instantaneous center," however, is inadequate because of the possibility of confusion
with points of zero acceleration.
Rolling means no slipping, according to the definition we adopt in this book (see Sec­
tion 3.6).
Answer 3.8 If Equation (3.8) says that v = v ; that is, all points of have the
0 P

same velocity vector. This common velocity vector is then zero only if the body is at rest.
Incidentally, some think of as having an instantaneous center at infinity when
Page 1 5 0

2. T h e point exists (and is unique) b e c a u s e is therefore seen to


be in order that

Question 3.9 Why is below Q in Figure 3.11 instead of being


the same distance above Ql

W e h a v e thus verified the existence o f the i n s t a n t a n e o u s center (unless


), b e c a u s e w e k n o w h o w to get t o it from a n y arbitrary point Q o f
the b o d y w h e n e v e r the angular velocity o f a n d the velocity v of Q

the point Q are k n o w n . W e also n o t e again that the velocity magnitude


of every point o f (in the reference plane!) equals the product o f and
the distance to the point from
S o m e t i m e s b e c a u s e o f a constraint o n the m o t i o n w e k n o w the loca­
tion o f at the outset. This is the case for the rolling w h e e l o f Figure 3 . 1 0
in w h i c h the b o t t o m point grips the ground and is held at rest (thereby
becoming ) for the instant o f its contact. I f the radius o f the w h e e l is 1 5 "
a n d the velocity o f its center t h e n from the a b o v e
discussion, the angular speed o f the w h e e l is

N o t e that w h e n v i e w e d from the senses o f the velocity direction


o f a n y point a n d the angular velocity direction o f the b o d y must always
agree. For e x a m p l e , these are possible situations:

T h e s e are not:

Answoi 3 . 9 If point I were above then would give an incorrect direction


for the velocity v — it would be opposite to the actual direction. The senses of v and
D 0

to when viewed from must agree, as we point out later in this section.
Page 151

Therefore the direction of o f the w h e e l in Figure 3 . 1 0 is clockwise


in order that the k n o w n velocity direction o f point C a n d the
angular velocity of the b o d y b e in a g r e e m e n t as the b o d y rotates about
at the instant s h o w n .
E v e n i f the angular velocity o f is not k n o w n , w e can still easily find
the instantaneous center o f zero velocity if w e k n o w t h e velocities —
or really, just the directions o f the velocities — o f two points A a n d B of
Constructing perpendicular lines to the velocities o f A (at A) a n d o f B
(at B) as s h o w n in Figure 3 . 1 2 , w e immediately recognize as the
Figure 3.12 intersection point o f the two lines.

Question 3.10 Why?

F r o m Figure 3 . 1 2 a n d the discussion a b o v e , w e k n o w that:

(Recall that there is only o n e for the b o d y . )

w h e r e w e are abbreviating by
,and by .
We n o w present three e x a m p l e s o f the use o f the a b o v e procedure for
locating w h e n t w o velocity directions are k n o w n in advance.

Answer 3 . 1 0 The point is unique. Since there is only one common point on the lines
drawn perpendicular to the velocities (to v at B and to v,, at A), that point is the instan­
B

taneous center.

EXAMPLE 3.6

Ladders commonly carry a warning that for safe placement, the distance B in
Figure E3.6a should be of the length L (i.e., of ). Let us suppose that a
careless painter temporarily set a ladder against a wall in a dangerous position
with B/l = 0.5, and went off to get his paint and brushes. Suppose further that
the ladder began to slip, with the top of the ladder, point P, sliding down the wall
and the bottom, point Q, slipping along the ground as shown. When B is 15 ft,
find the instantaneous center of zero velocity of the ladder, and discuss the
path of in space as the ladder falls further.
Solution
When B = 15 ft, the normals to v at P and to v at Q intersect at point
P Q as
shown in Figure E3.6b on the next page.
If we imagine a rigid sheet of very light plastic glued to the ladder as in
Figure E3.6c, then the ladder has been "rigidly extended." Note that only for this
Figure E3.6a instant, we can think of the extended body as rotating about an imaginary pin at
the intersection point of the normals to two velocities as shown. Note further
from Figure E3.6c that the velocities of all points of the rigid sheet are perpendic­
ular to lines drawn to them from . The velocity magnitude of each point is
proportional to the distance from that point to , with the proportionality
Page 1 5 2

Line normal
to vr

Line normal
to v Q S

Figure E3.6b Figure E3.6c

constant being of the body at the instant. Hence all triangles like the three
shaded in Figure E3.6c are similar.
As the ladder falls, the location of the point _ changes on the imagined

rigid extension (sheet) as time passes, because the perpendiculars to v and v„


A

intersect at different points of the sheet, as seen below in Figure E3.6d:

Figure E3.6d

Note that as point P (and thus all of the ladder) gets closer and closer to the
ground, point _ gets closer and closer to point Q. Thus even though increases,

gets smaller and smaller, until, in the limit (as P contacts the ground) Q
becomes (the intersection of the perpendiculars) and V is then zero.
Q

W h e n t h e b o d y h a s a pivot (a point that n e v e r m o v e s throughout


the body's m o t i o n , such as a pin), it is clearly always ; in this case the
motion is called pit re rotation. But otherwise, the point is not the s a m e
Page 1 5 3

point o f t h r o u g h o u t t h e motion, as w e h a v e already s e e n with t h e wheel,


a n d ladder. In e a c h o f t h e n e x t t w o e x a m p l e s , o n e o f the bodies h a s a,
pivot.

EXAMPLE 3.7

Rework Example 3.3 using instantaneous centers. (See Figure E3.7a) The crank
arm turns about a horizontal z axis, through its pinned end O, with an angular
velocity of 10 rad/sec clockwise at the given instant. Find the velocity of the
piston pin B.
Solution
Since O is the point for body , we have

Figure E3.7a

Next we find the of , using the fact that it is on lines perpendicular to the
velocities of A and B as shown in Figure 3.7b. If we next find the distance D from
to A, then will be By similar triangles:

Then

so

Again by similar triangles:


Figure E3.7b

so that i n . / s e c to the right, as we have seen be­


fore.

EXAMPLE 3.8

At the instant shown in Figure E3.8a, the angular velocity of bar is


rad/sec. Find the velocity of pin B connecting bar to the slider
block, constrained to slide in the slot as shown.

Solution
As seen in Figure E3.8b on the next page, the point for is O, since it is
pinned to the reference frame. The velocity of A is perpendicular to the line from
to A (that is, from O to A) and has a direction in agreement with the angular
Figure E3.8a velocity of as the body turns about O. Its value is
Page 1 5 4

Next we sketch body and note the position of for , as explained in


Figure E3.8c. Similar triangles yield the height H of above A:

Figure E3.8b
Question 3.11 Why does v have to be "southwest" along the slot and
B

not "northeast"?
is on each of these
lines since they are each
normal to the velocity of We may now use of to get the angular velocity of ; using vectors this
a point of £
2 time,

Substituting, we get

Solving gives

Figure E3.8c Note that when we write we are saying that is counterclockwise in
accordance with the sign convention adopted for the problem in the figure if its
value turns out positive. Thus when its value is now found to be negative, we
know that is turning clockwise at the given instant. Of course, as we have seen,
we do not have to use vectors on such a simple problem; we can use what we
know about the instantaneous center in scalar form to get a quick solution:

where we assign the direction in accordance with the known velocity direction of
A and the position of
Next we use of to obtain v : B

The velocity of B is thus

Note that the arrow in this sketch is just as descriptive of the direction of the
vector velocity of B as is the unit vector —

Answer 3.11 The known velocity direction of A dictates that is turning clockwise
around so v has to be "southwest" for this to be the case.
B
Page 155

T h e Special Case in W h i c h the Normals Do Not Intersect

T w o things can go w r o n g with the procedure w e h a v e b e e n following o f


intersecting t h e n o r m a l s to two p o i n t s ' velocity vectors to find . T h e
first of t h e s e is that t h e t w o perpendicular lines m a y b e parallel a n d h e n c e
n o t intersect, as suggested b y Figure 3 . 1 3 b e l o w for t h e points A a n d B o f
bar N o t e that A a n d B are e a c h at t h e top o f t h e vertical p l a n e circles on
w h i c h t h e y m o v e , a n d since their velocity vectors are tangent to their
paths, e a c h is horizontal at this instant.

Perpendiculars
d o not
intersect!

Figure 3.13

Let us e x a m i n e E q u a t i o n 3.8 for this case:

Since v a n d v h a v e o n l y -components w h i l e
B A has both
and then m u s t b e zero at this instant. T h i s does n o t m e a n t h e b o d y
is translating; that occurs w h e n is zero all the time. Rather, in this case
t h e b o d y is just stopped for o n e instant in its angular motion, as its ang­
ular velocity is c h a n g i n g from clockwise to counterclockwise (see Fig­
ure 3 . 1 4 ) .
T h e equation a b o v e also s h o w s that at such an instant w h e n
all points o f t h e b o d y h a v e identical velocities. S o in Figure 3 . 1 3 ,
v = v = v
A B at t h a t instant. C o n v e r s e l y , a n y time t w o points o f a
B n y p o j n l ofe

b o d y h a v e equal velocities in p l a n e motion, t h e b o d y ' s angular velocity


vanishes at that instant.

is just before A and E Circular path


reached highest points... of B

Circular path
of A

...but is just after A and B leave highest


points. At the highest points,
Figure 3.14
Page 1 5 6

T h e Special Case in W h i c h the Normals are Coincident

T h e s e c o n d exceptional case occurs w h e n the perpendiculars to two


velocities are o n e a n d the s a m e line (see Figures 3.15(a,b)):

(a) (b)
Figure 3.15

In this case, w e c a n find the i n s t a n t a n e o u s center using similar triangles


as s h o w n . T h i s simple procedure works, b e c a u s e , for the case s h o w n in
Figure 3 . 1 5 a ,

so that

If w e s h o u l d get coincident n o r m a l s w i t h the directions o f v a n d v


P Q

opposite, as in Figure 3 . 1 5 b , t h e n I lies between P a n d Q, a n d it m a y


again b e f o u n d b y similar triangles. T h i s time,

In the e x a m p l e s w e h a v e p r e s e n t e d in this section, n o t e that use of


the i n s t a n t a n e o u s center m a y b e m a d e w i t h or w i t h o u t vector algebra. Its
a d v a n t a g e is in finding a n d using points o f zero velocity in order to
simplify the resulting m a t h e m a t i c s . Instantaneous centers never have to
be u s e d to effect a solution. S o m e t i m e s they are helpful, b u t at other
times, t h e y m a y b e m o r e trouble to locate t h a n t h e y are worth!
E x a m p l e s o f b o t h the a b o v e special cases occur in our last e x a m p l e in
this section. It involves four different positions o f the s a m e system:

EXAMPLE 3.9

Figure E3.9a shows a rolling wheel of a large vehicle that travels at a constant
speed of . Find the velocity of the piston when equals: (a) 0°, (b) 90°,
(c) 180°, (d) 270°.
Page 157

Figure E3.9a

Solution
First we solve for the velocities by using several approaches. In each case, the
speed of the wheel's center is the same as the speed of the vehicle: 60 mph or
88 ft/sec. And since the piston translates, all its points have equal velocities
and equal accelerations at every instant.

Case (a): As we shall see in detail in Section 3.6, the instantaneous center of a
wheel rolling on a fixed track is at the point of contact. Since velocities increase
linearly with distance from this point , we have for the point E of :

If we draw lines at E and P perpendicular to v and v (P is constrained to move


E P

horizontally), they are parallel and thus will not intersect (see Figure E3.9b).
Therefore at that instant. Thus

Figure E3.9b Figure E3.9c

Case (b): This time we shall use vectors; on the wheel (see Figure E3.9c),
Page 1 5 8

or

or

On now (after noting the trigonometry results in Figure E3.9d):

Equating coefficients of and then of we obtain:

Therefore

so that

Figure E3.9d

Figure E3.9e

Case (c): In all four cases, rad/sec. This time, then (see Fig­
ure E3.9e),

Again, as in Case (a), body has so that all points of have the same
velocity. Thus
Page 1 5 9

Figure E3.9f

Case (d): Using Figure E3.9f, we see that, on body ,

We shall now use the instantaneous center of . From the similar triangles in the
above figure,

Therefore, on body ,

and

N o t e from the four answers in the a b o v e e x a m p l e that the piston is


moving faster during the parts o f the wheel's revolution in w h i c h v E

makes small angles with r o d ; conversely, it m o v e s slower w h e n v is £

making a large angle with . This is b e c a u s e the c o m p o n e n t s of v a n d v


E P

along must always b e t h e s a m e , as w e s a w earlier.


Page 1 6 0

PROBLEMS • Section 3.4

3.31 The angular velocity of in Figure P3.31 is 3.35 Solve Problem 3.6 by using instantaneous centers.
r a d / s = constant. Trace the five sketches and then
3.36 Solve Problem 3.24 by using instantaneous
show on parts (a) to (d) the position of for the rod .In
centers.
part (e), using the proper length of , draw the positions
of at the two times when v = 0 . Rod
B has length 3.37 Solve Problem 3.11 by using instantaneous
0.9 m. centers.
3.38 In Figure P3.38 the crank arm is 4 in. long and
3.32 Solve Problem 3.16 by using instantaneous
has constant angular velocity rad/sec. It is
centers.
pinned to the triangular plate, , which is also pinned to
3.33 Solve Problem 3.7 by using instantaneous centers.
the block in the slot at D. Find the velocity of D at the
3.34 Solve Problem 3.8 by using instantaneous centers. instant shown.

i.n (b) (c)

(d) (e)
Figure P3.31
Page 161

3.42 Rods and are pinned at B and move in a verti­


cal plane with the constant angular velocities shown in
Figure P3.42. Locate the instantaneous center of, forthe
given position, and use it to find the velocity of point C.
Then check by calculating v by relating it (on ) to the
c

velocity of B. Note that sometimes is more trouble to


locate than it is worth!

Figure P3.38

3.39 See Figure P3.39. The angular velocity of rod is a


constant: rad/s. Determine the angular veloc­
ities of plate ^ and bar in the indicated position.
3.40 The pin at B (Figure P3.40) has a constant speed of
51 c m / s and moves in a circle in the clockwise direction.
Find the angular velocities of bars and in the given Figure P3.42
position.
3.41 Solve Example 3.5 by using the instantaneous
center of the rod. 3.43 The linkage shown in Figure P3.43 is made up of
rods and . Rod has constant angular velocity
rad/sec. Determine the angular velocities of
and when the angle is equal to 90° as shown.

Figure P3.43

3.44 A bar of length 2L moves with its ends in contact


with the planes shown in Figure P3.44. Find the velocity'
Figure P3.39 and acceleration of point C, in terms of and its deriva­
tives, by writing and then differentiating the position
vector of C. Then check the velocity solution by using the
instantaneous center.

Figure P3.40 Figure P3.44


Page 162

3.45 The piston rod of the hydraulic cylinder shown in 3.48 The roller at B, which moves in the parabolic slot, is
Figure P3.45 moves outward at the constant speed of pinned to bar as shown in Figure P3.48. Bar is
0.13 m / s . Find the angular velocity of at the instant pinned to at A. The angular velocity of, at this instant
shown. is shown. Find the angular velocity of , at this time.
3.49 Bars and (see Figure P3.49) are pinned to­
gether at A. Find the angular velocity of bar , and the
velocity of point B when the bars are next collinear. Hint:
To find this configuration, draw a series of rough sketches
of and as turns counterclockwise from the position
shown, and you will see and coming into alignment.
3.50 Repeat the preceding problem at the second instant
of time when the bars are collinear. Follow the same hint,
but this time start just past the first collinear position,
found in Problem 3.49.
3.51 The constant angular velocity of wheel is
rad/sec. It is in rolling (i.e., "no slip") contact
Figure P3.45 with , which means the contacting points have the
same velocity. Find the angular velocity of the bar at
the instant shown in Figure P3.51.
3.46 Using the method of instantaneous centers, find
the velocity of point B in Figure P3.46, which is con­
strained to move in the slot as shown. The angular veloc­
ity of is r a d / s at the indicated instant.
3.47 The center of block in Figure P3.47 travels at a
constant speed of 30 mph to the right. Disk is pinned to
at A and spins at 100 rpm counterclockwise. Find: (a)
the velocity of P; (b) the instantaneous center of
and (c), using , the velocities of Q, S, and R.

Figure P3.48

Figure P3.49
Figure P3.4G

Figure P3.47 Figure P3.51


Page 163

3.5 Acceleration and Angular Acceleration Relationship for


T w o Points of the Same Rigid Body
T h e a n g u l a r a c c e l e r a t i o n vector o f a rigid b o d y in plane m o t i o n in a
frame is defined as the derivative in o f the angular velocity a n d is
called a :

or

where is a constant vector in b o t h and N o t e that, as with , we


delete the subscript w h e n there is n o confusion a b o u t the frame o f refer­
e n c e being used.

Development of the Acceleration and Angular Acceleration Relationship

W e n o w develop the relationship b e t w e e n the accelerations o f t w o points


P a n d Q o f a rigid b o d y Differentiating b o t h sides o f Equation ( 3 . 8 )
yields

N o w using Equation ( 3 . 7 ) , * w e m a y rewrite t h e last term as

or

Question 3.12 Why is the dot product in Equation (3.17)


zero?

Equations ( 3 . 1 6 ) and ( 3 . 1 8 ) t h e n yield the desired relation b e t w e e n the


accelerations o f P a n d Q:

W e n o t e that the s a m e three rules spelled out in E q u a t i o n ( 3 . 1 0 ) also


h o l d for the use o f Equation ( 3 . 1 9 ) . U n l i k e velocities, h o w e v e r , the accel­
erations o f P a n d Q do n o t generally h a v e equal c o m p o n e n t s along the
line P Q joining t h e m ; t h e s e c o m p o n e n t s differ b y . Likewise,
the c o m p o n e n t s perpendicular to PQ differ b y n fin the s a m e w a y as
the velocity c o m p o n e n t s n o r m a l to PQ differ b y ).

Answer3.12 Because r P C lies in the (xy) reference plane and is therefore perpendicular
to
* And the vector identity A X (B X C) = B(A • C) - C(A • B).
Page 164

Fi gure 3,1

If t h e acceleration of point A of a b o d y is & , for e x a m p l e , t h e n t h e


A

acceleration of a n y point B is the s u m o f the three vectors s h o w n in


Figure 3 . 1 6 . If A is a p i n n e d point, or pivot,* t h e n a h a s two c o m p o n e n t s :
B

o n e along t h e line from B t o w a r d A a n d t h e other perpendicular to it a n d


t a n g e n t to t h e circle (on w h i c h it necessarily travels w h e n there is a pivot
at A ) . N o t e that the direction o f the tangential c o m p o n e n t depends o n the
direction o f a, but t h e " r a d i a l " c o m p o n e n t is always i n w a r d t o w a r d t h e
Figure 3.17
pivot. T h i s case is s h o w n in Figure 3 . 1 7 . W e n o w illustrate the use o f
Equation ( 3 . 1 9 ) with several examples.

EXAMPLE 3 . 1 0

In Figure E3.10, let the links have length 1 m and let them each have
J
r a d / s and r a d / s at a time when they make an angle of 45°
with the ceiling. Find the acceleration of block (That is, find the acceleration
of any of its points — they are all the same since is translating.)
Solution
All points of have the same v and a as point A. Using Equation (3.19) for the
link OA, we get

Figure E3.10

EXAMPLE 3 . 1 1

In Example 3.3 find the acceleration of the translating piston at the given instant
2
if rad/sec and r a d / s e c . See Figure E 3 . l l .

* Again, pivot means a point of that does not move throughout a motion. It includes,
Figure E3.11 but is not limited to, a hinge.
Page 1 6 5

Question 3.13 What does it mean when a is in the opposite direction


from that of ?

Solution
Relating O and A on body by Equation (3.19), we have

And relating A and B on body , we have

In the previous example we found

Noting that the piston translates horizontally, we get

The coefficients of yield

The coefficients of then give our answer:

Let us find the acceleration o f the i n s t a n t a n e o u s center o f zero v e l o c ­


ity o f the rod in E x a m p l e s 3.7 a n d 3 . 1 1 . Using those e x a m p l e s and
Equation ( 3 . 1 9 ) , w e h a v e

W e see from this result that the i n s t a n t a n e o u s center o f zero velocity


does n o t generally h a v e zero acceleration. U n l e s s the point is a pivot
(i.e., a p e r m a n e n t l y fixed point, s u c h as a pin c o n n e c t i n g the point to the
reference frame), it s h o u l d never b e a s s u m e d that is zero.*

Answer 3.13 It means that the angular speed of the body is decreasing.
* Actually, in nontranslational cases there is a point of zero acceleration, but unless it is
a pivot point, or a point of rolling contact at an instant when , it is more trouble
to find than it is worth. See Problem 3.83.
Page 166

EXAMPLE 3 . 1 2

In Example 3.5, find the angular acceleration of the rod and the acceleration of its
endpoint A. See Figure E3.12a.
Solution
Relating the accelerations of B and A with Equation (3.19),

The acceleration of point B is

Figure E3.12a
where since = constant = 0.3 m / s . The curvature formula from calcu­
lus gives us the radius of curvature at point B:

Therefore

Therefore,

The unit vector is seen in Figure E3.12b to be

was 63.4°
(From Example 35] Therefore

Figure E3.12b

Substituting a into Equation (1) gives


B

where has also been substituted.


Next, the radius of curvature of the path of A at the instant of interest is

Substituting , and from Example 3.5 (along with


and at point A), we obtain the vector equation:
Page 1 6 7

Writing the component equations, we have

The equation gives

And the equation then yields

Therefore

M a n y m o r e examples of the use of Equation (3.19) will be found in


the next section, after w e h a v e discussed the topic of rolling.

PROBLEMS • Section 3.5

3.52 In Figure P3.52 the angular velocity of the bent bar


is 0.2 r a d / s counterclockwise at an instant when its angu­
2
lar acceleration is 0.3 r a d / s clockwise. Find the accel­
eration of the endpoint B in the indicated position.

Figure P3.53

3.54 End B of the rod shown in Figure P3.54 has a con­


stant velocity of 10 ft/sec down the plane. For the posi­
tion shown (rod horizontal) detenrtine the velocity and
Figure P3.52 acceleration of end A of the rod.

3.53 The acceleration of pin B in Figure P3.53 is 9.9 f t /


2
sec down and to the left, and its velocity is 4 f t / s e c up
and to the right, when passes the horizontal. At this
instant, find the angular acceleration of Figure P3.54
Page 168

3.55 The velocities and accelerations of the two end- 3.60 IfinProblem 3.11 the bar whose angular velocity is
points A and B of a rigid bar in plane motion are as shown given to be 2 rad / sec has an angular acceleration of zero
in Figure P3.55. Find the acceleration of the midpoint of at that instant, find the angular acceleration of the 5-inch
the bar in the given position. (horizontal) bar.
3.56 In Problem 3.42 find the acceleration of C in the * 3.61 The angular velocity of in Figure P3.61 is a con­
position given in the figure. stant 3 rad/sec clockwise. Find the velocity and accelera­
3.57 At the instant given, the angular velocity and an­ tion of point C in the given configuration, and determine
gular acceleration of bar are rad/sec and the acceleration of point C when v = 0.
c

2
r a d / s e c . (See Figure P3.57.) Find the angular accel­
erations of and at this instant.
3.58 Bar rotates with a constant angular velocity
of rad / sec. Find the angular velocities and angular
accelerations of and at the instant shown in Fig­
ure P3.58.
3.59 In the position indicated in Figure P3.59, the slider
block has the indicated velocity and acceleration. Find
the angular acceleration of the wheel at this instant.

Figure P3.61

Figure P3.55

Figure P3.64

3.62 Refer to the preceding problem. At the instant of


time when , find the acceleration of point C.
Figure P3.57
3.63 If in Problem 3.24 the angular velocity of the
Figure P3.58 5-inch bar is constant throughout an interval which
includes the instant shown, find at that instant the accel­
eration of the guided block.
3.64 At the instant of time shown in Figure P3.64, the
angular velocity and angular acceleration of rod are
2
rad/sec and r a d / s e c . At the same time, find the
angular acceleration of bar
3.65 In Problem 3.26 find the equation for the angular
acceleration of the connecting rod a a function of r,
Figure P3.59 , and .
Page 169

3.66 In the preceding problem, plot versus 3.72 InProblem 3.49 find the acceleration of point Band
(0 < 6 £ In) for = 1, 2, and 5. the angular acceleration of body at the described in­
3.67 Crank in Figure P3.67 is pinned to rod ; the stant.
other end of slides on a parabolic incline and is at the 3.73 In Problem 3.5 0 find the acceleration of point B and
origin in the position shown. The angular velocity of is the angular acceleration of body at the described in­
r a d / s = constant. Determine the acceleration of A stant.
and the angular acceleration of at the given instant. 3.74 In Problem 3.22 find the acceleration of S and the
Hint The radius of curvature p of a plane curve y = y(x) angular accelerations of and at the instant when
can be calculated from = 30°.
3.75 In Problem 3.38 determine the angular accelera­
tion of the plate and the acceleration of pin l? in the
indicated position.
Use this result in computing the normal component of a A

as in Example 3.11. 3.76 In Problem 3.45 determine the angular accelera­


tions of and in the indicated position.
3.77 The motion ofarotating element in a mechanism is
controlled so that the rate of change of angular speed
with angular displacement is a constant K. If the angular
speed is when both and the time t are zero, determine
and the angular acceleration a as functions of time.
• 3 . 7 8 Rod in Figure P3.78 is pinned to disk at A and
B. Disk rotates about a fixed axis through O. The rod
makes an angle radians with the line as shown, where
Figure P3.67 = sin t. The time is given by t in seconds. Deteirmine the
horizontal and vertical components of acceleration of the
midpoint P of segment AB when rad.
3.79 A rod is pinned at A and B to the centers of two
3.68 The 10-ft bar in Figure P3.68 is sliding down the
small rollers. (See Figure P3.79.) The speed of A is kept
13-ft-radius circle as shown. For the position shown, the
constant at v even after B encounters the parabolic sur­
0
bar has an angular velocity of 2 r a d / s e c and an angular
2
face. Find the acceleration of B just after its roller begins to
acceleration of 3 rad/sec , both clockwise. Find the x and
move on the parabola.
y components of acceleration of point B for this position.
3.69 In Problem 3.23, find the acceleration of B when
x = 10 m.
3.70 In Problem 3.31 find the acceleration of the upper
end B of rod in position (a).
3.71 Find the acceleration of P in Problem 3.58 if
at the same instant the body has angular acceleration
2
r a d / s e c instead of zero.

Figure P3.78

Figure P3.68 Figure P3.79


Page
170

* 3.80 In the preceding problem, find a just after the left


B • 3 . 8 2 The right end P of bar is constrained to move to
roller has begun to travel on the parabola. the right on the sine wave shown in Figure P3.82 at the
* 3.81 The bent bar shown in Figure P3.81 slides on the constant speed in./sec. The left end A of is con­
vertical and horizontal surfaces. For the position shown, strained to slide along the x-axis. At the instant when
2
A has an acceleration of 4 f t / s e c to the left, while the bar in., find (a) (b)a . P

has an angular velocity of 2 rad/sec clockwise and an • 3 . 8 3 Show that for a rigid body in plane motion, as
angular acceleration of (X. long as and a are not both zero there is a point of
having zero acceleration. Hint: Let P be a reference point
a. Determine a for the position shown. with acceleration See if you can find
b. Find, for the position shown, the angle and a vector from P to a point T of zero ac­
the distance PA such that point P has zero ac­ celeration. That is, solve
celeration.

for x and y.

y — 3 sin x in

Bar shown at in

Figure P3.81 Figure P3.82

3.6 Rolling
Let and b e two rigid bodies in m o t i o n . W e define r o l l i n g to exist
between and if during their motion:

1. A c o n t i n u o u s s e q u e n c e o f points on t h e surface o f c o m e s into


o n e - t o - o n e contact with a continuous s e q u e n c e o f points o n the
surface o f
2. At e a c h instant during t h e interval o f t h e motion, t h e contacting
points h a v e t h e s a m e velocity vector.

N o t e that according to this definition there can b e n o slipping or sliding


b e t w e e n t h e surfaces o f and if rolling exists. M a n y authors, h o w ­
ever, use t h e p h r a s e "rolling without slipping" to describe t h e m o t i o n
defined h e r e . In their context, "rolling a n d slipping" w o u l d denote turn­
ing without t h e contact points h a v i n g equal velocities; in our c o n ­
text, rolling means n o slipping, so w e shall s a y " t u r n i n g a n d slipping" in
cases o f u n e q u a l contact-point velocities.
In this section w e consider three classes o f p r o b l e m s involving rolling
contact:

1. Rolling o f a w h e e l o n a fixed straight line


2. Rolling o f a w h e e l o n a fixed p l a n e curve
3. Gears
Page 171

Rolling of a W h e e l on a Fixed Straight Line

If the w h e e l s h o w n in Figure 3 . 1 8 is rolling o n t h e g r o u n d ( , t h e


reference frame in this case), t h e n t h e c o n t i n u o u s (shaded) s e q u e n c e s of
points o f a n d , are in contact, o n e pair o f points at a time. S i n c e the
points o f are all at rest, e a c h point P o n t h e rim o f
1 c o m e s instanta­
n e o u s l y to rest as it contacts a point P o f 2 (and is gripped for a n instant
b y t h e ground). In this case, t h e velocities o f P , a n d P are e a c h zero,
2

although t h e y n e e d n o t vanish in general for rolling; all that is required is


t h a t v p , = vp .*
2

Figure 3.18 Rolling wheel, illustrating contacting sequence of points.

Using E q u a t i o n ( 3 . 8 ) a n d n o t i n g that t h e center point C m o v e s only


horizontally a n d that t h e c o n t a c t point is a l w a y s t h e i n s t a n t a n e o u s center
of ,

For t h e rolling w h e e l , therefore, t h e velocity o f its center C a n d t h e b o d y ' s


angular velocity are related quite simply b y

Question 3 . 1 4 H o w would this expression be different w e r e to h a v e


been chosen so that it would increase with counterclockwise turning of
the body?

T h e relation b e t w e e n the d i s p l a c e m e n t o f C a n d the rotation of


follows from integrating Equation ( 3 . 2 3 ) :

w h e r e t h e integration constant is zero if w e c h o o s e x c = 0 when

* If is the flatbed of a truck, for example, itself in morion with respect to a ground
reference frame then but E still rolls on
x

Answer 3.14 Then we would have


Page 172

A n o t h e r a p p r o a c h to rolling is to b e g i n with a small d i s p l a c e m e n t


while the base point grips t h e ground. I f t h e angle o f rotation
p r o d u c e d is , then

if w e envision t h e b o d y turning about its i n s t a n t a n e o u s center. Dividing


b y a small time i n c r e m e n t a n d taking t h e limit, w e get

as w a s o b t a i n e d in Equation ( 3 . 2 3 ) .
N e x t w e consider accelerations. F r o m Equation ( 3 . 2 2 ) :

a n d the c e n t e r point C accelerates parallel to the p l a n e , as it m u s t since it


(alone a m o n g all points o f t h e w h e e l ) h a s rectilinear m o t i o n . N o w let us
c o m p u t e t h e acceleration o f t h e i n s t a n t a n e o u s c e n t e r of the wheel.
Using Equation ( 3 . 1 9 ) , w e obtain

T h u s t h e c o n t a c t point o f a w h e e l rolling o n a flat, fixed p l a n e is


accelerated t o w a r d its c e n t e r with a m a g n i t u d e . W e see o n c e again
that a point o f zero velocity n e e d not b e a point o f zero acceleration,
although o f course it will b e s u c h if it is p i n n e d to t h e reference frame. T h e
point in t h e e x a m p l e is at rest instantanously, but it h a s an
acceleration — w h i c h is w h y its velocity c h a n g e s from zero as s o o n as it
moves and a new takes its place in t h e rolling. W e n o w take u p several
examples,* e a c h o f w h i c h deals w i t h a r o u n d object rolling on a flat
surface.

EXAMPLE 3 . 1 3

At a given instant, the rolling cylinder in Figure E3.13 has r a d / s and


2
r a d / s . Find the velocity and acceleration of points N and E.
Solution
We shall relate the velocities and accelerations of N and E to those of point C.
Calculating these, we have, because of the rolling (with S being ),

Figure E3.13

* The examples of this section make continued use of Equations (3.8), (3.13), and (3.19),
with the added feature that a rolling body is involved in each problem.
Page 173

and

Therefore

Note the agreement of this result with

Continuing, we get

EXAMPLE 3 . 1 4

In Example 3.9 find the piston acceleration when 6 = 9 0 ° . See Figure E3.14.
Solution
In Case E,
For point (b) of Example 3.9 we found rad/sec,
ft/sec, rad/sec, a n d f t / s e c . On b o d y , the
Figure E3.14 velocity of C is constant so that a = 0. Also, a = ra so that a = 0.
c c t x

iThe ,reader
and which may
as we
wish
have
to obtain
seen isthe results
not for
zero.
a. and a by relating them instead to
N E
Page 174

Therefore

Equating the coefficients first eliminates the unknown a :


P

Then the coefficients of yield our answer:


EXAMPLE 3 . 1 5

The wheel in Figure E3.15 rolls to the right on plane At the instant shown in
the figure, has angular velocity r a d / s . Rod is pinned to at A,
and the other end B of slides along a plane Q parallel to p. Determine the
velocity of B and the angular velocity of at the given instant.

Figure E3.15

Solution
We shall use Equation (3.8) together with the results we have derived for rolling.
We first seek the velocity of A; when we have v we shall then relate it to v on
A B

rod We may find v in either of two ways, each on wheel


A
Page 1 7 5

We note for interest that when point A is beneath its velocity is to the
left — that is, it is going backwards!
Next, on

Point B is constrained to move horizontally; therefore

or

so that

EXAMPLE 3 . 1 6
so the
In thatpreceding example, at the same instant, 2
r a d / s . Find the accel­
eration of point B and the angular acceleration of rod fi • 2

Solution
The reader is encouraged to mentally locate the point for £ and from it deduce
2

thatwe
As the
diddirections
with the of andofvA, are
velocity B we correct.
can relate a to the acceleration of either
A

orC:
Page 176

Now we relate a to a on the rod; note that the acceleration of B is con­


A B

strained by the plane to be horizontal:

so that

Rolling of a W h e e l on a Fixed Plane Curve

In the s e c o n d class o f rolling p r o b l e m s to b e c o n s i d e r e d in this section, t h e


csoo nthat
t a c t surface is curved. L e t and b e the principal unit t a n g e n t a n d
n o r m a l vectors for t h e c e n t e r point C o f the w h e e l . ( S e e Figure 3.19.)
T h e n , since w e are again defining a m o t i o n s u c h that t h e c o n t a c t point
h a s zero velocity, E q u a t i o n ( 3 . 8 ) yields

w h i c h gives us the velocity o f C. Differentiating E q u a t i o n ( 3 . 2 7 ) , w e get

Figure 3.19 Wheel rolling on curved path


concave upward.

in w h i c h w e h a v e u s e d Equations ( 1 . 4 2 ) a n d ( 1 . 4 5 ) , w h e r e p is t h e instan­
t a n e o u s radius o f curvature o f t h e p a t h o n w h i c h C m o v e s . If this p a t h is a
circle, as is often the case, t h e n p = constant = radius o f t h e circle.

T h e acceleration o f is interesting, a n d it follows from Equation ( 3 . 1 9 ) :

or
Page 1 7 7

Comparing the accelerations of the contact points of Figures 3.18


and 3.19, we observe from our results (Equations 3.26 and 3.31) that the
point contacting the curved track has a greater acceleration than the one
touching the flat track, due to the r/p term of Equation (3.31). This term
represents the normal component of acceleration of C, which was zero on
the flat track.
It is also interesting to examine the acceleration of point Q at the top
of the wheel in Figure 3.19:

Note that it is possible for the (normal) component of AQ to be either


away from or toward C (or even zero), depending on whether r > pat
r <p(orr = p), respectively (seeFigure 3.20).

Figure 3.20

If the track is concave downward as shown in Figure 3.21, similar


results may be obtained for the accelerations of C, , and Q. From
Equation (3.29), which still holds:

After relating and ac on , we obtain:


Figure 3.21

And relating Q to either C or gives


inward
Notenormal
the that
in this
this
component
time
case.the radius
of the
r cannot
acceleration
exceed,of Q is_ seen
is always
to beoutward;
always
Page 178

T h e reader m a y find it useful to r e m e m b e r t h e following form that


doesn't d e p e n d u p o n a n y particular c h o i c e o f coordinate system:

If w e k n e w either a or t h e tangential c o m p o n e n t o f a , for e x a m p l e , w e


c

could get t h e other w i t h o u t c o n c e r n for expressing t h e n o r m a l c o m p o ­


n e n t o f a . W e n o w consider several e x a m p l e s that feature a b o d y rolling
c

on a curved surface.

EXAMPLE 3 . 1 7

The cylinder shown in Figure E3.17a is rolling on the fixed, circular track with
the indicated angular velocity and acceleration when is at the bottom of the
track. Rod is pinned to the center C of , and its other end, B, slides on track
Find the velocity and acceleration of B.
Solution
Figure E3.17i As we have seen in Section 3.5, problems like this one have two parts. The
"velocity part" must be solved before the "acceleration part" because the veloci­
ties as well as angular velocities are needed in the expressions for acceleration.
We shall use instantaneous centers to get v ; the steps are:
B

1. The contact point of is its point, since the body is rolling (see Fig­
ure E3.17b).
2. v is determined as in the diagram from
c

3. The velocity of B is vertical (tangent to the path of the point).


4. Thus of is at the intersection of the normals to v and v , namely at
c B

point O (see Figure E3.17c).


5. Then rad/s
6. Finally, m/s

Next, to find a we shall relate it to the acceleration of C, which is, from Equa­
B
Figure E3.17b
tion (3.29),

Relating a to a , we get, using Figure E3.17c:


c B

Figure E3.17c

In substituting for a we have used the facts that for B we have


R (direction
vector of v ) and
B (always toward the center of curvature of the path of
Page 1 7 9

the point). Substituting we then have two


scalar equations in the two unknowns and a : 2

Therefore

W e wish to m a k e a very important point regarding the preceding


example. Y o u m a y h a v e n o t i c e d that the values o f a a n d2could h a v e
b e e n o b t a i n e d m o r e quickly from

and

TTizs shortcut is very dangerous because it is not always valid! It is essential to


u n d e r s t a n d w h e n a n d w h y this procedure works. (It is not b e c a u s e O is
the instantaneous center o f zero velocity o f !) For a counterexample,
consider the results o f E x a m p l e s 3 . 3 , 3 . 7 , a n d 3 . 1 1 :

fiqure 3.22

If w e were to divide (see Figure 3 . 2 2 ) , w e w o u l d erro­


2
n e o u s l y obtain 2 6 . 6 r a d / s e c with a direction indicator . But a 2

2
is r a d / s e c ! T h e a n s w e r to the question o f w h e n the procedure
is legitimate is covered b y the following text question:

Question 3.15 When can we use to obtain the correct accel­


eration component of B normal to line i B?

Answer 3.15 From Figure 3.13 we can see exactly when the component of a„ normal to
line AB is given by r a It is when a* has no component normal to AB. (This result does
AB

not require that A be the point , incidentally.)


Page 180

EXAMPLE 3 . 1 8
2
The velocity magnitude of G in Figure E3.18a is v = t ft/sec, and G moves on
G

the 8-ft circle in a clockwise direction. The position shown is at t = 2 sec. Find the
acceleration of B at this instant.

-Circular rolling surface

Figure E3.18a

Solution
We shall relate a to ac on bar
B

First, we determine a . Since


G in this problem, we have
2
= 2(2) = 4 f t / s e c . Therefore

From Equation (1)

in which the acceleration of B is an unknown magnitude in a known direction as


signified by the unit vector / 1 7 down the slot.

Question 3.16 If the direction of a turns out to be up the slot, will the
B

solution be valid?

Answer 3.16 Yes; this will be manifested by a turning out neeative. Note that (neg­
B

ative a ) •
B / l 7] is the same as (positive a ) •B
Page 181

We have two equations coefficients) in the three unknowns a ,ct , B 2

and but the angular speed may be found from the instantaneous center of
. From the geometry and similar triangles, we use Figure E3.18b to obtain:

(7) of (intersection of normals to two velocity vectors)


Figure E3.18b

Thus

Note that until of is located, we do not know whether v is up or down


B

the slot; however, the normal to v is the same line in either case. Once
B is
established, v to the right gives
G clockwise — and then we know that v is B

down the slot. Substituting, we get

Eliminating a gives a = —181, so that


2 B

The final example in this part of Section 3 . 6 will b e very helpful to us


later in situations such as t h e one s h o w n in Figure 3 . 2 3 . Suppose w e n e e d
Figure 3.23 to k n o w the location of point C of cylinder after h a s rolled on to
Page 182

t h e l o w e r position.* T h e p r o b l e m is to find t h e angle t u r n e d t h r o u g h b y


for a given rotation o f t h e line OQ. T h e procedure to b e followed in
solving this p r o b l e m is illustrated b y t h e n e x t e x a m p l e .

EXAMPLE 3 . 1 9

Find the relationship between the angle (locating the line OQ in Figure E3.19)
and the angle of rotation of the rolling cylinder
Solution
Treating C as a point whose path is a known circle, we get

Alternatively, we may also treat C as a point on the cylinder with instantaneous


center at

Figure E3.19

Thus, equating the two expressions for v , we have c

Integrating, and letting when , we get

or

F r o m E x a m p l e 3 . 1 9 , i f w e let R = 2r, t h e n w e see (Figure 3 . 2 4 ) that


. E v e n t h o u g h t h e circumferences o f cylinder a n d track are and
the curvature forces t h e angular velocities o f the line O C a n d
the cylinder to b e t h e s a m e . I f t h e outer track w e r e straight a n d o f length
, the cylinder w o u l d turn two revolutions in s p a c e instead o f just o n e
in traversing it.
It is seen that t h e line AC o f ( a n d h e n c e itself) revolves o n c e as C
completes its circular p a t h for the c a s e R = 2r. W h e n R > 2r, t h e n
S u c h a c a s e is s h o w n in Figure 3 . 2 5 . I f w e n o w let R = 7r, t h e n E q u a ­
tion ( 3 . 3 6 ) gives a n d n o w re v o l v e s o n c e in s p a c e for e a c h 6 0 ° of
turn o f line OC.

Figure 3.24

This need will arise, for example, in kinetics problems in which we seek the work
done by gravity on if its mass center is offset from its geometric center.
Which at first glance might lead one to believe that the cylinder would revolve twice
per revolution of OC.
Page 183

Start End (6 revs)

I rev 5 revs

\
1 revs 4 revs

3 revs
Figure 3.25

a n d the wheel would turn in space three times as fast a n d as far as line OC
(see Figure 3 . 2 6 ) .

End
(3 revs Start -
completed)

2 revs
1 rev
completed
completed

Figure 3.26

Gears

The final class of rolling problems is c o n c e r n e d with gears. Gears are used
to transmit power. The teeth of the gears are cut so that they will give
constant speed to the driven gear w h e n the driving gear is itself turning at
constant angular speed.
H o w e v e r , gears violate the rolling condition; there is necessarily
some sliding since the contacting points do not h a v e equal velocities
(except at ) , as c a n be seen in Figure 3 . 2 7 for spur gears; n o n e t h e ­
less, the teeth are cut so that w e m a y correctly treat the gears for dynamic
Page 184

,Pitch circles

Figure 3.27

purposes as if t h e y w e r e t w o cylinders rolling on e a c h other at t h e pitch


circles. Thus w h e n the centers are pinned as in Figure 3 . 2 7 , w e m a y use
the relation

where r , a n d r are the respective pitch radii of the gears


2 and . We
m a y also use the derivative of Equation 3 . 3 7 (since it is valid for all t):

W e note that the radius ratio is inversely proportional to the ratio of


angular speeds (and directly proportional to the ratio of n u m b e r s of gear
teeth, since the shape a n d spacing of teeth must m a t c h ) . W e n o w consider
several examples.

EXAMPLE 3 . 2 0

Find the angular speed of the front sprocket (rigidly fixed to the pedal crank) of
the bicycle in Figure E3.20a if the man is traveling at 10 mph. There are 26 teeth
on the front sprocket and 9 on the rear sprocket (which turns rigidly with the rear
wheel). The wheel diameters are 26 in.
Spokes Solution
not
The velocities of A and B, the two ends of the straight upper length of chain, are
shown
equal. (As shown in Figure E3.20b, A is just leaving the rear sprocket ; B is just
about to enter the front sprocket .) To prove this, we note that the translating
section AB of chain is behaving as if rigid, so that, calling this "body" ,

1 Speed
Chain But = 0, so that
clunker
Figure E 3 . 2 0 a
Page 1 8 5

Next we relate the equal velocities of A and B to the respective centers of their
sprockets and :

Now the velocities of C and C are each equal to the "velocity of the bike,"
t 2

meaning the common velocity of all the points on the translating part of the bike,
such as points of the frame and seat. Therefore v and v cancel, leaving
Figure E3.20b Ci C2

This says simply that (see Figure E3.20c):

Now if the speed of the bike is to be 10 mph, we have

Figure E3.20c

or

Thus

(Radii are proportional to number of teeth!)

So the rider must turn the pedal crank at rev/sec.

EXAMPLE 3.21

Frame is a fixed ring gear with internal teeth (not shown in Figure E3.21a) that
mesh with those of the planetary gear . The teeth of, also mesh with those of
the sun gear , which is pinned at its center point O to frame The crank arm
is pinned at its ends to O and to the center point P of . The arm has angular
speed counterclockwise. Find the angular velocity of in terms of R, r,
and
Figure E3.21a Solution
We take to be our reference frame, to which all motions are referred. We work
first with the crank , since we know its angular velocity and the velocity of one
of its points ( v = 0). From the sketch of
0 (Figure E3.21b), we see that we can
write

(Note that we align parallel to OP for convenience; we need not always draw it
to the right.)
Next, the points of and that are pinned together at P have the same
Figure E3.21b velocity at all times. Furthermore, the points of and at D (see Figure E3.21c)
Page 186

are in contact and each has zero velocity* since D is fixed in . Thus

or

Solving for gives

We are now in a position to obtain the velocity of point Q of , the point in


Figure E3.21 c
contact with the tooth of

Substituting for in terms of from Equation (2), we get

Note that Q has twice the speed of P since it is twice as far from the instantaneous
center D of as is P.
Finally, we come to the body of interest (see Figure E3.21d). Knowing that
Pitch circle the points Q and Q' (the respective tooth points in contact on and ) have
equal velocities as they move together tangent to the pitch circle, we obtain

Thus the angular speed of in is

Figure E3.21d
and the angular velocity of in is

N o t e in t h e previous e x a m p l e that t h e point o f passing o v e r point


Q h a s velocity , w h i c h is o f course less t h a n t h e velocity v o f t h e gear
Q

teeth in c o n t a c t at Q. T h i s velocity is , w h i c h is m o r e
t h a n twice as fast as
W e also r e m a r k that since t h e a n s w e r s

• As we have pointed out, the contact points of gear teeth necessarily slide relative to
each other. The points used in the analysis are actually not tooth points, however, but
imaginary points on the pitch circles of the gears. Furthermore, the radii given in the ex­
amples and problems are the radii of these circles.
Page 187

are c o m p l e t e l y general functions o f time, the angular accelerations are


obtainable i m m e d i a t e l y b y differentiation, w i t h :

R a t h e r t h a n differentiating, h o w e v e r , w e shall obtain t h e s e t w o results in


t h e following e x a m p l e in a n o t h e r m a n n e r : b y r e p e a t e d u s e o f Equation
( 3 . 1 9 ) . T h e purpose is to gain insight into its u s e in gearing situations
involving several b o d i e s . T h e procedure in t h e n e x t e x a m p l e w o u l d h a v e
to b e followed if t h e previous e x a m p l e h a d b e e n w o r k e d using instanta­
n e o u s values instead o f generally (with s y m b o l s ) .

EXAMPLE 3.22

Find the angular accelerations a , and a of the planetary and sun gears in the
3

previous example in terms of R, r, and and a , which are given functions of the
2

time t.
Solution
Relating the accelerations of P and 0 on body gives (see the bar in Figure
E3.22a):

This acceleration is then carried over to the coincident point P on (again see
Figure E3.22a). Relating D and P on the planetary gear , we have

Recalling that

the coefficients of then give

Figure E3.22a

and the coefficients yield

We now need a , where Q is again the tooth point of


Q in contact with the sun
gear :
Page 188

We now go to body to complete the solution. Relating the tooth point Q' to O
on gives the components of . (see Figure E3.22b). The tangential accelera­
tion components of Q and Q' are equal* as the teeth contact and move together:

Thus

Figure E3.22b
N o t e that w e m a y express the n o r m a l acceleration c o m p o n e n t o f Q '
in terms o f b y using t h e result for from E x a m p l e 3 . 2 1 :

W e also n o t e that w e could h a v e alternatively o b t a i n e d the accelera­


tions o f D a n d Q as points o n rims o f w h e e l s rolling o n curved tracks by
using Equations ( 3 . 3 1 ) a n d ( 3 . 3 2 ) . Noting that p for P is (R + r), w e
present these partial c h e c k s on our solution:

* This is in fact true even when neither body's center is fixed and the geometry is irregu­
lar. As long as there is rolling, the acceleration components of the contacting points in
the plane tangent to the two bodies are equal in plane motion at all times. See "Contact
Point Accelerations in Rolling Problems," D. J. McGill, Mechanics Research Communica­
tions, 7(3), 1 7 5 - 1 7 9 , 1980.
Page 189

PROBLEMS • Section 3.6

3.84 The wheel in Figure P3.84 rolls on the plane with 3.87 Figure P3.87(a) shows the manner in which a train
constant angular velocity rad/ sec. Find the velocity of wheel rests on the track. If the train travels at a constant
point Q by using the instantaneous center of zero speed of 80 mph and does not slip on the track, determine
velocity. Then check by using Equation (3.8) to relate v c
the velocities of points A, B, D, and £ on the vertical line
to v .
P
through the center C in Figure P3.87(b). Which point is
traveling backward? Why?

Figure P3.84
W

3.85 In the preceding problem, suppose that the plane


on which the wheel rolls is not fixed to the reference
frame but instead translates on it (this time the reference
frame is ) at constant velocity 3 f t / s e c to the left. (See
Figure P3.85.)
a. Find the instantaneous center of zero velocity
b. Find v Q again.

(b)
Figure P3.87

Figure P3.85 3.88 In the preceding problem find the velocities of


points F, G, and H.
3.89 Find the velocities of points B and C in Figure P3.89
3.86 The wheel in Figure P3.86 rolls on the bar. If at a if the cylinder does not slip on the translating bodies
certain instant the bar has a velocity of 2 m / s to the right and
and the wheel has counterclockwise angular velocity of
0.5 rad/s, determine the velocity of (a) the center of the
wheel and (b) point P.

Figure P3.86 Figure P3.89


Page 1 9 0

Figure P3.92

Figure P3.90

3.90 Two men, a tall one and a short one, travel up


identical inclines, pulling identical spools by means of
ropes wrapped around the hubs. (See Figure P3.90.) The Figure P3.93
men travel at the same constant speed , and the ropes
are wrapped in the opposite directions indicated. If the
spools do not slip on the plane, one of the men will be run eration (These values are v and a for all points in the
over by his own spool. Prove which one it is, and show body of the tank and for the centers of its wheels.) Find
how long it will take, from the instant depicted, for the the velocities of the five points P , P , P , P , and P if
1 2 3 4 5

spool to roll over him. there is no slipping. The wheels have radius R.
3.94 The wheel rolls on both and (See Figure
3.91 A cylinder of radius r rolls over a circular arc of
P3.94.) The constant angular velocity of the wheel in
constant radius of curvature R (see Figure P3.91). What is
frame is shown in the figure. Find:
the ratio of the angular speed of the cylinder to
a. The velocity of the points of relative to
b. The constant velocity of C in for which the
velocities of T (on ) and B (on ) in are
equal in magnitude and opposite in direction.

3.92 A disk with diameter 1.2 m rolls along the plane as


indicated in Figure P3.92. Its center point C has velocity

Figure P3.94
where t is the time in seconds. Find the velocity of the
point that lies 0.3 m directly below C when ( a ) f = 2 s and 3.95 An inextensible string is wrapped around the cylin­
(b) t = 5 s.
der in Figure P.3.95, fitting in a small slot near the rim.
3.93 The tank shown in Figure P3.9 3 is translating to the The center C is moving down the plane at a constant
right, and at a certain instant it has velocity and accel­ speed of 0.1 m / s . Find the velocities of points A, B, D, and
Page 1 9 1

E. Hint: The cylinder is not rolling on the plane, but it is 3.98 The cylinder in Figure P3.98 is rolling to the left
rolling on ?. with constant center speed . A stick is pinned to the
cylinder at B, and its other end A slides on the plane. Find
the velocity of A when 0 = 0 ° , 9 0 ° , 180% and 2 7 0 ° .

Figure P3.98

3.99 Wheel in Figure P3.99 has angular velocity 3


Figure P3.95
rad/sec. Find the angular velocities of wheel and the
bent bar

3.96 At the instant shown in Figure P3.9 6, point B of the


block (to which rod is pinned) has ft /sec.
Find the angular velocity of the rolling cylinder.

Figure P3.99

* 3.100 The cart in Figure P3.100 travels from left to


Figure P3.96 right, with its rear wheels rolling at constant angular
velocity 0.2 rad/sec. The front wheels are rolling up
the parabolic surface shown. The wheels have radius
0.4 m, and are pinned to the cart. Find the angular veloc­
3.97 Figure P3.97 shows a circular cam and an oscilla­ ity of cart at the given instant. Hint:
ting roller follower consisting of the roller (which rolls
on ) and the follower bar . If the cam rums at the
constant angular velocity 0.3 rad/s, find the angular
velocity of the follower bar and of the roller at the given
instant.

Figure P3.97 Figure P3.100


Page 1 9 2

3.101 The constant angular velocities of the ring gear 3.105 The shaded arcs on and (Figure P3.105(a)) are
and the spider arm shown in Figure P3.101 are 2 always equal if the two bodies are in rolling contact; how­
r a d / s and 10 rad/s, respectively. Determine the angu­ ever, the converse is not necessarily true. Just because the
lar velocity of gear and the velocity of the point of contacting arclengths are equal does not mean that rolls
having maximum speed in the given position. The centers on . For the wheel on the plane shown in Figure
of and are pinned to the reference frame P3.105(b), give constant values and for which the
arclengths of contact are equal but the velocities of the
contact points are not. Hint: Look at the shaded arcs on
and in Figure P3.105(b).

Figure P3.101

3.102 The rod which is pinned to cylinder , translates


upward in the y-direction at the constant speed 4 ft/sec
(see Figure P3.102). The rod is pinned to the reference Figure P3.105
frame at O and rests against the rim of as shown in the
figure. There is rolling contact between and . Find the
angular velocities of and at the instant shown. 3.106 If in Problem 3.99 the angular acceleration of is
at the given instant, find and at
3.103 If the given velocities of P and Q in Problem 3.89
that time.
are constant, find the accelerations of C and B.
3.107 In Problem 3.93 find the accelerations of the same
3.104 If, in Problem 3.89, the respective accelerations of P
five points P , P , P , P , and P . (See Figure P3.107.)
1 2 3 4 5

and Q are , find the angular ac­


celeration of the cylinder. 3.108 During startup of the two friction wheels (see Fig­
ure P3.108), the angular velocity of is rad/
sec. Assuming rolling contact, compute the acceleration
of the point T, which is at the top of when f = 3 sec.

Figure P3.102 Figure P3.107


Page 1 9 3

3.112 In Problem 3.88 find the accelerations of points F,


G, and H.
3.113 A moment applied to gear in Figure P3.113 re­
sults in a constant angular acceleration rad/
2
sec . The other gear, , is fixed in the reference frame.
Determine:

a. The time required for C to return to its starting


point after one revolution around from rest
b. The number of revolutions turned through in
Figure P3.108 space by during the complete revolution.

3.114 The center of the rolling wheel in Figure P3.114


moves to the right at a constant speed of 10 in./sec. The
3.109 The wheel in Figure P3.109 rolls, its center having a bar is pinned to the wheel at A, and end B always stays in
constant velocity of 10 ft/sec to the right. Find at contact with the ground. Find the acceleration of B at the
the instant shown. instant shown.
3.115 Calculate the acceleration of the instantaneous
center of rod in Example 3.15.

Figure P3.109

3.110 A wheel rolls on a 10-cm-radius track. (See Figure


P3.110.) At the instant shown the wheel has an angular
velocity of r a d / s and an angular acceleration of —
2
r a d / s . At the instant shown, find:

a. The velocity and acceleration of C


b. The velocity and acceleration of A
c. Figure P3.114
d. The center of curvature of the path of point T.
3.111 In Problem 3.87 find the accelerations of points A,
3.116 Gear and crank have angular speeds and
B, C, D, and E.
angular acceleration magnitudes at the instant shown
in Figure P3.116 in the indicated directions. Find the an­
gular velocities and angular accelerations of gears and
at the same time, if is pinned to , and

Figure P3.110 Figure P3.116


Page 1 9 4

• 3.117 The center C of the small cylinder in Figure 3.120 Point A of the slider block has, at the instant
2
P3.117 has a speed of 0 . l t m / s as it moves clockwise on shown in Figure P3.120, and a„
a circle. Body rolls on the large cylinder . In the = . Find the angular acceleration of bar
position given in the figure, t = 10 s. Find the accelera­
3.121 Two 5-in.-radius wheels roll on a plane surface.
tion of point B of the stick that is in contact with at the
(See Figure P3.121.)A13-in. bar is pinned to the wheels
given instant.
at A and B as shown. If C has a constant velocity of 20
ft/sec to the right, find, for the position shown, the accel­
eration of A.

Figure P3.121

Figure P3.117
3.122 See Figure P3.122. The velocity of the pin in block
is and its acceleration is in the
given position. Find at this instant the angular velocity
3.118 In Problem 3.89 let and a Q

and angular acceleration of the cylinder if there is suffi­


= at the instant shown. Again assuming no
cient friction to prevent it from slipping.
slipping, find the accelerations of B and C.
3.119 The two identical cylinders and (Fig­ 3.123 A wheel rolls along a curved surface. In the position
ure P3.119) are connected by bar (which is pinned to shown in Figure P3.123, its angular velocity and angular
their centers), and they roll on the surface as shown. If the acceleration are and . De­
angular velocity of is , find the termine at this instant the angular acceleration of bar
angular accelerations of both and at the given in­ and the acceleration of pin B of the slider block.
stant.

Figure P3.119

Figure P3.122

Figure P3.120 Figure P3.123


Page 1 9 5

3.124 In Example 3.14 find the piston acceleration when


, 180°, and 2 7 0 ° .
3.125 The center point C of gear in Figure P3.125
moves in a horizontal plane at constant speed . The ring
gear is fixed in the reference frame, and the constant
angular velocity of is clockwise. Find the acceleration
of point Q of

Figure P3.127

3.128 The angular velocity of crank in Figure P3.128 is


a constant 3 r a d / s . In the given position, find the ve­
locity of the center C of wheel and also determine the
angular acceleration of , which rolls on the circular
track.
Figure P3.125
3.129 Cylinders and in Figure P3.129 have a radius
of 10 in. each and roll on the respective planes. Bar has
length 48 in. and is pinned to the centers of the cylinders.
3.126 If the ball in a ball bearing assembly (Figure P3.126) The center G of has velocity . If
neither slips on the shaft nor on the fixed housing, find the time at the instant shown is t = 5 sec, find and at
the velocity and acceleration of the center of the ball in that instant.
terms of the angular velocity and angular acceleration of
the shaft

Figure P3.126

3.127 The ball in Figure P3.127 rolls on the fixed surface


Figure P3.128
and at the instant shown has angular velocity
r a d / s and angular acceleration . At this
instant find:
a. The velocities of A and B
b. The accelerations of A and B
c. Figure P3.129
Page 196

3.130 Bar in Figure P3.130 is 25 cm long and is pinned to • 3 . 1 3 4 Gears and in Figure P3.134 have 25 and 50
the rolling cylinder at B. The other end of is pinned to teeth, respectively. Rod is 2 ft long, and the radius of
the roller at A as shown. The center of the cylinder has is 1 ft. Determine the acceleration of point A when t = 0 if
2
= 11.2 c m / s and a = 16.8 c m / s down the plane at
c Bx = 0.2 sin ft (positive to the left) with = 90° at
the given instant; at this time line is vertical and t = 0.
is horizontal, and BC is parallel to the plane beneath it.
Find the acceleration of point A and the angular accelera­
tion of body at the given instant.

Figure P3.134

• 3.135 At the instant shown in Figure P3.135, bar has


2
rad/sec and r a d / s e c ; and the
2
gear has = rad/sec and = rad/sec .
Figure P3.130 At this instant, determine the accelerations of each of the
two gear tooth contacting points.
• 3.136 The outer gear in Figure P3.136 is stationary.
3.131 Figure P3.131 shows a 10-ft-radius disk that rolls
Crank turns at the constant angular velocity of 10 r a d /
on a plane surface. It has, at the instant shown, an angular
sec counterclockwise and is pinned at its ends to the
velocity of 2 rad/sec and an angular acceleration of 3
2
centers of the sun gear (at S) and the planetary gear
rad/sec , both counterclockwise. Find a point on the disk
(at P). Find the accelerations of the points of and that
or the disk extended that has zero acceleration at this
are in contact with each other, if the radii of and are,
instant.
respectively, 3 in. and 10 in.
3.132 Find the acceleration of point B, the pin connecting
• 3.137 Point O is pinned to the reference frame (See
rod to the blockin Figure P3.132, at t = 1 sec. Therodis
Figure P3.137.) The pitch radii of gears and are each
horizontal at f = 0, and the velocity of C is v = 2t c
2

0.2 m. The angular velocities of and are 2 rad/s,


ft/sec.
clockwise for and counterclockwise for , and both
constant. Find the maximum acceleration magnitude ex­
perienced by any point of

Figure P3.135
Figure P3.131

Figure P3.132

3.133 Referring to Problem 3.121, if at the instant shown


v = 20
c i n . / s e c and a = 5
c i n . / s e c , find a at this
2
A

time. Figure P3.136


Page 1 9 7

c. Observe that P is at its highest point when


and that there. In this configura­
tion, show that the following two expressions
for the acceleration of P agree:

• 3.140 Show that for a rigid body in the plane motion, as


long as there is a circle of points P of whose
accelerations pass through any point C of Hint: Write
and dot both sides with the
vector Assume that and see if you can
exhibit r . For the rolling wheel, show that the points are
CP
Figure P3.137
as shown in Figure P3.140.
• 3.141 Show that for the rolling uniform cylinder in Figure
P3.141 there is a point of zero acceleration at the indi­
cated position J if and are in the given directions and
are not both zero. You should find that the coordinates of
J are
Figure P3.138

• 3.138 The wheel in Figure P3.138 rolls on the plane. Find


the radius of curvature and the center of curvature of the
path of point T at the given time in terms of r.
• 3.139 A cycloid is the curve traced out by a point on the
rim of a rolling wheel. The equations for the rectangular
coordinates of a point on the cycloid, in terms of the
parameter (the angle shown in Figure P3.139), are
Figure P3.139

where a is the wheel's radius. Recall from calculus that


the curvature of a plane curve is Points accelerating
through C

where p is the radius of curvature. Figure P3.140

a. Use the chain rule

and show that, for the cycloid,

b. Explain what the minus sign in the expression


for means. Figure P3.141
Page 198

3.7 Relationship Between the Velocities of a Point with


Respect to T w o Different Frames of Reference

W h i l e Equation ( 3 . 8 ) gives us the relationship b e t w e e n the velocities in


of t w o points o f the same rigid b o d y , w e often n e e d a n o t h e r equation
relating the velocities o f the same point relative to t w o different frames or
bodies. This relationship ( t o g e t h e r w i t h a c o m p a n i o n equation for accel­
erations to b e developed in the n e x t section) will b e essential in solving
kinematics p r o b l e m s involving bodies m o v i n g in special w a y s relative to
others (such as a pin o f o n e b o d y sliding in a slot o f another).

Relationship Between the Derivatives of a Vector in T w o Frames

T o develop this equation, w e m u s t first find the relationship b e t w e e n the


derivatives o f an arbitrary vector A in t w o frames a n d T o do this, w e
begin b y e m b e d d i n g axes X a n d Y in a n d x a n d y in B, as suggested b y
the h a t c h m a r k s in Figure 3 . 2 8 . Further, w e let and b e pairs of
unit vectors, always respectively parallel to (X, Y ) a n d to (x, y).
W e n o t e that if again locates the x axis relative to X as s h o w n ,
then

and

Figure 3.28 Differentiating in frame a n d noting that and are constants there, w e
obtain

N o w let the arbitrary vector A b e written in frame (meaning that A


is expressed in terms o f its c o m p o n e n t s t h e r e — i . e . , in terms o f unit
vectors fixed in direction in ):

Differentiating this vector in w e get

W e n o w n o t e that the first t w o terms o n the right side o f Equation ( 3 . 4 3 )


add u p to the derivative o f vector A in b e c a u s e a n d do n o t c h a n g e in
magnitude or direction w i t h time there. T h u s

Substituting the derivatives o f and in from Equations ( 3 . 4 1 ) and


( 3 . 4 2 ) yields
Page 1 9 9

or

(3.44)

T h e angular velocity h a s reappeared, a n d Equation ( 3 . 4 4 ) s h o w s


us that this vector h a s a m o r e general purpose t h a n m e r e l y relating
velocities in kinematics. It is in fact the link that allows us to relate the
derivatives of a n y vector in two different frames. (This s a m e result is in
fact true in general three-dimensional motion, w i t h three-dimensional
vectors a n d a m o r e general expression for angular velocity substituted, as
will b e seen in C h a p t e r 6.)

Velocity Relationship in T w o Frames

W e n o w use Equation ( 3 . 4 4 ) to relate the velocities o f a point P in t w o


frames a n d F r o m Figure 3 . 2 9 , the position vectors of P in t h e s e two
frames are related b y

(3.45)

Differentiating this equation in we have

Figure 3.29 (3.46)

T h e first t w o vectors in Equation ( 3 . 4 6 ) are the velocities o f P a n d in


b y definition:

(3.47)

Question 3.17 (a) Why is the last vector in Equation (3.47) not the
velocity of P in (b) Why is it not the velocity of P in

T o replace b y a vector that w e can operate with, w e use E q u a ­


tion ( 3 . 4 4 ) , with b e c o m i n g the vector A :

(3.48)

Therefore, recognizing that is a n d substituting Equation ( 3 . 4 8 )


into ( 3 . 4 7 ) , w e obtain

(3.49)

A n o t h e r w a y o f expressing Equation ( 3 . 4 9 ) is to think o f frame as a


" m o v i n g f r a m e " with respect to a " f i x e d " reference frame T h e n the
velocities of P can b e written as simply v w h e n the reference is (thus
P

and as v w h e n the reference is the " m o v i n g f r a m e "


r e l (thus

Answer 3.17 (a) It is not because the origin of the position vector is not fixed in
(b) And it is not because the derivative is not taken in
Page 200

H e n c e w e c a n write E q u a t i o n ( 3 . 4 9 ) in abbreviated notation


as

(3.50)

where the position of P in t h e m o v i n g frame, a n d is the


angular velocity o f relative to frame T h e reader m a y find this form of
Equation ( 3 . 4 9 ) easier to use w h e n there is just o n e " m o v i n g f r a m e . "
N o w let us d e n o t e b y the point o f (or e x t e n d e d ) that is coin­
cident with P. T h e n a n d t h e last two terms o f Equation ( 3 . 4 9 )
or ( 3 . 5 0 ) are s e e n ( b y E q u a t i o n 3 . 8 ) to b e the velocity of in

(3.51)

In words, Equation ( 3 . 5 1 ) says that at a n y time, t h e velocity o f P in is


the s u m o f the velocity o f P in plus the velocity in o f the point o f
coincident with P.*
As a preliminary e x a m p l e , consider Figure 3 . 3 0 , in w h i c h pin Q is
m o v i n g to the right. Let P b e the center o f the o t h e r pin, w h i c h is attached
to frame T h e n w e m a y write

N o w the center o f the m o t i o n in is necessarily along a straight line


within the slot o f T h e r e f o r e the velocity o f (the point o f extended
coincident with P ) is seen to be also parallel to the slot a n d in a direction
Figure 3.30 opposite to that o f . W e n o w consider several detailed e x a m ­
ples of t h e use o f Equation ( 3 . 4 9 ) .

EXAMPLE 3 . 2 3

A yellowjacket walks radially outward at a constant 2 i n . / s e c in a straight line


relative to a record turning at (See Figure E3.23.) Find the velocity of the
yellowjacket in frame which is the cabinet on which the stereo rests.
Solution
We shall treat the yellowjacket as a point Y, and we note that the unit vectors in
Figure E3.23 are fixed in our "moving frame" Also, the "moving" and
"fixed" (O) origins are coincident. Then we have, using Equation (3.49):

Note how the second term grows linearly with the radius.

Figure E3.23 * This latter term is sometimes called the vehicle velocity of P.
Page 2 0 1

In t h e n e x t e x a m p l e , there is m o r e t h a n o n e " m o v i n g f r a m e . " T o


avoid three levels o f subscripts, w e shall n a m e t h e bodies and
rather t h a n the usual Thus becomes a n d so on.

EXAMPLE 3 . 2 4

Collar in Figure E3.24 is pinned to rod at P and is free to slide along rod
The angular velocity of is 0.2 r a d / s at the instant shown. Find the angular
velocity of at this time, and determine the velocity of P relative to
Solution
We relate the velocities of P in and in

Figure E3.24

Thus the pin is moving outward on which is turning clockwise.

Question 3.18 Will always have the same as does

N o t e that in t h e preceding e x a m p l e , w e did n o t n e e d to k n o w the


angular velocity o f N o n e t h e l e s s , it is important for t h e reader to realize
that . T h e reason is that a n d can o n l y translate relative to each
other; t h u s lines fixed in e a c h will turn at the s a m e time rates in This
observation will s o m e t i m e s be n e e d e d (as in P r o b l e m s 3 . 1 4 7 a n d 3 . 1 4 9 ) .

EXAMPLE 3 . 2 5

Block translates in a horizontal slot (see Figure E3.25a) and is pushed along by a
bar that turns at angular velocity rad/sec about the pin at point O.
Find the velocity of the contact point of when =60°.

Figure E3.25a Answer 3.18 Definitely not! This happened because of the geometry at the given instant.
Page 202

Solution
Let the ground be the reference frame and note that T is the point of
coincident with Q at the given instant (the point we have been calling in the
theory). Using Equation (3.51), we obtain

Therefore

(1)

Note that the velocity of Q relative to bar has an unknown magnitude


but a known direction (along ). Now we also know the direction of
so that

(2)

Equating the x components of Equation (2), we get

(3)

And equating the y components:

(4)

Equation (4) gives = 13.3 ft/sec, from which Equation (3) then yields

The correct triangle relating the velocities of Q and T is shown in Figure E3.25b.
Figure E3.25b As a check, and
ft/sec.

Question 3 . 1 9 Can the preceding example also be worked by using

Answer 3.19 Yes, provided we recognize that the direction of is along the axis of
the rod (see Problem 3.146). That is, must be tangent to the surface at which and
touch. It is important to realize, however, that while the path of Q in is a straight
line, the path of T in is not. Thus would not be in the direction of the axis of

EXAMPLE 3 . 2 6

Disk of Figure E3.26a, with its attached pin P, has limited angular motion. After
a 45° clockwise rotation from the original position (see Figure E3.26b), disk has
angular velocity rad/sec. At this time, find of the slotted triangular
Figure E3.26a body and determine the velocity of pin P relative to
Page 2 0 3

Figure E3.26b

Solution
Calling the ground frame we relate the velocities of pin P in and in

(1)
where is the point of coincident with P.
We may find by relating it on body to (which vanishes):

0
To find we need the orientation of the slot. At 4 5 , the configuration
is as shown in Figure E3.26c. Note that the slot center S moves on a circle about 0
and that a tangent to this circle at S must pass through P at all times, since P must
stay within the slot. From the diagram at the left we get, from geometry and
trigonometry,

Figure E3.26c

Using to form in Equation (1), we have:

in which we have used the fact that we know the direction but not the magnitude
of (It moves in the slot at the angle calculated earlier.) Further, relating the
velocities of points and O on we have
Page 204

or

Substituting, and equating the coefficients of and of we get

In w o r k i n g out the following p r o b l e m s , the student is urged to begin


Solving
by these carefully
thinking equations about
gives the =selection
1.61, so that:
o f a point w h o s e velocities in
t w o frames are to b e related with E q u a t i o n ( 3 . 4 9 ) .

PROBLEMS Section 3.7

3.142 Boat in Figure P3.142 departs from A and is sup­ 3.144 Cylinder in Figure P3.144 rolls on a circular sur­
posed to arrive at point B some 100 ft downstream and on face. When it is at the lowest point of the circle, its angular
the other side of a river with a current of 5 ft/sec. If can velocity and acceleration are =0.2 r a d / s and
2
move at 10 ft/sec relative to the water, and if it travels on = 0.02 r a d / s . Rod is pinned to at and is also
a straight line from A toward B, how long will it take? pinned to a block at P that slides in the slot of The
constant angular velocity of is 0.3 rad/s. Find the
velocity of P in and the angular velocity of at the given
instant.

Figure P3.142

3.143 Bar in Figure P3.143 is turning clockwise with


angular speed 0.25 rad/s, pushing bar as it goes. Find
at the given instant.

Figure P3.144

3.145 Rod is pinned to the ceiling at A and slides on


wedge at B. (See Figure P3.145.) The wedge moves to
the right with constant velocity of 5 ft/sec. Find the an­
gular velocity of the rod in the position shown.

3.146 Referring to Example 3.25 for the meanings of the


Figure P3.143 symbols, show that
Page 205

3.149 Rod in Figure P3.149 has angular velocity 5


rad / s. It is pinned to another rod which passes through
a slot in as shown. At the given instant, find the angular
velocity of body and the velocity of any point of
relative to Hint: All points of translate in —what
does this mean about the angular velocities of and ?
3.150 A mechanism consists of crank pinned to
rocker pinned to , and a small body that is pinned to
and slides in the slot of (See Figure P3.150.) The
length of is , where D is the distance between O
and . If has constant angular velocity over a
Figure P3.145
range of its motion, find when: (a)

3.147 Bar slides through a collar in body (see Figure 3.151 Collars and in Figure P3.151 are pinned to­
P3.147) and is pinned at P to a second bar Both and gether at C, and they slide on rods and respectively.
are pinned to the reference frame as shown, and rotates Rod has a constant angular velocity for
with limited motion at constant angular velocity 45°. Find the velocity of point C relative to as a
= 1 r a d / s counterclockwise. Find the angular velocity of function of D, , and in this range of angles.
when point P is at the top of the circle on which it
travels.

3.148 Plank slides on the floor at A and on block at


Block moves to the right with a constant velocity of
6 ft/sec while end A moves to the left with a constant
velocity of 4 ft/sec. For the position shown in Figure
P3.148, find the angular velocity of the plank.

Figure P3.149

Figure P3.150
Figure P3.147

Figure P3.148 Figure P3.151


Page 206

3.152 Figure P3.152 shows a circular cam and a flat- 3.155 Rods and in Figure P3.155 are pinned at O and
face translating follower If rotates with constant to a reference frame Rod is also pinned to the
angular velocity find the maximum velocity in ref­ slotted body at B. The upper end of is pinned at P to a
erence frame of any point of , in terms of and the roller that moves freely in the slot of The angular veloc­
offset distance ities of rod and link are constants:

Determine the velocity of P in and the angular velocity


of at the given instant.

Figure P3.152

3.153 Figure P3.153 illustrates a "Geneva mechanism,"


in which disk is driven with a constant counterclock­ Figure P3.155
wise angular speed and produces an intermittent (starting
and stopping, waiting, then repeating) rotational motion
of the slotted disk Pin P is fixed to and drives disk * 3.156 In Figure P3.156 collar isfixedtoarm andslides
by pressing on the surfaces of the slots. Show with the use along rod Arm is pinned to a second collar at A;
of Equation (3.49) that disk will have zero angular speed this collar slides on rod At the given instant, =0.2
in the two positions shown, a varying angular speed in rad/s, = 0 . 1 rad/s, and the velocity of all points of
between these positions, and zero angular speed while P relative to rod is 0.3 m / s outward (along OC). Give
is returning to the = 135 ° position. Note that the oper­ the angular velocity of by inspection, and find the ve­
ation of the mechanism requires that the distance be­ locity of the points of relative to
tween O and be

Figure P3.156
Figure P3.153

3.157 Show that the rigid-body velocity equation (3.8)


• 3 . 1 5 4 In the preceding problem let R = 0.1 m and can be derived from Equation (3.49). Hint: Fix P to and
= 5 r a d / s = constant. Find the angular velocity of the the equation will relate the velocities in of the two points
slotted body at the instant when =160°. P and of
Page 2 0 7

3.8 Relationship Between the Accelerations of a Point with


Respect to T w o Different Frames of Reference
W e shall n o w derive the c o m p a n i o n equation to ( 3 . 4 9 ) , this o n e relating
the accelerations o f P in and Differentiating Equation ( 3 . 4 9 ) in
w e get
(3.52)

Using Equation ( 3 . 4 4 ) to " m o v e the derivative" in the first a n d last terms


o n the right side o f Equation ( 3 . 5 2 ) gives

(3.53)

Recognizing that a n d that a n d rearranging


terms, from Equation ( 3 . 5 3 ) w e obtain

(3.54)

in w h i c h as w e h a v e already seen in Section


3.5.
T h e parenthesized term in Equation ( 3 . 5 4 ) is seen from Equation
( 3 . 1 9 ) to b e w h e r e , as before, is the point o f (or extended) that
is coincident with P. T h e r e f o r e w e h a v e the following for our result:

(3.55)

In words: T h e acceleration o f P in equals its acceleration in plus the


acceleration in o f the point o f coincident with P, plus the C o r i o l i s
acceleration, T h e Coriolis acceleration is seen to provide an
u n e x p e c t e d b u t essential difference b e t w e e n the forms o f Equations
(3.51) and (3.55).

Question 3.20 If we differentiate Equation (3.51) instead of (3.49), we


might (erroneously!) obtain

and we come up (incorrectly) short by half on the Coriolis term. What is


wrong with this approach?

In the s a m e procedure w e u s e d for velocities, w e can simplify the


notation o f Equation ( 3 . 5 4 ) if there is b u t o n e " m o v i n g f r a m e " in­
volved, w h i c h is in m o t i o n relative to the reference frame

(3.56)

Answer 3.20 The error is that the derivative is not equal to because de­
notes a succession of points of which are at each instant coincident with P.
Page 2 0 8

In this equation, a a n dP are t h e respective accelerations o f P in a n d in


T h e vectors and are the angular velocity a n d angular acceleration
of in (equal to and )and (the position vector o f P in t h e
" m o v i n g f r a m e " ) . W e n o w consider several e x a m p l e s s h o w i n g t h e use o f
Equations ( 3 . 5 4 ) a n d ( 3 . 5 5 ) .

EXAMPLE 3.27

Find the acceleration in frame of the yellowjacket of Example 3.23.


Solution
Using Equation 3.54, we obtain:

The in this example is the Coriolis acceleration. Note that the yellow­
jacket has two nonzero acceleration components, even though both and are
zero in this example.

EXAMPLE 3 . 2 8

If in Example 3.24 we have the additional data that at the


given time, find and (See Figure E3.28.)
Solution
Relating the accelerations of P in and we have

in which the answers to Example 3.24 are used in the Coriolis term. Filling in the
variables, we get

Figure E3.28
Page 2 0 9

Thus point P is slowing down as it moves outward along and is slowing


down as it rotates clockwise.

T h e reader should note in the preceding e x a m p l e that and are


identical. T h e sleeve forces the two bodies a n d to translate relative to
e a c h other, so that, as w a s pointed out at the e n d of E x a m p l e 3 . 2 3 ,
By differentiating this equation, w e see that the angular accel­
erations are also always equal.

EXAMPLE 3 . 2 9

In Example 3.25, suppose that at the given instant ( = 6 0 ° ) all the data are the
2
same and in addition = 3 0 rad/sec . Find the acceleration of block (see
Figure E3.29).
Solution
Note that if we again use Q as our point (of the block ) that is moving relative to
two bodies ( and ), we know the direction of (It is along since point Q
moves on a straight line in )
Equation (3.55) thus gives

Figure E3.29 Noting that point is the point we obtain

This result yields the following scalar component equations:

Thus the acceleration of the translating block is:


Page 2 1 0

EXAMPLE 3 . 3 0
2
If in Example 3.26 we add = 10 r a d / s e c to the data, find the angular
acceleration of and the acceleration of P relative to (See Figures E3.30a,b.)

Figure E3.30a

Figure E3.30b

Solution
Again we apply Equation (3.55):

and again we equate the coefficients, and then the coefficients, to obtain the
scalar component equations:

2
Solving these equations results in = 5 . 1 2 r a d / s e c , so that:
Page 2 1 1

EXAMPLE 3 . 3 1

Pin P in Figure E.3.31 is attached to cart and slides in the smooth slot cut in
wheel The wheel rolls on the rough plane The cart's position is given by
2
= 0.3t , with in meters when t is in seconds. Find and at the given
instant (which is at t = 3 s), and determine the acceleration of P in the slot at this
time.

Figure E3.31

Solution
2
Since = 0.3t , we have and these are the velocity and
acceleration vectors of all points of the cart in particular of P. At t = 3, we have
and

Relating the velocities of P in frames and we obtain

Relating the accelerations using Equation (3.54), we get

Note that we have related (the point of -extended coincident with P) and the
center of (call it C) to get

Solving, we obtain
Page 2 1 2

Thus the acceleration of P in the slot (which is its acceleration in ) is


2
1.8 m / s ; it is upward because we assumed it to be in the positive y direction
and got a negative answer. Note that although P is momentarily stopped
in the slot ( = 0 ) , it has to be "getting ready" to move outward since is
translating to the right and is rolling that way. This is what is indicated by
2
= 1.8 m / s .

PROBLEMS Section 3.8

3.158 A bug is crawling outward at a uniform speed 3.160 Bar in Figure P3.160 has angular velocity 0.25
relative to the rotating arm of 3 f t / sec. In the position r a d / s and angular acceleration 0.15 2
r a d / s at the
shown in Figure P3.158, for the arm = 2 rad/sec and given instant. Find the angular acceleration of at this
2
= 4 rad/sec , both counterclockwise. What is the ac­ time. (See Problem 3.143.)
celeration of the bug? Indicate the direction in a sketch.

figure P3.1S8

Figure P3.160

3.159 The mechanism shown in Figure P3.159 is used to 3.161 In Problem 3.144 find the acceleration of P in and
raise and lower hammer The 26-cm crank turns the angular acceleration of
clockwise at the constant rate of 30 rpm. It is pinned to 3.162 In Problem 3.155 find, at the same instant of time,
block which slides in a slot in If at t = 0 point A is the acceleration of P in and the angular acceleration of
directly above find the velocity and acceleration of
as a function of time. (The block and hammer are slightly 3.163 In Figure P3.163,
offset from so they do not interfere with the pin at O.)
and

If the bar stays in contact with both the step and circular

Figure P3.159 Figure P3.163


Page 2 1 3

trough, find its angular acceleration. Hint: Treat point Q • 3.167 Extending Problem 3.149, suppose that the angular
(fixed to the step) as the "moving point," and note that Q velocity of rod 5 rad/s, is constant in time. (See
moves on a straight line relative to the bar. Figure P3.167.) Find, at the given instant, the angular
acceleration of and the acceleration of any point of
3.164 In Problem 3.151 determine the acceleration of C
relative to
relative to as a function of D, and
3.165 Rods and (see Figure P3.165) pass smoothly
through the short collars, which can turn relative to each
other by virtue of the ball-and-socket connection. If bars
and turn with constant angular velocities 0.4 rad/
sec and 0.2 rad/sec, respectively, find the velocity and
acceleration of the ball-and-socket connection with re­
spect to and to in the indicated position.

Figure P3.167

3.168 Referring to Example 3.29 for the meanings of the


symbols, show that
* 3.169 A circular turntable (see Figure P3.169) rotates
about a vertical axis through O (normal to the plane of the
paper) with changing at a constant rate A block
rests in a groove cut in the turntable. If the cable is reeled
in at a constant velocity relative to find expressions
for the radial and transverse components of the block's
acceleration. Check your answers using the expression
Figure P3.165 for acceleration in cylindrical coordinates.

* 3.166 Wheel in Figure P3.166 has a constant clockwise


angular velocity of 2 rad/sec. It is connected by link to
block End B of rod slides in a vertical slot in block
For the position shown, find the angular velocity and
angular acceleration of rod if block translates.

Figure P3.169

3.170 Show that the rigid-body acceleration equation


(3.19) can be derived from Equation (3.54). Hint: If you fix
P to the equation will then relate the accelerations in
of the two points P and of
• 3.171 In Problem 3.153 let R = 0.1 m and = 5 rad/s
as in Problem 3.154. This time, again at = 160°, find: (a)
the angular acceleration of (b) the acceleration of P in
Figure P3.166
Page 2 1 4

• 3.172 The 26-ft rod in Figure P3.172 slides on a plane


surface at A and on the fixed half-cylinder at Q as shown
below. If end A is moved at a constant velocity of 4 ft/sec
to the right along the plane, determine the vertical com­
ponent of the acceleration of B for the position shown.
• 3.173 Pin P in Figure P3.173 moves along a curved path
and is controlled by the motions of the slotted links and
At the instant shown, each point of has a velocity of Figure P3.173
2
5 ft/sec and an acceleration of 20 f t / s e c , both to the
right, while each point of has a velocity of 3 f t / s e c and
2
an acceleration of 30 f t / s e c in the direction shown in the
figure. Find the radius of curvature of the path of P in this (on the rod extended)
position.
• 3.174 Considering the instantaneous center of the bar
in Figure P3.174 as a point of extended, find the
acceleration of _ at the instant shown, if v = in./
A

(on the rod extended)


sec = constant. Also find the acceleration of the point
of passing over the pin, and note that it is not along
the slot.

Figure P3.174

Figure P3.172

COMPUTER PROBLEMS Chapter 3

• Crank in Figure P3.175 is driven at a constant


angular speed clockwise. Show that the speed of the
piston is maximum when satisfies the equation

Figure P3.175
Solve with a computer for the first root of this equation
when the lengths of and are 8 cm and 20 cm, respec­
tively. Note that the answer is independent of the value of
You may wish to read Appendix B and make use of the
Newton-Raphson method described there.
• Crank in Figure P3.176 rotates at constant angu­
lar velocity . Use a computer to generate data for a plot
of the following two quantities as functions of for the
case in which D = 21:

a. The angle that locates slider


b. The ratio of the angular speeds of and Figure P3.176
Page 2 1 5

SUMMARY Chapter 3
This chapter h a s been devoted to presentation of t h e velocity a n d accel­
eration relationships that pertain to a rigid b o d y in plane motion.
If A a n d B are t w o points of the b o d y lying in the s a m e plane of
motion, then their velocities are linked through the angular velocity
( is perpendicular to the plane of motion) by

If a point instantaneously has zero velocity, call it , then:

w h i c h s h o w s that the speed of a n y point is the product of the angular


speed a n d the distance between the point a n d the instantaneous center.
For accelerations, w h e r e is the angular acceleration, w e h a v e

W h e n o n e b o d y rolls on another, the points of contact h a v e the s a m e


velocity. Rolling of a wheel on a fixed surface m e a n s that the point on the
wheel in contact with the surface is the instantaneous center of velocity,
a n d with C denoting the center of the wheel,

and
+ (a part n o r m a l to the p a t h of C )

Finally, w e h a v e investigated the relationships between velocities


a n d then accelerations of a single point P relative to t w o reference frames.
The results c a n be expressed c o m p a c t l y if w e think of the underlying
basic frame of reference as "fixed" a n d a second body as moving. Then
with

v P = velocity of P relative to the fixed frame,


v = velocity of P relative to the m o v i n g body,
rel

= angular velocity of the moving body,

a n d similarly for accelerations a n d with being a point fixed in the


moving body, w e have:

and

The last term in t h e s e c o n d equation is called t h e Coriolis acceleration.


Page 2 1 6

REVIEW QUESTIONS Chapter 3


True or False?

In these questions, P a n d Q are points in the reference plane of a rigid


body in plane motion.
1. For a rigid body in plane motion with angular velocity depends
on a certain choice of points w h o s e velocities are to be related.
2. For a rigid body in plane motion, there is always an instantaneous
center of zero velocity located a finite distance from the body.
3. a is not necessarily zero.
4. if the angular velocity of is not zero.
5. For a rigid b o d y in rectilinear translation, the velocities of all points
of are equal, a n d so are the accelerations.
6. For a rigid body in curvilinear translation, the velocities of all points
of are equal, but the accelerations are not.
7. A t a given instant, a n y point can be considered to lie on a rigid
extension of any rigid body.
8. T h e smallest n u m b e r of scalar parameters required to locate a rigid
body in plane motion is four.
9. F o r a rigid body in plane motion,

10. A point P can h a v e an angular velocity.


11. T h e c o m p o n e n t s of v a n d v
P Q along the line P Q are not always equal.
12. T h e c o m p o n e n t s of a a n d a along the line PQ are not always equal.
p Q

Answers: 1.F 2 . F 3 . T 4. T 5. T 6. F 7. T 8. F 9 . T 10. F 11. F 12. T


4 KINETICS OF A RIGID BODY
IN PLANE MOTION/
DEVELOPMENT AND SOLUTION
OF THE DIFFERENTIAL
EQUATIONS GOVERNING
THE MOTION
4.1 Introduction
4.2 Rigid Bodies in Translation
4.3 Moment of M o m e n t u m (Angular Momentum)
Inertia Properties
4.4 Moments and Products of I n e r t i a / T h e Parallel-Axis T h e o r e m s
Examples of Moments of Inertia
The Parallel-Axis Theorem for Moments of Inertia
The Radius of Gyration
Products of Inertia
Transfer Theorem for Products of Inertia
4.5 T h e Mass-Center Form of the Moment Equation of Motion
Development of the Equations of Plane Motion
Helpful Steps to Follow in Generating and Solving the Equations
of Motion
4.6 Other Useful Forms of the Moment Equation
Moment Equation in Terms of a c

Moment Equation in Terms of a p

Moment Equation for Fixed-Axis Rotation (The "Pivot" Equation)


Rotation of Unbalanced Bodies
SUMMARY
REVIEW Q U E S T I O N S

Page 217
Page 218

4.1 Introduction

In this chapter w e apply Euler's laws to the plane motions of rigid bodies.
Motion of the mass center of any body, rigid or not, is governed by Euler's
first law as discussed in Chapter 2. The rotational motion of a rigid body is
governed by Euler's second law. We saw in Chapter 2 that this law can be
expressed in terms of a moment of momentum for any body, rigid or not.
However, the moment of momentum for a rigid body can be expressed
in a particularly compact way that involves moments and products of in­
ertia of the body and its angular velocity; because of this, the term an­
gular momentum is used synonymously with moment of momentum.
There is, however, one type of plane motion of a rigid body ßthat can
be immediately studied, prior to the introduction of angular momentum.
This class of motions, called translation, is characterized by the angular
velocity of B being always zero. The translation problems treated in the
next section (4.2) differ from the particle/mass-center motion problems
of Chapter 2 in that a moment equation is required for their solution in
addition to the mass-center equation = ma .
c

In Section 4.3, w h e n w e develop the expression for the angular


momentum of a rigid body, the moments and products of inertia sud­
denly appear — in the same way the mass center did, back in Chapter 2.
Thus w e spend some time in Section 4.4 studying these inertia properties
before moving on.
In Sections 4.5 and 4.6 w e deduce several especially useful forms of
Euler's second law in terms of the inertia properties. After each form of
the equation, w e illustrate its use with a set of examples. Some of these
examples might be termed "snapshot" problems; in these w e investigate
the relationships between external forces on a body and its accelerations
at a single instant. These problems are rather natural extensions of those
the student has encountered in statics — that is, w e know the geometri­
cal configuration and seek information about forces on the body. Other
problems might be called "movie" problems; in one class of these the
geometry is of sufficient simplicity that Euler's laws produce differential
equations which w e can readily integrate so as to predict the motion of
the body during some interval of time.
In Section 4.7, w e take up the very special problem of rotation of an
unbalanced body about a fixed axis. Here w e establish the criteria for the
technologically important problem of balancing.
In the next chapter, w e will continue our study of plane-motion
kinetics of rigid bodies by investigating the use of three special solutions
(which can be obtained in general) to the differential equations of mo­
tion. These special integrals are known as the principles of work and
kinetic energy; linear impulse and momentum; and angular impulse and
angular momentum.
Finally, w e mention to the reader that it is possible to obtain all the
results of this chapter on plane motion of rigid bodies from the general
three-dimensional results developed in Chapter 7. It is not necessary to
Page 219

travel this complex route in order to l e a m p l a n e motion, however, a n d in


this chapter w e take a simpler p a t h . It is w o r t h noting that while the
planar case covers a restricted class of motions, it does in fact contain a
large n u m b e r of problems with important engineering applications.

4.2 Rigid Bodies in Translation


In Chapter 2 we presented the equation

(2.34)
w h i c h is valid b o t h for any b o d y a n d any point P. In particular, if the
b o d y is translating, t h e n b y definition all its points h a v e the same accel­
eration — including its m a s s center — a n d if w e label t h a t c o m m o n ac­
celeration a, t h e n it m a y be factored, leaving:

(using t h e definition
of t h e mass center)
(4.1a)

a n d therefore

(4.1b)

In this section, w e shall apply this simple equation to several examples of


translating rigid bodies in plane motion. The reader should note, h o w ­
ever, that t h e t w o forms of Equation (4.1) apply w h e t h e r the motion is
plane or not. It also d o e s n ' t even require the b o d y to be "physically
rigid," although if it is translating, it is necessarily b e h a v i n g like a rigid
b o d y during that motion.

Question 4.1 If point P is arbitrary in Equation (4.1a), where does the


inertial frame come into the equation?

Before w a d i n g into t h e examples, w e w i s h to n o t e w h a t is n e w here.


In Chapter 2, we were concerned with the mass center motions of bodies,
w h e t h e r they were rigid or not. In those sections there w a s n o n e e d to
s u m m o m e n t s , a n d that is w h a t will distinguish this chapter from those
preceding sections. N o w t h e simplest problems b y far in w h i c h a m o m e n t
equation is sometimes n e e d e d are those involving translation. Transla­
tion is simple because it can b e studied prior to the introduction of
angular m o m e n t u m forms for rigid bodies with their accompanying in­
ertia properties a n d angular velocities.
We n o w examine three examples of translation, a n d in each, the
reader is urged to note t w o things: (1) h o w the problem could not be

Answer 4.1 T h e acceleration a is t h e s e c o n d derivative, t a k e n in a n inertial frame, of t h e


position vector from an ortgin in t h a t frame to a n y p o i n t of t h e b o d y .
Page 220

solved without the use of Equation (4.1); and (2) h o w for translation, the
moments generally do not sum to zero. They do sum to zero at the mass
center, and also at points lying on the line of a drawn through C (for then
r is parallel to a)*, but not otherwise. (Equations for translation at con­
pc

stant velocity are trivial and identical to the equilibrium equations; these
were studied in statics and are not considered in this book.)

EXAMPLE 4 . 1
m, L
Find the angle 6 for which the bar in Figure E4.1a will translate to the right at the
given constant acceleration "a." Then find the force P required to produce this
motion.
Solution
P 0 We sum moments at the contact point A, using the FBD in Figure E4.1b and
Equation (4.1):
Figure E4.1a

Therefore 0 in terms of a is given by:

mg
P o
µN To determine P, we write the mass-center equations:
A
N (1)

Figure E4.1b
and

Substituting this value of N into Equation (1) gives

P —µmg= ma
or
P =m(µg+ a)

Note that part of P balances the friction and the rest, the unbalanced force in the
x-direction, produces the "ma."

a n d of course at times w h e n t h e e q u a l accelerations of all p o i n t s , a, are zero.


Page 221

1ft 2 ft
EXAMPLE 4 . 2

A 300-lb cabinet is to be transported on a truck as shown in Figure E4.2a.


4 ft
Assuming sufficient friction so that the cabinet will not slide on the truck, for
forward acceleration find
4 ft a. the forces exerted by the truck bed on the cabinet for 5 ft/sec accel­
2

eration;
b. the maximum acceleration for which the cabinet will not tip over.
Solution
Figure E4.2a a. The acceleration is so using the free body diagram (Fig­
ure E4.2b) and putting = ma into component form,

1 ft

To locate the line of action of N, that is, to find the distance d, we


4 ft 300 lb need the moment equation:

4F - (1 - d)N = 0
4(46.6) - (1 - d)(300) = 0
Figure E4.2b
1 - d = 0.621
d = 0.379 ft
We might note at this point that the minimum coefficient of friction
for which this motion is possible is

b. Assuming we still have adequate friction to prevent slip, but


treating the acceleration magnitude, a, as an unknown, we can
retrace our steps as in part (a) to obtain

and
4F - (1 - d)N = 0
When the cabinet is on the verge of tipping, the line of action of N is
at the left comer, d = 0, so for that condition:
4F - (1 - 0)(300) = 0
F = 75 lb
Page 222

and then from

we find

The reader should note that this tendency to tip over "backwards" is a
phenomen uniquely of dynamics; there's nothing quite like it in stat­
ics. In addition we sometimes tend to think of friction in oversimpli­
fied terms, as perhaps "always opposing motion"; of course it is pre­
cisely the friction that here provides the motive force to cause the
cabinet to accelerate.

In the preceding example w e saw h o w the state of translation could


be jeopardized by the tendency of a body to "rock." In the next w e
Bar additionally explore a tendency to slip.
0
60
EXAMPLE 4 . 3

The coefficient of friction at both ends of the uniform slender bar in Figure E4.3a
is 0.5. Find the maximum forward acceleration that the truck may have without
Figure E4.3a the bar moving relative to the truck.
Solution
One possibility is that the upper end separates from the truck body. The free-
body diagram in Figure E4.3b shows the situation when the end is barely about to
break away. Because the bar is in translation, we may write:

0
60

Figure E4.3h

But this much friction cannot be generated because µ= 0.5 < 0.577. Therefore
motion of the rod relative to the truck will not be initiated by this mechanism.
The other possibility, as shown in the second free-body diagram, Fig­
ure E4.3c, is that the bar is on the verge of slipping where it contacts the truck;
this must occur at the two surfaces simultaneously. The equations of motion are
(with a =

(1)

Figure E4.3c (2)


Page 223

(3)

With µ= 0.5, Equation (3) yields


N, = 16.7N 2

and from (2)


N = 0.0581 mg
2

so that
N = 16.7N = 0.971 mg
1 2

Equation (1) then gives


0.5(0.971 mg) - 0.0581 mg =
= 0.427 g
2 2 2 2
which is 13.8 ft/sec (for g = 32.2 ft/sec ) or 4.19 m / s (for g = 9.81 m / s ) .

The reader is encouraged to rework the previous example using


Equation (4.1a) to sum moments about the bottom point of the translat­
ing bar.

PROBLEMS • Section 4 . 2

4.1 For what force P is it possible for the uniform


slender bar (Figure P4.1) to translate across the smooth
floor in the position shown? The bar has mass m and
1ft
length
40 lb
1 ft

1 ft
Figure P4.2

Figure P4.1

4.2 A 100-lb cabinet, rolling on small wheels, is sub­


jected to a 40-lb force as shown in Figure P4.2. Neglecting
friction, find (a) the acceleration of the cabinet; (b) the
reactions of the floor on the wheels.
4.3 Repeat the preceding problem for the case where
the 40-lb force is applied 1 ft above C,
Mass m
4.4 Find the value of F for which one of the wheels of
the door in Figure P4.4 lifts out of its track. Which one?
Assume negligible friction. Figure P 4 . 4
Page 224

4.5 The force P causes the uniform rectangular box of 4.9 The cords in Figure P4.9 have a tensile strength of
weight W in Figure P4.5 to slide. Find the range of values 12 N. Cart has a mass of 35 kg exclusive of the 10-kg
of H for which the box will not tip about either the front or and 1.2-m vertical rod which is pinned to it at A. Find
rear lower corner as it slides on the smooth floor, if P = W. the maximum value of P that can be exerted without
breaking either cord if: (a) P acts to the right as shown; (b)
P acts to the left. Neglect friction, and assume negligible
tension in each cord when the cart is at rest.
4.10 A force F, alternating in direction, causes the car­
2b
riage to move with rectilinear horizontal motion defined
by the equation x = 2 sin ft, where x is the displace­
ment in feet and t is the time in seconds. (See Fig­
ure P4.10.) A rigid, slender, homogeneous rod of weight
Figure P4.5 32.2 lb and length 6 ft is welded to the carriage at B and
projects vertically upward. Find, in magnitude and di­
rection, the bending moment that the carriage exerts on
the rod at B when
4.6 Repeat the preceding problem for a coefficient of
sliding friction of 0.2. 0.9 m 1.1 m

4.7 A 400-lb cabinet is to be transported on a truck as Cord


shown in Figure P4.7. Assuming sufficient friction so that
the cabinet will not slide on the truck, what is the maxi­
mum forward acceleration for which the cabinet will not
tip over?
1 ft 2 ft
Small wheels
Figure P4.9
4 ft

4 ft

Figure P4.10

Figure P4.7 4.11 A child notices that sometimes the ball does not
roll down the inclined surface of toy when she pushes it
along the floor. (See Figure P4.11.) What is the minimum
acceleration of to prevent this rolling?
4.8 The uniform bar in Figure P4.8 weighs 60 lb and is
pinned at A (and fastened by the cable DB) to the frame Solid
2
If the frame is given an acceleration a = 32.2 ft/sec as sphere 111
shown, determine the tension T in the cable and the force
exerted by the pin at A on the bar.

2
a = 32.2 ft/sec
Figure P4.11

45°
6 ft 4.12 In the preceding problem, suppose the acceleration
Cable
of is 2 a . What is the normal force between the
m i n

smooth vertical surface of and the ball? The ball's


Figure P4.8 weight is 0.06 lb.
Page 225

4.13 A can that may be considered a uniform solid


cylinder (see Figure P4.13) is pushed along a surface 2ft
by a moving arm If it is observed that translates to the
right with = g/10, what must be the minimum coeffi­ 4 ft
cient of friction between and ? The coefficient of fric­
tion between and

Figure P4.16

4.17 The block of mass m in Figure P4.17 is resting on


Figure P4.13 the cart of mass M. Force P is applied to the cart, starting
it in motion to the right. The wheels are small and fric-
tionless, and the coefficient of friction between and is
Find the largest value of P for which and will
4.14 The force Pis applied to cart and increases slowly
move together, considering all cases.
from zero, always acting to the right. (See Figure P4.14.)
Point C is the mass center of B. At what value of P will the
bodies and no longer move as one?

2 ft-
|B(3 lb)

1.8 ft
µ= 0.3
0.6 ft 6 (2 lb)

Figure P4.14 Figure P4.17

4.18 The 25-lb triangular plate is smoothly pinned at


4.15 A nonuniform block rests on a flatcar as shown in vertex A to a small, light wheel (see Figure P4.18). Find
Figure P4.15. If the coefficient of friction between car and the value of force P so that the plate, in theory, will trans­
block is 0.40, for what range of accelerations of the car late along the incline. Also find the acceleration.
will the block neither tip nor slide?
2 ft 1 ft
1 ft
0.5 ft

1 ft

4 ft
2 ft

2 ft

Figure P4.15

4.16 The truck in Figure P4.16 is traveling at 45 mph.


Find the minimum stopping distance such that the 250-lb
crate will neither slide nor tip over. Figure P4.18
Page 226

4.19 Repeat the preceding problem if the angle of the


plane is changed from
4.20 The monorail car in Figure P4.20 is driven through
its front wheel and moves forward from left to right. If
the coefficient of friction between wheels and track is
= 0.55, determine the maximum acceleration possible
for the car. Figure P4.23
8 m- Sm

4.24 A uniform rod. of length L and weight W is con­


nected to smooth hinges at E and D by the light members
B and each of length L. In the position shown in Fig­
ure P4.24, has an angular velocity of rad/sec clock­
2 5m wise. Find the forces in members ßand and deter­
Figure P4.20 mine the acceleration of center C of rod in terms of
the given variables.

4.21 A dragster is all set for the annual neighborhood


race. (See Figure P4.21.)
a. In terms of the dimensions b, H, and d and the
coefficient of friction find the maximum pos­
sible acceleration of the car. Neglect the rota­
tional inertia of the wheels.
b. How would you adjust the four parameters b,
H, d, and to further increase the driver's ac­
celeration?

Figure P4.24

4.25 Find the range of accelerations of that F can pro­


duce without moving in any manner relative to (See
Figure P4.25.) Note carefully the position of the mass
center of
Figure P4.21
1 ft
1 ft 2 ft

4.22 Rework the preceding problem for a car with (a)


front-wheel drive and (b) four-wheel drive.
3 ft
4.23 In an emergency the driver of an automobile ap­
plies his brakes; the front brakes fail and the rear wheels 8
15
are locked. Find the time and distance required to bring
the car to rest. Neglect the masses of the wheels, and
express the results in terms of the coefficient of sliding
friction the initial speed v, the gravitational acceleration
and the dimensions shown in Figure P4.23. Figure P 4 . 2 5
Page 227

• 4.26 A slender homogeneous rod weighing 64.4 lb and


20 ft long is supported as shown in Figure P4.26. Bars
and are of negligible mass and have frictionless pins at
each end. The system is released from rest with
a. Derive expressions for the angular velocity and
acceleration of bars and as functions of
b. Derive expressions for the axial force in bars
and as a function of

Figure P4.26

4.3 Moment of Momentum (Angular Momentum)


We recall from C h a p t e r 2 that the m o m e n t of m o m e n t u m of a n y b o d y
with respect to a point P (not necessarily fixed in either the b o d y or in the
reference frame) w a s defined by Equation (2.35):

w h e r e in this section R is the vector from P to the element of m a s s dm


of the body, a n d v is the velocity of dm in the reference frame (See Fig­
ure 4.1.) W e note that H d e p e n d s o n the location of P as well as the distri­
P

butions of m a s s a n d velocities in the body; it is seen to b e the s u m of


the m o m e n t s of m o m e n t a of all the m a s s elements of

body B

reference frame
Figure 4.1

We shall n o w restrict the general b o d y above to b e rigid, place it in


plane motion, a n d recall from Chapter 3 that the kinematics of a rigid
body in plane motion can be described very simply. If w e k n o w the
velocity of just one point a n d the angular velocity of the body, t h e n we
k n o w the velocity of every point of — quite a bargain. Because of this
simplicity, w e shall see that compact a n d yet completely general expres­
sions can b e written for the m o m e n t of m o m e n t u m (or angular m o m e n ­
tum, as it is often called for rigid bodies). We shall further restrict the
generic point P in the foregoing to b e a point of
The rectangular axes (x, y, z) h a v e origin at P as s h o w n in Figure 4.2,
a n d the xy-plane is the reference plane, or plane of motion, as described
in Sections 3.1, 2.
Page 228

reference plane

Figure 4.2

Recall also from Section 3.1 that each point of t h e rigid b o d y h a s a


" c o m p a n i o n p o i n t " in t h e reference p l a n e w h i c h always h a s t h e same
a n d as t h e point, a n d h e n c e also h a s t h e same velocity a n d acceleration.
For t h e point at "dm" in Figure 4.2, t h e c o m p a n i o n point is Thus, using
Equation (3.8),

Substituting this expression for v a n d the coordinates of dm into R,


Hp becomes:

or

The integral R dm in t h e first term above is equal to mr b y the defini­ PC

tion of t h e m a s s center. Making this substitution, a n d also carrying out


the cross products in t h e second term, gives t h e m o m e n t of m o m e n t u m
vector in terms of v , a n d certain m a s s distribution integrals:
P

Inertia Properties

We call the integrals in this equation inertia properties. Specifically:

m o m e n t of inertia of mass of
about z axis t h r o u g h P (4.2a)
product of inertia of m a s s of
with respect to x a n d z axes
through (4.2b)

*If t h e p r o d u c t s of inertia a r e defined w i t h t h e m i n u s sign as a b o v e , t h e n a n d o n l y t h e n


will t h e inertia properties t r a n s f o r m as a t e n s o r — a topic b e y o n d t h e scope of this book,
however.
Page 229

product of inertia of mass of


with respect to y a n d z axes
through P (4.2c)

Thus,

(4.3)

We are n o w at t h e point in our d e v e l o p m e n t w h e r e t h e inertia properties,


like the m a s s center in Chapter 2, h a v e arisen naturally. W e shall s p e n d
t h e next section studying the m o m e n t s a n d products of inertia; readers
already familiar with inertia properties m a y wish to skip Section 4.4.
Before leaving Equation (4.3), w e note for future reference that its
first term, r mv , vanishes if P is t h e m a s s center or h a s zero veloc­
pc P

ity* In b o t h these cases, w h i c h will prove valuable to us, H takes the P

form

(4.4)

T h u s in these cases t h e m o m e n t of m o m e n t u m can b e expressed in terms


of the angular velocity of (hence its other n a m e : angular momentum),
along w i t h three m e a s u r e s of its mass distribution.

Question 4.2 In Equation (4.3), does the mass center have to lie in the
reference plane with P? How about in Equation (4.4) when P has zero
velocity?

4.4 Moments and Products of Inertia /


The Parallel-Axis Theorems
Examples of M o m e n t s of Inertia

From the definition of m o m e n t of inertia,

w e see that is a m e a s u r e of " h o w m u c h m a s s is located h o w far" from


t h e 2 axis t h r o u g h P. In cylindrical coordinates w e h a v e dm, a n d
thus measures t h e s u m total of mass times distance squared over the
body's volume. The quantify is t h u s seen to b e always positive. We
n o w c o m p u t e the mass-center m o m e n t s of inertia of a n u m b e r of com­
m o n shapes. In Examples 4 . 4 - 4 . 1 2 , w e are seeking

* O r if r
ps is parallel to v , a case w e n e e d n o t consider here.
p

Answer 4.2 No. No.


Page 230

EXAMPLE 4 . 4

Homogeneous solid cylinder if 2 is its axis (see Figure E4.4).


Solution
Noting that dm— p dV, where p = mass density,

Figure E4.4

EXAMPLE 4 . 5

Homogeneous solid cylinder if 2 is an axis normal to the axis of the cylinder (see
Figure E4.5a).
Solution

Figure E4.5a

It may be confusing at first to know which moment of inertia to use for a


cylinder in a plane kinetics problem; the answer is that it is always the value
associated with the axis normal to the xy plane of the motion. If the problem is a
rolling cylinder, If we have a cylinder turning around a diametral
axis (see Figure E4.5b), then
Figure E4.5b
Page 2 3 1

EXAMPLE 4 . 6

Two special cases of Example 4.5: Slender rods and disks (see Figures E4.6a, b).
Solution

1. If the body is "pencil-like" — that is, L the moment of inertia for a


2
lateral axis through C is approximated as mL /12. This is also a correct result
even if the cross section is not circular but has a maximum dimension within
the cross section much less than L. Such a body is called a slender bar or rod.
2. If the body is a disk, however, we have R L and the moment of inertia is
2
approximately m R / 4 .
For a cylinder with the dimensions of a pencil, for example, with in.
and L = 7 in., we see that (see Figure E4.6a)
Figure E4.6a

2
where the second term is less than 0.1 of 1 percent of the retained mL /12 term.
For a typical coin, on the other hand, with in. and L we obtain (see
Figure E4.6b).
Figure E4.6b

2
This time it is the mL /12 term that is negligible; it is less than 0.15 of 1 percent of
2
the m R / 4 term. We emphasize, however, that with respect to the axis of any
sobd homogeneous cylinder (disk, rod, or anything in between), the moment of
2
inertia is mR /2.

EXAMPLE 4 . 7

A uniform rectangular solid (see Figure E4.7).


Solution

Figure E4.7
Page 232

This integration yields

EXAMPLE 4 . 8

Special case of Example 4.7: A rectangular plate.


Solution
If the rectangular solid is a plate — that is, it has one edge much smaller than the
other two dimensions — then, referring to Figure E4.8a-c, we have:

Figure E4.8a Figure E4.Bb Figure E4.8c

Again it depends on how the body's plane morion is set u p as to which axis is z
(normal to the plane of the motion) and hence which formula to use.

EXAMPLE 4 . 9

Solid, homogeneous, right circular cone about its axis (see Figure E4.9).
Solution
Here we encounter a variable limit, Noting that C and O are on the
same z axis, then (see Figure E4.9):

Figure E4.9
From similar triangles,
Page 233

which gives the varying radius in terms of z. Then

EXAMPLE 4 . 1 0

Hollow, homogeneous cylinder about its axis (see Figure E4.10).


Solution

Figure E4.10

The same result can be obtained by substracting the moment of inertia of the
"hole" (H) from that of the "whole" (W). The basis for this procedure is that we
may integrate over more than the required region provided we subtract away the
integral over the part that is not to be included:

Note that if the wall thickness is small, we have a cylindrical shell (or a hoop if the
length is small) for which and

(It is obvious that if all the mass is the same distance R from the axis z we should
2
indeed get mR .)
Page 234

EXAMPLE 4 . 1 1

A uniform solid sphere about any diameter.


Solution

Also:

and

Adding:

= 3 / since they are all


equal by symmetry

Thus (see Figure E4.ll for the spherical coordinates):

dV i n s p h e r i c a l c o o r d i n a t e s

Figure E4.11

A less tricky way to do the sphere is to use spherical coordinates directly; the
integral is

2
which yields the same result of (2/5)mR , as the reader may show by carrying out
the integration.

EXAMPLE 4 . 1 2
An example in which the density is not constant. Sometimes a body's density
varies; if it does, it must stay inside the integral when we calculate the inertia
properties. An example is the earth; we now know that the density of the solid
central core of the earth is about four times that of the outermost part of its crust
and, moreover, that this central density is nearly twice that of steel!
Page 2 3 5

Let us imagine a sphere with the same mass and radius as in the preceding
example but with a density that varies linearly and is twice as high at r = 0 as at
r = R. We shall find I about any diameter. (See Figure E4.12(a).)

Density variations

Solidn
onhomogeneous
spheres
(a) (b)
Figure E4.12

Solution
The mass of the body is

Letting the density at R be p , then


1

Substituting and integrating with the same limits as before, and then equating the
new mass to the old, gives

Thus to have the same mass as the uniform sphere, the density varies from
to going outward from the center. Then, integrating to find I,

which gives

(slightly less than )

Alternatively, if the density varies linearly but is only half as much at


the center as at R, then the results, if m and R are again the same as in the uniform
case, are (see Figure E4.12(b)):

(slightly more than )

T h e Parallel-Axis T h e o r e m for Moments of Inertia

In many applications the body consists of a number of different smaller


bodies of familiar shapes. In such cases, there is fortunately n o need to
integrate in order to find the inertia of each part with respect to a common
axis of interest, thanks to what is called the parallel-axis theorem or
transfer theorem. If w e know the moment of inertia about an axis
Page 236

through the mass center C of any body w e can then easily find it about
any axis parallel to C by a simple calculation. The theorem states that the
moment of inertia of the mass of about any line is the moment of inertia
about a parallel line through C plus the mass of times the square of the
distance between the two axes:

To prove this theorem, w e let (x, y, z) and (x , y , z ) be rectangular


1 l 1

cartesian coordinate axes through P and C with the corresponding axes


respectively parallel, as s h o w n in Figure 4.3. Then w e have, by defini­
tion,

Figure 4.3

Since it is seen that

substitution gives

(4.5)

or
(4.6)

in w h i c h the square of t h e distance b e t w e e n z axes t h r o u g h


C a n d P.

Question 4.3 Why are the last two integrals in Equation (4.5) zero?

Equation (4.6) is the parallel-axis t h e o r e m for m o m e n t s of inertia. But

Answer 4.3 A n y integral s u c h as dm, w h e r e x is m e a s u r e d from a n origin at, say, Q


is equal to this is t h e definition of t h e m a s s center. So if t h e origin is C, t h e n = 0
and dm = 0.
Page 237

note: We can only transfer from the mass center C and not from any other
point A about which we may happen to know

EXAMPLE 4 . 1 3

For the uniform slender rod shown in Figure E4.13 find the moment of
inertia of the mass of with respect to a lateral axis through one end. (This
exercise will be useful in pendulumlike applications in which a rod is pinned at
one end.)
(mass m)
Solution

We next consider an example of the buildup of the moment of inertia


for a composite body.

EXAMPLE 4 . 1 4

Find for the body shown in Figure E4.14. The mass densities each =
= constant, so that the respective masses are:

Sphere Half-cylinder
Slender bar

Side view

Top view
Figure E4.14
Page 238

and

Solution
First we observe that since the inertia integral may be
carried out over the bodies separately, so long as we cover all the elemental
masses of the total body. Filling in the separate integrals, we get

in which, for the half-cylinder,

Note that we cannot correctly transfer the inertia of the semicylinder from
it must be done from the mass center C . So first we go "through the back door"
3

to find (since we know the moment of inertia already with respect to and
only then may we transfer to O.

EXAMPLE 4 . 1 5

A closed, empty wooden box is 5 ft X 3 ft X 2 ft and weights 124 lb (see Fig­


ure E4.15).
a. Find its moment of inertia about an axis through C parallel to the 2-ft
dimension.
b. If the box is then filled with homogeneous material weighing 240 lb
(excluding the box), how much does the moment of inertia about the
axis of part (a) increase?

3 ft

5 ft 2 ft

Figure E4.1S

Solution
a. The masses of the various sides of the box are proportional to their
areas (thickness and density assumed constant):
Page 239

total
area

Therefore, taking the contributions from the three pairs of sides,

2 2
= 15.9 slug-ft (or lb-ft-sec )

2
slug-ft
contents

Even though the box weighs only about half as much as the contents, the position
of its mass makes its moment of inertia over three-fourths that of the contents.
2
The total moment of inertia is 37.0 slug-ft .

T h e Radius of Gyration

There is a distance called the radius of gyration that is often used in


connection with moments of inertia. The radius of gyration of the mass of
a body about a line z (through a point P) is called , or just k if the axis is
P

understood to be z, and is defined by the equation

(4.7)

If one insists on a physical interpretation of k , it may be thought of as the


P

distance from P, in any direction perpendicular to z, at which a point


mass, with the same mass as the body, would have the same resulting
moment of inertia that the body itself has about axis z. For example, a
solid homogeneous cylinder has a radius of gyration with respect to
its axis of since The usefulness of k is seen
c

here, since regardless of the mass of a cylinder (and hence of its density)
k will be the same for all homogeneous cylinders of equal radii.
c

Note further that (using the parallel-axis theorem)

Thus

(4.8)
Page 240

a n d w e see from Equation (4.8) that the radius of gyration, like the
m o m e n t of inertia itself, is a m i n i m u m at C.

Products of Inertia

We n o w turn to t h e other t w o measures of mass distribution that h a v e


arisen in our study of p l a n e motion of a rigid b o d y — n a m e l y and
taken here to b e with respect to axes (x, y, z) t h r o u g h a n y point P.*
O u r first step is to gain insight into t h e m e a n i n g of products of inertia
as w e s h o w that they in fact vanish for t w o large classes of c o m m o n l y
occurring symmetry. These classes are defined b y the two conditions
(with p constant in both): (1) z is a n axis of symmetry a n d (2) xy is a plane
of symmetry. Let us examine w h y t h e t w o products of inertia and
are zero in these cases. We recall that their definitions are

(4.9)

Class 1: z Is an Axis of Symmetry. For each dV at (x, y, z) there is a cor­


responding dV at (—x, —y, z). T h u s t h e contributions of these t w o ele­
m e n t s cancel in b o t h t h e and integrals. Since each point of has
a "canceling p o i n t " reflected t h r o u g h t h e z axis, and are each
zero for this class of bodies. (See Figure 4.4.)
Class 2: xylsa Plane of Symmetry. In this case each differential v o l u m e dV
at (x, y, z) necessarily h a s a mirror image at (x, y, — z). T h u s the contribu­
tions of these t w o elements cancel in b o t h integrals, a n d taken over the
whole of w e see again that and are each zero. (See Figure 4.5.)

Figure 4.4

Figure 4.5

* I n general (three-dimensional) motion, t h e r e arise six distinct inertia properties: three


m o m e n t s of inertia a n d t h r e e p r o d u c t s of inertia.
Page 2 4 1

Just because a b o d y does n o t belong to either of these t w o classes


does not m e a n it cannot h a v e zero products of inertia. However, these are
simply c o m m o n cases w o r t h y of note.

Question 4.4 Think of a rigid body for which both products of inertia
are zero, but which does not fall into either of the two classes.

Transfer T h e o r e m for Products of Inertia

There is a transfer t h e o r e m for products of inertia, just as there is o n e for


m o m e n t s of inertia. To derive it, w e write from Figure 4.3:

The last t w o terms v a n i s h b y virtue of t h e definition of t h e mass center


(for example, dm = m times t h e z distance from C to C, w h i c h is zero).
Thus
(4.10a)

W e note that t h e factor of m in Equation (4.10a) is alternatively the


product of t h e x a n d z coordinates of P in a n axis system w i t h origin at C.
Similarly, w e h a v e

(4.10b)

Answer 4.4 Both p r o d u c t s of inertia are z e r o for t h e b o d y s h o w n in t h e d i a g r a m at t h e


left. T h r e e cylindrical b a r s of a n y l e n g t h s lying a l o n g t h e (x, y, z) axes are joined at t h e
origin t o form a rigid b o d y . N e i t h e r t h e z axis n o r t h e xy p l a n e is o n e of s y m m e t r y , yet
and a r e zero.

EXAMPLE 4 . 1 6

Find and for the body shown in Figure E4.16; it is composed of eight
identical uniform slender rods, each of mass m and length

2 3
4
1 8

7
5
6
Figure E4.16
Page 242

Solution
We have since xz is a plane of symmetry. Recall that when this happens,
the two products of inertia containing (as a subscript) the coordinate normal to
the plane are zero.* With superscripts identifying the various rods, we then have
the following for the other product of inertia:

Note that by symmetry each rod has zero about axes through its own center of
mass parallel to x and z. Therefore the eight terms listed above will consist only of
transfer terms in this problem.
Furthermore, since is zero for rods 4 and 5, and since is zero for rods 1 and
8, only four rods contribute to the overall

Note that the "unbalanced" masses lie in the second and fourth quadrants in
this example; hence the sign of is positive since its definition carries a minus
sign outside the integral. We shall return to this example later in the chapter and
examine the reactions caused by the nonzero value of when the body is spun
up in bearings about the z axis.

PROBLEMS • Section 4.4

4.27 An ellipsoid of revolution is formed by rotating the 4.28 In Figure P4.28, the area bounded by the x and y
ellipse about the x axis as in Figure P4.27. Find the mo­ 2
axes and the parabola y = 1 — x is rotated about the x
3
ment of inertia of this solid body of density 15 slug/ft axis to form a solid of revolution. The density is p
about the x axis. 8 3
= 1000 • (1 — x) k g / m . Find the moment of inertia of
the solid mass about the x axis.

1 ft
1

1
2 ft
1
Figure P4.27

Figure P4.28

* Thus is also z e r o in this e x a m p l e , b u t d o e s n o t a p p e a r a n y w h e r e in t h e p l a n e -


m o t i o n equations, as w e h a v e seen.
Page 243

4.29 The slender rod in Figure P4.29 has a mass density 4.32 Use the parallel-axis theorems and the results of the
given by preceding problem to find, for that plate, the moments
and products of inertia at the center of mass.
4.33 For a uniform thin plate with xy axes (and origin O)
in the midplane, show that
in which and are constants. The rod has length L.
Find its moment of inertia about the line defined by x = 0,
y-L/2. Confirm this statement with the results of Problem 4.31
4.30 Find for the semielliptical prism shown in Fig­ for the case when that plate is thin.
ure P4.30 (density length normal to plane of paper 4.34 Find for a uniform thin plate in the form of a
= L)• pie-shaped circular sector as shown in Figure P4.34.
4.31 The midplane of a uniform triangular plate is
shown in Figure P4.31. Find, by integration:

Figure P4.34

What would be good approximations were the plate thin? 4.35 Find for the plate in the preceding problem.
4.36 The surface area of a solid of revolution is formed
2
by rotating the curve y = x (for 0 x 2 m) about
the x axis. (See Figure P4.36.) The density of the material
varies according to the equation p = 20x, where is in
3
k g / m when x is in meters. Find and tell why your
answer is also
4.37 In the preceding problem, find and
4.38 See Figure P4.38. (a) Show that the moment of in­
ertia for a solid homogeneous cone about a lateral
2 2
Figure P4.29 axis through the base is = (m/20)(3R + 2H ). (b)
Using the transfer theorem, find the expression for

2a
Figure P4.30

Figure P4.36

Density = p, thickness = t
Figure P4.31

* Asterisks identify t h e m o r e difficult problems. Figure P4.38


244

•4.39 In the previous problem, find 1° for the body in the Find Ig for the semicircular ring Bin Figure P4.48.
figure if the part above z = H/2 is cut away to form a Hint: If the dashed portion were present, Ig would be
truncated cone. 2
(2m)R ; by symmetry, our semicircular ring contributes
4.40 The body shown in Figure P4.40 is composed of a half of this, so that for B we have
slender uniform bar (m = 4 slugs) and a uniform sphere Ig = mR 2

(m = 5 slugs). Find Ig for the body, where z is normal to


the figure. Now use the transfer theorem to complete the solution
without integration.
y*. A rod ^of length 1 m is welded on its ends to a disk
2> and a sphere S. (See Figure P4.49.) The uniform bodies
have masses m = 10 kg, m = 5 kg, and m = 15 kg.
e 3 s

The radii of 2) and J ' a r e 0.3 m and 0.1 m, respectively.


Find 7^.
4.50 The three bodies shown in Figure P4.50, welded
Figure P4.40 Figure P4.41 together to form a single body B, have masses of 64 (rec­
tangular plate), 56 (rod), and 48 (disk), all in kilograms.
Find the moment of inertia of with respect to the z axis.
Q

4.41 Two bars, each weighing 5 lb per foot, are welded


together as shown in Figure P4.41. (a) Locate the center of y t
mass of the body, (b) Find Ig.
4.42 Find 1 ^ in Problem 4.41.
4.43 Use the result of the preceding problem, together
with the transfer (parallel axis) theorem, to find Ig.
Find Iyy in Problem 4.41.
4.45 Use the result of the preceding problem, together
with the transfer theorem, to determine Igj.
4.46 Find the moment of inertia of the mass of B about
2
axis z if 7£ = 40 kg • m . (See Figure P4.46.)
B

4.47 Find the moment of inertia of a uniform hemi­


spherical solid about the lateral axis x through its mass
c

center. (See Figure P4.47.)

X
Figure P4.49

'-•—0.8 m — - \

Figure P4.47 Figure P4.50


Page 245

4.51 In Figure P4.51, is a solid sphere, is a solid 4.54 The bent slender rod in Figure P4.54 is located with,
cylinder, and and are slender rods. The center lines the axes of the rods parallel to the x- and z-axes, as shown.
of and pass through the mass centers of and Find the value of
respectively. Find for the system of four bodies.
0.3 m

10 kg 2.4 m

4.5 kg
2 4 kg 0.3 m

12 k g 2.6 m

(3 kg)
0.7 m 0.4 m

Figure P4.51

(4 k g )
4.52 Find (which equals for a thin plate (density
thickness t) in the shape of a quarter-circle. (See Fig­ 4.0 m
ure P4.52.)

Figure P4.54

4.55 The cylinder in Figure P4.55 has a mass of 6 kg


Figure P4.52 and a radius of 0.4 m. Show that the moment of inertia
2
about an axis z normal to the page is 0.48 kg • m , and
c

that the corresponding radius of gyration is m.


4.56 In the preceding problem, show that it is possible to
4.53 Find the moments of inertia of the pendulum about drill a hole through that is below the geometric center
axes x, y, and z. (See Figure P4.53.) Axes and are in the and satisfy all of the following:
plane of the pendulum, is a slender rod, and is a a. the remaining mass is 5.5 kg;
semicircular disk, each of constant density. b. the distance between and the new mass
center C (see Figure P4.56) is 0.02 m.
Find the radius r of the hole, the center distance d, and the
new value of

(1 kg)
1 m 04 m

0.2 m
[2 kg)
Figure P4.53 Figure P4.55 Figure P4.56
Page 246

4.57 The antenna in Figure P4.57 has a moment of 4.61 Show in the following three ways that the moment
inertia about of and the counterweights have a of inertia of a uniform, thin spherical shell, about any line
collective moment of inertia about of . Points C and G 2
through its mass center, is (2/3) mr . (See Figure P4.61.)
are the respective mass centers of the antenna and its Which of the three approaches would work if the object
counterweights. The purpose of the counterweights is to were hollow but not thin — that is,
place the combined mass center at O to reduce stresses.
a. Use spherical coordinates:
Thus MD = md, where we neglect the mass of the con­
necting frame for this problem. Compute the values of M
and D that will minimize the total moment of inertia I a

(Iof about 0 plus 7 of. about O). Use where


is a constant. where r is the average radius,

b. Use the idea:

Counterweights
(mass M)

Mass m Let R increase by and compute the change,


by using differential calculus with
Note that this change is the moment
of inertia of the shell!

Figure P4.57 2R

4.58 Find the product of inertia for the ring of Prob­


lem 4.48.
4.59 Find for the welded body of Problem 4.50.
4.60 Determine in Problem 4.49. The xy plane con­
tains the centers of and Figure P4.61

4.5 The Mass-Center Form of the Moment Equation


of Motion
Development of the Equations of Plane Motion

In Chapter 2 w e developed several different forms of Euler's second law.


In this section w e will continue to study t h e mass-center form, Equa­
tion (2.43):
(2.43)

w h e r e C is the mass center of a n arbitrary b o d y If w e n o w restrict the


body to b e rigid a n d t h e reference frame to b e a n inertial frame in
Equation (4.4), then w e m a y substitute from that equation for H into c

Equation (2.43) above a n d obtain:

(4.11)
Page 247

At this stage w e m u s t m a k e a decision with regard to h o w the a n d


axes of Equation (4.11), w h i c h h a v e their origin at C, will b e allowed to
change relative to t h e inertial frame of reference w h e r e the derivative in
Equation (4.11) is to be taken, a n d to which of the b o d y is referred.
Note that the direction of the z axis h a s already b e e n fixed perpendic­
ular to the reference plane. If w e fix the directions of x a n d y relative
to t h e inertial frame, t h e n a n d (as well as are constant relative to
that frame b u t and are in general time-dependent. This choice is
very difficult to deal with.

Question 4.5 However, will not change in this case. Why not?

A m u c h m o r e convenient choice is to let the axes a n d z all b e fixed in


t h e b o d y so that t h e m o m e n t s a n d products of inertia are all constant.
N o w a n d are t i m e - d e p e n d e n t relative to the inertial frame Figure 4.6
s h o w s the unit vectors a n d (fixed to expressed in terms of a n d
(which are fixed in t h e inertial reference frame

Figure 4.6

The derivatives of these unit vectors, obtained earlier as Equations (3.41)


a n d (3.42), are:

(4.12a)

Answer 4.5 Let (x, y, z) b e fixed in t h e reference frame A s t h e b o d y rotates w i t h r e ­


spect to frame its m a s s is t h e n distributed differently, at different times, w i t h respect
to a n d But in p l a n e m o t i o n t h e z axis is fixed in direction in both t h e b o d y a n d in
2
space. T h u s since t h e s q u a r e of t h e distance from t h e z axis is a l w a y s t h e s a m e r re­
gardless of t h e orientation of a n d w e see t h a t d o e s n o t c h a n g e as t h e b o d y t u r n s .
Page 2 4 8

(4.12b)

Therefore, carrying out the differentiations in Equation (4.11), w e have

or
(4.13)
This expression represents three scalar equations:

(4.14a)
(4.14b)
(4.14c)

Equations (4.14a,b), along with tell us about the nature of reac­


tions necessary to maintain the plane motion. If and are both zero,*
then and the system of external forces (loads plus
reactions) has a planar resultant, with this plane containing the mass
center. Thus, with a coplanar system of external loads, the resultant
reactions must have a resultant in that same plane. This is the basis for
"two-dimensionalizing" the analysis of problems for which the two
products of inertia above vanish; w e shall first work with symmetrical
bodies for which this is the case. Then later in this chapter w e shall
examine some problems in which and are not both zero. In any case,
however, the rotational motion of the body is governed by the simple
kinetics equation
(4.14d)
We see that while force produces acceleration with the "resistance"
being the mass, it is also true that moment produces angular acceleration
with the "resistance" being the body's moment of inertia. Note also that
the moment of the external forces about the z axis through the mass
center is and w e have Thus the resultant m o ­
ment about this axis equals the moment of inertia about the axis multi­
plied by the angular acceleration of the body, regardless of whether or
not the products of inertia vanish.
We will n o w restrict our attention in the remainder of this section to a
special class of problems of plane, rigid-body motions. This class is de­
fined by the following pair of conditions:

1. (usually because the body is either symmetric about


the plane of motion of the mass center or else has an axis of physical
symmetry which remains normal to the reference plane of motion); and
2. The external loads have a planar resultant with the plane contain­
ing the mass center.

When or is not zero, t h e r e h a s to b e a n o n z e r o p r e s e n t to m a i n t a i n


t h e m o t i o n ; t h e s e are u s u a l l y f o r m e d b y lateral forces (such as b e a r i n g reactions) at dif­
ferent positions a l o n g t h e z-axis.
Page 249

These two conditions necessitate external reactions likewise equipollent


to a coplanar (in the same plane as the loads) force system. Euler's laws
are then effectively reduced to the following three scalar equations:

(4.15a)
(4.15b)
(4.15c)

where x and y are coordinates of the mass center in a rectangular


c c

coordinate system fixed in the inertial frame.* Equations (4.15a,b) are of


course the x and y components of the mass center equation of motion
valid for any body, rigid or not. Since there will be n o confusion about the
axis in question, w e shall often write Equation (4.15c) as simply

or
2M C = Iac
(4.16)

Helpful Steps to Follow in Generating and Solving the Equations of Motion

In some instances Equations (4.15a-c) will yield a differential equation(s)


that can be readily integrated so that w e predict the ongoing motion of a
body. More commonly w e shall be dealing with what w e might call
"snapshot" problems where at a specific instant w e calculate forces and
accelerations. These problems are rather natural extensions of statics, all
of the operations in the analysis being algebraic. For problems of either
class the following steps are recommended:

1. Draw a free-body diagram (FBD) of each body in the problem.


2. Define a set of unit vectors, or, equivalently, a coordinate system,
in terms of which unknown forces and accelerations may be expressed.
3. Substitute into the three equations of motion; some may prefer to
do this in vector algebraic form, explicitly displaying previously defined
unit vectors, while others may prefer to initiate the analysis using the
three component equations (4.15a,b and 4.16).
4. Often the number of scalar unknowns will exceed the number of
independent equations (maximum of three), and w e must look for sup­
plementary information. This might mean nothing more than applying
the Coulomb law of friction, but often the supplementary information
will be in the form of a kinematic constraint. For example, for a wheel
having its mass center at the geometric center, rolling would imply (with
appropriate definitions of variables) If some point A has
its motion constrained, then the restriction o n its acceleration along
with a = a +c A can be used to relate acceleration of

W h i c h c o m p o n e n t e q u a t i o n s a r e used a n d t h e specific forms they take d e p e n d , of


course, o n w h a t t y p e of c o o r d i n a t e system is used to describe t h e m o t i o n of C. A rectan­
gular coordinate s y s t e m is t h e n a t u r a l choice for m o s t of t h e p r o b l e m s t a k e n u p in this
chapter. A polar c o o r d i n a t e s y s t e m is t h e n a t u r a l choice, h o w e v e r , for p r o b l e m s of or­
bital m e c h a n i c s (see Section 8.4).
Page 2 5 0

the mass center and angular acceleration of the body (presuming veloci­
ties, and thus are known). (Of course, a and a are the kinematical
c

variables which naturally appear in the equations of motion, as w e have


seen.)
5. Solve for unknown forces a n d / o r accelerations. All problems
appearing in this text are "rigid-body dynamically determinate," so this
will always be possible. In the case of "movie" problems, integrate accel­
erations to get velocities and then velocities to get positions as functions
of time.
6. Be sure to check on dimensional consistency of results and, in the
case of numerical results, check to see that units are correct (especially
important in these days of transition from U.S. to SI units). Also, see if
your answer seems to make sense.

We n o w proceed toward a set of examples which will make use of the


equations (4.15a,b and 4.16), which w e have developed in this section.
These examples are designed to illustrate the kinds of problems the
student should learn to solve. The first one is a "movie" problem in which
one resultant force component and the moment are zero, while the other
force component is not.

EXAMPLE 4 . 1 7

A horseshoe pitcher releases a horseshoe with = 4 . 7 1 rad/sec in the posi­


tion shown in Figure E4.17a. If the horseshoe turns exactly once in plane motion
and scores a ringer, find the initial velocity of the horseshoe's mass center.

CLANK

3 5 ft

4 0 It

Figure E4.17a

A free-body diagram of the horseshoe in flight is shown in Figure E4.17b. We


write and then integrate Equations (4.17a-c):

(the initial (the initial velocity of C


velocity of C in y-direction)
in x-direction)

since x = 0c
since horseshoe is released
Figure E4.15b at t = 0 at y = 3.5 ft
c
Page 251

Now when the horseshoe lands, 6 = 2n, so

Thus

and

Hence the initial velocity of the horseshoe's mass center is

The next example involves rolling, and is a "movie" (ongoing time)


problem:

EXAMPLE 4 . 1 8

The cylinder (mass m, radius r) is released from rest on the inclined plane shown
in Figure E4.18a. The coefficient of friction between cylinder and plane is µ.
Determine the motion of C, assuming that µis large enough to prevent slipping.
(How large must it be?)
Solution

As in statics, a good first step is to draw a free-body diagram (see Figure E4.18b).
Figure E4.18a We are to assume that the cylinder rolls. In this case the friction force is an
unknown and has a value satisfying

where = µN from the study of Coulomb friction in statics. We shall also use a
kinematic equation expressing the rolling. After solving for we shall then
impose the condition that it be less than µ.N, since we know the cylinder is not
slipping.
We choose x, y, and Øas shown, motivated by the fact that C will move down
the plane and the cylinder will rum counterclockwise. The equations of motion
are

Figure E4.18b (1)


(2)
(Note that, kinematically, y is constant so that
c vanishes.)

(3)
We can solve (2) for N, getting N = mg cos There remain two equations in the
three unknowns andØ.We must therefore supplement our equations of mo-
tion with the remaining kinematics result, which comes from the rolling condi-
tion:

(4)
Page 252

Solving Equations (1), (3), and (4) gives

Integrating twice, and noting that the integration constants vanish, we get

And since we are told that

that is,

then we have

for the rolling motion to occur.

In the preceding example, things would be quite different if the value


of the friction force that satisfies the equations, namely = (mg sin ) / 3 ,
were larger than = = mg cos So if the solution were to yield

then there would be insufficient friction to permit rolling. We would then


need to abandon* the rolling condition, replacing it by the known maxi­
m u m value of f,i.e.,

but n o w w e know

where N still equals mg cos And since Equation (3) still holds,

or

Therefore, integrating twice, w e get

* W h e n y o u a s s u m e s o m e t h i n g a n d later arrive at a contradiction of fact, t h e n t h e logi­


cal conclusion is t h a t t h e a s s u m p t i o n w a s invalid.
253

w h e r e t h e integration constants are zero since = 0 at t = 0. Equa­


tion (1) n o w yields, with

or

Thus

a n d t h e solutions for t h e motion [x^f) a n d (f)] are i n d e e d quite different


w h e n t h e cylinder turns a n d slips t h a n they are w h e n it rolls. If = 0.5
a n d = 3 0 ° , for example, then

a n d the cylinder rolls. But if = 0.2 a n d = 60°, t h e n

a n d t h e cylinder slips. N o t e that in general (as one w o u l d expect) the


cylinder rolls for larger a n d smaller
Finally, note that if w e wish to distinguish b e t w e e n static a n d kinetic
coefficients of friction and t h e n the rolling assumption w o u l d b e
correct if tan . But if t a n w e w o u l d t h e n use in
t h e r e m a i n d e r of t h e solution, a n d t h e n in the answers for x a n d would
c

become
Next w e take u p a n o t h e r rolling problem, only this time t h e m a s s
center a n d geometric center are n o t t h e s a m e point. As a result, the
kinematics equations are m o r e difficult. This example is a " s n a p s h o t "
(occurring at one instant of time) problem.

EXAMPLE 4 . 1 9

The rigid body in Figure E4.19a consists of a heavy bar of mass m welded to a
light hoop; the radius of the hoop thus equals the length of the bar. Find the
minimum coefficient of friction between the hoop and the ground for which the
body will roll when released from rest in the given position.
Figure E4.19a
Solution
The free-body diagram is shown in Figure E4.19b along with the base vectors
adopted for the problem.

Question 4.6 Why is this a good choice of base vectors to use in this
problem?

Answer4.6Because t h e initial acceleration of t h e m a s s center C will b e to the right a n d


Figure E4.19b downward a n d t h e initial a n g u l a r acceleration will b e clockwise
Page 254

Note that the gravity force resultant acts through the center of the bar since we
are neglecting the weight of the hoop.
Next we write the three differential equations of motion, letting a = c

+ and a =

(1)
(2)

(3)

These equations contain the unknowns f, N, and a. We draw upon kine­


matics for two more equations. We know that the accleration of the geometric
center of the hoop is = so that

or

Therefore, equating the i coefficients and then the coefficients, we find

(4)

(5)

The student may wish to verify that (4) and (5) also result from relating a to c

= true for any round body rolling on a flat, fixed plane. In this problem,
is then zero at release because, until time passes, is still zero.
Solving Equations (1) to (5) for the five unknowns gives the results:

To complete the solution, we must get the coefficient of friction µinto the picture.
We know that for any friction force

Therefore, in our problem,

so that

This means that for the body to roll, a friction coefficient of at least = 6 / 1 3 is
required; this is then the desired minimum.
We emphasize that students should always make "eyeball checks" of their
answers — glancing over the results to be sure they make sense physically. In this
problem, for instance, note that:
Page 255

1. N < mg as expected, for otherwise the mass center could not begin to move
downward as the body rolls.
2. is positive, and therefore in the correct direction to (since it is the only
force in the x-direction) move the mass center to the right.
3. and a are all positive and therefore in the expected directions.

The next example features another "snapshot" problem — one


which w e are examining only at one instant. It also involves the interest­
ing constraint of a taut string.

EXAMPLE 4 . 2 0

A uniform rod is supported by two cords as shown in Figure E4.20a. If the


right-hand cord suddenly breaks, determine the initial tension in the left cord AD.
("Initial" means before the rod has had time to move and before it has had time to
generate any velocities.)
Solution
Using the free-body diagram in Figure E4.20b, the equations of motion are, with
64.4 Ib
a =
c + and a =
Figure E4.20a

(1)

(2)

30'

64 4 lb (3)

Figure E4.20b Unfortunately, Equations (1-3) contain four unknowns (T, , , and a). Thus
we seek an additional equation in these unknowns from kinematics. The point A
is constrained to move (see Figure E4.20c) on a circle of radius I about D. Thus
point A has the tangential and normal components of acceleration shown (see
Section 1.7). Furthermore, v = 0 at the instant of interest (nothing is moving
A

yet!).
We may relate this a to a :
A c

(4)

Figure E4.20c

Here now is a nice shortcut:


If we dot Equation (4) with a unit vector in the direction , we will elimi­
nate a (because it is perpendicular to that direction). This is easier than solving
A

the two and equations." Such a unit vector is


Page 256

so that, doing the dotting,

or

(5)
Equations (1), (2), and (3) yield:

Substituting these three results into Equation (5) results in:

or

Note that before the right-hand string was cut, the tension, from statics, was:

Forces in inextensible strings (ropes, cables, cords) are capable of changing "in­
stantaneously," and indeed we see that this is the case in this problem.

Question 4.7 Can spring forces change instantaneously in this way?

In the preceding example, back-substitution immediately yields

Thus the mass centeT will


start to m o v e off to the
left and d o w n .

Thus the body will


start to turn clockwise.

The acceleration of A follows from Equation (4):

a n d , as a check, t h e direction of a , A , is 3 0 ° .

Answer 4.7 N o , t h e l e n g t h m u s t c h a n g e a n d t h a t takes time.


Page 257

2
The m a g n i t u d e of n isA = 4.97 f t / s e c , which
is (see Section 1.7) at t h e initial instant. It is interesting to n o t e that
the initial angular acceleration of t h e string DA is or 4 . 9 7 /
Finally, w e close t h e section with a n example containing t w o bodies
in rolling contact. The plate is simply translating, b u t t h e pipe h a s a more
complicated motion: it rolls o n t h e pipe, b u t not o n t h e inertial frame
(ground).

EXAMPLE 4 . 2 1

Force P is applied to a plate that rests on a smooth surface. (See Figure E4.21a.)
Mass m Find the largest force P for which the pipe will not slip on the plate.
Solution
Mass m For the pipe (Figure E4.21b), with a =

(1)
(2)
Figure E4.21a
(3)
(Note that If the thickness (r — r ) is not o i

given, assume it is small.)


For the plate (Figure E4.21c), we note that only the x equation of motion is of
help; = 0 gives N = N + Mg = (m + M)g as expected, and dimen­
2

sions are not given so moments cannot be taken. (The moment equation would
only give us the location of N , anyway.) Therefore
2

(4)
Figure E4.21b
Elirninating f between (1) and (3) gives

(5)
And between (1) and (4) gives

(6)
We note that if m = m + M and C is the mass center of pipe plus plate, then
T T

Figure E 4 . 2 1 c Equation (6) could have been written immediately from for the
combined system. Here = P; the right side follows from two derivatives of
the definition of the mass center = mx + Mx ). c G

The kinematics equation is tricky here. It is a rolling condition, but we must


remember that x and x are necessarily measured relative to an inertial frame,
G c

here assumed to be fixed in the ground. Thus it is that is related to a For


no slip, which, when differentiated, yields

(7)
Substituting a from (5) into (7) relates the accelerations of the two mass
centers:

(8)

*S o m e t i m e s G is u s e d to d e s i g n a t e a m a s s center.
This difference is just t h e acceleration of C in t h e frame consisting of t h e translating plate.
Page 258

Then (6) and (8) may be combined to give


(9)
And combining (9) and (1) gives us the relationship between P and

(10)

Since N for no slip, (10) gives:

Any larger P than (m + 2M) , will cause the pipe to slip on the plate.

PROBLEMS • Section 4.5

4.12 A uniform sphere (radius r, mass m) rolls on the


plane in Figure P4.62. If the sphere is released from rest at
f = 0 when x = L, find x(t).
4 63 A symmetric body has mass m and radius R; a
cord is wrapped around it as shown in Figure P4.63,
Compute the downward acceleration of the center C if is
(a) a cylinder; (b) a sphere (with a small slot to accommo­
date the cord); (c) a thin ring. Hint, Work the problem just
once with radius of gyration k ; then substitute the three
c Smooth
values and 1R far k . c
0
60

Fioure P4.64

4.65 Sally Sphere, Carolyn Cylinder, Harry Hoop, and


Wally Wheel each have mass m and radius R. Wally's
spokes and rim are very fight compared to his hub. (See
Figure P4.65.) They are going to have a race by rolling
Figure P4.62 down a rough plane. Give (a) the order in which they
finish and (b) the times.

Figure P4.63

4.64 The cord in Figure P4.64 is wrapped around the


cylinder, which is released from rest on the 60° incline
shown. Find the velocity and position of as a function of Sally Carolyn Harry Wally
time t. Figure P4.64
Page 259

4.66 In the preceding problem, Wally and Carolyn are 4.69 The uniform sphere (mass m, radius r) in Fig­
connected by a bar of negligible mass and released from ure P4.69 is at rest before P is applied. If µis the coeffi­
rest on the same incline. (See Figure P4.66.) Determine cient of friction between sphere and floor,
the force in the bar.
a. find the maximum P for there to be no slip;
b. for P twice that found in (a), find a and a.
c

Note that does not equal as it would if


there were no slipping, i.e., if the sphere were
rolling.

Rod
Figure P4.69

Figure P4.66 4.70 The uniform cylinder in Figure P4.70, of mass m


and radius r, is at rest before it is subjected to a couple of
4.67 Repeat the preceding problem, but suppose that moment M The coefficient of friction between cylinder
0

Wally and Carolyn switch places. and floor is


4.68 The two pulleys and and the block in Fig­ a. Find the largest value of M for which there is
0

ure P4.68 each have mass m and are connected by the no slip.
cord. b. For M twice the value found in (a), find a and a.
0 c

a. Write a brief paragraph explaining in words


why the system cannot be in equilibrium. Start
with, "If the system were in equilibrium, the
tension in the rope above would equal mg."
Then follow the rope around the pulleys until
you reach a contradiction.
b. Find the acceleration of C . Figure P4.70
1

4.71 Find the ratio of r to R for which the force T in


Cord Figure P4.71 will cause the wheel to roll (no slip) no mat­
ter how small the friction. Treat the wheel as a uniform
cylinder.

Figure P4.68 Figure P4.71


Page 260

4.72 Force T is given to be small enough, and the friction 4.76 The bowling ball in Figure P4.76 is released with
coefficient large enough, that both wheels in Fig­ = 22 ft/sec and co = 0 as it contacts the surface of the
ure P4.72 will roll on the plane. alley. Neglecting the effect of the three finger holes, and
using a coefficient of friction of 0.3, find the distance
a. Give arguments why one wheel rolls left and
traveled by the center of the ball before slipping stops.
the other right.
b. Find the ratio of r to R for which the accelera­ 4.77 The force P = 60 N is applied as shown in Fig­
tions of C are equal in magnitude. ure P4.77 to the 10-kg cylinder originally at rest be­
neath the mass center of the thin, 5-kg rectangular plate
The coefficient of friction between and is 0.5, and
the plane beneath is smooth. Determine: (a) the initial
acceleration of C; (b) the value of x when is slipping on
both surfaces. The length of is 2 m.
4.78 The constant force F is applied to the cylinder,
0

initially at rest, as shown in the two drawings constituting


Figure P4.78. Show in the following two ways that the
Figure P4.72 cylinder will slip provided that

4.73 The uniform sphere in Figure P4.73, of mass m and


radius r, is at rest when it is subjected to a couple of
a. Assume rolling; then obtain the inequality from
moment M . If there is no slip, find the acceleration of the
0
after solving for and N.
center of the sphere.
b. Assume slipping; then integrate x and to
c

4.74 A cylinder spinning at angular speed rad/sec obtain x and 0, and then find the velocity of
c

clockwise is placed on an inclined plane. (See Fig­ the contact point B; if it is to the right (that is,
ure P4.74.) Show that the cylinder center will begin mov­ positive), this is consistent with to the
ing up the plane if tan Why does this result have left and we have slipping.
nothing to do with the size of ?
4.75 The cylinder of weight W and radius r shown in
Figure P4.75 has an angular velocity of 100 r a d / s clock­
wise. It is lowered onto the rough incline. If its center C is
observed to remain momentarily at rest, determine the
coefficient of sliding friction. Find how long the center C
remains at rest.
Figure P4.76

Figure P4.73 Figure P4.74


Figure P4.77
100 rad/s
0.1 m

0
30

Figure P4.75 Figure P4.78


Page 261

Cylinder,

Drum

Coefficient of
friction = µ

4.5 in.
Bar 6 in.
3 lb 3 1b Bar Figure P4.82

Figure P4.79

4.79 Two drums of radius 4.5 in. are mounted on each


end of a cylinder of radius 6 in. to form a 40-lb rigid body 3 ft
with radius of gyration = 5 in. (See Figure P4.79.) 1 ft Cord
Ropes are wrapped around the drum and tied to a hori­
zontal bar to which a 3-lb force is applied. As rolls from
rest, tell (a) the number of inches of rope wound or un­
wound (tell which) in three seconds and (b) the minimum
friction coefficient needed for the rolling to take place.
4.80 Find the range of possible values of the couple M 0
Figure P4.83
for which the cylinder in Figure P4.80 will not slip in
either direction when released from rest on the incline.
The mass is 15 kg; the radius is 0.2 m; and the coefficient
of friction is = 0 . 2 . 4.82 A hula hoop (mass m, radius r) is thrown forward
4-. 81 An airplane lands on a level strip at 200 mph. (See with backspin; = to the right and = counter­
Figure P4.81.) Initially, just before the wheels touch the clockwise. (See Figure P4.82.)
runway, the wheels are not turning. After they touch the
a. How long and how far does the mass center
runway they will skid for some distance and then roll free.
move before stops slipping?
If during this skidding the plane has a constant velocity of
200 mph and the normal force between the wheel and b. Find the relationship between and such
the runway is 10 times the wheel weight, find the length that when stops slipping: (i) it rolls right; (ii)
of the skid mark. (The coefficient of friction is the radius it rolls left; (iii) it stops.
of gyration of the wheel is three-fourths of its radius.) 4.83 The strong, flexible cable shown in Figure P4.83
is wrapped around a light hub attached to the 130-lb cyl­
inder Find the angular acceleration of upon release
from rest. Note that it is impossible for the wheel to roll
down the plane (meaning without slipping); to do so
the cord would have to break.
4.84 Repeat the previous problem for = 0.25.
4.85 In Figure P4.85, find how far down the incline C
Figure P4.8G travels in 5 s if the 20-kg cylinder is released from rest.
Airplane
v = 200 mph

Wheel µ= 0.45
R = 0.5 m

o
60

Figure P4.81 Figure P4.85


Page 262

4.86 The 50-lb body in Figure P4.86 may be treated as 21b S(32.21b)
a solid cylinder of radius 2 ft. The coefficient of friction
1 ft
between and the plane is = 0 . 2 , and a force P = 10 lb
2 ft
is applied vertically to a cord wrapped around the hub.
Find the position of the center C, 10 sec after starting from
6 4 . 4 lb
rest.
3 1b
4.87 Given that the slot (for the cord) in the cylinder in
Figure P4.87 (mass 10 kg) has a negligible effect on I , c

find:
a. The largest for which no motion down the
plane will occur 100 ft
b. The time required for C to move 3 m down the
incline if = 6 0 ° . Figure P4.88

1 lb (each of 8) Thin rim (3 lb)

Figure P4.86 Hub (2 lb)

cord

Figure P4.89
0.5 m

0.8 m

4.90 The wheel shown in Figure P4.90 has a mass of


10 kg, a radius of 0.4 m and a radius of gyration with
Figure P4.87
respect to the z-axis through C of 0.3 m. Determine the
angular acceleration of the wheel and how far the mass
center C moves in 3 seconds if the wheel starts from rest.
4.88 A light 100-ft cord is wrapped around the 32.2-lb
spool which is pinned at C to the cart (see 40 N
Figure P4.88). The radius of gyration of with respect
to an axis normal to the figure at C is 1.3 ft. The cart (with­
out has weight 64.4 lb. The wheels of are small and 60 N
light, so that friction beneath them is negligible. The 2- 4m
and 3-lb forces are applied to the system at rest. If upon µ= 0.5
complete unwrapping the cord is to end up between
points P and Q in the lower figure, where should be
Figure P4.90
originally parked along PQ?
4.89 A child pulls on an old wheel with a force of 5 lb by
means of a rope looped through the hub of the wheel.
(See Figure P4.89.) The friction coefficient between wheel
and ground is = 0 . 2 . Find I for the wheel, and use it to
c 4.91 Rework the preceding problem if the friction coef­
determine the location of C after 3 sec. ficient is changed to = 0 . 2 .
Page 263

4.92 Two cables are wrapped around the hub of the (64.4 lb)
10-kg spool shown in Figure P4.92, which has a radius of
gyration of 500 mm with respect to its axis. A constant (96.6 lb) 10 lb
40-N force is applied to the upper cable as shown. Find 1 ft
the mass center location 5 s after starting from rest if: (a) Cord
= 0.2; (b) = 0.5. 3 ft

Cable
600 m m
Figure P4.94
40 N

Cable

200 mm
4.95 Assume that enough friction is available to prevent
the cylinder in Figure P4.95 from slipping.

Figure P4.92
a. Show that
(i) rolls to the right if (r/R).
(ii) rolls to the left if (r/R).
(iii) is in equilibrium if (r/R) (and
4.93 A sphere of radius ft and weight 16.1 lb is pro­ will translate if P increases enough to over­
jected onto a horizontal plane (Figure P4.93). Its center come friction).
has initial velocity at t = 0 and the sphere has initial b. Find and a if r = 0.2 m, R = 0.4 m,
angular velocity defined as shown. If the coefficient P = 20 N, mg = 40 N, and 6 = 45°.
of sliding friction between the sphere and the plane is
0.15, plot graphs of distance gone (x ) against time t up to
c
Cord
t = 3 sec for the following cases:
a. = 10 ft/sec; = 100 rad/sec
b. = 1 0 ft/sec; = 50 rad/sec
c. = 1 0 ft/sec; = 30 rad/sec
Figure P4.95

A homogeneous spool of weight W rolls on


an inclined plane; a string tension of amount 4W acts
up the plane as shown in Figure P4.96. With I given c
2
approximately by WR /2g, find the acceleration of C.
Assume unlimited friction.
Figure P4.93

4.94 The cylinder in Figure P4.94 has a thin slot cut in


it which doesn't affect its moment of inertia appreciably.
A cord is wrapped in the slot and connects to the cart
which rests on the plane on small, light wheels. The force
of 10 lb is applied to with the system initially at rest.
Find the length of unwrapped cord after 4 seconds 30 c

elapse. Assume enough friction to prevent slip of on the


plane. Figure P4.96
Page 264

4.97 Pulley in Figure P4.97 weighs 100 pounds and 4.99 Wheel is made up of the solid disk rim and
has a radius of gyration about the z-axis through O of four spokes Masses and radii are given in Figure P4.99
k = 7 in. Pulley weighs 20 lb and has k = 3 in. Find
0 c and the table.
the angular acceleration of just after the system is re­
a. Compute for the wheel.
leased from rest. Assume the rope doesn't slip on but
that there is no friction between and the rope. Is this the b. The coefficient of friction between and the
angular acceleration for later times as well? plane is = 0.3. If a cord is wrapped around
the disk and connected to the 50-kg body de­
termine the acceleration of the mass center C
8 in. of
1
4 in.
Part M a s s (kg)

20
5 (each)
(100 lb) 10
Rope

(20 lb)
Cord

2 in. radius
0.2 m

(50 lb) 50 kg

Figure P4.99
Figure P4.97

4.98 The 32.2-lb body in Figure P4.98 is a spool hav­


ing a radius of gyration k = 6 in. about its axis. Cords are
c

wrapped around the peripheries; one is connected to a


ceiling, the others to the 48.3-lb block Find the acceler­ The radius of gyration of the 20-kg wheel in
ations of the centers C (of and B (of Figure P4.100 with respect to its axis is 0.3 m. Motion
starts from rest. Find the acceleration of the mass center
C, and determine how far C moves in 5 s.

4in 20°

8 in. 0.5 m

I50N

0.15

Figure P4.98 Figure P4.100


Page 2 6 5

4.101 A string is wrapped around the hub of the spool 0.4 m


1m
shown in Figure P4.101. There are four indicated string
directions. For the direction that will result in the largest 0.4 m

displacement of C in 3 s, find this displacement. Assume 10 kg; k c = 1 m


0.2 m
sufficient friction to prevent slipping. The spool has a
mass of 12 kg and a radius of gyration about z of 0.6 m.
c

Each force equals 10 N, and the spool starts from rest.


Cord

Disk, 7 kg
0.5 m
1m

5 Kg

Figure P4.103
Figure P4.101

4.104 Disks and each weigh 64.4 lb and are rigidly


4.102 Cylinder in Figure P4.102 has a mass of 4 slugs, attached to the light shaft that joins their centers. (See
and the effect of the hub on its moment of inertia is negli­ Figure P4.104.) A 96.6-lb cylinder has a hole drilled
gible. It is connected by means of a cord to the 1-slug along its axis, through which passes. A force of 20 lb is
block The mass of the pulley is negligible. The coeffi­ applied horizontally to an inextensible string wrapped
cient of friction between and the plane is = 0.5, and around If friction is negligible between and and if
the radii of are given in the figure. If the system is and roll on the plane, find:
released from rest, determine the time that will elapse
a. The angular acceleration of the cylinder
before hits the ground.
b. The angular acceleration of the disks
c. The minimum coefficient of friction between
disks and plane for no slipping.

1 ft
2 ft

Cord 2 ft

1 ft

2 0 ft

Figure P4.104
Figure P4.102

4.103 Find how long it takes for to roll off the plane in 4.105 Rework the preceding problem, but this time as­
Figure P4.103, assuming sufficient friction to prevent sume that the string is wrapped so that it comes off the
slipping. The system is released from rest. bottom of
Page 266

4.106 The two wheels are identical 16.1-lb cylinders with W r a p p e d cord
smooth axles at their centers. (See Figure P4.106.) The
carriage weighs 32.2 lb and has its mass center at C. The
cylinders do not slip on the inclined plane. Find the accel­
eration of point Q.

Figure P4.109

4.110 The semicylinder in Figure P4.110(a) is released


0
from rest, and there is enough friction to prevent slipping
30
throughout the ensuing motion (Figure P4.110(b)).
Figure P4.106
a. Find
b. Write the three differential equations of motion
of the body (good at any angle
4.107 Cylinder inFigure P4.107 weighs 100 lb; it is roll­
c. Find the two equations relating and to
ing on the plane and is pinned at its center C to the 10-lb
and
rod If is initially 10 ft/sec to the left, and if the
coefficient of kinetic friction between the plane and each d. Eliminate N, and and obtain the single
body is = 0.4, determine how long it will take the sys­ differential equation in the variable (t). Note
tem to come to rest. the complexity of the equation!

Mass m

Figure P4.107

4.108 A uniform half-cylinder of radius r and mass m is


held in the position shown in Figure P4.108 by the string
tied to B. Find the reaction of the floor just after the string (a) (b)
is cut. There is sufficient friction to prevent slipping. Figure P4.110

4.111 The 15-lb carriage shown in Figure P4.111 is sup­


A ported by two uniform rollers each of weight 10 lb and
radius 3 in. The rollers roll on the ground and on the
carriage. Determine the acceleration of the carriage when
Figure P4.108
the 5-lb force is applied to it.

4.109 In Figure P4.109 the force P is applied to the cord at


t = 0, when the 25-N cylinder is at rest. Find the position
of the mass center when t = 6 s. Figure P4.111
Page 267

4.112 The 128.8-lb homogeneous plank shown in Fig­ 4.116 The pipe in Figure P4.116 has a mass of 500 kg and
ure P4.112 is placed on two homogeneous cylindrical rests on the flatbed of the truck. The coefficient of friction
rollers, each of weight 32.2 lb. The system is released between the pipe and truck bed is = 0 . 4 . The truck
from rest. Determine the initial acceleration of the plank if starts from rest with a constant acceleration a . 0

no slipping occurs. Is this the acceleration for later times


a. How large can a be without the pipe slipping
0
as well?
at any time?
b. For the value of a in part (a), how far has the
0

truck moved when the pipe rolls off the back?


10 ft

2 m

0
30

Figure P4.112

Figure P4.116

4.117 The uniform sphere (mass = 1 slug, radius = 1 ft)


Figure P4.113 and the slab (mass = 2 slugs) shown in Figure P4.117 are
at rest before the force P = 24 lb is suddenly applied to
the slab. The coefficient of friction is 0.2 between the
4.113 Body in Figure P4.113 is a rigid plate of mass M, sphere and slab and between the slab and horizontal
resting on a number n of cylinders each of mass m and plane, (a) Does the sphere slip on the slab? (b) What is the
radius R. Force F is constant and starts the system moving acceleration of the center of the sphere?
from the position shown. If there is no slipping at any
surface, find: (a) the acceleration of the plate and (b) its
position x as a function of M, m, F, n, and time t.
c

4.114 A 6-ft gymnast makes a somersault dive into a net


by standing stiff and erect on the edge of a platform and
allowing himself to overbalance. He loses foothold (with­ Figure P4.117
out having slipped) when the platform's reaction on his
feet becomes zero; he preserves his rigidity during his fall.
Show that he falls flat on his back if the drop from the 4.118 The homogeneous cylinder in Figure P4.118
platform to the net is about 43 ft. weighs 64.4 lb. The acceleration of the 96.6-lb cart is
2

The homogeneous cylinder in Figure P4.115 is at 10 ft/sec to the right.


rest on the conveyor belt when the latter is started up with a. Determine the acceleration of the center C of
2
a constant acceleration of 3 ft/sec to the right. If the the cylinder and the friction force exerted on
cylinder rolls on the belt, find the elapsed time when the by if there is sufficient friction to prevent
cylinder reaches the end A . slipping.
b. How large does the friction coefficient have to
10 ft
be for this to occur?
2.5 ft
1 ft

Figure P4.115 Figure P4.118


Page 268

4.119 The homogeneous cylinder . in Figure P4.119 4.123 After release from a slightly displaced position, the
weighs 64.4 lb and rolls on the 96.6-lb truck The mass rod in Figure P4.123 will remain in contact with the floor
of the truck rollers may be neglected. Find the force P throughout its fall. Describe the path of C and find the
such that C does not move relative to the plane. reaction onto the floor just before the rod becomes hori­
zontal.
6 4 . 4 lb 4.124 The uniform slender bar of mass m and length L is
released from rest in the position shown in Figure P4.124.
Find the force exerted by the smooth floor at this instant.

Rod

0
30

Figure P4.119

Smooth plane
Figure P4.123 Figure P4.124

Small wheels
Figure P4.120 4.125 A thin rod AB of length and mass m is released
from rest in the position shown in Figure P4.125. Point A
of the rod is in contact with a surface whose coefficient of
4.120 The system shown in Figure P4.120 is initially at friction is
rest. A force P is then applied that varies with time ac­
a. Determine the minimum value of say
2
cording to P = 7f , where P is in newtons and t in seconds.
, required of
If the coefficient to prevent
friction end A fromcylinder and cart is
between
= 0.5, find how much time elapses before the cylinder slipping upon release.
starts to slip on the cart. b. Find the acceleration of the mass center of the
rod immediately after release for and
In the previous problem, determine how much time for
passes (from t = 0) before the cylinder leaves the surface
of the cart. Initially, the center of the cylinder is 2 m from B
the right end of the cart.
4.122 A slender homogeneous bar weighing 193 lb has 0
30
an angular velocity of 2 rad/sec clockwise and an angu­ A
2
lar acceleration of 8 r a d / s e c clockwise when in the posi­
tion shown in Figure P4.122. The wall at B is smooth; the Figure P4.125
coefficient of sliding friction at A is 0.10. Find the reac­
tions at A and B on in this position. Hint: The force P can 13m
be found. U6m

Figure P4.126

4 ft 4.126 The 30-kg sphere and 15-kg rod in Figure P4.126


are welded together to form a single rigid body. Deter­
mine the angular acceleration of the body immediately
after the right-hand string is cut.

2 ft
4.127 If the right-hand string in Figure P4.127 is cut, find
1 ft the initial tension in the left string. The slender rod has
Figure P4.122 mass m and length L.
Page 269

4.131 The uniform slender bar of mass m is released from


rest in the position shown in Figure P4.131. Find a and A

the tension in the inextensible cord at the instant after


release.

Figure P4.127

4.128 Repeat the preceding problem if the rod is replaced


by a rectangular plate suspended from the two upper
comers. The width (between strings) is B and the height
is H. s
30
4.129 A uniform slender rod, 10 ft long and weigh­
ing 90 lb, is supported by wires attached to its ends. (See Figure P4.131
Figure P4.129.) Find the tension in the right wire just
after the left wire is cut. Assume the wires to be inexten-
sible.
4.130 The left end of a slender uniform bar is attached to a
light inextensible cable as shown in Figure P4.130. If the
bar has mass m and length L and is released from rest in 4.132 The uniform 20-lb bar is three feet long and has an
the position shown, find the angular acceleration of the angular velocity = 3 rad/sec with v = 0 at the in­
c

bar at the instant after release. stant shown in Figure P4.132. Neglecting interaction
with the air, what is the angular velocity of the bar after its
center has dropped 10 feet?

0
30

Figure P4.132
Figure P4.129

4.133 The disk shown in Figure P4.133 has mass m and


radius r. Show that at the instant the right-hand string is
cut, the tension in the other string changes to mg, so that
the acceleration of the mass center is
0
45

300 30

Figure P4.130 Figure P4.133


Page 270

4.134 A uniform rod is supported by two cords as 4.142 A slender bar weighing 64.4 lb is attached by
shown in Figure P4.134. If the right-hand cord suddenly massless cables to a fixed pivot A as shown in Fig­
breaks, determine the initial tension in the left cord AD. ure P4.142. The system is swinging about A as a pendu­
("Initial" means before the rod has had time to move and lum. At = 0 the angular velocity is 2 rad / sec and
before it has had time to generate any velocities.) cable AD breaks. Find the tension in cable AB just after the
break.
4.135-4.140 The six equilateral triangular plates (Fig­
ures P4.135-4.140) are each supported on their right­ 4.143 A beam of length L and weight W per unit length is
most comer B by a string; each has a different support supported by two cables at A and B. (See Figure P4.143.)
condition at the left comer A. At the instant when If the cable at B should break, find the shear force V and
the string at B is cut, find a and a in each case. The
c bending moment M at section xx just after the cable
length of each side is s. breaks. Hint: Euler's laws apply to every part of the body.
4.141 The uniform 10-lb bar in Figure P4.141 is sus­ 4.144 See Figure P4.144. Assuming that sufficient fric­
pended by two inextensible cables. At the instant shown, tion is present to prevent slipping between and the
when each point in the bar has a velocity of 10 ft/sec, plane, find the angular accelerations of and just after
the right cable breaks. Find the force in the left cable force P is applied to the bodies at rest. They are connected
immediately after the break. by a smooth pin.

0
30
13 ft

64.4 lb
Figure P4.134

Figure P4.142
String Spring A Moment
spring
A B A B B
c c C

Figure P4.135 Figure P4.136 Figure P4.137

A
A B
A
A B B B
C c C
Figure P4.143

Figure P4.138 Figure P4.140


Counterweight
Cylinder,
(pulley is light)
mass = 5 kg
Figure P4.139

0.3 m Slender bar,


2 ft 2 ft
mass = 2 kg

Starts here at t = 0
4ft at rest
Figure P4.141 Figure P4.144
Page 271

4.145 Rods and each have mass m. (See Fig­ Determine the angular acceleration of if is fixed in the
ure P4.145.) Upon release from rest in the horizontal po­ inertial frame of reference. Treat the gears as uniform
sition indicated, find the reactions at O, and at A, disks. The plane of the page is horizontal.
onto
Mass M
Pin

A B

Figure P4.145 60 0 (mass m)

D Figure P4.147
A
Figure P4.146

4.146 Two uniform bars and are released from rest in Figure P4.148
the position shown in Figure P4.146. Each bar is 2 ft long
and weighs 10 lb. Determine the angular acceleration of
each bar and the reactions at A and D immediately after 4.148 Cylinder in Figure P4.148 rolls down a wedge
release. The rollers are light and the pins smooth. that can slide without friction on a smooth floor. Show
that the acceleration of wedge is a constant given by the
4.147 A constant torque T is applied to the crank arm
0
equation
of the planetary mechanism shown in Figure P4.147. The
axes of the identical gears and are vertical, and the
ends of the crank are pinned to the centers of and

EXTENDED PROBLEMS

4.149 The stick of mass m, shown in Figure P4.149, origi­ (1)


nally at rest with = 0, is disturbed slightly and begins to
slide on a smooth wall and floor. Derive the differential (2)
equation of motion of the stick. Integrate the equation
and find the angle 6 at which contact with the wall is lost. (3)

Then note that there are more unknowns than equa­


tions. Use kinematics to relate a to a , and use the
c B

component" of that equation to obtain a fourth equa­


tion, containing . Then relate a to a and get a fifth
c T

equation, containing . From your five equations, elim­


inate N , and N ,obtaining the following differen­
T B

tial equation governing

(4)

Multiply (4) by 6 and integrate, using /dr. Use


the initial condition = 0 at = 0 to evaluate the con­
Figure P4.149 stant of integration. You will now know as a function of
. Then, with N expressed as a function of , set N = 0
T T

in (1) to find the angle where contact is lost. Your answer


Hints: First verify the following equations of motion: should be
Page 272

Disks and each weigh 64.4 lb and are rigidly


attached to the light shaft that joins their centers. (See
Figure P4.150.) A 96.6-lb cylinder has a hole drilled
along its axis, through which passes. Let represent 2 ft

the rigid body comprised of and


While the body is held fixed on the plane, the 1 ft
cylinder is spun up to an angular velocity 8 rad / sec,
and the system is then released. Assume that part of the
reaction between the axle and the wall of the cylindrical
hole in the cylinder is a friction couple proportional to the
difference in angular velocities, with proportionality con­
stant k. The friction couple acting on the axle will cause
to roll to the right; the opposite couple on will slow its
angular speed down. As time passes, the bodies and Figure P4.150
will approach the condition of moving as one. Show this,
and find the common, limiting-case "terminal" angular
velocity shared by and . There is sufficient friction
between and the ground to prevent slipping there.

4.6 Other Useful Forms of the Moment Equation


For a rigid b o d y in plane motion, there are several other forms of t h e
m o m e n t equation of m o t i o n [besides t h e translation equation (4.1) a n d
w h i c h are w o r t h y of special study. T h ei d e a is that it is often
convenient a n d helpful to s u m the m o m e n t s about a point other t h a n the
m a s s center C. We will study three of these forms o n e - b y - o n e a n d present
examples of each as w e go along.

Moment Equation in T e r m s of a c

To develop this form, w e begin with Equation (2.45):

(2.45)
w h e r e w e recall that in this form there is no restriction at all on t h e location
of point P, t h e type of b o d y being studied, or t h e type of motion. Thus,
specializing for a rigid b o d y in plane motion a n d using the r i g h t - h a n d
side of Equation (4.13) to replace H for this case,
c

(4.17)

W h e n e v e r the products of inertia in Equation (4.17) vanish, this equation


takes the particularly simple form

(4.18)
N o t e that t h e translation Equation (4.1) results from Equation (4.18) if
a = 0.
Note further that if P a n d C are in t h e same (reference) p l a n e of
motion, t h e n is perpendicular to t h e p l a n e of motion — that is,
Page 273

it is parallel to k. W h e n this is the case, w e will rewrite Equation (4.18) in


scalar form*:

(4.19)

in w h i c h ( ) m e a n s the coefficient of the unit vector


z within the
parenthesis.
W e n o w present t w o examples of t h e use of Equation (4.18). In the
first one, w e eliminate reactions at a pin b y s u m m i n g m o m e n t s there:

EXAMPLE 4 . 2 2
A C The uniform rod in Figure E4.22a (length 80 cm, mass 20 kg) is smoothly
pinned to cart (50 kg) at point A. Force P, applied to with the system initially
P at rest, causes to translate with the acceleration . Find the initial
angular acceleration of the rod.
Solution
By using Equation (4.19), we can sum moments about A of the forces on and
R
avoid having to use in this case. We obtain, using the free-body in
Figure E4.22a
Figure E4.22b,
(1)
2 2

A
We note that I = c /12 = 20(0.8) /12 = 1.07 kg • m , and we get a from c
y

kinematics:
A
Ax

C
20(9.81) N
so that, substituting into (1),
Figure E4.22b

Thus the rod starts off with

Question 4.8 Why would the solution to the preceding example be


much more complicated using the mass center form of the moment
Equation (4.16)?

Next w e use Equation (4.19) to rework a n example from t h e previous


section.

* When the products of inertia and both vanish, then by Equations 4.14a,b),
. This leads, when P and C are both in the reference plane, to
= 0= since X M a l w a y s equals
P and .
Answer 4.8 T h e p i n reaction A w o u l d a p p e a r in
x . T h u s w e w o u l d n e e d to also
write Equation (4.15a) to eliminate A . x
Page 274

EXAMPLE 4 . 2 3

Find the starting angular acceleration for the body of Example 4.19, shown again
in Figure E4.23a.
Solution
We sum moments about point with the help of Equation (4.19) and the
free-body diagram in Figure E4.23b:

Figure E4.23a

From the kinematics in the earlier example, we know


Substituting these into the above equation after cancelling "m".

Therefore,

Figure E4.23b
as before.
Note that the equations of motion, (4.15a,b), were not needed in
this example. They would have been in Example 4.19, however, even if all we
had been seeking was a.

Question 4.9 Why?

Moment Equation in T e r m s of a P

Another form of the moment equation of motion that is sometimes useful


involves the acceleration and inertia properties at some point other than
the mass center. Recalling that if P is a point of a rigid body in plane
motion, then by Equation (4.3),

(4.3)

And for any point P, w e know from Equation (2.36) that:

(2.36)

Equating these two expressions for H P and then differentiating with


respect to time,

Answer 4.9 w o u l d h a v e i n c l u d e d m o m e n t s of b o t h a n d N.
Page 275

(4.20)

By Equations (2.8) a n d (2.43), w e k n o w that m a c

Using these results a n d Equation (2.42), w e m a y replace b y t h e first


t w o terms on t h e left-hand side of this equation a n d obtain

(4.21)

On the right side of Equation (4.21) the first term vanishes since
and the third term is of the same form as except that here the
inertia properties are with respect to axes with origin at P. T h u s retracing
t h e steps b e t w e e n Equations (4.11) a n d (4.13), w e obtain

(4.22)

Question 4.10 Why can we say, as was done above, that


and why does that cause the first term on the right side of
Equation (4.16) to vanish?

W h e n the products of inertia and vanish, Equation (4.22) sim­


plifies to

(4.23)
As in Section 4.5, w h e n P a n d C are in t h e s a m e (reference) p l a n e t h e a n d
components of this equation vanish. T h u s t h e scalar form of t h e equa­
tion is

(4.24)
w h e r e ( ) again m e a n s t h e coefficient of within t h e parentheses.
z

This equation is very useful if w e h a p p e n to k n o w the acceleration of


a point other t h a n the mass center, as in the following examples:

Answer 4.10 Let and be position vectors for P and C; differentiating


gives The cross product of parallel vectors always vanishes.

EXAMPLE 4 . 2 4

Solve the problem of Example 4.22 by using Equation 4.24.


Solution
Equation (4.24) makes a problem such as this even simpler than did Equa­
tion (4.19):
Page 276

To determine other information in the preceding example, for in­


stance the starting value of force P, we can write the x-component of the
mass center equation of motion for the cart and then for the bar (see
Figure 4.7):

50 k g

20 k g ,
80 c m

Figure 4.7

For

For

where

or
Page 277

Thus

and

EXAMPLE 4 . 2 5
Repeat Examples 4.19,23, using Equation (4.24) this time to find the starting
angular acceleration.
Solution
We again sum moments about point

0 initially

once again. Notice that the cross-product term is simpler this time, but the mo­
ment of inertia at has to be calculated by the parallel axis theorem. In Prob­
lems 4.153,154, this problem is to be reworked one last time using the geometric
center of the hoop as point P in Equations 4.19 and 4.24, respectively.

In the preceding example, it accidentally happened that


This is not generally true. For example, an instant later co would not be zero
and the cross-product term would not be zero.

Moment Equation for Fixed-Axis Rotation (The "Pivot" Equation)


Consider n o w the case in which the acceleration of a point P of the rigid
body in plane motion is identically zero. Equation (4.22) tells us that for
such a point,
(4.25)
If w e take the dot product of this equation with w e see that whether or
not the products of inertia and vanish, w e always have:
(4.26)
Page 278

W h e n t h e point P is fixed in t h e inertial frame as well as in t h e body, w e


will t h e n usually label the point as O for emphasis. In this case, t h e only
w a y t h e b o d y can m o v e is to rotate about t h e z-axis t h r o u g h O, a motion
w h i c h w e call fixed-axis rotation. The point O, w h i c h does n o t m o v e
during t h e motion of interest, is called a pivot, a n d w e abbreviate Equa­
tion (4.26) as

(4.27)

or as

(4.28)

W h e n the products of inertia vanish at a pivot O, as they will for all


problems in this section, t h e n of course can be further abbreviated
to simply for t h e n only t h e z-component of is non-zero. This is
not always the case, as w e shall later see in the final section 4.7 of this
chapter. W h e n it is, h o w e v e r , w e simply write

(4.29)

Question 4.11 Since Equation (4.4) applies for any point P having
zero velocity, why can we not use equations such as (4.29) for the
instantaneous center of B when is not a pivot?

Because of t h e importance of fixed-axis rotation in engineering, w e


present m o r e examples of it t h a n w e did for t h e earlier forms in this
section. In t h e first example, w e examine a composite b o d y rotating about
a pivot:

Answer 4.11 A l t h o u g h H m a y a l w a y s b e w r i t t e n as
0 whenever v D

is zero, its derivative is only equal to w h e n v is identically z e r o — in o t h e r w o r d s ,


D

zero all the time.

EXAMPLE 4 . 2 6

The rod and sphere in Figure E4.26a are welded together to form a combined
rigid body which is attached to the ground at O by means of a smooth pin. Find
the force exerted by the pin onto the body, upon release of the system from rest.

Figure E4.26a
Page 2 7 9

Solution
Because O is a pivot of the combined body, we use Equation 4.29:

Figure E4.26b

so that, at release, using the FBD in Figure E4.26b,

or

To calculate the pin reaction we shall use 2F = m a . We first locate the mass
c

center, C, of the body. The distance from O to C is

At this instant we have

Thus

So

An alternate approach to the mass center calculation in the preceding


example would not require that w e explicitly locate C, because

or

from which
Page 2 8 0

and

as above.
In our second example, w e feature distinct bodies connected by an
unwinding rope. One body has a pivot and the others don't.

EXAMPLE 4 . 2 7

A rope is wrapped around the 10 -lb cylinder as indicated in Figure E4.2 7a. The
rope passes through a hole in the 5-lb annular disk and is then tied to the 15-lb
block When the system is let go from rest with the rope just taut, what is the
Rope reaction exerted on by
Solution
Using the free-body diagrams in Figures E4.27b, 4.27c, and 4.27d, we write the
following equations of motion of the respective bodies. For using the "pivot
equation" (4.29):

Figure E4.27a

which gives us the tension T in terms of

(1)
For the block by itself,
Figure E4.27b

(2)

where R is the force exerted by onto Now, on the disk

(3)

Figure E4.27c
At this point we have three equations in the five unknowns and
One constraint is that the vertical component of a (see Figure E4.27b) is the
0

same as the acceleration of the points of the straight portion of rope, and these
accelerations are each

(4)
Figure E4.27d
Also, the accelerations of and are equal. Without any rope tension, they
would each be with this tension, the acceleration of is slowed, guaranteeing
continuing contact of the two bodies. Therefore:

(5)
Page 281

Adding Equations (1) and (2) and using (4),

or
(6)
Equations (5) and (3) give:

(7)
Adding Equations (6) and (7),

so that, by (6),
R = - 15 + 0.621(25.8) = 1.02 lb
2
Note that the acceleration of is less than "g" (32.2 ft/sec ), as it must be,
and that R is less than the weight of thereby allowing it to fall, but not freely.

In our third example, w e again have two bodies, but this time both
have pivots and one is in fact in equilibrium. The contact between these
bodies involves sliding friction:

EXAMPLE 4 . 2 8

Just after the brake arm in Figure E4.28a contacts the top of the cylinder the
0.6 m
cylinder is turning at 1000 rpm The coefficient of kinetic friction between
0.2 m
and is . Find (a) how long it takes for to come to rest under the
constant force P = 40 N; and (b) the pin reactions exerted on to at the pin O.
Solution
Brake arm
Since body is in equilibrium, we may find the normal force between it and the
cross section
is 2 cm X 2 cm; cylinder by statics. The bar's weight is proportioned between its horizontal and
mass = 2 kg Cylinder vertical parts as shown in Figure E4.28b. Note that equilibrium requires that since slipping is taking plac
mass = 10 kg;
radius = 0 2 m
Figure E4.28a

0.1 m
0.3 m 0.2 m
0.02 m
40 newtons

newtons

newtons
Figure E4.28b
Page 282

Summing moments about we have


0.401N + 0.2(0.3N) - 40(0.401) - 1.5(9.81)(0.301) = 0

from which we get

newtons

and
newtons

Question 4.12 Would the solution for the normal force N be any
different if were counterclockwise?

The motion of body is one of pure rotation. Its free-body diagram is shown
in Figure E4.28c. We use for a since we are ultimately interested in equating to
zero.

Figure E4.28c
Integrating, we get

where the initial condition on allows us to calculate the integration constant C . 1

Body stops when at a time that we are now in a position to


calculate:

Note that the mass center O = C of is fixed in the inertial frame, so that the pin
reactions follow from the mass center equations:

newtons
newtons

In the fourth example, we are concerned with internal forces and with the
fact that the equations of motion can be applied to a part of a body,
considered as a body in itself:

Answer 4.12 Yes, for then the friction force would be in the opposite direction and'.
Page 283

EXAMPLE 4 . 2 9

The slender bar, pinned smoothly at O, is released from rest in the position shown
in Figure E4.29a. After a rotation of its angular speed is
Find at that instant the axial force, shear force, and bending moment in the bar at
Figure E4.29a point A, which is one-fourth the length of the bar from its free end.
Solution
First, we find the angular acceleration, making use of the FBD in Figure E4.29b:
0
45

Figure E4.29b
Next we expose the desired forces and moment by drawing a free-body diagram
(Figure E4.29c) of the lower fourth of the bar, and writing the equations of
motion for just that body ( C is its mass center):

so that

Figure E4.29c where we note that and are the same for this "sub-body" as they were for the
whole bar, and that O is a pivot of the sub-body extended. The other mass center
equation is:

Finally, the "moment equation of motion," written for the sub-body this time, is

The final three results are summarized pictorially on the cut section in Fig­
ure E4.29d.
Figure E4.29d
Page 8 4

We n o w examine ot problem in which the mass center of the body is


offset from the fixed axis of rotation:

EXAMPLE 4 . 3 0

How must the applied couple C in Figure E4.30a vary with time in order to turn
the unbalanced (but round) wheel at constant angular velocity ? What is
the initial angular acceleration if the couple is absent and the wheel is released in
the position shown in the figure? The moment of inertia of the mass of with
respect to its axis of rotation is , and the mass center of is located at G.
Mass m
Solution
Free-body diagrams of and are shown in Figure E4.30b. Since for
Mass M body we have:

Mass center = G
(1)
Figure E4.30a
And for the translating block we may write*

(2)
The point Q of located where the rope leaves the rim has the same velocity
and tangential acceleration component as does the rope itself at that
point. Since the rope is assumed inextensible, this acceleration has the same
magnitude as Therefore our kinematics gives us the following additional
equation for the translating block:

(3)
Figure E4.30b
Substituting (3) into (2) gives

And substituting T into the moment equation (1) for then yields

Since constant, we have and that

and the required couple varies harmonically.


If there is no couple C and the system is released from rest in the position
shown, then the equations are still valid and, with couple C = 0,

The initial angular acceleration of (with ) is thus seen to be

which is positive if M > 2m.

• N o t e t h a t since translates, t h e acceleration of its m a s s center is


Page 285

Question 4.13 What happens if M < 2m? If M = 2m?

In t h e preceding example, it is interesting to write the equations of


m o t i o n in t h e following form for t h e wheel:

(4)
(5)

(6)

Equations (4) a n d (5) are useful if t h e pin reactions are desired,* but
Equation (6) is n o w h e r e near as h a n d y to use as w h i c h we
have u s e d earlier in the example since the b o d y h a s a pin. The student
m a y wish to eliminate O , O a n d T from (6) b y using (4), (5), a n d the
x y

previous equation for a n d to s h o w that t h e s a m e result


is obtained (after a good deal m o r e work t h a n in the example) for
(Kinematics m u s t also b e u s e d to relate and to and )
In our last " p i v o t - e q u a t i o n " example, w e take u p a m u c h longer
problem, involving t w o bodies neither of w h i c h is translating. O n l y one
of t h e bodies h a s a pivot, a n d so w e shall h a v e to use b o t h the pivot
equation (4.29) and the m a s s center form of the m o m e n t equation (4.16)
before finally getting t h e problem solved:

Answer 4 . 1 3 If M = 2 m , t h e n ( w h e n t h e couple is n o t present) T = mg a n d are


solutions to t h e p r o b l e m a n d t h e r e is no morion. If M < 2 m , t h e block m o v e s downward
a n d is negative

0.5 m

EXAMPLE 4 . 3 1
The two uniform, slender rods and in Figure E4.31a, each of mass 2 kg, are
0.5 m pinned together at P, and then is suspended from a pin at O. (This arrangement
is called a double pendulum.) The counterclockwise couple C , having moment 0

150 N • m, is applied to beginning at t . Find the angular accelerations of


Figure E4.31a
and of upon application of the couple, and the force exerted on at P.
Solution
The equations of motion for and using the respective free-body diagrams in
Figures E4.31b and 4.31c, are:

(1)
(2)

(3)

* For instance, t h e p i n s m u s t b e designed strong e n o u g h to take t h e forces c a u s e d b y the


Figure E4.31 h accelerations.
Page 286

(Note in Equation (3) that O is a pivot of B .)


1

(4)
2(9.81) N (5)
150N • m
(6)

Figure E4.31 c Thus far we have six equations in the ten unknowns 0 ,O ,P ,P
I y x yl

, and . Kinematics gives the four additional equations we will need:

(7)
(8)
Also,

But

Thus
(9)
(10)
Solving these equations gives:

As a point of interest, the values of the other four variables (besides


2 2
are: O = - 1 2 9 N, O = 39.2 N,
x y m / s , and m/s .

Finally, w e remark that any of the alternative moment equations


discussed in the preceding two sections may be used. However, the
student is cautioned to realize that, just as in statics, once the vector
"force equation" and "moment equation" have been written, n o n e w
(independent) information will arise from summing moments at a differ­
ent point.
Page 287

PROBLEMS • SECTION 4 . 6

4.1 I I The rod is pinned to the light roller, which moves in 4.152 If in the preceding problem we replace the 6-N
the smooth slot, and with the system at rest as shown in force by a force F which produces a constant acceleration
2
Figure P4.151, the 6-N force is applied. Find the angular of pin P of 0.5 m / s , again find the initial value of a, this
acceleration of the rod at the given instant, by using time using only Equation (4.24). Explain why the answers
Equation (4.19) together with Then check are the same. (Hint: Solve for F at the given instant.)
your solution using only 4.153 Solve the problem of Examples 4.19, 23, 25 using
Equation (4.19), with the point P (about which moments
are taken) being the geometric center of the hoop.
4.1&4 Solve the problem of Examples 4.19, 23, 25 using
Equation (4.24), with the point P again (see the preceding
problem) being the geometric center of the hoop.
4.155 The cylinder shown in Figure P4.155 is made of
0.5 m two halves of different densities. The left half is steel,
3

48 kg
with mass density slug/ft ; the right half is
3
wood with slug/ft . Recalling that the mass
center of each half is located 4r/3n from the geometric
center find the acceleration of when the cylinder is
Figure P4.151 released from rest. Assume enough friction to prevent
slipping. Hint: Use Equation (4.17) with as point P.
4.156 (a) Use Equation (4.24) to categorize the restrictions
on point P for which we may correctly write
Show that there are only three cases, and that the mass
center form is one of them, while the "fixed-axis-of-
rotation form" is but a special case of one of the other two.
(b) Note that the instantaneous center of zero velocity
is not a point P for which, in general, (c) Fi­
Radius r - 1 ft
30 0
nally, determine in which of the problems in Fig­
ure P4.156 (a-e) it is true that
Figure P4.155

Geometric center of circle

Mass center
Cutout
No slip
N o slip (rolling)
(rolling)

Figure P4,156(b)
Figure P4,156(a)

N o slip
Slipping (rolling)

Figure P4,156(c) Figure P4,156(d) Figure P4,156(e)


Page 288

4.157 Let E be a rigid body in plane motion, in con­


stant contact with a surface Let Q be the point of in
contact with (Q can be different points of at differ­
ent times). Use the result of Problem 4.156 to prove
that if the following four conditions hold, then
at all times: (1) is fixed in an inertial frame;
(2) is rolling on (3) is round; and (4) the mass
center of is at its geometric center.
4.158 In the preceding problem, suppose that conditions
(1) and (2) hold, and that at a certain instant, of is
zero. Using the result of Problem 4.156, show that, at 30 cm
that instant, regardless of whether condi­
tions (3) or (4) hold.
4.159 The thin-walled hollow sphere of Figure P4.159
(moment of inertia about any diameter has
average radius r = 0.5 m and mass 50 kg. It is pinned at
Figure P4.160 Figure P4.161
the bottom of the cart. The force F applied to the cart at
rest produces a constant acceleration of all points of the
cart of 4.161 The cylinder in Figure P4.161 has mass 10 kg. The
a. Find the maximum value of a if the sphere is to 9-N force is applied to a string wrapped around a thin slot
translate and the breaking strength of each of near the surface of the cylinder. Find the angular acceler­
the cords is 100 N. ation of the cylinder, assuming enough friction to prevent
slip, using (a) Equation (4.19); (b) Equation (4.24). Ob­
b. Suppose a is twice the result of (a), so that one 2
serve how both right-hand-sides add up to mr a even
cord breaks at t = 0. Find the angular accelera­
though the individual terms are different.
tion of the sphere at the instant it has turned
through 90°, using Equation (4.24). Does the 4.162 In Problem 4.152 at a later time, let 6 be the angle
answer to (b) depend on the value of a? between the vertical and the rod (see Figure P4.162).
Using Equation (4.24), find the angular acceleration of
Cord 1 Cord 2 the rod as a function of

Figure P4.162

Figure P4.159
4.163 The crank arm OP is turned by the couple M at 0

constant angular velocity In the position


shown in Figure P4.163, determine the reaction of the
smooth plane onto the 20-kg slender bar PQ. Hints: First
solve for a and for a of PQ using kinematics, then use
4.160 The rectangular door of a railroad car has mass m P

Equation (4.24).
(Figure P4.160); it is of uniform width and has its
hinges on the side of the doorway closest to the engine. 4.164 A body weighing 805 lb with radius of gyration
Initially the door makes an angle with the train, which 0.8 ft about its z axis (see Figure P4.164) is pinned at its
c

begins to move forward from rest at constant acceleration mass center. A clockwise couple of magnitude e' lb-ft is
a . Find the initial resultant horizontal reaction compo­
0
applied to starting at t = 0. Find the angle through
nent that the hinges exert on the door. which has turned during the interval
Page 289

0.5 m

0.3 m
Figure P4.164

Figure P4.166 Figure P4.167


1.2 m

4.168 Figure P4.168 shows a scene from Edgar Allen


Poe's "The Pit and the Pendulum." Find the reaction of
12 ft the pin onto the bar if the pendulum is instantaneously at
rest in a horizontal position.

Figure P4.163 Semicircular disk 5 l b

Smooth pin
Figure P4.165

4.165 The slender homogeneous rod in Figure P4.165 is


12 ft long and weighs 5 lb; it is connected by a rusty hinge
to a support at A . Because of friction in the hinge, the
hinge exerts a couple of 9 lb-ft on the rod when it rotates.
If the rod is released from rest with , find: (a) the
angular acceleration of the rod when , 60" and
90°; (b) the angle at which the angular acceleration of
the rod is zero.
4.166 A wagon wheel spinning counterclockwise is
placed in a comer and contacts the wall and floor. (See
Figure P4.166.)
a. Show with a free-body diagram that the wheel
cannot climb the wall.
b. Show with a free-body diagram that the wheel
cannot move to the right along the floor either.
Figure P4.168
c. Therefore the wheel stays in the comer. Treat it
as a ring of mass m and radius R, with friction
coefficient at both surfaces of contact. Deter­
mine how long it takes for the wheel to stop, 4.169 The uniform slender bar of mass m is released from
and find how many radians it has turned rest in the position shown in Figure P4.169. Find the an­
through since first contacting the surfaces. gular acceleration when the bar has turned through 45 °
4.167 The cylinder in Figure P4.167 has a mass of 30 kg
and rotates about an axis normal to the clevis at O. At the
2
instant shown, and rad/s .
Find the force P that acts on the cylinder, and determine
the reactions exerted by the pin onto the clevis at O, all at
the given instant. Figure P4.169
Page 290

4.170 The uniform slender bar in Figure P4.170, of mass 4.173 Find the angular acceleration of cylinder in Fig­
m and length is released from rest at zero plus a tiny ure P4.173. The rope passes over it without slipping and
increment. Find the magnitude of the bearing reaction ties to and as shown.
when
4.174 Body is a pulley made of the cylinders and
which are butted together and rigidly attached. (See Fig­
ure P4.174.)The combined body is smoothly pinned to
the ground through its axis of symmetry (which passes
through its mass center). Ropes wrapped around and
are tied to bodies and respectively. If the system is
released from rest, what will be the angular acceleration
of
40 cm

20 cm
Figure P 4 . 1 7 0
50 N

4.171 Body is a slender bar bent into the shape of a 20 N


quarter-circle (Figure P4.171). Find the tensions in strings
OA and OB when the system is released from rest.

100 N 45 N
Figure P 4 . 1 7 4
(mass m)

4.175 The 32.2-Ib particle P rests on the 128.8-lb plank as


shown in Figure P4.175. If the cord at B suddenly breaks,
Figure P 4 . 1 7 1 find the initial acceleration of the particle, and the force
exerted on it by the bar.
4.172 Two weights W and W in Figure P4.172 are con­
1 2

nected by an inextensible cord that passes over a pulley.


Cord
The pulley weighs W and its mass is concentrated at the 32.21b 128.8 1b
rim of radius R. Show that if the system is released and the
cord does not slip on the pulley, the acceleration magni­
tude of W and W , is
1 2
2 ft

6 ft
Figure P 4 . 1 7 5

4.176 In Example 4.29, note that pivot point O is also a


point of the "subbody" used in that example. Thus for
that body, Separately compute and
(20 kg)
for the subbody and show that their values are the
same.
4.177 The uniform slender bar of weight W and length L
in Figure P4.177 is released from rest at and pivots
on its square end about comer O.
10 kg 30 kg a. If the bar is observed to slip at find the
Figure P 4 . 1 7 2 Figure P 4 . 1 7 3 coefficient of limiting static friction
Page 2 9 1

1 ft
4 in.

R C = 3 in

9 in

Figure P4.177

4 in
b. If the end of the bar is notched so that it cannot
slip, find the angle at which contact between
bar and comer ceases. Hint: Write the moment Figure P4.179
equation of motion about the pivot O, multiply
it by and integrate, obtaining as a function
of Use this relation together with the compo­
nent equation of in the direction. 4.180 The slender, homogeneous rod in Figure P4.180 is
supported by a cord at and a horizontal pin at B. The
4.178 The chain drive in Figure P4.178 may be consid­ cord is cut. Determine, at that instant, the location of pin B
ered as two disks with equal density and thickness. The that will result in the maximum initial angular accelera­
larger sprocket has a mass of 2 kg and a radius of 0.2 m. If tion of the rod.
the couple is applied starting from rest at t = 0, find the
angular speed of the smaller sprocket at t = 10 s. Hint:
What does a dentist look at?

Figure P4.180
2 kg

4.181 The uniform rod of mass m is released from rest in


Figure P4.178 the horizontal position indicated in Figure P4.181. Con­
sider the force exerted by the smooth pin.
4.179 Cylinder in Figure P4.179 with four cutouts is a. How does the magnitude of the force vary with
rotating at 200 rpm initially. A uniform 100-lb cylinder
the angle through which it has turned?
is placed in the position shown, and the friction produces
b. What is the maximum value of this magnitude?
a braking moment that will stop The friction coefficient
is and before the four holes were drilled the uni­
form body weighed 200 lb. For whichever rotation di­
rection of results in a quicker stop, find the stopping
time. Figure P4.181

4.7 Rotation of Unbalanced Bodies


W h e n a rigid b o d y is m o u n t e d in bearings a n d m a d e to rotate b y m e a n s of
a m o m e n t about the axis of the bearings, it is said to be balanced (for
rotation about that axis) if the external reactions exerted b y the bearings
Page 292

onto t h e b o d y are only w h a t are required to support t h e weight of t h e


body. The bearing reactions accompanying imbalance result in vibration
a n d w e a r of rotating machinery a n d are the reason, for example, for
balancing automobile tires.
There are t w o distinct causes for a rotating b o d y to b e out of balance.
The first is if t h e m a s s center is located (a distance "d") off t h e axis of
rotation. T h e n as t h e b o d y turns, there will b e forces at the bearings
producing a n d equaling m a . Clearly, these forces will be constantly
c

changing in direction (relative to t h e inertial frame) if not also in m a g n i ­


tude.

Question 4.14 What would cause them to change in magnitude?

Moving t h e mass center onto the axis of rotation b y addition or deletion


of mass is called static balancing. It carries that n a m e because only t h e n
will the b o d y remain in equilibrium w h e n t u r n e d to a n y position a n d
released, this being true regardless of the orientation of the axis.
The second cause of imbalance is n o n z e r o products of inertia
and/or w h e r e z is the axis of rotation a n d P is a point on that axis. In
the s a m e w a y that a n off-axis mass center causes bearing forces w h i c h
produce m a , these products of inertia likewise cause bearing forces to
c

exist; for a system w h i c h h a s b e e n statically balanced, they produce t h e


"bearingmoments"(seeEquations4.14a b) / and .Andsimilar
to r e m o v i n g t h e offset "d," w e can also a d d or delete material to force t h e
values of and to be zero. W h e n this is done, in addition to h a v i n g
ensured that C lies o n t h e axis, the b o d y is t h e n said to b e dynamically
(and of course also statically) balanced. We shall develop t h e equations to
accomplish this in w h a t follows.

Bearings in the
inertial frame

Figure 4.8

Answer 4.14 C h a n g e s in rotational speed.


Page 293

We show body in Figure 4.8, set in ball bearings at D and E. T is


an externally applied torque about z, the axis of rotation. Let us say that
T is the driving torque less any frictional resistance moments from the
air or the bearings. Finally, note that the and axes are also fixed in
and are the coordinates of C in this system.
For an unbalanced body rotating about a horizontal axis, the bearing
reactions required to support the weight of the body (when it is not
rotating) may be simply added to the dynamic reactions that would be
generated were there n o gravity. Therefore, for the sake of simplicity w e
shall ignore the effects of gravity in our discussion here.

Figure 4.9

N o w consider the free-body diagram, Figure 4.9, and let the bearing-
reaction components be referred to the body-fixed axes (x, y, z).
Then yields the component equations

(4.24a)
(4.24b)
(4.24c)

Using Euler's second law in the form

(4.20)

we obtain the following component equations:

(4.25a)
(4.25b)
(4.25c)

We observe, then, that if w e know and the geometric and


inertia properties of the body, w e can solve Equations (4.24a,b) and
(4.25a,b) for the bearing reactions D , D , E , and E . We n o w illustrate
x y x y

such a calculation with an example before taking up the issue of h o w to


balance a body.
Page 294

EXAMPLE 4 . 3 2

The body in Figure E4.32 has mass m = 2 slugs, and its mass center is off-axis by
the amount d = 1/64 in. in the x-z plane so that in. and . Its
2
products of inertia are slug-ft . If the body is spun up to a
constant angular speed of 3000 rpm, what then are the dynamic reactions at
bearings D and E?
Solution
By the parallel-axis theorem

Figure E4.32

Also,

Thus by Equations (4.24) and (4.25) we obtain

and

Let u s n o w assume that w e h a v e a rotating b o d y a n d a not both


zero) m o u n t e d in bearings. For such a body, w e can s h o w that the bear­
ing reactions vanish if a n d only if and
The "if" proof is simple, for if Equations (4.25 a,b)
yield E = 0 = E , a n d substituting these zero values along with
x y

into Equations (4.24 a,b), w e find D = 0 = D . x y

For t h e " o n l y if" part of t h e proof, if t h e bearing reactions are all zero,
Kramer's rule applied to Equations (4.24 a,b) gives a n d to
Equations (4.25 a,b) yields W h e n these t w o products of
inertia are zero at a point D, t h e n z is called a principal axis of inertia at D;
this concept is discussed in considerable detail in Chapter 7.
In s u m m a r y , then, w e can say that t h e bearing reactions vanish, a n d
h e n c e t h e b o d y is balanced, if a n d only if t h e axis of rotation is a principal
axis of inertia containing t h e m a s s center of t h e body.
N o w let u s see w h a t can b e d o n e about correcting imbalance. S u p ­
pose values of and of a b o d y are k n o w n , w h e r e P, lying on
t h e axis of rotation, is t h e origin of t h e coordinates. W e can, for example,
determine t h e coordinates (x , y ) a n d (x , y ) a n d masses (m a n d m ) of
A A B B A B
Page 295

a pair of weights which, w h e n placed in two "correction planes" A (at


z = z ) and B (at z = z ), will ensure that the shaft is dynamically bal­
A B

anced. All w e have to do is (a) force the mass center C* of the combined
system (m plus m and m ) to lie on the axis of the shaft, and (b) force the
A B

products of inertia of the combined system to vanish:

x-coordinate of (4.26a)
y-coordinate of (4.26b)
(4.26c)
(4.26d)

Note that w e assume that the "balance weights" are small enough to
be treated as particles.
These four equations (4.26) may be solved for the four quantities
and . Thus there is some freedom to select two of
the six quantities and provided there is n o other
condition linking them; for an example of such a constraint, the weights
might have to be placed o n a circle of given radius (such as w h e n tires are
balanced and weights are clamped to a rim). In this case w e would

and

and n o w there are six equations in six unknowns. Let us illustrate the use
of these equations in the following example.

EXAMPLE 4 . 3 3

In Example 4.32, suppose that we are to balance the body by adding weights in
two correction planes midway between C and the two bearings. Furthermore, the
weights are each to be placed on a circle of radius ft. Find the masses and
coordinates of the weights.
Solution
2
We had m = 2 slugs, in., and slug-ft . If we
choose P to have the same axial position as C, then ft, and

Solving these we get


Page 2 9 6

Similarly,

from which we obtain

Squaring and adding,

slug
So the weight of B is or 0.932 oz. For the
coordinates,

and

(These add vectorially to as a check.)

Plane A

0 . 0 9 6 1 ft
Plane B

490 ft

0 . 1 9 1 ft

0 . 4 6 2 ft
Figure E4.33

Now, for the mass and coordinates of m in plane A, we have, by again


A

squaring and adding,


Page 297

Thus the weight of A is or 1.85 02. The


coordinates are

Again checking,

The above results are all shown in Figure E.4.33.

In our last example, w e shall add mass in the form of two rods to balance
the body in Example 4.16.

EXAMPLE 4 . 3 4

For the body of Example 4.16, find the length L of the pair of rods, each of
mass 3 m, that will dynamically balance the shaft when attached to it as shown in
Figure E4.34.

Figure E4.34

Solution
In the previous example, was computed to be Note that the mass center
of the original and modified systems is at P, so the system is already statically
balanced. Thus since (all the mass is still in the xz-plane), all we need for
dynamic balance is:
Page 298

PROBLEMS Section 4.7

4.182 Explain why the uniform plate in Figure P4.182 is a. Find the dynamic bearing reactions
dynamically at A and B
balanced.
in terms of the system parameters shown in the
4.183 A light rod of length with a concentrated end figure.
mass M, is welded to a vertical shaft turning at constant b. If find the radius of a hole (in terms
(See Figure P4.183.) Find the force and moment exerted of r) at Q that will eliminate these bearing reac­
by the rod onto the shaft. Include the effect of gravity. tions.
4.186 Two thin disks are mounted on a shaft, each mid­
way between the center and one of the bearings, as indi­
cated in Figure P4.186. The disks are each mounted off
center by the amount in. as shown. Determine
the x and y locations of two small 4-oz magnetic weights
(one for each disk), which when stuck to the disks will
balance the shaft. Neglect the thicknesses of the disks,
and treat the weights as particles.
4.187 In Figure P4.187, is the axle of a bicycle,
Figure P4.182 mounted in bearings 2d apart. The cranks are rigidly
connected to the axle and also to the pedal shafts and
Figure P4.183
. If the rigid body consisting of axle the cranks and
the pedal shafts is turning freely about axis z at constant
c

4.1 B4 The shaft in Figure P4.184 rums at constant angu­ angular speed find the forces exerted on the bearings
lar velocity 10 rad/sec. If the bars are light compared in the given configuration.
with the two weights, determine the bending moment
exerted on (length by at the point where they are
welded together. Sketch the way the shaft will deform in
reality under the action of this couple. Ignore gravity.

Each:
W = 5 lb ;

R = 6 in.

Figure P4.186

Figure P4.184

Figure P4.185

4.185 The circular disk in Figure P4.185 is offset by the


amount 6 from the shaft to which it is attached. Figure P4.187
Page 299

4.188 A solid cylinder (of mass m, radius r, and length Show that, if a body mounted on a shaft is statically
and a light rod are welded together at angle as balanced, and if and are zero for any point D on the
shown in Figure P4.188. The rigid assembly is spun up to shaft, it follows that and are zero for any other point
rad/sec and then maintained at that speed. Find the Q on the shaft.
dynamic bearing reactions after 4.192 The shaft in Figure P4.192 supports the eccentri­
cally located weights W (0.1 lb) and W (0.2 lb)as shown.
1 2

It is desired to add a 0.3-lb weight in plane A and a 0.4-lb


Bearing Bearing weight in plane B to balance the shaft dynamically. De­
termine the x and y coordinates of the added weights.

0
45
3 ft 3 ft 3 ft
0
60
Figure P4.188
Plane A

Plane B
4.189 Repeat the preceding problem if the lower half
(shaded) of the cylinder is missing. (The mass is now
Figure P4.192
m/2.)
4.190 The S-shaped shaft in Figure P4.190(a) is made of
two half-rings, each of radius R and mass m / 2 . Find the 4.193 Rotor in Figure P4.193 has a mass of 2 slugs,
dynamic bearing reactions for the instant given. Hint: For and its mass center C is at a 5-in. offset from its shaft
a half-ring, the mass center is located as shown in Fig­ as shown in., in., and in. in the coor­
ure P4.190(b). dinate system fixed in the shaft at the point B).
The products of inertia of with respect to the center
2
of mass axes are lb-in.-sec and
the two correction planes and balance the rotor.
That is, determine the x and y coordinates of each of the
two added masses by ensuring that the mass center of
the final system is on the shaft and that the products of
inertia vanish.

(a)

(b)
Figure P4.190

Figure P4.193
Page 300

3 in. 4 in. 5 in.


0.01 slug
0.02 slug

Plane B
Plane of m

Plane of m
Plane A

View C-C
1

Figure P4.194

4.194 Balance the shaft in Figure P4.194 by adding a


2 ft mass of 0.003 slug in plane A and a mass of 0.004 slug in
plane B.
4.195 Two plates, each weighing 32.21b, are welded
to a light shaft as shown in Figure P4.195. A torque T of
10 lb-ft is applied about the z axis until the assembly is
2 ft
turning at angular speed then T is removed. If the
bearings can hold a force perpendicular to the shaft of
no more than 320 lb, find the maximum value that
1ft
can be without failure. Note that xz is the plane
4ft of the plates and (x, y, z) are fixed to the assembly.
2 ft

2 ft

4 ft

2ft

Figure P4.195

COMPUTER PROBLEM • Chapter 4

4.196 A cylinder of mass m and radius R is rolling to the where have been substi­
left and encounters a pothole of length s, as shown in tuted.
Figure P4.196(a). The angular velocity when the mass b. Multiply Equation (3) by and integrate, ob­
center C is directly above O is We are interested in taining
the condition(s) for which there will be no slip at O while
the cylinder pivots prior to striking the comer at A.
a. Show that for no slip at O, the equations of mo­
tion are (see Figure P4.196(b)): Solve these equations for and N, and show that
the no-slip condition requires that
sin
Note that for very low this is easily satisfied if
is not too small and s (and therefore is not too
large. But, for example, if then the cyl-
Page 301

Figure P4.196(a) Figure P4.196(b) Figure P4.196(c)

inder will slip regardless of the value of because pivoting if


then the inequality cannot hold. Note too that if
the cylinder is not slipping just prior to impact, it
has not slipped at all.
c. Next, use Figure P4.196(c) to compute the angle Finally, use the computer to create data for plots of
for which the cylinder will strike the left the minimum required for no slip at O versus for
comer A of the depression, and show that no three values of s/2R: 0.1, 0.2, 0.5. Draw the three curves
slip will have occurred at any time during the on the same graph.

EXTRA CREDIT PROJECT PROBLEM • Chapter 4

4.197 Construct a round object with a challenging to inclined plane (see Figure P4.197(b)), roll your object
calculate (as an example, see Figure P4.197(a)). On an down a 5-ft, 15° grade and with a stop-watch measure
the descent time. Do this twice and average the times.
Then explain the experiment, calculate the expected time,
compare with the actual time, and give possible reasons
for the difference in a brief report. (Note: It is fun to do all
the students' experiments in the same session.)

5 ft

15°

Figure P4.197(b)

Figure P4.197(a)

SUMMARY • Chapter 4
In this chapter w e h a v e developed compact forms for the r i g h t - h a n d
sides of m o m e n t equations for plane motion of a rigid body. The most
general forms w e h a v e studied are, for a n arbitrary point P,
Page 302

and

where moments and products of inertia are defined by

and for which there are the very useful parallel-axis theorems

and
and

Except for the topic of balancing of rotating bodies (Section 4 . 7 ) w e have


restricted our attention to situations in which the products of inertia
vanish, usually because of the body having an xy plane of symmetry. In
those cases,

and
Important special cases are
a. Translation (in which every point has the same acceleration, a,
and of course a = 0) for which:

and so

b. Summing moments at the mass center C:

or, more simply,

c. P is a pivot (body rotates about a fixed axis), so that a = 0:


P

or, more simply,

It is more important to realize that while w e have a number of


options as to form for the moment equation, one moment equation plus
the force equation, are all w e can bring to bear independently
for a given (free) body. That is, the situation is the same as in statics: we
may sum moments wherever w e like, but the two vector equations —
one force and one moment — give us all the independent relationships
involving external forces on the body. Many practical problems are
solved by augmenting these equations with kinematic constraint condi­
tions that can be invoked to generate relationships between a and c
Page 303

A body rotating about a fixed axis is said to be statically balanced if


the mass center is located on the axis. It is said to be dynamically balanced
(no bearing reactions induced by the rotation) if in addition the products
of inertia associated with the rotation axis all vanish. Industrial equip­
ment and automobile tires are modified by the addition of "balance
weights" so as to ensure these conditions.

REVIEW QUESTIONS • Chapter 4


True or False?

These questions all refer to rigid bodies in plane motion.


1. Euler's second law enables us to study the rotational motion of rigid
bodies.
2. The moment of inertia is always positive, whereas the products of
inertia can have either sign.
3. The formula gives the exact value of the moment of inertia of
a slender rod about a lateral axis through its mass center.
4. Euler's second law, is valid only in an inertial frame
(meaning that the position vectors and velocities inherent in H , the 0

origin O, and the time derivative are all taken in an inertial frame).
5. In the quantity d is the distance between the points P
and C. (C is in the reference plane, whereas P is any point of the
body.)
6. Euler's second law, applies to deformable bodies, liq­
uids, and gases, as well as to rigid bodies.
7. If represents the instantaneous center of zero velocity, then
in general.
8. Products of inertia are not found in the equations of plane motion.
9. is just as general as where O is fixed in an
inertial frame.
10. In translation problems, the moments of external forces and couples
taken about any point add to zero.
11. Suppose you buy a n e w set of automobile tires and a dynamic bal­
ance is performed on each wheel by adding weights in two planes
(inner and outer rims). The products of inertia and have thus
been eliminated, which otherwise would have, caused bearing reac­
tions and vibration.
12. applies to deformable as well as to rigid bodies, as long as
they are in plane motion.
13. For two bodies and the sum of the equations written
for each will be for the combined body.
14. If the bodies of Question (13) are turning relative to each other, it
makes n o sense to talk about a combined equation.

Answors : 1.T 2. T 3. F 4. T 5. F 6. T 7. T 8. F 9. T 10. F 11. T 12. F 13. T 14. T


5 SPECIAL INTEGRALS OF THE
EQUATIONS OF PLANE
MOTION OF RIGID BODIES:
WORK-ENERGY AND IMPULSE-
MOMENTUM METHODS
5.1 Introduction
5.2 The Principle(s) of Work and Kinetic Energy
Kinetic Energy of a Rigid Body in Plane Motion
An Alternative Form for Kinetic Energy
Derivation of the Principle W= Power and Work of Systems
of Forces and Couples
Restriction of W= to a Rigid Body; A Notable Exception
Computing the Work Done by Various Types of Forces and Moments
Examples Solved by the Principle W=
Two Subcases of the Work and Kinetic Energy Principle
Potential Energy, Conservative Forces, and Conservation of
Mechanical Energy
5.3 The Principles of Impulse and Momentum
The Equations of Impulse and Momentum for the Rigid Body in
Plane Motion
Conservation of Momentum
Impact
The Center of Percussion
SUMMARY
R E V I E W QUESTIONS

Page 304
Page 305

5.1 Introduction
Just as in Chapter 4, the framework here is rigid bodies in plane motion.
But we shall focus our attention now on problems which most efficiently
can be attacked by using work-and-kinetic-energy and/or impulse-and-
momentum principles. We shall employ these principles, rather
broadly stated in Chapter 2, taking advantage of the simple forms that
kinetic energy and angular momentum take when the body is rigid and
constrained to plane motion.
In Chapter 2 we defined the kinetic energy of a body to be the sum
of the kinetic energies of the particles making up the body; that is,
T= Because we have found in Chapter 3 that velocities of dif­
ferent points in a rigid body are related through the body's angular
velocity, the reader should not be surprised to find kinetic energy for such
a body to be expressible in terms of the velocity of one point and the
angular velocity. Moreover, in Chapter 2 we observed that, for a body in
general, change in kinetic energy equals work of external and internal
forces. But for a rigid body the net work of internal forces vanishes, so
that the work W in W = T is the work only of external forces. We shall
derive this work-and-kinetic-energy relationship directly from the force
and moment equations (Euler's laws) as studied in Chapter 4, but it is
helpful to recall the discussion of Chapter 2 and note the consistency of
that material with the result we shall develop here.
The relationship between angular impulse and angular momentum
developed in Section 5.3 takes on a quite useful form for a rigid body in
plane motion, owing to the fact, as shown in Chapter 4, that the angular
momentum can be expressed then in terms of inertia properties and
angular velocity. Thus we shall find ourselves in a position to evaluate
sudden changes in rates of turning for colliding bodies and to study
quantitatively the relationship between the spin rate and the arm-trunk
configuration of a skater.
It is very important for the reader to always keep in mind that the
principle of work and kinetic energy and the principles of impulse and
momentum do not stand as principles somehow separate from Newton's
laws or their extensions to bodies of finite size, Euler's laws. Rather, here
it will be seen, as was observed before in Chapter 2, that these relation­
ships, which involve velocities, are really just special first integrals of the
more fundamental second-order expressions relating forces and acceler­
ations. Thus the principles of this chapter allow us to begin our solutions
halfway between accelerations and positions. They therefore involve
velocities but not accelerations.

5.2 T h e Principle(s) of W o r k and Kinetic Energy

Kinetic Energy of a Rigid Body in Plane Motion


There is a principle, derived from the equations of motion, that will help
us to solve for unknowns of interest in kinetics problems. In this section
Page 306

we shall see that this principle arises from first deriving and then differ­
entiating the kinetic energy of the body.
Kinetic energy, which we have examined in Chapter 2, is usually
denoted by the letter T; for any body or system of bodies, it is defined as
the summation of over all its elements of mass:

(5.1)

In this section we need to specialize Definition (5.1) for a rigid body in


plane motion. To this end we kinematically relate the velocity v of the
differential mass to the velocity of the mass center C. Using the fact that v
is at all times equal to the velocity of its companion point in the reference
plane containing C (see Figure 5.1), we may write

C o m p a n i o n point of
dm in plane of mass
center C

Rigid body
in plane m o t i o n

Figure 5.1

We note that the x and y axes are fixed in with their origin at C. Forming
2
v , that is, we have
(5.2)
Thus the kinetic energy becomes, substituting (5.2) into (5.1),

Recognizing the moment of inertia integral in the second term, we obtain

(5.3)

in which the square of the magnitude of the veloc­


ity of C.

Question 5.1 Why do dm and dm both vanish?

We note that the (scalar) kinetic energy has two identifiable parts (not
components!): one relating to the motion of the mass center

* H e n c e f o r t h , w e shall u s e t h e a b b r e v i a t i o n t h r o u g h o u t this c h a p t e r .
Answer 5.1 B y t h e definition o f t h e m a s s c e n t e r .
Page 307

and the other to the motion of the body relative to C This


clear division of T even exists in general motion of rigid bodies (that is, in
three dimensions), though there are more terms in then.

EXAMPLE 5.1
Calculate the kinetic energy of the round rolling body in Figure E5.1, which has
mass m, radius R, and radius of gyration k with respect to the z axis. The mass
c c

center C (see Figure E5.1) lies at the geometric center.


Solution

Figure E5.1

Note that if is a solid cylinder, then k =


c and

In this case, two-thirds of the kinetic energy rests in the translation term of T
2
If is a ring (or hoop), however, I = mR so that k = R and
c c

and this time half the kinetic energy is in each of the translational and
rotational terms.

EXAMPLE 5.2
Work Example 5.1 for the case when the mass center C is offset by a distance r
from the geometric center of a round rolling body (See Figure E5.2.)
Solution
In order to use our equation for kinetic energy,

Figure E5.2
we must first calculate

Therefore
Page 308

Substituting, we get

Note that if r = 0, the answer agrees as it should with Example 5.1,

An Alternative Form for Kinetic Energy


There is an alternative means of writing the kinetic energy T of a rigid
body in plane motion by making use of the instantaneous center of zero
velocity (see Figure 5.2):

Thus by using the parallel-axis theorem we obtain

(5.4)

The translational (T ) and rotational


v terms composing the scalar T are
thus seen to collapse into the one term if we choose to work with
instead of C.

Figure 5.2 Figure 5.3

As an example, we consider a rolling cylinder again (see Figure 5.3):


Page 309

We have noted that two-thirds of the cylinder's kinetic energy is asso­


ciated with the "translational part" of T and one-third with the "rota­
tional part." If we now use we get all of T at once:

(as above)

As a second example of the use of Equation (5.4), consider the


slender rod swinging about a pivot at A as shown in Figure 5.4. The
kinetic energy of may be found in either of two ways:

Figure 5.4

Derivation of the Principle W = Power and Work of Systems of


Forces and Couples
Returning now to the derivation of our principle, we next compute the
rate of change of kinetic energy:

Therefore

(5.5)

Recalling that and that the z component of for rigid


bodies in plane motion, we may write

(5.6)
Page 3 1 0

Question 5.2 Since can contain x and y components (see Equa­


tion 4.13) why may we substitute the total vector for just the z
component in Equation 5.5?

Our next goal is to get the individual external forces and couples
acting on the body into the equation. (See Figure 5.5.) Note the abbre­
viations r = r , r = , and so on, of the vectors to the points of
CP1 1 CP2 l2

application of F , F , and so on. We assume that the external mechanical


l 2

actions on the body arise from a system of forces ( F , F . . .) and couples


l 2

with moment vectors (C C , . . .), as shown in Figure 5.5. Further, we


l 2

let (V , v , . . .) be the velocities of the material points (P ,P , . . .) on


l 2 l 2

which the forces act instantaneously.

Figure 5.5

Clearly, then, the resultant of the external forces is

(5.7)
and the moment of the s and s about C is

(5.8)
Substituting Equations (5.7) and (5.8) into (5.6) gives

(5.9)
Now since the dot and cross may be interchanged without altering the
value of a scalar triple product,

(5.10)

Answer 5 . 2 Because is z e r o !
Page 311

But the velocities of P and C are related: 1

so that

(5.11)

The right-hand side of Equation (5.11) is called the power, or rate of


work, of the external system of forces and couples acting on the body.
The power of a force is its dot product with the velocity of the point on
which it acts; the power of a couple is its dot product with the angular
velocity of the body on which it acts:

Rate of work of force F = 1 F 1 = power of F 1


(5.12)
Rate of work of couple C = 1 C 1 = power of C 1
(5.13)
Hence one form of the principle of this section is

Power (5.14)

or

Integrating, we obtain another principle.*

or

(5.15)

where the integral of the power is called the work W of the external forces
and couples. It is the work done by the s and s on the body between
the two times t and t . Hence we have a principle that can be stated in
1 2

words:
Work done by external forces Change of kinetic
and couples on energy of

Restriction of W = to a Rigid Body; A Notable Exception


It is essential to recognize that our derivation of the principle of work and
kinetic energy depends crucially on the body being rigid. In fact the work of
external forces on a def ormable body is not in general equal to the change
in its kinetic energy. That is the case even when the "deformable" body is

* Sometimes is u s e d t o d e n o t e t h e t i m e i n t e r v a l , r a t h e r t h a n t h e subscripts
s t a n d f o r "initial" a n d "final" v a l u e s .
Page 312

composed of several individually rigid parts. However, there are a num­


ber of special circumstances, usually easy to recognize, for which the
principle is valid for such a system of rigid bodies. To give an example for
which this is true, suppose we have two rigid bodies, and making
up the system, and suppose the bodies are connected by a pin (or hinge)
with negligible friction as shown in Figure 5.6.

Figure 5.6

Let F be the force exerted by on at the pin, and consequently — F


is the force exerted by on . Furthermore, let

v = common velocity of attachment points in the two bodies


= power (rate of work) of forces acting on that are also
external to system
= power of forces acting on that are also external to system
= kinetic energy of
= kinetic energy of
Now if we apply Equations (5.11) and (5.14) to each of the bodies, we
obtain

and

which may be added to yield


Page 313

or

where P is the power of the external forces on the system and T is the
kinetic energy of the system.
With friction in the pin, however, we also would have interactive
couples C and — C, and the sum of their work rates would be

which in general would not vanish.* This net rate of work of friction
couples would be negative, reflecting the fact that the friction will reduce
the kinetic energy of the system. We can expect the principle of work (of
external forces) and kinetic energy to be valid for a system of rigid bodies
whenever the interaction of the bodies leads neither to dissipation of
mechanical energy by friction nor to a storing of energy as in a spring.
When in doubt, follow the procedure we have just been through — that
is, apply Equation (5.14) to each of the bodies, add the equations, and see
whether the rates of work of interactive forces cancel out.

Computing the Work Done by Various Types of Forces and Moments


Before we can put Equation (5.15) to use, it is essential to demonstrate
how to compute the work W done on by a number of common types of
forces and moments:
Type 1: F is constant. In this case, as in Chapter 2,
1

(5.16)
Type 2: F acts on the same point P of
1 1 throughout its motion In
this case,

(5.17)

where r = r and i and f denote starting (initial) and ending (final)


O P 1 1

times and positions. It is true, of course, that the velocity v , which 1

combines with f to produce its power, is at each instant the derivative of


1

some position vector. If the force acts on different material points of at


different times throughout a motion (such as friction from a brake),
however, the path integral F • d i has no real functional utility and
1 O P 1

the general F • V dt must be used.


1 1

Type 3: F is due to gravity. This is an example of both Types 1 and 2.


1

Thus, letting z be positive downward, we get

* It w o u l d v a n i s h , o f c o u r s e , if t h e friction w e r e e n o u g h t o p r e v e n t r e l a t i v e r o t a t i o n s o
that t h e n t h e s y s t e m w o u l d b e h a v e a s a single rigid b o d y !
W h i c h w a s necessarily t h e c a s e in C h a p t e r 2!
** T h e w o r k d o n e b y any c o n s t a n t f o r c e F a l w a y s a c t i n g o n t h e s a m e p o i n t w i t h p o s i t i o n
v e c t o r r is t h u s F •
Page 314

Expressing the differential of the position vector in terms of rectangular


cartesian coordinates, we get

and substituting we obtain a simple result for the work of gravity:

(5.18)

as we observed in Chapter 2. Note that gravity does positive work if the


body moves downward. (Indeed, a good rule of thumb to remember is
that a force does positive work if it "gets to move in the direction it wants
to" — that is, it has a component in the direction of the motion of the
point on which it acts. If it does not, it does negative work during the
motion of that point.)
Type 4: F is the normal force exerted at the point of contact on a rigid
1

body that is maintaining contact with a fixed surface, whether rolling or


slipping. Note in the lower portion of Figure 5.7 that the normal force F ,
is always perpendicular to the velocity of P. That is,
Figure 5.7

Type 5: F is the friction force exerted at the point of contact when a


1

rigid body rolls on a fixed surface (Figure 5.8). This time, the force F 1

(which may or may not be zero) does zero work because it always acts on
a point of zero velocity:

Figure 5.8
Type 6: F is the force in a linear spring connected to the same two
1

points P and Q of bodies and during an interval of their motions. (See


Figure 5.9.) We denote:
k = spring modulus (which when multiplied by the stretch yields
the force in the linear spring)

(inertial
reference
frame)

Figure 5 . 9
Page 315

unstretched length
stretch if compressed)
unit vector along spring toward body
We first note that the work of spring S on body is

Using

we may differentiate and obtain

Therefore, substituting for v , we get P

Since the derivative of a unit vector is perpendicular to the unit vector,


the last integral vanishes and we obtain

Thus

(5.19)

If Q is fixed in the inertial reference frame, the work of S on B alone is


given by the right side of (5.19);* if Q moves, however, we can only say
Figure 5.10 that the total work on both bodies by S is given by
We note from the spring's force-stretch diagram (Figure 5.10) that
the work done by the spring is in fact the negative of the change in energy
E stored in it; namely, in stretching from
E = (area of triangle OCB) - (area of ODA)

Type 7: We now consider the work done by the force in an inextensi-


ble cable (or rope, string, cord) connected to two points P and Q of bodies
and during an interval of their motions (Figure 5.11). The cable
under consideration may pass over one or more light, f rictionless pulleys

Figure 5.11 * A n d its w o r k o n R is o f c o u r s e t h e n z e r o .


Page 316

between the bodies, but it is assumed to remain taut throughout the


motion.
The work done by the cable tension on the system of plus is
zero, which we proceed to prove as follows:
We write v and v in terms of their components parallel and per­
Q P

pendicular to the cord:

Noting that the perpendicular components and have zero dot


products with the unit vectors and (see Figure 5.11), we obtain for the
works of the tensions:

and

But by the cable's inextensibility, and

so that
W by cable on both bodies = 0

Type 8: We have a couple C. In this case, the work of the couple in


plane motion is given by

(5.20)

Thus if C is constant, the work of the couple is given by


(5.21)

That is, the work of C is the strength of the couple times the angle through
which the body turns. As with the work of forces, the couple's work is
positive if it "gets to move" in the direction in which it acts (or turns, in
this case).

Examples Solved by the Principle W —


We are now in a position to solve some problems by using the principle of
work and kinetic energy. A number of examples follow. In the first, work
is done only by gravity, and W = is used to supplement the equations
of motion.
Page 317

EXAMPLE 5 . 3
Find the pin reaction at O when the uniform bar in Figure E5.3a has fallen
0
through 4 5 from rest.
Solution
Figure E5.3a
0
We first find the angular speed in thefinal(45 ) position by using the principle

Letting T be the kinetic energy in thefinalposition, and noting that the work
2

done by gravity we find:

(1)

We must now return to the differential equations to obtain equations in the


desired reaction. (Note that W = alone can only give us the solution to one
scalar unknown!) In the final position, we have:

Expressing this equation in its tangential and normal components with the help
of the free-body diagram (Figure E5.3b),

(2)

But
Figure E 5 . 3 b

and with k defined as

so that, from the component of Equation (2),

where we have substituted from Equation (1). Therefore, the normal compo­
nent of the reaction is
(3)

Next, from the component of Equation (2),

(4)

Also, since point O is a pivot of the rod, we know:

(5)
Page 318

Substituting from Equation (5) into (4) gives the tangential component of the
reaction:

Thus the pin reaction is

In the second example, work is done only by a spring; however, calculat­


ing the final stretch is tricky. *

EXAMPLE 5 . 4
One end of the linear spring in Figure E5.4a is attached to a thin inextensible cord
that is lightly wrapped around a narrow groove in the wheel (mass = 1 slug,
radius of gyration about center = 1 . 5 ft). If the wheel rolls, and starts from rest
when the spring is stretched 1 ft,findthe velocity of the center of the wheel when
the center has moved 2 ft. The mass center of the wheel coincides with the
geometric center.
Figure E5.4a Solution
Wefirstnote that the cord is not attached to a specific material point on the wheel.
However, as time passes, the various "wrapping points" on the end of the
straight portion of the cord (such as in Figure E5.4b) have, at every instant the
cord is taut, the same velocity as the coincident point of the wheel at Q. Thus
Equation (5.19) gives the work done on the wheel by the spring.
We have at all times, by kinematics (see the figure),

and
so that
Figure E5.4b

or

Thus the net shortening of the spring when C has moved 2 ft to the right is ft.
Another way to see this is to let C move to the right the amount x
This com­
presses the spring (if it were able to do so!) the same amount, x . Then turn the
c

wheel clockwise about C through the angle radians, until the


correct point is on the ground. ("Correct" means the point that would be on the
ground had the wheel rolled normally over to the final position.) The rotation
wraps of string around the inner radius and "takes back" of
the compression. Thus = is the reduction in the original 1 ft of stretch,
leaving ft, as before. Hence
and
Page 319

We then find, noting that gravity does no work here,

We note that when the center has moved 3 ft, then and all the
stretch is gone. At this time, the spring would simply drop out of the problem.

The next example is an actual application of W = from industry.

EXAMPLE 5 . 5

This example involves a practical application in the antenna industry of the work
and kinetic energy principle. The antenna positioner in Figure E5.5 is equipped
with a mechanical stop spring so that if the elevation drive overruns its lower
limit, the antenna motion (a pure rotation about the horizontal elevation rotation
axis) will be arrested before the reflector strikes another part and is damaged.

Counterweights place mass


center of rotating part on Elevation rotation axis
elevation axis

M e c h a n i c a l stop
spring

Positioner

Reflector

Figure E5.5

The elevation motor has an armature rotational mass moment of inertia of


2 2
0.01 lb-ft-sec (or slug-ft ) and drives the reflector through a gear reducer with a
700:1 gear ratio. The combined moment of inertia of the reflector, its counter­
2
weights, and the supporting structure is 12,000 slug-ft = I . 0

It is desired to arrest a rotational speed of 30°/sec during a rotation from


0
contact to full stop of 3 . The radius from the elevation rotation axis to the stop
spring is 1.5 ft. The spring is unstretched at initial contact and may be assumed to
have linear load-deflection behavior. It is further assumed that the motor is
Page 320

switched off but remains mechanically coupled while the rotation is being
arrested. Find:
a. The required stiffness of the spring.
b. The maximum force induced in it.
c. The rotational position when it sustains its maximum force.
d. The angular accelerations of the reflector and motor armature at the
position of maximum force. (Are these the maximum accelerations?)
Solution
Since the spring is linear, its greatest force is the spring stiffness times the maxi­
mum deflection. This is also the position for which motion is completely arrested.
At this position the kinetic energy has been brought to zero with the stop spring
storing the energy; the principle W = gives

Note that point O is for the rotating body and that gravity does no work
between contact and stop.

Question 5.3 Why does gravity do no work?

The values of and needed in the equation are calculated as follows:


total moment of inertia at axis of rotation

(Note that over the very small angle of 3 ° the spring compression is approxi­
mately the arclength
Solving for the spring's stiffness, we get

The maximum spring force = = 753,000 X 0.0785 = 59,100 lb. The rota­
tional position is 3 ° beyond contact — that is, the position at full stop. The

Answer 5.3 S i n c e t h e c o u n t e r w e i g h t s p l a c e t h e m a s s c e n t e r o n t h e e l e v a t i o n axis, t h e


mass center does not move.
• A s t h e r e a d e r m a y w i s h to p r o v e , m o m e n t s o f inertia reflect t h r o u g h g e a r t r a i n s f r o m
i n p u t t o o u t p u t w i t h t h e g e a r r a t i o s q u a r e d a s a f a c t o r ; also, t h e t o r q u e i n c r e a s e s ( w h i l e
the speed decreases) with the gear ratio as the multiplying factor.
Page 3 2 1

angular acceleration of the reflector is

2
and that of the motor armature is 5.25 X 700* = 3680 rad/sec .
These are the maximum accelerations, since here the force (and torque) are
greatest. In closing, we note that motor torque and friction, omitted in this

The next example illustrates work done by a force acting on different


points of the body as time passes. A shortcut for calculating this work is
presented.

EXAMPLE 5 . 6
This example illustrates the work done by forces and couples belonging to Types
1, 2, and 8 on the preceding pages. The force F (52 lb) is applied to the uniform
cylinder C at rest in Figure E5.6a at the left. (This type of force might be applied by
a cord on a hub, as is suggested by Figure E5.6b.) If force F continues to act with

a. The work done by F during transit to the dashed position


b. The velocity of C and the angular velocity of the cylinder in the

Figure E5.6a Figure E5.6b

Solution
We shall work part (a) in two ways. First, the definition of the work of F is

where Q is the point of in contact with F at any time. The geometry in Fig­
ure E5.6c gives an angle of 45.1° between F and v , since
Q

Figure E5.6c * See preceding footnote.


Page 322

Also,

Therefore

and

A second and simpler approach is to note that F at Q may be moved to C,


using the idea of resultants, as in Figure E5.6d. The force at Q is replaced by the
force and couple at C which produce the same effect on therigidbody. Since, at C,
force F always acts on the same point of the body (it didn't at Q!) we may write
W = work of F at Q = (work of F at C) + (work of couple on

= 6.53F = 340 ft-lb (as before)

Note that the work of a constant couple in plane motion is simply the moment of

FT sin 22.4°

Figure E5.6d

For part (b) we equate the work to the change in the kinetic energy of C:

Note that the gravity, friction, and normal forces do no work in this problem, for

The hardest part of the next example is finding where the mass center
is in the final position!
5.2 / The Principle(s) of Work and Kinetic Energy 323

EXAMPLE 5 . 7
The unstretched length of the spring in Figure E5.7a is ( = 0.3 m. The initial
u

angular velocity of body J in the top position is <w = 2.5 ? rad/s. There is
f

enough friction to prevent slipping of JoxxBdX all times. Determine the modulus
of the spring that will cause ^4 to stop in the <p = 90° position.

Figure E5.7a

Solution
Part of the work W in this problem is done by gravity. To express this work, we
must determine where the mass center C is located when ^4 reaches the final
position. Reviewing the kinematics, we find that the velocity of the geometric
center Q of J (see Figure E5.7b) is expressible in two ways:
0
1. As a point of J, v = yfe + ROe,.
Q

0
2. As a point of 2>, v = y£ + tye,.
Q

Thus we see that R0 = lip. Integrating, we get


Rd = lq>
in which the constant of integration is zero if we select 6 = 0 when g> = 0.
Therefore when <p = n/2, we may find the orientation of body J:
8 = angle that body -J turns through in reference frame (angle seen by
stationary observer in body B)
t 0.6 71 ,
= = X = 3 j I
] ^ oT 2-
Page 324

And so the final position of C is to the left of Q (see Figure E5.7c). We can now
write the work of gravity W because we now know the h moved through by C:
g

C is now here

C w a s initially to t h e right of
but d u r i n g t h e rolling, line
(and e v e r y o t h e r line o n b o d y
turned r a d c l o c k w i s e in s p a c e

rad
Figure E5.7c

The work done by the linear spring is always given by

where k is our unknown and the initial and final stretches are computed as
follows:
(unstretched length plus initial stretch =
initial length of spring)
and

so that the stretches are

For the kinetic energy side of W we need the moments of inertia;firstwe


consider body

Figure E5.7d We shall use the "short form" of T — namely (always valid when­
ever in plane motion). Thus we need I and * Note that when is a
At different point of a body in the initial and final positions, the value of is
is now here
generally different in the two configurations, as is the case in this problem.
and no longer Using Figures E5.7d and E5.7e, we find:
the instantaneous
center of
2
= 0.392 + 80(0.1 + 0.05 ) 2 2
= 0.392 + 80(0.1 - 0.05)
2
= 1.39 k g - m 2
= 0.592 kg • m

* Since = 0, w e d o n o t h a v e t o c a l c u l a t e h e r e , b u t w e d o s o t o illustrate t h e p r o ­
Figure E5.7e c e d u r e in g e n e r a l .
Page 325

Therefore the kinetic energies of we need are

(since the final angular


speed is to be zero)
For the bar is the same in any position since is point 0 , which is pinned
to the reference frame. Therefore, using we have

Applying the work and kinetic energy principle, we get

This is equivalent to 829 lb/ft of stiffness in the U.S. system of units, since
1 lb/ft is the same stiffness as 14.6 N/m.

Question 5.4 What happens if k is larger than the calculated value?


What happens if it is smaller?

In the next example work and kinetic energy is used to help deter­
mine the point where rolling stops and slipping starts.

Answer 5 . 4 L a r g e r : t h e r o d will n o t r e a c h t h e h o r i z o n t a l position; smaller: t h e s y s t e m


p a s s e s t h r o u g h this p o s i t i o n w i t h o u t s t o p p i n g .

EXAMPLE 5 . 8
Mass m As we found in Example 5.3, sometimes it is useful to combine the work and
kinetic energy principle with one or more of the differential equations of motion
in order to obtain a desired solution. This example involves such a combination.
The small cylinder starts from rest at in the dotted position (see Fig­
ure E5.8a) and begins to roll down the large cylinder. Find the angle at which
slipping starts, and show that the small cylinder will always slip before it leaves
Figure E5.8a the surface for a finite coefficient of friction.
Page 326

Solution
Using the free-body diagram in Figure E5.8b, the equations of motion are

(1)

(2)

(3)

Figure E5.8b Just prior to slipping, the friction force while is still equal to and v is
c

still equal to Therefore the equations can be rewritten as

(la)

(2a)

(3a)

These equations may be supplemented with the work and kinetic energy equatio

(4)

Equations (1) to (4) may now be treated as four equations in the unknowns
2
to , and a , where the last three quantities are the angle, angular velocity, and
s

angular acceleration at slip. Solving them for yields the equation

Writing for sin and then squaring and solving the resulting
quadratic for cos 6., gives

which plots as shown in Figure E5.8c.

Asymptote

Figure E5.8c

The curve in the diagram gives the slipping angle as a function of the friction
coefficient; this is not the angle at which body leaves the surface. We note that if
we were to look for the angle at which the small cylinder leaves the surface of the
large cylinder, assuming no slipping has occurred, we would be trying to solve a
Page 327

problem with no solution if the friction coefficient is finite. The curve clearly
_1
shows that for C to reach the angle cos (4/7), an infinite coefficient of friction is
required. Since the solution to the "leaving without slip" problem is precisely
_1
cos (4 /7), shown below, then regardless of the friction coefficient (so long as it
is finite) will have to slip before it leaves. Assuming now that the cylinder leaves withou
the (simpler) solution:

Equation (1)

Equation (4)

Eliminating gives

As we have noted, this solution is valid only for an infinite coefficient of friction
between the cylinders. If were a particle (no rotational kinetic energy) with a
_1
smooth surface, we would obtain (Example 2.13) = cos (2/3) = 48.2°.Note
the differences between these solutions.

T w o Subcases of the Work and Kinetic Energy Principle


There is an important subcase of the principle of work and kinetic energy
that we have already seen in Chapter 2. Using

we obtained

(5.22)

This principle states again that the work done by the external force
resultant, when considered to act on the mass center, equals the change
in the translational part of the kinetic energy:
(5.23)
The integral of Equation (5.6) is

(5.24)

If we subtract (5-23) from (5.24), we obtain yet another result:

or
(5.25)
Page 328

This second subprinciple says that the work done by the external
moments (about C) on the body, as it turns in the inertial frame, is equal to
the change in the rotational part of the kinetic energy. We may use the
"total" W = principle or either of its two "subparts" (Figure 5.12).

Figure 5.12
Let us now examine an example in which these subparts may be seen
to add to the "total" W = equation:

EXAMPLE 5 . 9
As an illustration of the two subcases of the principle of work and kinetic energy,
we consider the cylinder of mass m rolling down the inclined plane shown in
Figure E5.9a. If the cylinder is released from rest, find the velocity v of its mass
c

center as a function of the distance x traveled by C.


c

Figure E5.9a Figure E5.9b

Solution
Referring to the free-body diagram in Figure E5.9b, we see that the normal and
friction forces do no work because, as the cylinder rolls on the incline, they
always act on a point at rest. That is,

and
Applying the principle that W = only the component of the gravity force W
that acts parallel to the plane does any work:

(1)

Since W always acts on the same point (C) of and since

(2)
Page 329

The mass center's velocity is therefore

(3)

Now suppose we apply Equation (5.23):

(4)

In this problem the resultant force acting on is

and Equation (4) becomes

(5)

We see that, as expected, the friction force (though it does no net work)
retards the motion of the mass center C while turning the cylinder, as can be seen

(6)

(7)

or

(8)

And the sum of Equations (5) and (8) indeed gives Equation (2): the total W =
equation!

Potential Energy, Conservative Forces, and Conservation of


Mechanical Energy
In Section 2.4 we introduced the concept of potential energy, or the
potential of a force. When the work done by a force on a body is inde­
pendent of the path taken as the body moves from one configuration to
another, the force is said to be conservative and the work is expressible
as the decrease in a scalar function the potential (energy). Thus as a
body moves from a configuration at time f j to a second configuration at
time t , the work done by an external conservative force is
2

or simply

If all the external forces that do work on a rigid body are conservative and
Page 330

is now the sum of the potentials of those forces, Equation (5.15) yields

or

or
constant
which expresses the conservation of mechanical energy.
From Chapter 2 and earlier in this section we can easily identify two
common conservative forces: (1) the constant force acting always on the
same material point in the body and (2) the force exerted on a body by a
linear spring attached at one end to the body and at the other to a point
fixed in the inertial frame of reference.
In the case of the constant force, a potential is = — F • r, where r is a
position vector for the point of application. When the force is that exerted
by gravity (weight) on a body near the surface of the earth,
mgz
where h is the altitude of the mass center of the body.
For the linear spring, we recall that = where k is the spring
modulus, or stiffness, and 8 is the stretch. It is important to recognize that
when a spring is attached to, or between, two bodies that are both moving
(relative to the inertial frame), then is a potential for the two
spring forces taken together (see Equation 5.19). That is, while neither of
the forces acting on the bodies can be judged by itself to be conservative,
the net work done on the two bodies by the two forces is expressible as a
decrease in the potential, This is helpful in the analysis of
problems in which we have two or more interacting rigid bodies. We have
already noted earlier in this section that the work of the external forces on
a system of rigid bodies is not in general equal to the change in kinetic
energy of the system; this is because there may be net work done on the
rigid bodies by the equal and opposite forces of interaction. Suppose now
that our system is made up of two bodies joined by a spring, and suppose
the spring forces are the only internal ones that produce net work on the
system. We may then write W = for each rigid body. Upon adding
these equations there results

(Work of forces external to system) + (Work of pair of spring forces)


= (Change in kinetic energy of system)
If the forces external to the system that do work are conservative, we
may add the various potential energies associated with them to that for
the pair of spring forces and conclude that
- constant
That is to say, in this case the mechanical energy of the system is con­
served.
Page 331

An example of a nonconservative force is sliding friction. A potential


cannot be found for friction, since the work it does depends on the path
taken by the body on which it acts. In this case, must be used,
and it is seen to be more general than the principle of conservation of
mechanical energy.

EXAMPLE 5 . 1 0
Show that the same equation for the spring modulus in Example 5.7 is obtained
by conservation of mechanical energy. (See Figure E5.10.)

R- 0.1 m

D
(Uniform,
slender bar of
length l = 0.6 m
and mass - 35 kg)

B (Fixed cylinder! inertial


frame for the problem)

Figure E5.10

Solution
The potentials for gravity and for the spring are

Therefore, measuring z from O, we have


c

For the spring, using i for the initial and f for the final configuration, we have
Page 332

Thus, adding the potentials we get

The kinetic energies were T = 4.34 + 0.365 = 4.71 J and T = 0. Therefore


i f

574 + 0.0242k + 4.71 = 0.0722k + 0


or, rearranging,
574 - 0.0480k = - 4 . 7 1
This is the same final equation that resulted from in the earlier Ex­
ample 5.7.

PROBLEMS Section 5.2

5.1 Find the kinetic energy of the system of bodies , 5.3 Upon application of the 10-N force F to the cord in
, and at an instant when the speed of is 5 ft/sec. Figure P5.3, the cylinder begins to roll to the right. After C
(See Figure P5.1.) has moved 5 m, how much work has been done by F?

Figure P5.3

5.4 The suspended log shown in Figure P5.4 is to be


used as a battering ram. At what angle should the ruf­
fian release the log from rest so that it strikes the door at
Figure P5.1
with a velocity of 20 ft/sec?

5.2 See Figure P5.2. (a) Explain why the friction force f
does no work on the rolling cylinder if the plane f is the
reference frame, (b) If, however, f is the top surface of a
moving block (dotted lines) and the reference frame is
now the groundG,doesfthen do work on ? Why or why not?

Figure P5.4

5.5 The 20-kg bar in Figure P5.5 has an angular veloc­


ity of 3 rad/s clockwise in the horizontal configuration
Figure P5.2 shown. In that position the tensile force in the spring is
Page 333

30 N. After a 90° clockwise rotation the angular velocity 5.9 Bar is smoothly pinned to the support at A and
has increased to 4 rad/s. Determine the spring modu­ smoothly pin-jointed to at B. (See Figure P5.9.) End D
lus k. slides on a smooth horizontal surface. If D starts from rest
at , determine the angular velocities of the rods just

Figure P5.5
Figure P5.9
5.6 A uniform 40-lb sphere (radius = 1 ft) is released
from rest in the position shown in Figure P5.6. If the
sphere rolls (no slip), find its maximum angular speed. 5.10 The prehistoric car shown in Figure P5.10 is pow­
ered by the falling rock m, connected to the main wheel (a
cylinder of mass M) by a vine as shown. If the weights of
the frame, pulley, and front wheel are small compared
with Mg, find the velocity v of the car as a function of y if
c

Figure P5.6

5.7 The unbalanced wheel of radius 2 ft and weight


64.4 lb shown in Figure P5.7 has a mass center moment
2
of inertia of 6 slug-ft . In position 1, with C above O, the
wheel has a clockwise angular velocity of 2 rad/sec. The
wheel then rolls to position 2, where OC is horizontal.
Determine the angular velocity of the wheel in position 2.

Figure P5.10

5.11 A truck body weighing 4000 lb is carried by four


solid disk wheels that roll on the sloping surface. (See
Figure P5.ll.) Each wheel weighs 322 lb and is 3 ft in
Figure P5.7
diameter. The truck has a velocity of 5 ft/sec in the posi­
tion shown. Determine the modulus of the spring if the
truck is brought to rest by compressing the spring 6 in.

Figure P5.8

5.8 Determine the spring modulus that will allow the


2-kg bar in Figure P5.8 to arrive at the position at
zero angular velocity if it passed through the vertical
(where the spring is compressed 0.1 m) at 8 rad/s Figure P5.11
Page 334

5.12 For the cylinder of Problem 4.70, assuming no slip 5.16 To the data of Problem 5.15 we add a constant
and that the cylinder starts from rest, use work and kinetic counterclockwise couple of moment acting as
energy to find the speed of its center in terms of the dis­ shown in Figure P5.16. Repeat the problem.
placement of the center.
Ideally, the following five problems should be worked
sequentially:
5.13 A cylinder with mass 6 kg has a 20-N force applied
to it as shown in Figure P5.13. Find the angular velocity
of the cylinder after it has rolled through 90° from rest.

Figure P5.16

5.17 To the data of Problem 5.16, we add a spring, at­


tached to a cord wrapped around a second slot in the
cylinder near its outer rim as shown in Figure P5.17. The
spring has modulus 6 N/m and is initially stretched
Figure P5.13 0.2 m. Repeat the problem.

5.14 Rework Problem 5.13 if a slot is cut in the cylinder


and a cord is wrapped around the slot, with the 20-N
force now applied to the end of the cord as shown in
Figure P5.14. Neglect the effect of the thin slot on the
moment of inertia of the cylinder.

Figure P5.17

5.18 The 5-lb cylinder in Figure P5.18 rolls on the in­


cline. If the velocity of the mass center C is 5 ft/sec down
the plane in the upper (starting) position, find v in the
C

bottom position.
Figure P5.14

k = 2 lb/ft
5.15 Suppose in Problem 5.14 we remove some material
from the cylinder so as to offset the mass center C from Unstretched
length = 4 ft 2 in
the geometric center Q as shown in Figure P5.15. The
removal reduces the mass to 5.5 kg and makes the radius
of gyration with respect to the axis through C normal
to the plane of the figure k = 0.286 m. Repeat the
C

problem.
Figure P5.18

5.19 The spring in Figure P5.19 has an unstretched


length of 0.8 m and a modulus of 60 N/m. The 20-kg
wheel is released from rest in the upper position. Find its
angular velocity when it passes through the lower
Figure P5.15 (dashed) position if its radius of gyration is k = 0.2 m.
C
Page 3 3 5

5.22 The wheel in Figure P5.22 has a mass of5slugs and


a radius of gyration for the z axis through C of 0.7 ft. The
spring has modulus 20 lb/ft and natural length 4 ft. The
wheel is released from rest, and it rolls without slipping
on the plane. Find how far down the plane the mass
center C will move.

Figure P5.19

5.20 Bar, in Figure P5.20 is initially at rest in the vertical


position, where the spring is unstretched. The wall and
floor are smooth. Point B is then given a very slight dis­
placement to the right, opening up a small angle ­
a. Draw a free-body diagram of the slightly dis­
placed bar and use it to show that the bar will
Figure P5.22
start to slide downward if
b. Find the angular velocity of as a function of
for such a spring. 5.23 The wheel in Figure P5.23 weighs 200 N and has a
radius of gyration 0.3 m with respect to the z axis. It is
C

released from rest with the spring stretched If there is

a. up, and
b. down the plane in the subsequent motion.

Stiff rod
Modulus k

Figure P5.20

20 kg (eachI

Figure P5.23

Figure P5.21
5.24 Show that if the rolling body in Example 5.8 is a
sphere instead of a cylinder, it will slip at the angle
5.21 Find the spring modulus k that will result in the satisfying the equation
system momentarily stopping at after being re­
leased from rest at if the initial stretch in the
spring is zero. (See Figure P5.21.) Hint: Use symmetry!
Page 336

5.25 Use in Problem 4.87(b) tofindthe velocity 5.30 For the data of Problem 4.113 use to find
of C when it has moved 3 m down the incline. the speed of the plate as a function of the distance x itC

• 5.26 In Problem 4.134 use the principle of work and has traveled to the right. Use the x = x (t) result to check
C C

energy to obtain an upper bound on the rod's angular your answer; differentiate and eliminate t to produce the
speed in its subsequent motion after the right-hand string same result. is cut.
5.31 Figure P5.31 shows a fire door on the roof of a
5.27 A thin disc of mass m and radius a is pinned building. The door , 4 ft wide, 6 ft long, and 4 in. thick,
3

smoothly at A to a thin rod of mass m/2 and length 3a is wooden (at 30 lb/ft ) and can rotate about a friction-
(see Figure P5.27). The rod is then pinned at B. If the body less hinge at O. A cantilever arm of negligible weight is
is held in equilibrium in the configuration shown, then its free end. During a fire the link melts and the door
swings open 45°. Find the angular velocity of the door
released from rest, find the velocity of point A as the
just before the 150-lb weight hits the roof: (a) with no
system passes through the vertical. 2
snow on the roof; (b) with snow at 1 lb/ft on the roof.
5.28 Repeat the preceding problem if the pin at A is re­
placed by a weld.

Root

Hinge
Figure P5.31

5.32 Block in Figure P5.32 is moving downward at


Figure P5.27
5 ft/sec at a certain time when the spring is compressed
1 ft. The coefficient of friction between block and the
5.29 The 10-lb wheel shown in Figure P5.29 is attached plane is 0.2, and the radius of cylinder is 0.5 ft. Weights
at its center to a spring of modulus 20 lb/in. The radius of
gyration of the wheel about the center is 2.5 in. The wheel a. Find the distance that falls from its initial po­
rolls (no slip) after being released from rest with the sition before coming to zero speed.
spring stretched 1 in. Find: (a) the maximum magnitude b. Determine whether or not body will start to
of force in the spring; (b) the maximum speed of the move back upward.
center of the wheel during the ensuing motion.

Figure P5.32

Figure P5.29 5.33 The system in Figure P5.33 consists of a cylinder


(100 kg) and (equilateral) triangular plate (20 kg)
pinned together at the mass center C of the cylinder. The
1

* Asterisks identify the more difficult problems. other two vertices of the plate are connected to springs,
Page 337

the left one of which (S ) remains vertical in the slot.


1

(Spring S is shown only in its initial position.) The initial


1

stretches of the two springs (in the position shown) are


0.2 m for S and 0.04 m for S . The moduli are 40 N/m
1 2

for S and 10 N/m for S . If the system is released from


1 2

rest in the given position, find the velocity of C when


1

vertex B reaches its lowest point in the slot. Assume suffi­


cient friction to prevent, from slipping on the plane. The
moment of inertia of an equilateral triangular plate of
2
side s about its z axis is ms /12.
C

Figure P5.35
Figure P5.36

Figure P5.37

Figure P5.33

5.34 The mass center C of a rolling 2-kg wheel of radius


R = 15 cm is located 5 cm from its geometric center Q. Figure P5.38
(See Figure P5.34.) The spring is attached at C and is not
shown in position 2; its unstretched length is 0.3 m, 5.36 The center of mass of a uniform triangular plate is
and its modulus is 3 N/m. The radius of gyration is two-thirds of the distance from any vertex to the opposite
k = 0.09 m. Find the angular speed in position 2 (one-
C
side. The moment of inertia of an equilateral triangular
quarter turn from position 1). 2
plate is ms /12 with respect to the z axis through C. For
the plate shown in Figure P5.36, with mass 30 kg and
side 2 m, find its angular velocity when it reaches the
dotted position where C is beneath O. The spring has
unstretched length 0.5 m and modulus 20 N/m, and the
plate is released from rest.
5.37 The bar in Figure P5.37 weighs the same (W) as the
hoop to which it is welded. The combined body is re­
leased from rest on the incline in the position shown. If
Figure P5.34 there is no slipping, determine the velocity of Q after one
revolution of the hoop.
• 5 . 3 5 The three rods shown in Figure P5.35 are pinned 5.38 Cylinders and in Figure P5.38 are released
together with one vertex also pinned to the ground. The from rest and turn without slip at the contact point. A cord
length of the bar labeled is given by 2b = 0.4 m, and is wrapped around an attached hub of each, which has
3
the density of the material of all bars is 7850 kg/m . Their negligible effect on the moment of inertia. There is
2
cross-sectional area is 0.002 m . Find the angular velocity enough friction to prevent the rope from slipping on the
of the combined body after it swings 90° from rest if: (a) pulley. Find the velocity of the mass center of the pulley
after body has fallen 20 ft.
Page 338

5.39 A 5-lb cylinder is raised from rest by a force


P = 20 lb. (See Figure P5.39.) Find the modulus of the
spring that will cause the cylinder to stop after its center
has been raised 2 ft. Will it then start back down? The
spring is initially unstretched.

Figure P5.41

Figure P5.39

Figure P5.42
5.40 Body in Figure P5.40 rolls to the right along the
plane and has a radius of gyration with respect to its axis
of symmetry of k = 0.5 m. The corresponding radius of
c
5.42 The 20-lb wheel in Figure P5.42 has a radius of
gyration for is 0.12 m. The spring is stretched 0.6 m at gyration of 4 in. with respect to its (z ) axis. A cable
C

an instant when Find after C has trav­ wrapped around its inner radius passes under and over
eled 1 m to the right. (C is an externally applied couple
0
two small pulleys and is then tied to the 50-lb block
acting on .) The spring has a modulus of 90 lb/ft and is constrained
to remain horizontal. There is sufficient friction to pre­
5.41 The cylinder and the block each weigh 100 lb. They vent from slipping on the plane, (a) If the system is
are connected by a cord and released from rest on the released from rest, find the angular speed of after the
inclined plane as shown in Figure P5.41. The spring, con­ block then falls 1 ft. (b) Would the answer be different if
nected to the center C of the cylinder, is initially stretched block were replaced by a device that keeps the cable
6 in. Find the velocity of the block at the instant the spring force constant at 50 becomes unstretched,
lb? Why or why not? if there is sufficient friction be­
ping.
* 5.43 Rod and disk in Figure P5.43 have weights
W = 5 lb and W = 6 lb. The rod's length is 8 in., the
1 2

disk's radius is 4 in., the mass center offset of the disk is


2 in. from Q, and the radius of gyration of the mass of
with respect to the z axis through C is 3 in. It is desired to
attach a spring between point Q and afixedpoint so that

Figure P5.40 Figure P5.43


Page 3 3 9

the disk and rod come to a stop (in the dotted position) 5.47 In Problem 4.108 determine the velocity of corner
after turns 90° clockwise from rest. The spring has a B of the half-cylinder when the diameter AB becomes
modulus of 25.5 lb/ft and an unstretched length of 4 in.; horizontal for the first time.
it is to be unstretched initially. Find the final spring 5.48 Body translates in the slot without friction. (See
stretch, and from this result determine where to attach the Figure P5.48.) Disk (radius R) is pinned to block
fixed end of the spring. (There are two possible points!) through their mass centers at G. Body and body each
5.44 The bodies in Figure P5.44 have masses m = 0.3 1
has mass m; body has mass 2m. The system is released
slug, m = 0.5 slug, and m negligible. A spring is at­
2 3
from rest a distance D above the floor. Find: (a) the start­
tached to A that is stretched 25 in. in the dotted position ing accelerations of and G; (b) the velocity of when it
when everything is at rest. Find the spring modulus if hits the floor, using
when is horizontal.
5.45 A vertical rod is resting in unstable equilibrium
Light pulley
when it begins to fall over. (See Figure P5.45.) End A
slides along a smooth floor. Find the velocity of the mass
center C as a function of L, g, and its height H above the Cord

floor.
5.46 Pulley weighs 100 lb and has a centroidal radius
of gyration k = 7 in. (See Figure P5.46.) The disk pulley
C

weighs 20 lb. Find the velocity of weight (50 lb) after


it falls 2 ft from rest. (Assume that the rope does not slip
on the pulleys.)

Figure P5.48

N o slip
5.49 Link weighs 10 lb and may be treated as a uni­
Initial position form slender rod (Figure P5.49). The 15-lb wheel is a cir­
cular disk with sufficient friction on the horizontal sur­
face to prevent slipping. The spring is unstretched as
shown. Link is released from rest, and the light block
slides down the smooth slot. Neglecting friction in the
pins, determine: (a) the angular velocity of the link as A
Figure P5.44
strikes the spring with horizontal; (b) the maximum
deflection of the spring. (The modulus k of the spring is
10 lb/in.)

Figure P5.45 Figure P5.46 Figure P5.49


Page 340

5.50 The masses of four bodies are shown in Fig­ a. Find how far to the right the mass center moves
ure P5.50. The radius of gyration of wheel with respect in the ensuing motion, assuming sufficient fric­
to its axis is k = 0.4 m. Initially there is 0.6 m of slack
C tion to prevent slipping.
in the cord between and the linear spring. (Modulus b. When stops instantaneously at its farthest
k = 1000 N/m, and the spring is initially unstretched.) right point, what increase in the 50-N force and
Determine how far downward body will move. what minimum friction coefficient are needed
to keep it there?
5.58 The slender nonuniform bar in Figure P5.58 (the
mass is m and the radius of gyration with respect to the
mass center C is L/2) is supported by two inextensible
wires. If the bar is released from rest with , find the
tension in each wire as a function of
5.59 The system depicted in Figure P5.59 is released
from rest with 2 ft of initial stretch in the spring. There is
sufficient friction to prevent slipping at all rimes. Deter­
mine whether will leave the horizontal surface during
the subsequent motion. Note that the string S goes slack if
the stretch tries to become negative.

Figure P5.50
Wrapped
cords .

Figure P5.I1
Figure P5.57

5.51 Cylinder, in Figure P5.51 is moving up the plane


with v = 0.3 m/s at an initial instant when the spring is
C

stretched 0.2 m. If does not slip at any time, determine


how far down the plane the point C will move in the
subsequent motion. Note: The spring, connected to the
cord, cannot be in compression.
5.52 Solve Problem 4.64 for v as a function of x using
C c

Figure P5.58
5.53 In Problem 4.165, find the angular velocity of the
rod when
5.54 Solve Problem 4.65 (a) by
5.55 For each of the wheels in Problem 4.65, solve for v C

as a function of x using
C . The wheels start from
rest.
• 5.58 Solve Problem 4.177 with the help of . Ig­
nore the hint.
5.17 The radius of gyration of the wheel and hub in
Figure P5.57, with respect to its axis of symmetry through
C, is k = 2.5 m. The springs are unstretched at an initial
cC

position of rest, when the 50-N force is applied. Figure P6.59


Page 341

5.60 The system is released from rest in the position 5.63 The cord connects the slotted cylinder to the cyl­
shown in Figure P5.60. Force P is constant, 60 lb, and the inder as shown in Figure P5.63. Assume that neither
cord is wrapped around the inner radius of . Note the body slips after the system is released from rest. The
mass center of is at C. Find the normal force exerted spring is initially unstretched, and is stiff and guided so it
onto by the plane (after using to get ) at the can take compression. Find the angular velocity of after
instant when has rotated . The spring is initially its center C has moved 1 m.
unstretched, and there is enough friction to prevent slip­ 5.64 The 12-ft, 32.2-lb homogeneous rod shown in
ping. Figure P5.64 is free to move on the smooth horizontal
and vertical guides as shown. The modulus of the spring
is 15 lb/ft and the spring is unstretched when in
the position shown. Rod is released from rest with
and nudged to the right to begin motion, (a)
Determine the angular velocity of the rod when it be­
comes horizontal, (b) What is the angular acceleration of
the rod in this position ?
5.65 The 50-kg wheel in Figure P5.65 is to be treated as a
Figure P5.60 cylinder of radius R = 0.2 m. If it is rolling to the left with
v = 0.07 m/s at an initial instant when the spring is
c

unstretched, find: (a) the distance moved by C before v is c


5.61 The two identical links and in Figure P5.61, instantaneously zero; (b) the minimum coefficient of fric­
each of mass m and length are pinned together at A, and tion that will prevent slip.
is pinned to the ground at B. The end C of slides in the
vertical slot. Friction is negligible, and the system is re­
leased from rest. Find the velocity of point C just before
point A reaches its lowest point.
5.62 The body of mass 2m in Figure P5.62 is composed
of two identical uniform slender rods welded together. If Cylinder 1 0 0 N
friction in the bearing at O is neglected and the body is
released from rest in the position shown, find the magni­ Figure P5.63
tude of the force exerted on the rod by the bearing after 120 N/m
the body has rotated through 90°.

Figure P5.64
Figure P5.61

Figure P5.62 Figure P5.65


Page 342

5.66 The cylinder in Figure P5.66 is rolling at 5.69 A slender uniform rod of weight W is smoothly
rad/sec in the initial (i) position, where the spring is hinged to a fixed support at A and rests on a block at B.
unstretched. Other data are: (See Figure P5.69.) The block is suddenly removed. Find:
(a) the initial angular acceleration and components of re­
m = 2 slugs
action at A; (b) the components of reaction at A when the
r = 3 ft rod becomes horizontal.
k = 3 lb/ft 5.70 Two quarter-rings are pinned together at P and re­
leased from rest in the indicated position (Figure P5.70)
on a smooth plane. Find the angular velocities of the rings
when their mass centers are passing through their lowest
points. Hint: By symmetry, point P always has only a
vertical velocity component; this means that no work is
done on either ring by the other, because (again by sym­
metry) the force between the rings has only a horizontal
component normal to the velocity of P. More generally, as
long as the pin is smooth, the work done by two pinned
bodies in motion on each other will be the negative of
each other because the velocities will be equal whereas
the forces will be opposites.
• 5.71 A slender rod is placed on a table as shown in Fig­
ure P5.71. It will begin to pivot about the edge E and, at
Figure P5.66 some angle , it will begin to slip. Find this angle, which
will depend on the coefficient of friction and on k. Hint:
Use all three equations of motion together with
Find the final position of C (x ) at which either the cylin­
C Eliminate and , obtaining expressions for f and N.
der has stopped (for an instant) or started to slip, which­ Setting then permits a solution for . Solve the
ever comes first. Hint: Try one, check the other! resulting equation when and k = 0.25.
5.67 The uniform slender rod in Figure P5.67 (mass = 5
slugs, length = 10 ft) is released from rest in the position
shown. Neglecting friction, find the force that the floor
exerts on the lower end of the rod when the upper end is
6 ft above the floor. Hint: First use a free-body diagram
and the equations of motion to deduce the path of the
mass center.
5.68 In Figure P5.68, the ends of the bar are constrained
Figure P5.69
to vertical and horizontal paths by the smooth rollers in
the slots shown. The bar, originally vertical, is very gently
nudged at its lower end to initiate motion. Find the reac­
tions onto the bar at A and B just before the bar becomes
horizontal.

Figure P5.70

Figure P5.67 Figure P5.68 Figure P5.71


Page 343

• 5.72 The uniform equilateral triangular plate ABE in Fig­


ure P5.72 weighs W and is pinned to a fixed point at A and
to a rope at E. The rope passes over a small, frictionless
pulley at D and is then tied to the (equal) weight W which
is constrained to move vertically. If the system is released
in the given position with the angular velocity of ABE
being , find the angular velocity of ABE in the Next, show that an identical result is obtained (as it must
dashed position (i.e., when side AE becomes horizontal).

• 5.74 The cylinder in Figure P5.74 has a spring at­


tached and is released from rest. Assume that there is
sufficient friction between and the plane to prevent slip
throughout the motion, and that the slot around which
the cord is wrapped has a negligible effect on the cylin­
der's moment of inertia. Find the velocity of B when the
unwrapped length of rope is completely vertical (that is,
when C has 0.3 m left to travel before it would be directly
above E). Assume also that the weight moves only
vertically — that is, that the rope does not start to sway.

Figure P5.72
Smooth,lightpulley

•5.73 This problem, continuing and using the results of


Problem 4.150, is to verify that for that solution.
First, note that only the friction couple does net work on
the system of plus , , and verify: Figure P5.74

5.3 T h e Principles of Impulse and Momentum


The Equations of Impulse and Momentum for the Rigid Body in Plane Motion
The principle of work and kinetic energy is very helpful when the prob­
lem is posed in terms of positions and velocities. When time, rather than
position, is the main concern, we often draw on a pair of principles
concerned with impulse and momentum vectors. Just like , these
principles are obtained by general integrations of the equations of mo­
tion, but now the integration is directly with respect to time, without first
dotting the equations with velocity. Thus they leave us with a set of
vector equations instead of a single scalar result.
We have encountered one of the principles in Section 2.5 in our study
of mass center motion: The impulse of the external forces imparted to any
system equals its change of momentum over the same time interval. In
Chapter 2 the system was general, so this principle holds for the rigid
bodies we are now studying. From Equation (2.27), so that
Page 344

(5.26)
Reviewing, we note that the integral dt is called the impulse (or
linear impulse) imparted to the system by external forces. The vector
is the change in the system's momentum (or linear momentum) from the
initial to the final time.
We need only the x and y components of Equation (5.26) for the rigid
body in plane motion:
(5.27)
(5.28)
There is also a corresponding principle of angular impulse and momen­
tum. From Equation (2.43),

so that

(5.29)

This equation may be put into a convenient form for rigid bodies in plane
motion by recalling Equation (4.4) for the angular momentum:

For symmetric bodies in which the products of inertia vanish and


this equation becomes

Therefore

or
(5.30)
The integral is called the angular impulse imparted to the
system by the external forces and couples, and the quantity is the
change in angular momentum, both taken about C.
A subtle but important point regarding Equation (5.30) must be
understood here. We note from Equation (5.29) that angular impulse
equals the change in angular momentum for any body (deformable as
well as rigid); therefore the use of Equation (5.30) only requires that the
body of interest behave rigidly at the start (t ) and end (t ) of the time
i f

interval (t , t ). At those times the moment of momentum is


i f

even though this simple expression for H may not apply between t, and
C i

t . A good example is an ice skater drawing in her arms to increase angular


f

speed, as we shall see later in Example 5.14.


Page 345

In summary, for the rigid body in plane motion we have the follow­
ing two principles at our disposal:
1. Linear impulse and momentum:

from which we get

(5.31)

(5.32)

where the directions of x, y, and z are fixed in the inertial frame.


2. Angular impulse and momentum:

(5.33)

We note also that if the products of inertia are not zero, we have
(5.34)
and
(5.35)

Oust lion 5.5 Are the coordinate axes associated with Equations (5.34)
and (5.35) the same as those of (5.31) and (5.32)?

In the remainder of this section we treat only examples of symmetric


bodies, for which Equations (5.31) to (5.33) are the impulse and momen­
tum equations. The first example deals with both linear and angular
impulse and momentum for a single body.

Answer S.5 Yes.

Figure E5.1 l a

EXAMPLE 5 . 1 1
A cylinder has a string wrapped around it (see Figure E5.11a) and is released
from rest. Determine the velocity of C as a function of time.
Solution
We choose the sign convention to be as shown in Figure E5.1 lb, since the cylin­
der turns clockwise as C moves downward. Applying the impulse and momen­

Figure E5.11b tum equations in the y and J directions (note that means so that
gives
Page 346

(1)

and

(2)

We note that it is itself an unknown and should be treated as such. (In this
problem, use of the equations of motion would separately tell us that T = mg/3,
so that the integral is in fact Tt. But sometimes T is time-dependent, in which case
Tt would be incorrect for the value of the integral.)

Eliminating

However, because the cylinder rolls on the rope, so that

which gives our result:

In the next example, several bodies are involved, making the solution
a little more difficult. Two limiting case checks are discussed at the end.

The cylinder falls with ' of a g" because of the retarding force of the rope.
EXAMPLE 5 . 1 2
The cart shown in Figure E5.12a has mass M exclusive of its four wheels, each of
which is a disk of mass m/2. The front wheels and their axle are rigidly con­
nected, and the same is true for the rear wheels. If the axles are smooth, find the
velocity of G (the cart's mass center) as a function of time. The system starts from
Figure E5.12a rest. Assume that there is enough friction to prevent the wheels from slipping.
Page 347

Solution
We first consider the free-body diagram (Figure E5.12b) of a wheel pair (either
front or back). Since the front and rear wheels are constrained to have identical
angular velocities at all times, the ffs (front and back) must produce identical
la's; hence the friction force is the same for the rear wheels as for the front. And
since the wheels' mass centers must always have identical velocities, the forces
acting down the Diane on each pair (front and back) must also be equal. These
resultants are so the reaction A is also the same on each pair x

Figure E5.12b of wheels.

Question 5.6 Are A and N also the same for front and rear wheels?
y

From the free-body diagram, we may write the following linear and angular
equations of impulse and momentum:

(1)

(2)

We may also isolate a free-body diagram of the translating cart (Figure E5.12c)
and write its equation of impulse and momentum in the x direction down the
plane:

(3)
Figure E5.12c
Next we note that Cj and C , the mass centers of the front and back wheel pairs,
2

are also points of the cart; thus Using this relation, and adding
Equation (3) to twice Equation (1), eliminates the unknown impulse of the reac­
tion A : x

(4)

Similarly, adding Equation (4) to twice Equation (2) results in an equation free of
the unknown friction force:

Answer 5.6 N o t in g e n e r a l . T h e y d e p e n d o n t h e p o s i t i o n o f G r e l a t i v e t o C a n d C .
x 2

* T h i s is t h e e q u a t i o n o f l i n e a r i m p u l s e a n d m o m e n t u m f o r t h e total ( n o n r i g i d ) s y s t e m o f
cart plus w h e e l s .
Page 348

Carrying out the integration and solving for , we get

Smooth

Figure I5.12d We note from this result that if the wheels are very light compared to the
weight of the cart (m M), then which is the answer for the
problem of Figure E5.12d. Thus light wheels on smooth axles makes the cart
move as if it were on a smooth plane, as expected.
The reader may wish to examine the other limiting case, that of the cart being
light compared to heavy wheels (M m). In this case the result, using the
free-body diagram in Figure E5.12e, is

Figure E5.12e
In the next example, the two bodies — one rolling and the other
translating — are connected by an inextensible cord.

EXAMPLE 5 . 1 3
Force P acts on the rolling cylinder C beginning at f = 0 with C at rest. (See
Figure E5.13a.) Force P varies with the time t in seconds according to

(positive to the left as shown)

:
Cylinder C and body t- respectively weigh 100 and 40 N. Find the velocity of G
(the mass center of l<) when f ™ 10 s. Neglect the effect of the hubs in Fig­
ure E5.13b (and the drilled hole to accommodate force P) on the moment of
inertia of C.

0.1m
P 0o m

Cord

Light
pulley

Figure E 5 . H a Figure E8.13b

Solution
Using the free-body diagrams (Figures E5.13c and E5.13d), we may write the
equations of impulse and momentum. On C, using Figure E5.13c,
Page 349

at

(1)

Also on C:

or
Figure E5.13c

(2)

On B, using Figure E5.13d,

or

Figure E5.13d (3)

Subtracting Equation (3) from (1), after integrating the sine function, we obtain

(4)

To obtain a second, independent equation that is also free of the integral of


the unknown tension T, we add Equation (1) to twice Equation (2):

(5)

Multiplying Equation (4) by 0.6 and subtracting from Equation (5) gives

(6)

Kinematics now relates and the lowest point of C has the same
velocity magnitude as does G because of the inextensibility of the cord (see
Figure E5.13e):

Kinematic conditions:

Substituting these expressions foi and into Equation (6) gives


Figure E5.13e

Hence the velocity of the mass center of # a t t = 10 s (when the force changes
direction) is
Page 350

We emphasize that Equations (1), (2), and (3) in the preceding exam­
ple are merely first integrals of the equations of motion studied in Chap­
ter 4.

Conservation of Momentum
As we saw in Section 2.5, if the force in any direction (let us use x, for
example) vanishes over a time interval, then the impulse in that direction
vanishes also:

Since this impulse equals the change in momentum in the x direction, we


have zero change when and thus the momentum is conserved in
that direction between r- and t :
( f

or
mx = mx^
C( (5.36)
We would of course also have conservation of momentum in the y (or
any other) direction in which the force resultant vanished.
Finally, if the z component of 2M,- is zero between t and t , then the
{ f

angular impulse vanishes and we have conservation of angular mo­


mentum:

or

(5.37)
Fnr nlane motion of symmetric bodies (l£ and J £ = 0) we have
there is then no need for the z subscript, and Equation (5.37)
may be rewritten
(5.38)
We now consider a well-known example of conservation of angular
momentum.

EXAMPLE 5 . 1 4
A skater spinning about a point on the ice (see Figure E5.14) draws in her arms
and her angular speed increases.
a. Is angular momentum conserved?
b. Is kinetic energy conserved?
Figure E5.14 c. Account for any gains or losses if either answer is no.
Page 351

Solution
We begin with part (a). Before the skater draws in her arms, we may treat her as a
rigid body and thus . The same is true after the arms are drawn in, so
that If we neglect the small friction couple at the skates and the small
drag moments caused by air resistance, then the answer to part (a) is yes because
XMc is then zero. Thus

Therefore

(showing an angular speed increase since 1 > I )


1 2

For part (b) the kinetic energies are

Thus kinetic energy is not conserved.


For part (c) the change in kinetic energy is seen to be positive:

Since there is no work done by the external forces and couples,* it is clear that this
kinetic energy increase is accompanied by an internal energy decrease within the
skater's body as her muscles do (nonexternal) work on her (nonrigid) arms in
drawing them inward. Since total energy is always conserved (first law of ther­
modynamics), the skater has lost internal energy in the process.

The next example is similar to Examples 2.18 and 2.19 except that
now the pulley has mass.

EXAMPLE 5 . 1 5
Two identical twin gymnasts, L and R, of mass m are in equilibrium holding onto
a stationary rope in the position shown in Figure E5.15a. The rope passes over the
pulley B, which has moment of inertia I with respect to the axis through O normal
to the figure. The gymnasts then begin to move on the rope at speeds relative to it
of y upward and y downward. When gymnast R reaches the end of the rope,
L rel R rel

he discovers he is in the same spot in space at which he began. How far up or


down (tell which) has L moved (a) relative to the rope? (b) in space?

Figure E5.15a * A s s u m i n g h e r a r m s a r e d r a w n in at t h e s a m e level.


Page 352

Solution
If we select our system to be "everything": L, R, the rope, and B, then
and thus angular momentum about O is conserved. (Note that the gymnasts'
gravity forces' moments about O cancel, and the pin reactions and weight of B
pass through O.) Therefore,
(since all bodies are at rest initially) (1)
After motion begins, the angular momentum about 0 of Bis simply \ o) \sL, since e
0 B

O is a pivot of B. For L and R, however, the situation is different and needs some
discussion. The gymnasts, of course, are not rigid bodies, and we shall resort to
Equation (2.38) to write their angular momenta with respect to O. For L, with
velocity v in the inertial frame (the ground), and mass center Q ,
L

We now assume that the gymnast is a particle; this is equivalent to neglecting


, the gymnast's angular momentum about his mass center, in comparison
with the "r X mv" term. This is a good assumption because whatever body
motions are not translatory are caused by parts (arms, mostly) in relative motion
fairly close to the mass center. Thus,

where P is shown in Figure E5.15b. Similarly,

Question 5.7 Why is r OCl[ X mv = r R 0 0 X mv„?

Now, if we locate (see Figure E5.15b) the gymnasts' position in space with the
coordinates y and y , then
L R

Note that if the gymnasts move in opposite vertical directions on opposite sides of
the pulley, their angular momenta about O will be in the same direction. Note
Figure E5.15b further that even though the "particles" L and R are in rectilinear motion, they
still have angular momenta about points such as O that do not lie on their lines of
motion.
Substituting the three angular momenta into Equation (1), we find:

(2)

Next, we know from the data that the rope moves counterclockwise around the
pulley. Therefore, calling its speed y^, we have (see Figure E5.15c)

and
Starting p o s t i o n of
end of rope at t = 0

Figure E5.15c Answer 5.7 Because r ^ , = r O Q + r Q C i , and r Q C < X mv Cm = 0 since T


QCK and v C | ( a r e parallel
Page 353

Substituting these into Equation (2), and simplifying the result, we obtain the
equation
(3)
Integrating,

ButC = 0 since y . = y = y = 0 at t = 0. Now at thetimewhen R is at the


1 rope r e l R rel
=
end of the rope, y = H, and y
R rel H also, so that
rope

Therefore

and gymnast L moves u p r e l a t i v e to the rope and in space from


his original position.

We can check Example 2.18 by going back to Equation (2) of the


preceding example, and letting J —» 0 (note that at this point nothing has
been said about the relative motions):

or

Therefore, regardless of relative motions (with respect to the rope), the


two gymnasts rise in space equal amounts. The reader may wish to show
that for the problem of Example 2.18 but with I > 0, the left gymnast is
pulled up less than the height of climb in space of the right gymnast.
We now consider an example in which (unusually) both angular
momentum and kinetic energy are conserved.

EXAMPLE 5 . 1 6

The 2-kg collar Cin Figures E5.16a,b turns along with the smooth rod I? (see
Figure E5.16a), which is 1 m long, has a mass of 3 kg, and is mounted in bearings
with negligible friction. The angular speed is increased until the cord breaks (its
tensile strength is 60 N), and at that instant the external moment is removed.
Determine the angular velocity of I? and the velocity of C (the mass center of C)
when the collar leaves the rod.
Page 354

Figure E5.16a Figure E5.16b

Solution
The string provides the force causing the centripetal (inward) acceleration until it
breaks. At that instant, we may solve for the angular velocity of C:

In the accompanying free-body diagram, Figure E5.16c, Nj and N are the verti­
2

cal and horizontal resultants of the pressures of the inside wall of exerted by
After the rope breaks at time f,, collar C moves outward in addition to
turning with /?; this is because there is no longer any inward force to keep it from
"flying off on a tangent." Between times t, and f (when it leaves R), we have the
2

following for the system


Figure E5.16c
1. Conservation of angular momentum H about z (because the external
a c

forces have no moment about z ). Q

2. Conservation of kinetic energy T (since no net work is done on the system).


Note that the normal forces between rod and collar, being equal in magni­
tude but opposite in direction, act on points with equal velocity components
in the direction of either force; hence their net work vanishes.
Condition 1 gives

Until the string breaks,


O is a point of both bodies!

Thus
Page 355

The component of v perpendicular to the rod /? is thus v = 1.05afy


c c

= 3.87 m/s. We can now obtain the radial component by conservation of T


(condition 2):

Ois for both e components of y parallel


c

and/Pinitially and perpendicular to *

Thus since the initial kinetic energy was 59.2 J and since

we see that 63 percent of the original energy has gone into the outward motion of
the collar.

Impact
We studied the impact of a pair of particles in Section 2.5. In this section
we shall extend this study to two bodies colliding in plane motion.
The large forces occurring during an impact between two bodies
and <B obviously deform the bodies. Because of vibrations and perma­
2

nent deformations that are produced, some of the mechanical energy will
be dissipated in the collision. However, it is often possible to treat a body
as rigid before, and then again after, the impact in order to gain informa­
tion of value. In impact problems we assume that:
1. Velocities and angular velocities may change greatly over the short
impact interval
2. Positions of the bodies do not change appreciably.
3. Forces (and moments) that do not grow large over the interval At are
neglected (such as gravity and spring forces). Such forces are called
nonimpulsive; the large contact forces are called impulsive. It is the
impulsive forces and moments that produce the sudden changes in
velocities and angular velocities.
In Chapter 2 we introduced the coefficient of restitution as a measure
of the capacity for colliding bodies to rebound off each other. We shall
continue to use this parameter in this section, where now the relative
velocities of separation and approach are of the impacting points of B x

and B . Thus rigid-body kinematics will be needed to relate these veloci-


2
Page 356

ties to those of the mass centers of the bodies. We emphasize again that
the coefficient of restitution "e" is not the best of physical properties to
measure; it depends upon the materials, geometry, and initial velocities.
But as long as we take "e" with a grain of salt and remain aware of the
limiting values e = 0 (bodies stick together) and e = 1 (no loss of energy),
the definition of e does provide an approximate, much-needed equation
that allows us to solve many problems of impact. We now consider two
forms of the angular impulse and angular momentum equation that are
applicable at the beginning and end of impacts involving the plane mo­
tion of bodies that may be regarded as rigid except during the collision
phase of the motion.
If the body has a pivot O, we recall that

With O fixed in the inertial frame we have

Thus we may replace the C by an O in the angular impulse and momen­


tum equation (5.33) for such pivot cases. The resulting equation about
the axis of rotation is

(5.39)

This formula is of considerable value in impact problems because impul­


sive pivot reactions have no moment about O and thus do not appear in
the equation. Note that if O is C, then Equation (5.39) is the same as our
previous Equation (5.33) written about the mass center.
Another useful equation follows from

(5.40)

In scalar form, for rigid bodies in plane motion this equation is

in which P is an arbitrary point. If we state that P is now a fixed point O of


the inertial frame J, we may integrate this equation, getting

(5.41)

* W e e m p h a s i z e a g a i n t h a t t h e a n g u l a r m o m e n t u m H (or H ) is n o t e a u a l t o its rigid


O C

body form during t h e i m p a c t , b u t t h e s e s u b s t i t u t i o n s m a y b e m a d e a t t,


b e f o r e t h e collision a n d a t t a f t e r w a r d .
f
Page 357

QUESTION 8 . 8 Why is the right-hand side not the integral of the right
side of Equation (5.40) if O is moving?

Answer 5.B If 0 is n o t fixed in then and


t h e s e c o n d t e r m is n o t z e r o t h e n !

We shall now use these principles to solve a pair of example prob­


lems.

EXAMPLE 5 . 1 7
An arrow of length L traveling with speed strikes a smooth hard wall obliquely
as shown in thefigure.End A does not penetrate but slides downward along the
wall without friction or rebound. Find the angular velocity of the arrow after
impact.
Solution
The only impulsive force acting on the arrow during its impact with the wall is the
normal force N shown in the free-body diagram in Figure E5.17. We note that the
gravity force over the short time interval is nonimpulsive:

This is negligible in magnitude if is very small, since mg does not grow large

and this is non-negligible since N grows large "impulsively" during the short
interval with average value . In what follows, we shall delete the
subscript and simply denote the impulse of
The impulse and momentum equation is then:
Figure E5.17

or

(1)

and
(2)
where we note that momentum is conserved in the y-direction during impact.
The angular impulse and angular momentum principle yields:

(3)

"No rebound" means the x-component of is zero; thus:


Page 358

Using , we find that the x-component of this equa­


tion is:

(4)

We have four equations in the unknowns and . Solving, we


find

Note the obvious, that a head-on impact brings the arrow to a dead
stop with and, from Equation also.

The next example, and the comments following it, constituted the
solution to an actual engineering problem.

EXAMPLE 5 . 1 8
A 770-ton steel nuclear reactor vessel is being transported down a 6.5 percent
grade using a specially designed suspended hauling platform together with
crawler transporters. (See Figure E5.18a.) Determine the maximum velocity at
which the reactor can be transported without tipping over if it should strike, and

Figure E5.18a (Courtesy American Rigging Co.)

Solution
The reactor vessel will tip over if there is any kinetic energy left after it pivots
about the front edge at O (see Figure E5.18b) and the mass center reaches its
highest point B, directly above O. Thus we solve for the velocity that will cause C
to reach B; any higher velocity will cause overturning.
Page 359

C = position of mass B = position of mass


center at instant of center at point of no return
contact at O
Figure E5.18b

We begin the solution with some preliminary geometric and trigonometric


calculations based on the diagram. The 6.5 percent slope means
.* The turnover height h, is given by the distance OC:

Also

and

The initial height h of C, above the horizontal line through O, is


t

and thus the vertical distance through which C will move in reaching B is

* W e u s e f o u r significant digits in this e x a m p l e .


Page 360

(If we had adhered to three significant digits, the subtraction would have reduced
us to just one good digit.)
There now remain two separate main parts to the solution of this problem.
We first have to consider that mechanical energy is lost during the impact of
with the obstacle at O. Thus we are prevented from using the principle of work
and kinetic energy over the short period of impact. What does apply, however, is
conservation of angular momentum about O. This is because the impulsive forces
(in both the x and y directions!) causing the sudden changes in the mass center
velocity v and in the angular velocity to are acting at O, so that
c

Therefore

or
(1)
We shall use Equation (2.36) to express this is the best formula for
for translation problems because in that case. For however, we have
a nonzero to. Thus we draw on the fact that since the vessel does not bounce at O,
we may consider O a fixed point of both and the inertial frame during and
following the short period of impact. This in turn means that is simply
after impact. Therefore Equation (1) becomes

(2)
Now since

and
the left side of Equation (2) is simply

(3)

By the parallel-axis (transfer) theorem for moments of inertia we have

The vessel is essentially a thick shell; considering then that

(4)

we see that

(5)

By substituting Equations (3) and (5) into (2), we thus obtain

and the angular velocity after impact is then

(6)
Page 361

We are now ready to proceed to the second part of the solution.


Between the start of pivoting (immediately following impact) and the arrival
at point B, the system is easily analyzed by work and kinetic energy:

To obtain the least possible value of v for no overturning, we set


ci . The
only work done in this phase of the vessel's motion is by gravity, so that

where is now an initial angular speed for this final stage of the problem and O
is still a pivot point for Thus

(7)

or

There are several important follow-on remarks to be made about the


preceding example. The first is that it can be shown (with a coefficient of
restitution analysis) that more energy is lost with no bounce at O than if
rebounding takes place. This energy loss for the case just studied is

where and refer to the instants just before and after the impact.
Substituting (for the case of no bounce), we get

For = 65 ft and r = 10 ft, we obtain

Thus 32.2%, or nearly a third of the original mechanical energy, is


lost during impact if the lower front comer of sticks to, and pivots about,
point O. Of great importance here is the fact that we do not know how
much rebounding would actually occur in the physical situation and
hence how much energy would be lost. This means that 9.4 f t / s e c may
Page 362

not be a conservative engineering answer for the safe speed. If the plane is
flat , for example, it can be shown that:
1. The speed corresponding to pivoting as in this example is
11.9 ft/sec. (It has farther to pivot so it can be going faster prior to
impact.)
2. At this speed initially, and with a no-energy-lost rebound, the vessel
will easily overturn even though the striking comer backs up.
A conservative safe speed of the vessel in the inclined plane case can
be obtained by assuming that no energy is dissipated during the impact
and that all the vessel's initial kinetic energy goes into tilting it up about
O. This approach gives

In practice, the engineers in this case decided not to exceed 3 ft/sec, in


view of the importance of the work and the danger involved.

The Center of Percussion


We now turn our attention to a new topic. Besides the mass center C (but
of much lesser importance), there is another special point of interest
associated with a rigid body in plane motion, a point that differs from C
in that it depends not only on the mass distribution of but also on the
motion of the body. This point lies along the resultant of the ma vectors of
all the body's mass elements. The point is called the center of percussion,
and it has value in certain applications such as impact testing.
Before getting into the theory behind the center of percussion, we
first illustrate its existence and demonstrate its value by means of an
example. If a youngster hits a baseball with a stick and does not translate
his hands too much, we may model the situation as shown in Figure 5.13
and ask where the ball should hit the stick in order to eliminate the
"sting" (transverse reaction of the stick onto the boy's hands). If the
boy hits the ball at just the right place, called the "sweet spot," he hits it a
long way while hardly feeling it and is said to have gotten "good wood"
on the ball. Assuming the stick to be rigid at the beginning (f = 0) and end
of the short impact interval, the two principles of impulse and
momentum are used as follows.
The impulse-momentum equation for the stick in the direction is

Figure 5.13 (5.42)

The angular impulse-angular momentum equation is

(5.43)
Page 363

Using for "no sting," setting by kinematics at


and and multiplying Equation (5.42) by d gives

(5.44)

Dividing Equation (5.44) by (5.43) gives

Thus if the ball is struck two-thirds of the way from O to the end of the
stick, the transverse reaction will be zero.
In this example the point at which the ball is struck is the
center of percussion of the stick. To show this, at least for the case when
the bat is rigid, we first recall that in Chapter 2 we saw that for any point P
(moving or not, fixed to or not), we can always write

in which R is the position vector from point P to a generic differential


mass element. Therefore, since the integral of the vectors ( R X a dm) over
the body in fact represents the resultant moment about P of the ma
vectors over the mass of this integral vanishes for all points on the
line along which the resultant of the ma vectors lies and hence
for these points.
Armed with the fact that for the center of percussion, we
can now derive the general equation for the distance from a pivot O to the
center of percussion in plane motion (Figure 5.14):

Figure 5.14 Therefore

(5.45)

and we see that, for the stick,

(as before)

Note from Equation (5.45) that the center of percussion is always farther
from the pivot than is the mass center. We make one final remark about
the center of percussion. If we treat the "a dm's" of as a collection of
vectors, its resultant may be expressed (for a rigid body in plane motion)
at the mass center (Figure 5.15), where

Figure 5.15
Page 3 6 4

In a manner identical to reducing a force and couple to its simplest


form, we may reduce this resultant of the ma vectors as shown in Figure
5.16, where the distance D is . We note that there is in fact a line
of points along the resultant of the ma vectors, making the location of a
single point ambiguous. However, the concept of the center of percus­
sion is usually used in conjunction with problems in which the body has a
pivot O (as in the previous example). In these problems is the well-
defined single point at the intersection of lines and OC as in Figure 5.16.

Figure 5.16

EXAMPLE 5 . 1 9
Find the center of percussion for a pendulum consisting of a rod plus disk,
each of which has equal mass m. (See Figure E5.19.)
Solution
The mass center of is located at a distance from O given by

(Note that with equal masses C lies halfway between the mass centers of the rod

Figure E5.19
Thus m and Equation (5.45) then gives us the location of :

Striking the pendulum at eliminates the horizontal pin reaction at O, as we


have seen.
Page 3 6 5

PROBLEMS • Section 5.3

5.75 Drum has a radius of gyration of mass with re­ 5.77 The sinusoidal force P is applied to the string in
spect to a horizontal axis through 0 of 1 m and a mass of Figure P5.77 for a half-cycle. If the cylinder (initially at
800 kg. Body has a mass of 600 kg and a velocity of rest) does not slip, find its angular velocity at the end of
20 m/s upward when in the position shown in Figure the load application
P5.75. Find the velocity of later. 5.78 A massless rope hanging over a frictionless pulley
of mass M supports two monkeys (one of mass , the
other of mass 2M). The system is released at rest at
as shown in Figure P5.78. During the following 2 sec,
monkey B travels down 15 ft of rope to obtain a massless
peanut at end P. Monkey A holds tightly to the rope dur­
ing these 2 sec. Find the displacement of A during the
time interval. Treat the pulley as a uniform cylinder of
radius R.
Figure P5.75

5.76 The hollow drum shown in Figure P5.76 weighs


161 lb and rotates about a fixed horizontal axis through O.
The diameter of the drum is 2.4 ft, and the radius of
gyration of the mass with respect to the axis through O is
0.8 ft. The angular speed changes from 30 rpm to
90 rpm during a certain time interval. Find the time
interval.

Figure P5.76

Weight = W
Figure P5.78

5.79 Force F in Figure P5.79 varies with time according


to newtons, where f is measured in seconds. If
there is enough friction to prevent slipping of the cylinder
on the plane, find the velocity of C at: (a) t = 3 s; (b)
f = 10 s. The cylinder starts from rest at t = 0.

Figure P5.77 Figure P5.79


Page 366

5.80 The cylinder in Figure P5.80 has mass m = 3 slugs 5.83 Acting on the gear is a couple C with a time-
and radius of gyration ft with respect to C. There dependent strength given by C = (6 + 0.8f)N-m, where t
is sufficient friction to prevent slipping on the plane. A is measured in seconds. (See Figure P5.83.) If the system
rope is wrapped around the inner radius, and a tension is released from rest at t = 0, find the velocity of block
T = 40 lb is applied parallel to the plane as shown. Use when (a) t = 3 s; (b) t = 10 s. The centroidal radius of
impulse / momentum principles to find the velocity of C gyration of the gear is 0.25 m.
after 3 sec if motion starts from rest.

Figure P5.83
Figure P5.80

Solve the following problems by making use of the im­


pulse and momentum, and/or angular impulse and an­
gular momentum, methods.
Figure P 5 . 8 1 5.84 Problem 4.107
5.85 Problem 4.166(c)
5.81 The 161 -lb round body is rolling up the plane with 5.86 Problem 4.179
at the instant shown in Figure P5.81. 5.87 Problem 4.102
The radius of gyration of the mass of the body with re­
spect to the axis through the mass center C normal to the 5.88 Problem 4.101
page is 0.7 ft. Find the time required for the mass center to 5.89 Problem 4.87(b)
reach its highest point. 5.90 A pipe rolls (from rest) down an incline (Figure
5.82 The cylinder , turning at , is brought to P5.90). Using the equations of motion, find:
rest by applying the 50-1b force to the light brake arm as a. at time t
shown in Figure P5.82. Friction in the bearings at O pro­
duces a constant resistance torque of 71b-ft,and the coef- b. after C moves the distance .
ficient of friction at the contact point A between and Then use work and energy to verify the answer to part (b)
is (a) Find the stopping time, and (b) find the and impulse and momentum to verify part (a). Finally,
number of revolutions turned by during the braking. give the minimum to prevent slipping.
5.91 A body weighing 805 lb with radius of gyration
0.8 ft about its axis (see Figure P5.91) is pinned at its
mass center. A clockwise couple of magnitude 1b-ft is

Figure P5.82 Figure P5.90 Figure P5.91


Page 367

applied to £ starting at t = 0. Find the angular velocity of * 5 . 9 6 A child pulls on an old wheel with a force of 5 lb by
B when t = 3 seconds. means of a rope looped through the hub of the wheel.
5.92 Given that the slot (for the cord) in the cylinder in (See Figure P5.96.) The friction coefficient between wheel
Figure P5.92 (mass 10 kg) has a negligible effect on I , and ground is Find I for the wheel, and use it to
c
c

find the velocity of the mass center C as a function of time, determine the velocity of C 3 sec after starting from rest.
if 1 lb (each of 8) T h i n rim ( 3 lb)

Figure P5.96

Figure P5.92

5.97 Two cables are wrapped around the hub of the


10-kg spool shown in Figure P5.97, which has a radius of
gyration of 500 mm with respect to its axis. A constant
40-N force is applied to the upper cable as shown. Find
the velocity of the mass center C 5 sec after starting from
rest if: (a) (b)
Figure P5.93 The cart B is given an initial velocity v to the right
x {

at t = 0. The rod B is pinned to B at its mass center G, as


2 x

shown in Figure P5.98(a). At t = 0, the mass center C of B 2

is heldfixedat the instant the cart starts off, then immedi­


ately released. At a later time (see Figure P5.98(b)), it is
observed that B has2 at an instant when B has 2

turned 90° clockwise. If M = m, find the velocity of G at


that instant. Use and an impulse and momentum
Smooth principle.

Cable
600 m m
Figure P5.94 Figure P5.95
40 N

Cable
5.93 A uniform sphere (radius r, mass m) rolls on the 200 m m
plane in Figure P5.93. If the sphere is released from rest at Cart B , 1

t = 0 when x = L, find i(f). Mass M

5.94 The cord in Figure P5.94 is wrapped around the


cylinder, which is released from rest on the 60° incline Figure P5.97
shown. Find the velocity of C as a function of time f.
* 5.95 The 50-lb body C in Figure P5.95 may be treated as Rod S 2

a solid cylinder of radius 2 ft. The coefficient of friction Mass m


between C and the plane is and a force P = 10 lb length /

is applied vertically to a cord wrapped around the hub. Figure P5.98(a)


Find the velocity of the center C 10 sec after starting from
rest. Figure P5.98(b)
Page 368

5.99 Two gymnasts at A and B, each of weight W, hold 5.101 Two disks are spinning in the directions shown in
onto the left side of a rope that passes over a cylindrical Figure P5.101. The upper disk is lowered until it contacts
pulley (weight W, radius R) to a counterweight C of the bottom disk (around the rim). Find how long it takes
weight 2W. (See Figure P5.99.) Initially the gymnast A is for the two disks to reach a common angular velocity, and
at depth d below B. He climbs the rope to join gymnast B. determine its value. Finally, determine the energy lost.
Detenrune the displacement of the counterweight C at the Show that if = I and2 your solution pre­
end of the climb. dicts that 100 percent of the energy is lost (as it should).
5.100 DiskB1and the light shaft in Figure P5.100 rotate Determine which of the three answers (time, energy
freely at 40 rpm. Disk B (initially not turning) slides down loss) are the same if the two disks are instantaneously
2

the shaft and strikes B ; after a brief period of slipping, locked together instead of slipping.
1

they move together. Find the average frictional moment 5.102 Figure P5.102(a) shows a rough guess at a skater's
exerted on B by B if the slipping lasts for 3 sec.
1 2 mass distribution. Calculate the percentage increase in his
angular speed about the vertical if he draws in his arms as
shown in Figure P5.102(b). Assume that his arms are
wrapped around the 6-in. radius circle of his upper body.

A'JIO lb)

-Sphere: 0 0 8 m r = 4 in
;

Figure P5.99 *,(15 lb)

Stick: 0 0 9 m (each arm)


1.5 h
Cylinder: 0 2 8 m r = 6 in
;

Figure P5.100
Cylinder: 0 13m (each thigh); r = 3 in

Cylinder: 0 0 7 m (each); r = 1 5 in

Each foot a concentrated mass of 0 0 3 m

Coefficient of Figure P5.102(a)


friction = n

Shoulders

Figure P5.101 Figure P5.102(b)


Page 3 6 9

• 5.103 A starving monkey of mass m spies a bunch of If either answer is yes, give the relationship between d
delicious bananas of the same mass. (See Figure P5.103.) and H in the figure for which overtaking the bananas is
He climbs at a varying speed relative to the (light) rope. possible.
Determine whether the monkey reaches the bananas be­ • 5.104 A circular disk of mass M rotates without friction
fore they sail over the pulley of radius R if: about O. (See Figure P5.104.) A string passed over the
a. The pulley's mass is negligible disk (and not slipping on it) carries a mass M at each end.
b. The pulley's mass is fm, where f > 0 and the The system is released at rest as shown with the right-
radius of gyration of the pulley with respect to hand mass carrying a washer of mass M. As the system
its axis is k. moves, the left-hand weight picks up a washer of mass M
at the same instant the right mass deposits its washer.
Find the velocity of the right-hand weight just after this
exchange of washers.
• 5.105 A bird of mass m, flying horizontally at speed v B

perpendicular to a stick, lands on the stick and holds fast


to it. (See Figure P5.105.) The stick (mass M, length i) is
lying on a frozen pond. Find the angular velocity of the
bird and stick as they move together. (Answer in terms of
m, M, l, and v .) Assume that the bird lands on the end of
0

the stick.

Figure P5.103

Figure P5.105

* 5.106 A hemispherical block of mass M and radius a


whose surfaces are smooth rests with its plane face in
contact with a smooth horizontal table. A particle of mass
m is placed at the highest point of the block and is slightly
disturbed. Show that as long as the particle remains in
contact with the block, the radius to the particle makes an
angle 6 with the upward vertical where
Figure P5.104
Page 370

Solid cylinder - Tlate


(m, l = 2r, r) (mass M)

Light bar

(at impact)

Figure P5.109

(at impact)
Figure P5.107

5.107 The bar in Figure P5.10 7 is welded to the end of the


cylinder, which is traveling downward in translation. The
bar strikes the tables at speed v , and the cylinder begins
0

to rotate about the bar without rebound.


a. Find the angular velocity of the cylinder when Figure P5.111
C is at its lowest point.
b. Find the percentage of energy lost during the
impact; that is 5.110 Work the preceding problem but assume that the
plate is initially free. If you work both problems, show
2
that the difference in the energies is I /(2M).
5.111 A bullet (see Figure P5.111) of mass m strikes a
1

5.108 A CARE package (Figure P5.108) consists of the square homogeneous block of mass m , where2

box plus contents described in Example 4.12. At impact The bullet is traveling with initial velocity v and be­
O

the crate has ft/sec and is translating. If there comes embedded in the block. After impact, the block is
is no rebound, find the angular velocity of the box and the observed to be pivoting about corner A. What is the maxi­
velocity of its mass center G just after the impact. mum speed of the bullet such that the block will not
tip all the way over?
5.109 The plate in Figure P5.109, supported by ball joints
at its top corners, is suddenly struck as shown with a force 5.112 The cylinder B (radius 10 cm, length 40 cm) swings
that produces the impulse I normal to the plate. Find the down from a position of rest where and strikes the
kinetic energy produced by the impact. particle P of mass 5 kg. (See Figure P5.112.) The coeffi-

Taxpayer

Figure P5.108
Page 371

5.115 A homogeneous cube of side a and mass M slides


on a level, frictionless table with velocity v . See Figure
0
21) k«
P5.115.) It strikes a small lip on the table at A of negligible
height. Find the velocity of the center of mass just after
impact if the coefficient of restitution is unity. (The cen-
troidal moment of inertia of a cube about an axis parallel
2
to an edge is Ma /'6.)
5.116 There is only one height H above a pool table at
which a cue ball may be struck by the stick without the
ball slipping for a while after the impact. (See Figure
P5.116.) Find this value of H, in terms of R, for which the
ball immediately rolls.

Figure P5.112

cient of restitution is e = 0.5, and at impact the particle


1
has Find the angle through which Bwill
turn about its pivot O after impact.
5.113 An equilateral triangular plate of mass 2 slugs and Figure P5.115
side 2 ft is released in the upper position from rest (see
Figure P5.113). It swings down and strikes the stationary
cylinder. The coefficient of restitution for the impact is
e = 1/2. Find elapsed time after impact until the cylinder
no longer slips on the plane.
5.114 A block slides to the right and strikes a small ob­
struction at a speed of 20 ft/sec. (See Figure P5.114.)
a. If the coefficient of restitution is zero, find the Figure P5.116
energy loss caused by the impact.
b. What is the minimum striking velocity required 5.117 Compute the error in in Example 5.18 that was
to overturn the block after collision? incurred in assuming the vessel to be a shell (so that
2 2
was m( /12 + mr /2). Use the weight, height, outer
radius, and density to compute the thickness of the vessel;
then calculate a more accurate and compare.
5.118 A uniform rod of length L is dropped and translates
downward at an angle with the vertical as shown in
Figure P5.118. If end A does not rebound after striking the
ground at speed v , find: (a) the energy lost during the
0

impact of A with the ground; (b) the speed at which


the other end B then hits the ground.

Figure P5.113

Figure P5.114 Figure P5.11B


Page 372

5.119 A uniform bar AB of length L and mass M is mov­


ing on a smooth horizontal plane with and
. when end B strikes a peg P (see Figure P5.119).
If / L and the coefficient of restitution find
the loss of kinetic energy.
5.120 The 80-lb solid block hits a smooth, rigid wall
(see Figure P5.120) and rebounds with a coefficient of
restitution of e = 0.2. Prior to impact the block had:
rad/secand ft/sec. Find the
angular velocity of the block immediately following the Figure P 5 . 1 2 3
impact.
5.121 The rod in Figure P5.121isfreely falling in a verti­
cal plane. At a certain instant it is horizontal with its ends
A and B having the velocities shown. If end A is suddenly
fixed, prove that the rod will start to rise around end A
provided thatv >,< 2v .
1 2

Figure P 5 . 1 2 4

Figure P 5 . 1 1 9

Figure P 5 . 1 2 5

Figure P5.120 5.124 A wooden sphere weighing 0.644 lb swings down


from a position where the rod is horizontal, and it impacts
the block. (See Figure P5.124.) The coefficient of restitu­
tion is e = 0.6. The block weighs 3.22 lb and is initially at
Figure P5.121
rest. Find the position of the block when it comes perma­
nently to rest. (Assume that the sphere is removed from
the problem after impact and that the spring cannot re­
5.122 In the preceding problem show that the energy loss bound past its original unstretched position.)
in instantaneously stopping point A is independent of v . 2
5.125 A 4-lb sphere is released from rest in the position
5.123 The bar in Figure P5.123 swings downward from shown in Figure P5.125, and two observations are then
the dotted horizontal position and strikes mass m. The bar made: (1) The sphere comes immediately to rest after the
has mass M and length The collision takes place with a impact; (2) the 5-lb block slides 3 ft before coming to rest.
coefficient of restitution of zero. If the coefficient of fric­ Using these observations, find the coefficients of restitu­
tion between m and the plane is find the distance tion (between sphere and block) and friction (between
moved by m before stopping. Treat m as a particle. block and floor).
Page 373

5.126 The rod-sphere rigid body in Figure P5.126 is re­ 5.128 Two toothed gear wheels, which may be treated as
leased from rest in the horizontal position. It swings down uniform disks of radii a and b and masses M and m, re­
and at its lowest point strikes the box. Find how far the spectively, are rotating in the same plane. They are not
box slides before coming to rest if the coefficient of resti­ quite in contact and have angular velocities w and w
1 2

tution is e = 0.5. The data are: about fixed axes through their centers. Their axes are then
slightly moved so that the wheels engage. Prove that the
1. Rod: length = 1 m; mass = 3 kg
loss of energy is
2. Sphere: radius = 0.2 m; mass = 10 kg
3. Block: b = 0.3 m; H = 0.35 m; mass = 5 kg
4. Coefficient of friction between block and plane
= 0.3 5.129 Sphere B has mass m and radius r, and it rolls with
1

mass center velocity on a horizontal plane. (See Fig­


Assume the sphere hits the block just once.
ure P5.129.) It hits squarely an identical sphere B that is
2

at rest. The coefficient of friction between a sphere and


the plane is µ, and between spheres it is negligible. The
impact is nearly elastic
a. Find v and w of each sphere right after impact.
C/ f

b. Find v of each sphere after it has started rolling


c

uniformly.
c. Discuss the special case when µ= 0.

Figure P5.126

5.127 The assembly in Figure P5.127 is turning at co, = 2


rad/sec when the collar is released. All surfaces are
smooth, and the disk is fixed to the bar; the light vertical Figure P5.129
shaft ends in bearings and turns freely. The data are:

•5.130 The 30-kg bent bar in Figure P5.130 falls from the
dashed position onto the spinning cylinder, which was
initially turning at 3000 rad/ If the bar does not
The collar moves outward and impacts the disk without bounce (coefficient of restitution is zero), find the stop­
rebound. Find: (a) the angular speed of the bar just before ping time for the cylinder following the impact.
and just after impact; (b) the percentage of energy lost
during impact. The radii of B and B are small compared
1 2

to their lengths. Treat B as a particle.


3

Figure P5.127 Figure P5.130


Page 374

* 5.131 In Example 5.18 assume that the lower front strik­ 5.133 Prove statement 1 near the end of Example 5.18 for
ing comer of the vessel B rebounds back up the plane the case when the plane is level and e = 0. Hint:
with velocity where e is the coefficient of restitution Note carefully that the angle of the plane does not affect
Use the equations of impulse and momen­ Equation (6), so you only need to alter the Ah in Equation
tum in the x and y directions, and the equation of angular (7) to obtain the new result.
impulse and angular momentum, to find the two mass-
5.134 Prove statement 2 near the end of Example 5.18.
center velocity components and the angular velocity of B
Again the plane is to be level in this problem, but now
after impact. Compare the results of for
Hint: The component of velocity is constant
e = 1 with those at e = 0. Show that no energy is lost
after impact, since with all external forces (mg and
when e = 1; that is, show
N) are vertical. To find this velocity use and the
velocity of the striking comer Q just alter impact (co is the
f

same as with in Problem 5.131 with


is Then use
• 5.132 After the impact in the preceding problem, show
that the equations of motion of the vessel are

to show that C reaches the top with energy to spare.


• 5.135 Show that the cylinder in Figure P5.135, following
release from rest, will reach the lower wall. Find the ve­
locity with which C will rebound up the plane following
where N is the normal reaction at the comer Q (see Figure impact, and determine the amount of energy lost. All data
P5.132). Observe that until there is another impact, the are shown on the figure.
mass center has a constant x component of acceleration. " 5.136 A sphere rolling with speed v on a horizontal sur­
c

Use kinematics to prove that face strikes an obstacle of height H. What is the largest
value that H can have if the sphere is able to make it over
the obstacle? Consider the coefficient of restitution to be
zero during the sphere's impact with the comer point O.
Use Equations (2) and (4) to eliminate N from (3), thus The answer will be a function of g, r, and v —in fact, H/r
c

obtaining a single differential equation in joveming the may be solved for as a function of the single nondimen-
rotational motion of B, and note its complexity. sional parameter (See Figure P5.136.)

Figure P5.132

Figure P5.135

Figure P5.136
Page 375

" 5.137 Using a density of wood of 673 kg/m , find the


3

mass center C of the baseball bat in Figure P5.137 and


then determine its moment of inertia with respect to the z 0

axis, perpendicular to the axis of symmetry of the bat. Use


the parallel-axis theorem to obtain and find the bat's
center of percussion if it is swinging about a fixed point O.

Figure P5.137 Figure P5.138

* 5.138 The hammer in Figure P5.138 strikes the sphere 5.139 Repeat the preceding problem but suppose that the
and imparts a horizontal impulse I to it. Determine the sphere and rod are welded together to form one rigid
initial angular velocity of the sphere. body.

COMPUTER PROBLEM • Chapter 5

* 5.140 The system in Figure P5.140 is released from rest in


the given position. With the help of a computer, generate
data for a plot of the angle turned through by wheel
beforefirststopping, as a function of the mass ratio M/m.
Hint: First show using that the equation govern­
ing 6 is

Figure P5.140

SUMMARY • Chapter5
For a rigid body in plane motion, the -kinetic energy, T, can be expressed
as

or as

With W standing for the net work over a time interval of all the
external forces on the body, the principle of work and kinetic energy
states

or more compactly
Page 376

The work done by a force, F, is defined as

where F • v is called the power, or rate of work of force F, with v being


the velocity of the material point being acted upon instantaneously by
the force. This definition is necessary to accommodate the possibility that
the force moves around on the body. When the force always acts on the
same material point for which a position vector is r,

The work done by a couple of moment C is

which, with is

For some special cases:


(a) F is constant:

and for weight mg, with y being elevation,

(b) Spring forces (on two bodies):

(c) Workless force:

if as with a normal force acting on a sliding body, or if v = 0


at each instant as for the contact point of a rolling body,
(d) Constant couple:

A conservative force does work independent of path and can have


associated with it a potential which we define so that its change is the
negative of the work done by the force. Examples are weight, for which
and a linear spring, The minus sign is used for the
convenience that follows when all forces acting on a body are conserva-
Page 377

rive, so that

or

which expresses the conservation of mechanical energy.


For a rigid body in plane motion, we found in Chapter 4 that

and similarly for a pivot. Concerning ourselves only with the case when
the products of inertia vanish,

As long as a body is behaving rigidly before and after an interval of time


of interest, the principle of angular impulse and angular momentum from
Chapter 2 gives

even though it may be that as in the example of the spin­


ning ice skater. The above is, of course, paired with the principle of linear
impulse and momentum,

to effect solution of collision problems.

REVIEW QUESTIONS • Chapter 5


True or False?
These questions all refer to rigid bodies in plane motion.
1. If you raise a 2-lb object 3 ft from rest and stop it there, gravity has
done — 6 ft-lb of work and you have done + 6 ft-lb on the object.
2. The work of a constant couple on body # i s always
where is the angle through which the body turns.
3. The work done by a linear spring is always where
and are the amounts of initial and final stretch. (If negative, they
represent compression.)
4. There are actually three separate work and kinetic energy principles;
two of the equations add to give the third.
5. The principles of work and kinetic energy, and (linear and angular)
impulse and momentum, result from general integrations of the
equations of motion, and thus they are free of accelerations.
Page 3 7 8

6. Not all forces acting on a body have to do non-zero work on it in


general.
7. The friction force beneath a rolling wheel does work on it if the
surface of contact is curved and fixed.
8. The normal force exerted on a rolling wheel by a surface, whether
fixed or in motion, never does work on the wheel.
9. The principle is valid for deformable
bodies.
10. The principle is valid for deformable
bodies.
11. Any problem that can be solved by can likewise be solved by
using "kinetic + potential energy = constant."
12. The formula gives all the kinetic energy of the rigid body
in plane motion, assuming the body is not translating.
6 KINEMATICS OF A RIGID BODY
IN THREE-DIMENSIONAL
MOTION

6.1 Introduction
6.2 Relation B e t w e e n D e r i v a t i v e s / T h e A n g u l a r Velocity Vector
6.3 Properties of A n g u l a r V e l o c i t y
The Derivative Formula
Uniqueness of the Angular Velocity Vector
The Addition Theorem
Simple Angular Velocity
Summary of Properties of Angular Velocity
6.4 T h e A n g u l a r Acceleration V e c t o r
6.5 V e l o c i t y and Acceleration i n M o v i n g Frames of Reference
The Velocity Relationship in Moving Frames
The Acceleration Relationship in Moving Frames /
Coriolis Acceleration
6.6 T h e Earth as a M o v i n g Frame
6.7 V e l o c i t y and Acceleration Equations for T w o Points of t h e
Same Rigid Body
Does A n Instantaneous Axis of Rotation Exist in General?
Describing t h e Orientation of a Rigid Body
The Eulerian Angles
6.9 Rotation M a t r i c e s

SUMMARY
REVIEW Q U E S T I O N S

Page 379
Page 380

6.1 Introduction
In this chapter w e study the kinematics of a rigid body in general
motion — w e n o w do for general motion what w e did in Chapter 3
for plane motion. There w e found that at any instant the velocities
of different points are linked together because of the rigidity of the
body, and the connecting link is the angular velocity of the body. We
also found there that it is angular velocity that links the derivatives
of the same vector relative to two different frames of reference. We n o w
wish to remove the plane-motion restriction.
The principal difficulty encountered in the study of general motion
of a rigid body is that the angular velocity does not always take the 6k
form of plane motion. The fact that can be changing in direction as the
body moves causes it to be difficult to visualize. A n efficient way to deal
with this abstract concept is to start with derivatives of the same vector in
different frames of reference (rigid bodies). Angular velocities will natu­
rally arise out of connecting these derivatives, and properties of relative
angular velocities amongst several bodies, almost self-evident for the
case of plane motion, will surface for the general case.
Application of the derivative/angular-velocity relationship to posi­
tion vectors, in two frames, of a point leads us to the velocities of the point
as observed in those frames, connected, in part, by the relative angular
velocity of the frames. Subsequent mathematical analysis leads to: a
relationship between accelerations of a point in two frames; a relation­
ship between velocities, relative to a frame, of two points fixed in the
same rigid body; and a relationship between accelerations, relative to a
frame, of two such body-fixed points.
We close this chapter with development of methods to describe the
orientation of a rigid body (relative, of course, to some reference frame).
This was easy to do in plane motion — an angle 8 is all that was required.
In the general case w e shall see the need for three angles, called Euler
angles in one popular scheme.
The reflective reader will notice that the sequence of coverages in this
chapter is almost precisely the reverse of its counterpart in Chapter 3,
which is after all just a special case. There w e were able to capitalize on an
ease of visualization not available to us here.

6.2 Relation Between Derivatives/The Angular Velocity Vector


In this section w e consider the relationship between the derivatives of a
vector taken in two different frames. In the process w e shall arrive at a
concise and useful definition of angular velocity. The reader is strongly
encouraged to persevere until this section and the next are fully under­
stood. Even though the angular velocity vector in three dimensions is a
difficult subject at first, it must be comprehended before w e can consider
the kinematics and kinetics of general rigid-body motion. The angular
velocity vector is the key to the subject. It will either make life easier for
Page 381

students of three-dimensional motion (if they work hard at understand­


ing it) or much more difficult (if they do not).
Let Q be an arbitrary vector. We may express Q in terms of its
components (Q , Q , Q ) associated with directions fixed in a frame by
x y 2

(6.1)

in which the unit vectors ( ) are parallel at all times to the respective
axes of a Cartesian coordinate system fixed in . N o w consider another
reference frame , in which w e wish to differentiate vector Q (see Fig­
ure 6.1). As an example, w e may wish to find the velocity of a point in
frame even though the point's location may be defined in (say by the
vector Q). In this case, part of the solution will require that w e be able to
differentiate Q in even though it is expressed in terms of its compo­
nents in .

Figure 6.1 Vector Q and frames and

Therefore it is n o w time to learn h o w to relate derivatives of a vector


taken in two different frames. We emphasize at the outset that these
vectors are completely arbitrary — they need not even be related to dy­
namics! Nor does the derivative have to be with respect to time, although
this is the independent variable of interest to us in dynamics and thus the
one w e shall use in the development to follow.
Letting represent (see Equation 1.8) the derivative of Q with
respect to time in , w e have

(6.2)

Recognizing the first three terms on the right of Equation (6.2) as the
derivative of Q in , w e have

(6.3)
Clearly the last three (parenthesized) terms in Equation (6.3) represent a
vector depending upon both Q and the change of orientation of frame
with respect to . We n o w proceed to obtain a useful and compact
expression for this vector; in the process, the angular velocity vector will
arise.
Since
(6.4)
it follows that
(6.5)
Page 382

so that the three derivatives of the unit vectors in Equation (6.3) are each
perpendicular to the respective unit vectors themselves.*

Question 6.1 Will this be true for any vector of constant magnitude
(not necessarily a unit vector)?

This means that there are three vectors and for which

(6.6)

The cross products ensure that and are each perpendicular to their
derivatives ( and so on) and the magnitudes of and give to
and their correct magnitudes.
In terms of their components in w e can write and as

(6.7)

Substituting these component expressions into Equations (6.6) results in

(6.8)

and w e see that and , at this point, remain arbitrary.

Question 6.2 Why do they remain arbitrary?

Here w e are seeking to relate the components of the vectors and


in the hope of finding a w a y to express the last three terms of Equation
(6.3). To this end w e note that, for all time t,

from the first of which, differentiation yields

(6.9)

* This assumes that the unit vectors are n o t constant in frame . If t w o of t h e m are con­
stant in , then all three are and the angular velocity vanishes, if only one is constant
in , w e have a simple special case to be considered later.
Answer 6.1 Sure, as w e h a v e seen in Section 1.6.
Answer 6.2 Since , then a can b e anything and n o t affect the first of Equa­
z

tions (6.6).
Page 383

Substitution of the first two equations of (6.6) into (6.9) yields

(6.10)

Interchanging the dot and cross in each term (which leaves the scalar
triple product unchanged) results in

(6.11)
so that
(6.12)
Similarly from
and (6.13)
w e respectively obtain (as the student should verify)

and (6.14)

The only components not involved in Equations (6.12) and (6.14) are
and , which were arbitrary. If w e n o w select them as follows,

(6.15)

then all three vectors are identical, and we call the resulting common
vector :
(6.16)

If w e n o w dot the three equations (6.8) respectively with and w e get


the three components of :

(6.17)

Thus the vector may be expressed as*

(6.18)
We call the vector defined by Equation (6.18) the angular
velocity of frame w i t h respect to frame or more briefly, the angu­
lar velocity of i n . It is clear that the angular velocity vector depends
intimately on the w a y frame is changing its orientation with respect to
. In the next section w e examine some special properties of this vector.
We shall see that is unique, which means that w e lost no generality
w h e n w e let , and in our develop­
ment above of angular velocity.

* This is the definition of angular velocity set forth by the dynamicist T. R. Kane. See his
books Dynamics: Theory and Applications ( N e w York: McGraw-Hill, 1985), p. 16 and
Spacecraft Dynamics ( N e w York: McGraw-Hill, 1983), p. 49.
Page 384

6.3 Properties of Angular Velocity


T h e Derivative Formula

We n o w return to Equation (6.3). Substituting from Equations (6.6) for


and and using Equation (6.16) to replace and by we
obtain
(6.19)

We call this the derivative formula, which may be expressed, using


Equation (6.1), as

(6.20)

Equation (6.20) will turn out to be of vital importance in this chapter


and, moreover, to be equally invaluable in our later study of the kinetics
of rigid bodies in general motion in Chapter 7. It permits us easily to
calculate the derivative of a vector in one frame if it is expressed in terms
of base vectors fixed in another; the only price w e have to pay is to add
the cross product . Thus the first property of is that it
allows us to relate (by Equation 6.20) the derivatives of any vector in two
different frames. We have already encountered this for the special case of
plane motion in Section 3.7, where Equation (3.44) may be seen to be the
plane motion counterpart of Equation (6.20).

Uniqueness of the A n g u l a r Velocity Vector

There remains the nagging question of whether there might be more than
one vector satisfying Equation (6.20); remember that w e arbitrarily se­
lected the components and in the preceding section in order to
make . We n o w proceed to s h o w that the angular
velocity vector is indeed unique. We do so by postulating that two vectors
and both satisfy Equation (6.20) and then showing that they
are necessarily equal.* We have

(6.21)
(6.22)

so that, subtracting, w e get

(6.23)

Finally, since Q is arbitrary, the parenthesized expression of Equa­


tion (6.23) must vanish, and thus the angular velocity vector has been
shown to be unique as the two 's are one and the same.
Nothing has yet been said about dynamics in this section or in the
preceding one; thus it is clear that angular velocity is a far more general

* Let be calculated with and as described in Section 6.2, for example, and let
be computed with another triad of unit vectors fixed in . The question of unique­
ness is whether the resulting 's are the same.
Page 385

vector than one that is simply useful in describing rotational motions of


rigid bodies. We have seen that angular velocity is in fact the vector that
may be used in relating the derivatives in two frames of any arbitrary
vector. Furthermore, even though w e have used time as the independent
variable, these derivatives may be taken with respect to any scalar vari­
able. Finally, w e note from the defining equation (6.18) for that
angular velocity is a vector relating two frames; thus it is meaningless to
talk about the angular velocity of a point.
N o w let us consider several additional properties of that will
prove useful in what is to follow. First w e note for emphasis that if two
frames and maintain a constant orientation (even if they are each in
motion in a third frame ), then .* The proof is simply to observe
that n o unit vector fixed in direction in can change with time in if
there is no change in orientation between and . Thus from Equation
(6.18),
Next w e shall prove that the angular velocity of in is the negative
of the angular velocity of in If w e add Equation (6.20) to the equation

(6.24)

we obtain

(6.25)

Again, since Q is arbitrary, w e have the expected result:

(6.26)

T h e Addition T h e o r e m

We n o w prove the addition theorem, which states that

(6.27)

For the proof, w e know from the first property of that


(6.28)
(6.29)
(6.30)
Adding Equations (6.29) and (6.30) yields
(6.31)
and subtracting Equation (6.31) from (6.28) gives
(6.32)

* Constant orientation m e a n s that and m o v e as if they were rigidly attached except


for a possible translation of o n e with respect to the other.
Page 386

Therefore, again since Q is arbitrary,

(6.33)
and the theorem is proved. It may seem intuitively obvious to the reader
that Equation (6.33) is true, but in the next section w e s h o w that such a
relationship does not exist for angular acceleration!
The addition theorem is an extremely powerful result. With it w e are
able to build up the angular velocity, one pair of frames at a time, of a
body turning in complicated ways relative to a reference frame. This
theorem makes it possible for us to avoid using the definition (6.18),
which has served us well but in practice is normally supplanted by the
properties described in this section.
We recall for emphasis that if and both move in and maintain
constant orientation with respect to each other, then Thus, by
the addition theorem,

so that, as expected, the angular velocities in of two frames and


maintaining constant orientation with each other are identical.
Conversely, if two frames and have equal angular velocities in
w e may s h o w that their relative orientation is constant. Using the addi­
tion theorem, thus with still fixed in direction in
then from Equations (6.6) and (6.16)

Therefore, are constant in so the orientation of in is


constant.
Thus, for two frames and , the descriptions "constant orienta­
tion" and " " are completely equivalent. Note also that the addi­
tion theorem can be extended to any number of frames by repetition of the
following procedure, two frames at a time:

Simple A n g u l a r Velocity

Next w e show that w h e n there exists a unit vector whose time deriva­
tive in each of two frames and vanishes (that is, ), then

(6.34)
in which is the angle between a pair of directed line segments
and fixed respectively in and each perpendicular to . The
angle is measured in a reference plane containing projections of the
two lines intersecting at point P as s h o w n in Figure 6.2. The sign of
the angle is given by the right-hand rule: If the right thumb is placed in
Page 387

the positive direction at P, then the direction of positive is that of the


right hand's fingers w h e n they curl from into as shown.

Figure 6.2 Simple angular velocity.

The type of rotational motion given by Equation (6.34) is called


simple angular velocity. One case in which Equation (6.34) holds is that
of plane motion; note, however, that there are more general cases of
simple angular velocity in which the body may also have a translational
motion in parallel to that would prevent the plane motion designa­
tion.

Figure 6.3 Unit vectors drawn in the


reference plane for simple angular velocity.

To prove Equation (6.34), w e make use of Figure 6.3. (The reference


plane is the plane of the paper, and the unit vector that is constant in
and in is k, perpendicular to the paper.) From this figure w e may write

(6.35)
Therefore, differentiating Equation (6.35), w e get

(6.36)
388

and Equation (6.18) yields, upon direct substitution of Equation (6.36),

(6.37)

which is the desired result.


One interesting final property of is that its derivative is the same
whether computed in or in Using Equation (6.20) and letting Q be
itself, w e get

This result is not true for any other nonvanishing vector, unless it hap­
pens to be parallel to

S u m m a r y of Properties of A n g u l a r Velocity

The properties of that w e have examined are summarized here:

1. It is a unique vector that satisfies

which is called "The Derivative Formula."


2. is synonymous with "the orientations of and do not
change." And if and maintain constant orientation, their angular
velocities in any third frame are equal.
3.
4. "The Addition Theorem":
which can be further extended to any number of frames.
5. "Simple Angular Velocity": If k is constant in both and then

where was defined earlier.


6.

In the following example w e use the addition theorem to write an angular


velocity, and then w e express it in three different frames.

EXAMPLE 6 . 1

Body in Figure E6.la rotates in frame about the vertical at constant angular
speed ; in disk rotates about its pinned axis at constant angular speed
relative to (The directions of rotation are as shown. Determine the angular
Figure E6.1 a velocity of in
Page 389

Solution
The coordinate axes shown are fixed in E. By the addition theorem,*

(1)
We see from this answer that expressing in terms of its components in the
intermediate ("between" and ) frame has yielded a neat, simple result. If
we had chosen instead to write in terms of its components in then (see
Figure E6.1b) with fixed in

Figure E6.1b

so that, substituting into Equation (1),

And if we had written in terms of its components along directions (


in Figure E6.1c) fixed in then with

we obtain, again using Equation (1),

Figure E6.1c
We see that expressing in terms of its components in either or gives a
lengthier expression than in moreover, these expressions become even more
complicated if or vary with time.

Question 6.3 Why?

The reader should note, however, that even though each of the three above
representations of appear to be different, they all yield the same vector.

* While the defining equation (6.18) is always available for directly computing the angu­
lar velocity, it is usually easier to build u p the vector b y using the addition theorem.
Answer 6.3 Because then the angles (arguments of the sines and cosines) are not simply
or but integrals of or with respect to time.
Page 390

In the next example w e illustrate the use of the "Derivative Formula"


(Equation 6.20) three times.

EXAMPLE 6 . 2

Two children are playing in the park on a seesaw mounted on a merry-go-round


as shown in Figure E6.2. The merry-go-round rotates about the ground-fixed
(frame ) vertical at rad/sec, and at the instant shown the seesaw turns at
rad/sec relative to the merry-go-round The vector from the girl to the
boy is always ( ) being fixed in the seesaw board Find
and at the given instant.

parallel to
board B

see-saw
board

radial line
fixed in

merry-go-
round

ground

Figure E6.2

Solution

is constant relative to so

To find we shall use the derivative formula:

where
Page 391

so

One way to find is to use

where

so

Had we not desired to obtain we might have used

where, by the addition theorem,

so that

as before.

We n o w present an extended practical example of the use of the


properties. In this example three separate bodies are in motion in a
reference frame, and their angular velocities are related by using the
simple angular velocity and addition theorem properties.

EXAMPLE 6 . 3
The Hooke's joint, or universal joint, is a device used to transmit power between
two shafts that are not collinear. Figure E6.3a shows a Hooke's joint in which the
shafts and are out of alignment by the angle
Each shaft is mounted in a bearing fixed to the reference frame . The shafts,
whose axes intersect at point A, are rigidly attached to the yokes and . A rigid
cross is the connecting body between the yokes. One leg of the cross (indicated
by the unit vector ) turns in bearings fixed in at D , and E , while the other leg
1 1

(unit vector ) turns in bearings fixed in at D and E . The arms of cross are
2 2

identical; they form a right angle with each other, and each is perpendicular to its
Figure E6.3a respective shaft.
Page 392

Figure E6.3b shows that measures the angular position of in If


Axis of (considered to be the drive shaft) has angular velocity and the
resulting angular velocity of is find the ratio of to in terms
of and and plot versus for 0,20,40,60, and 80°. Letting be
the rotation angle of , further investigate versus for the same five values.

Question 6.4 In Figure E6.3b note that has no component. Why


does this not represent a loss of generality?

Axis of Solution
Using the addition theorem, we may relate the angular velocities of the four rigid
bodies (shaft plus its yoke ), (shaft plus its yoke ), and

(1)
Figure E6.3b Since and both have simple angular velocity in we may write

(2)
We also know from Figure E6.3a that the cross has a simple angular
velocity in each of and . For example, the only motion that can have with
respect to is a rotation about DJEJ , the line fixed in both bodies. The same is true
for the motion of in . Thus

(3)
in which and are the unknown magnitudes of the respective vectors,
Next we must express all of and in terms of a common set of
unit vectors. Then we shall be able to obtain three scalar equations from (1) and
hence solve for in terms of . From Figure E6. 3b, three of the unit vectors are
obvious:

(4)
To obtain , we note that it is perpendicular to both and . Crossing
into then gives the assigned direction of (note that is opposite!);
is not generally a unit vector, however, so to get we divide this vector
by its magnitude:

(5)

Substituting Equations (4) and (5) for the four unit vectors into Equations (2) and
(3), and then substituting the resulting angular velocity expressions into Equa­
tion (1), we get a vector equation that has the following three scalar component
equations:
coefficients:
(6)

Answer 6.4 The (xy) plane of the paper can be chosen to be the plane containing and
without loss of generality.
Page 393

c o e f f i c i e n t s : ( 7 )
coefficients: (8)
in which . Eliminating between (6) and (8)
gives

(9)

and substitution of (9) into (7) yields

so that

(10)
A plot of this expression (Figure E6.3c) shows the manner in which
changes over a quarter-turn of in space. Note that since cos is squared, the
curves reflect around the vertical line at for ; between
180° and 360° we again have a mirror image, this time of the curves between 0°
and 180°. Note that for large misalignment angles shaft must turn very
rapidly at and near ; in fact, when the bodies reach a configuration
in which they cannot turn at all. This is called gimbal lock. Note further that a
misalignment of as much as 10° results in an output speed variation ( ) over a
revolution of only about 3 percent.

(degrees)
Figure E6.3c

Next we examine the angles of rotation (of ) and (of ). Since


and both have simple angular velocity in we have and ,
so that (from Equation 10):

(11)
394

Integrating (11) gives

(12)

The constant of integration is zero if we define when . Then


may be plotted as a function of for the same representative values of (see
Figure E6.3d). If , then and the curve is a 45° line. If there is

90

80

70

60

SO

40

30

20

10

0
0 10 20 30 40 ^0 60 70 80 90
(degrees)
Figure E6.3d

misalignment , then note from the curves that for , shaft is


always turned more than .Then catches up at , and from to
180° the angle of lags behind .

Question 6.5 Explain why this is so by using Equation (12).

From 180° to 360°, the cycle repeats and everything returns to the same starting
position at the same time.

Answer 6.5 For in the second quadrant, and with we see from Equa­
tion (12) that tan is a more negative number than tan (unless , in which case
). Therefore, and are angles between 90° and 180°, and
Page 395

PROBLEMS • Section 6.3

(See also the Project Problem 6.93 at the end of this chapter.)

6.1 Verify, in Example 1.1, that is indeed , 6.5 Note the three frames and and the vector A
where . (See Figure P6.1.) defined in Figure P6.5 expressed in terms of its com­
6.2 The angular velocities of and in a reference ponents in Also, and
frame are, respectively, rad/sec and rad/sec. Find at f = / 4 sec.
Find the angular velocity of in expressed in terms of 6.6 Review Problem 1.155 in which the unit tangent,
and (See Figure P6.2.) normal, and binormal of a curve in space are defined.
A vector v is given as a function of time t by v = Let be a frame moving relative to in such a way that
, where are unit vectors whose di­ , and are always fixed in Use the definition (Equa­
rections are fixed in a frame . The angular velocity of M tion (6.18)) of angular velocity to find the angular velocity
in frame is r a d / s . Find the deriv­ of in Note that
ative of v in frame that is, (a) as a function of t; (b) at
f = 1 s; (c) at t = 2 s.
6.4 In the preceding problem, find
6.7 The antenna in Figure P6.7 is oriented with the
following three rotations:
1. Azimuth, about y fixed in at the rate
2
= 3t rad/sec
2. Elevation, about Z fixed in a first intermediate frame
1

, at the constant rate


4 in.
3. A polarization rotation about the antenna axis x 2

(fixed in both the second intermediate frame


and in ) at = 4t rad/sec
If the structure is in the position at t = 0, find
at the time f = / 2 sec. Use these unit vectors fixed in
direction in parallel to y, parallel to z and
l7
Figure P6.1

6.8 Show that the output angle of the Hooke's joint


in Example 6.3 can be alternatively obtained by dotting
with

Figure P6.2

Azimuth
bearing

Figure P6.7

Figure P6.5
Page 396

6.9 A device for simulating conditions in space allows 2. The line AB (a base radius fixed in ) always lies in
rotations about orthogonal axes as shown in Figure P6.9. some vertical plane parallel to XY.
Determine the angular velocity in frame of the capsule 3. Cone slides on ; that is, there is always a line of
containing the astronaut. Express the result in terms of contact between O and a point of the base circle of
unit vectors fixed in the beam Note that the
4. Point A of revolves around the x axis in a vertical
rotation is about an axis fixed in and in but not in
circle at constant speed
this axis is parallel to y at t = 0, and is a constant.
Use the addition theorem to show that the angular veloc­
6.10 The outer cone in Figure P6.10 has the following
ity of in is given by
prescribed motion with respect to the fixed inner cone
I. The vertices remain together.

6.11 In the preceding problem find if the projection


of AB into the YZ plane through A is always aligned with
the radius. (See Figure P6.11.)
6.12 Find in the preceding problem if the outer cone
rolls on the fixed inner cone. (See Figure P6.12.)

A popular method of stabilizing shipboard antennas is by


means of pendulous masses together with the gyroscopic
effect of spinning flywheels. In Figure P6.13 the ship
(frame ) pitches (about x), rolls (about y), and yaws
(about z) in the sea (frame ). Frame just above a
Hooke's joint, is to form a stable platform on which the
antenna can then be easily positioned in azimuth (angle
A) and elevation (angle E). The frame remains level by a
"depitching" rotation P above the "derolling" rotation R.
The following three problems are based on this system.
The INMARSAT communications satellite system
Figure P6.9 required that shipboard antenna systems remain opera­
tional up to the following oscillatory limits:
Pitch: ± 1 0 ° in 6 sec
Roll: ± 3 0 ° in 8 sec
Yaw: ± 8 ° in 50 sec

Figure P6.11

Figure P6.10 Figure P6.12


Page 397

*6.13 Assume sine waves for each of these three motions able to extend (and retract) up to 5 in. in 30 sec. The wrist
and assume yaw over roll over pitch — that is, the as­ has two motions: It is able to pivot up to 180 ° about y' in
sumed order of ship rotations is (1) pitch, from frame to 10 sec and to rotate (about x') up to 350° in 4 sec. Axes
an intermediate frame ; and (2) roll, from to a second (x', y', z') are fixed in Finally, the gripper is able to
open (and close) 3.5 in. in 3 sec, but is assumed here to be
a closed circle with a 2.5-inch diameter. Approximate di­
mensions are shown in the figure.
For this problem, assume that all the robot's motions
(except the gripper opening) are occurring simultaneously
about positive axes with their respective average speeds.
Find the angular velocity of the gripper relative to and

Ship

Sea
(not to scale)

Figure P6.13 Figure P6.16(a)

intermediate frame ; and (3) yaw, from to frame


Write the angular velocity of the ship in the sea (earth-
fixed frame ), expressed in the ship-fixed axes (x, y, z).
Hint: For example, will be /180sin / 6 rad.
6.14 Write the angular velocity of in expressed in
the axes {x, y, z).
6.15 Write the angular velocity of in using the re­
sults of the preceding two problems together with the
addition theorem.
* 6.16 A robot manufactured by the Heath Company has
the mechanical arm shown in Figure P6.16(a) and ac­
companying photograph. Its shoulder extends from the
head which can itself rotate 350° about z in 30 sec
relative to the reference frame The arm is able to travel
150° about axis y in 26 sec. (The axes (x, y, z) are fixed in
the head ) The part of the arm to the left of point £ is

Figure P6.16(b) (Courtesy of the Heath


* Asterisks identify the more difficult problems, Company.).
398

expressed in terms of unit vectors in at an instant when 2. The wrist is pivoted 30°:
these two conditions hold:
I. The shoulder rotation angle is — 60 °:

6.4 The Angular Acceleration Vector


In applying what w e have learned about angular velocity to the kine­
matics of rigid bodies, w e also need to understand its derivative. The
angular acceleration of frame relative to frame is defined to be

(6.38)

(Note from the last property of in the previous section that the
derivative could equally well be taken in but generally not in any other
frame.)
It is important to note that the addition theorem (Equation 6.27) does
not hold for angular acceleration. Watch:

(6.39)
We see that there is an extra term (the cross product of two angular
velocity vectors) that prevents the simple theorem w e have derived for
from working for . This term is sometimes called a gyroscopic term;
note that it vanishes for plane motion, in which case w e do have an
addition theorem for the (which are then of the form ).
In each of the two examples to follow, the reader should notice h o w
the various properties of — simple angular velocity (Equation (6.34)),
the addition theorem (6.27), and the derivative formula (6.20) — are
used to great advantage.

EXAMPLE 6 . 4

Body in Figure E6.4 rotates in frame about the vertical at constant angular
speed ; in disk rotates about its pinned axis at constant angular speed
(The directions of rotation are as shown.) Determine the angular acceleration of
in
Solution
As we saw in Example 6.1,
Axes (x, y, z) embedded in
Figure E6.4
Page 399

Next, using Equation (6.20) and noting that is expressed in terms of axes
embedded in we "move the derivative" using the derivative formula and
obtain*

Note that the same result is obtained by using Equation (6.39) with frames
and replaced by and respectively.

Question 6.6 Why was in the above example?

EXAMPLE 6 . 5

Determine the angular acceleration of the cross relative to frame in Exam­


ple 6.3, for the case constant. Express the result in terms of unit vectors ,
and fixed in
Solution
Using the definition of angular acceleration [Equation (6.38)], the addition
theorem, and the derivative formula,

(1)
Let us first concentrate on the first term on the right side of Equation (1). In
Example 6.3 we had

(2)
where we are using the notation
and c = cos . We also know from Example 6.3 that
x

(3)

Substituting from Equation (3) into (2) and differentiating the result in (and
noting is constant there) yields, after simplifying,

(4)

* By "moving the derivative," w e m e a n shifting it from a frame in w h i c h it is i n c o n v e ­


nient to differentiate, to a frame in w h i c h w e desire to differentiate.
Answer 6.6 and are constant scalars, and and are unit vectors fixed in direc­
tion in
Page 400

The second term in Equation (1) is

which upon simplification is

(5)

The last term in Equation (1) is:

(6)

where we have differentiated Equation (3).


The solution for is therefore the sum of the vectors in Equations (4), (5),
and (6):

We note that up to this point in Chapter 6 w e have not mentioned


velocities or accelerations. As long as the angular velocities from one
body to another are all simple, w e can do a considerable amount of
angular velocity and angular acceleration computation merely by using
the definitions and properties of and

PROBLEMS • Section 6.4

6.17 We know that one of the properties of the angular


velocity vector is that . Show that this is not
a property of the angular acceleration vector
6.18 In Problem 6.6, find the angular acceleration of 8
in
6.19 The components of two angular velocity vectors are
shown in the following table as functions of time. The Figure P6.19
orthogonal unit vectors are fixed in direction in
frame Find the angular acceleration of in (a) as a
function of time; (b) at t = 0 sec; (c) at t = 0.5 sec. (See
(The unit vectors are
Figure P6.19.)
fixed in )

Find the restriction on frame for which (i = 1,


2,3).
2
4t 2t 6
sin t cos t 7t 6.21 The antenna in Figure P6.21 (see Problem 6.7) is
oriented with the following three rotations:
1. Azimuth, about y fixed in at the rate
2
6.20 Let the angular velocity and angular acceleration = 3f rad/sec
vectors and be expressed in terms of their com­ 2. Elevation, about ZJ fixed in a first intermediate frame
ponents in a third frame : , at the constant rate = 1 rad/sec
Page 401

6.23 In Example 6.2 find the angular acceleration of the


4t rad/sec see-saw board in the ground if in addition to the given
3t rad/sec data rad/sec and 2
= 1.5 rad/sec . 2

6.24 See Figure P6.24. Axes x, y, and z are fixed in body


which rotates in about the z axis with angular veloc­
1 rad/sec
ity The arm attached rigidly to supports a bear­
ing about which turns with angular velocity relative
to Finally, body turns about the direction (which
Azirmuth lies along axes of symmetry of both and with
bearing
relative to If and are all functions of time,
find the angular acceleration of in at an instant when
makes angles with x and z of 135 ° and 45 °, respectively.
Figure P6.21

3. A polarization rotation about the antenna axis x 2

(fixed in both the second intermediate frame and


in ) at rad/sec
If the structure is in the position at t = 0, find
at the time sec. Use unit vectors fixed in direction
in Figure P6.24

6.22 In problem 6.9 find the angular acceleration of the


capsule C in Take and to be constants.

6.5 Velocity and Acceleration in Moving Frames of Reference


In certain practical situations a point is moving relative to two frames (or
bodies) of interest. For example, a pin P may be sliding in a slot of a body E
that is itself in motion in another frame (see Figure 6.4). In problems
such as these, w e are often interested in the relationship between the
Pin P (fixed to ) velocities (and also the accelerations) of P in the two frames and We
moves in both
and studied this problem in plane motion in Sections 3.7 and 3.8, and w e now
wish to expand the treatment to three dimensions.

T h e V e l o c i t y Relationship in M o v i n g Frames
Figure 6.4 Example of a point moving
relative to two frames. We shall arbitrarily choose as a reference frame for the moving body
but w e emphasize that both are frames and both are bodies; as long as
they are considered rigid, the terms mean the same. Figure 6.5 shows the
general picture.

Figure 6.5 Point P moving with respect


to frames and
Page 402

Letting O and O' be fixed points of and respectively, w e differen­


tiate the connecting relation
(6.40)

in and obtain*
(6.41)

Because O is fixed in the first two of the three vectors in Equation (6.41)
are the velocities in of P and O':

(6.42)
The last vector in Equation (6.42) presents a problem. It is not the
velocity of P in because the point O' is not fixed in nor is it the
velocity of P in because the derivative is not taken there. To overcome
this dilemma, w e shall rewrite the term by moving the derivative from
to using Equation (6.20):

(6.43)

(6.44)
We are n o w in a position to derive the equation relating the velocities
of two points of the same rigid body from the (therefore more general)
equation (6.44). Temporarily let P be a fixed point of then and

(6.45)

or
(6.46)
which is the same as the plane motion equation (3.5), except that n o w the
r and v vectors may also have z components and the vector can have x
and y components in addition to z.
Returning to the general case in which P is not necessarily attached to
either or let us denote the point of (or extended) coincident with P
by . Then Equation (6.44) becomes

(6.47)
In words, w e may restate Equation (6.47) as follows:

Velocity in of
Velocity of Velocity of
the fixed point of
Pin Pin
coincident with P

Equation (6.47) has the virtue of compactness. However, it is a less con-

* The superscripts in Equation (6.41) are now necessary to denote the frame in which
Page 403

venient form than is (6.44) for differentiation to produce a corresponding


relationship of accelerations.

Question 6.7 Why?

Another common alternative means of stating Equation (6.44) is to


view body as a moving frame — that is, as a body moving relative to
another frame (See Figure 6.6.) We may then rewrite Equation (6.44)
as

(6.48)

in which

v = velocity of P in reference frame


P

= velocity of moving origin =


Figure 6.6
= velocity of P in moving frame =
= angular velocity of moving frame =
r = position vector of P in moving frame =

We n o w consider two examples involving the use of the velocity equation


for moving frames (Equation 6.44, 6.47, or 6.48).

EXAMPLE 6 . 6

Find the velocity in of point A at the bottom of the disk in Example 6.1. (See
Figure E6.6.)
Solution
We select as the moving frame, and Equation (6.44) gives the following (where
is and A is P):

In this case also from the previous example. The velocity of


A in is given by

Axes (x, y, z) embedded in


Figure E6.6 Therefore

Answer 6.7 If w e differentiate Equation ( 6 . 4 7 ) in w e encounter the term


which is not recognizable as a standard kinematic quantity; this is be
cause denotes a succession of points of w h i c h are at each instant coincident with P.
Page 404

EXAMPLE 6.7

Crank in Figure E6.7 rotates about axis z through point O. Its other end, Q, is
attached to a ball and socket joint as shown. The ball forms the end of rod
which passes through a hole in the ceiling find the velocity of point P of the bar,
which is passing through the hole when as shown.

Figure E6.7

Solution
We denote by H the point of at the center of the hole; then:

(1)
in which P is the point of coincident with point H. Knowing that the motion
of H in must be along the axis of we have:

(2)

or

(3)
Collecting the coefficients of and respectively, we find

(4)

(5)

(6)
Page 405

These three equations (4 - 6) obviously cannot be solved for unique values of


all four unknowns . However, an answer for is obtainable by
subtracting Equation (5) from (4) and then adding times Equation (6). The
result is

Substituting this result back into Equations (4-6) gives three equations in
and whose coefficient matrix is singular (has a zero determinant). Thus they
cannot be solved for the angular velocity components. A more physical reason
for this is that the component of along bar B cannot affect the answer for
because can turn freely in its socket about its axis without altering . Mathe­
matically this is manifested by this "axial" component of being parallel to
R and thus canceling out of Equation (2). These components are not needed
QP

for a solution, however, because from Equation (1) we can obtain our desired
result:

An added note is that although we cannot solve for the component of


along with the given information, we are able to calculate the component of
normal to It will be made up of values and that enforce the
relationship

That is,
(7)
This equation states that these components form a vector normal to the line
QP; again, this vector is the only part of that can affect
After adding primes to in Equations (4-6), the solution of Equa­
tions (4-7), as the reader may verify, is

And we may now calculate the velocity of P from Equation (1) as before or from
Equation (2) as follows:

(as before)

T h e Acceleration Relationship in M o v i n g F r a m e s / C o r i o l i s Acceleration

After these examples w e are n o w ready to derive the corresponding


relationship between the accelerations of P in two frames and . Differ­
entiating Equation (6.44) gives

(6.49)
Page 406

or, again using Equation (6.20) (once in each of the last two terms), w e get

(6.50)

Rearranging the terms, w e have

(6.51)

(6.52)

The middle three terms on the right side of Equation (6.51) make up the
acceleration of the point of (or extended) coincident with P. (The
proof is brief: If P is fixed to at point , then the other two terms vanish
since r becomes a constant vector in and what remains is necessarily
O'P

.) The term (which is , with both derivatives taken in ) is


clearly the acceleration of P in The last term, , is called the
Coriolis acceleration of P. Note that due to the presence of the Coriolis
acceleration it is not true that the acceleration of P in frame is its
acceleration in plus the acceleration of the point of with which it is
coincident (as was in fact the case with the velocity of P). This result is
interestingly analogous to the fact that the addition theorem for angular
velocity is not true for angular acceleration.
As w e did for the velocity equation, w e n o w restate Equation (6.52) in
words:

Acceleration in of
Acceleration Acceleration Coriolis
the fixed point of
of P i n of P i n acceleration
coincident with P

In the abbreviated notation of Equation (6.48) w e have

(6.53)

in which

a =
P acceleration of P in reference frame
= acceleration of moving origin =
= angular acceleration of moving frame =
a =
rel acceleration of P in moving frame =

and in which all other terms in Equation (6.53) are defined directly after
Equation (6.48).

EXAMPLE 6.8

Axes (x, y, z) embedded in Compute the acceleration in of point A in Examples 6.1, 6.4, and 6.6 (see
Figure E6.8 Figure E6.8).
407

Solution
We shall use Equation (6.51): is again the moving frame; the reference
frame is and the moving point Pis A:

The various terms on the right side are calculated as follows:

1.

2. Note that O' is fixed in in this example.


3. Note that since is a constant, and
does not change in direction in
4.
5.

Thus the answer for the acceleration of point A is

In Section 6.7, w e shall rework the above example using another


approach.

PROBLEMS • Section 6.5

6.25 In Example 6.2, the children are each 5 ft from 6.27 The large disk in Figure P6.27 rotates at 10 rad/
the fulcrum O. At a later time the girl slides a stone P sec counterclockwise (looking down on its horizontal sur­
along the see-saw board toward the boy. The velocity face). A small disk rolls radially outward along a radius
of t h e stone relative to the board is — ft/sec at OD of At the instant shown, the center C of is 4 ft
an instant when (a) the stone is 2 ft from the girl; from the axis of rotation of and this distance is increas­
(b) (c) rad/sec; and (d) rad/sec. ing at the constant rate of 2 ft/sec. Determine the velocity
Find and at this instant. and acceleration of point E, which is at the top of at the
given instant.
6.26 In Problem 6.25, if at the given instant
2
rad/sec , and the stone's acceleration relative
2
to t h e board is 0 . 8 ft/sec , find and

4 ft 1 ft

Figure P6.27
Page 408

6.28 Shaft in Figure P6.28 turns in the clevis at 2


2 rad/sec rad/sec in the direction shown. The wheel simulta­
neously rotates at 3 rad/sec about its axis as indicated.
Both rates are constants. The bug is crawling outward on a
(5 in. long) spoke at 0.2 in./sec with an acceleration of 0.1 in./sec 2

2 in. both relative to the spoke. At the instant shown, find:


(a) the angular velocity of the wheel; (b) the velocity of
the bug.
Find the angular acceleration of the wheel and the
acceleration of the bug in Problem 6.28.
1 in. 3 rad/sec 6.30 The crane in Figure P6.30 turns about the ver­
tical at rad/sec = constant, and simultane­
Figure P6.28 ously its boom is being elevated at the increasing rate
rad/sec. The (x, y, z) axes are fixed to the crane
zero. When the boom makes a 60° angle with the
horizontal, find: (a) (b) (c) (d)

6.31 A platform translates past a turntable at 6 mph.


(See Figure P6.31.) People step onto the turntable and
walk straight toward the center where they exit onto
stairs. There is rolling contact between platform and turn­
table. Suppose the people walk at the constant rate of
approximately 3 mph relative to the turntable. If it is de­
2
sired that they do not experience more than 3 ft/sec of
lateral acceleration, find the required turntable radius.

4 ft 6.32 The truck in Figure P6.32 moves to the left at a


Figure P6.30 constant speed of 7.07 ft/sec. At the instant shown, the
loading compartment has an angular speed rad/
2
sec and an angular acceleration of rad / sec . The
cylinder shown on the truck bed comes loose and rolls

6 mph

Figure P6.31

Figure P6.32
Page 409

toward the ground at an angular velocity, relative to the


compartment, of 1 rad / sec at this instant; it is speeding
2
up at a rate of rad/sec , also relative to the compart­
ment. Find the velocity and acceleration of the center of
the cylinder relative to the ground. Use the rotating refer­
ence frame shown.
6.33 The bent bar in Figure P6.33 revolves about the
vertical at rad/sec. The center C of the collar has
velocity and acceleration relative to of in./sec and
— respectively, where is in the direction of
the velocity of C in At the given instant, find the veloc­
ity and acceleration of C in the frame in which turns.
6.34 A centrifugal pump turns at 500 rpm in and
the water particles have respective tangential velocity
Figure P6.33 and acceleration components relative to the blades of
2
120 ft/sec and 80 ft/sec outward when they reach
the outermost point of their blades. (See Figure P6.34.)
At the instant before exit determine the velocity and
acceleration vectors of the water particle at P with
respect to the ground
6.35 A man walks dizzily outward along a sine wave
2 ft fixed to a merry go-round that is 40 ft in diameter and
turns at 10 rpm. (See Figure P6.35.) If the man's speed
relative to the turntable is a constant 2 ft/sec, what is the
1 ft magnitude of his acceleration when he is 10 ft from the
center?
Each blade is
parabolic: 6.36 Disk rotates about its axis at the constant angular
speed r a d / s . T h e round wire is rigidly affixed to
at points A and B as shown in Figure P6.36. A bug
crawls around the wire from A to B; its speed relative to
the wire (initially zero) is always increasing at the con­
2
stant rate of 0.001 m / s . Find the velocity and accelera­
Figure P6.34
tion of the bug, relative to the reference frame in which
the disk turns, when it arrives at B.

Wire

10 ft

Bug

10 rpm Figure P6.36

Figure P6.35
Page 410

Figure P6.38

Figure P6.37

3 ft

6.37 A bird flies horizontally past a man's head in a


straight line at constant speed v toward the axis of a
0

turntable on which the man is standing. The axes xy have


origin at the man's feet; the z axis is vertical and y always
points toward the center of the turntable, which has
10 ft
radius R and angular velocity Derive x(t) and 2 ft
y(t) of the bird in terms of R, v ,
0 , and the time t. (See Figure P6.39
Figure P6.37.) Use Equation (6.47), integrate, then check
your results by inspection.
* 6.38 In Figure P6.38 the axes x and y, and the origin O, astronaut's head will not exceed 5g at the given instant.
are fixed on the deck of a ship. The ship has an angular Let
velocity relative to the earth of *6.40 In Problem 6.16 find the velocity of point P at the
tip of the gripper at the instant given.
*6.41 Prove that if two bodies are in rolling contact, the
where x and y are fore-and-aft and athwartships axes,
shaded arcs in Figure P6.41, representing the loci of
respectively; thus is a rolling component and co a P
former contact points, are equal in length. (Note from
pitching component of angular velocity. Point T is a target
Problem 3.105 that the converse is not true.)
fixed relative to earth, such as a geosynchronous satellite.
Find the angular rates and in terms of and
that are required to track point T.
• 6.39 In Problems 6.9 and 6.22, let rad/sec and
rad/sec in the directions indicated. With the
dimensions given in Figure P6.39, find the maximum
value of for which the acceleration magnitude of the Figure P6.41

6.6 The Earth as a Moving Frame


In this section, w e shall make use of Equation (6.53) to set up the differ­
ential equation governing the position of the mass center C of a body
that is in morion near the earth. This equation will allow us to measure
Page 411

the position of C with respect to a desired site 0 ' at latitude which is


itself in motion as the earth turns on its axis from west to east. We assume
that for describing certain motions near the earth, a frame with origin at
the earth's mass center O is "sufficiently fixed" to be justifiably called
inertial. The frame moves as does the (assumed rigid) earth, except that
it does not share the earth's daily spin. Thus the site O' has an accelera­
tion in directed toward the earth's north-south polar axis.
North We set up the moving frame as shown as Figure 6.7. The frame
is the turning earth, and the (x, y, z) axes are embedded in it at O'
with x pointing east, y north, and z in the direction of local vertical.
The acceleration of the mass center C of a body moving near the
earth and whose position is desired relative to O', is known from
Equation (6.53) to be

where r is the position vector of C in and v and a are the velocity and
rel rel

acceleration vectors of C in Further, and We n o w


use the mass-center equation of motion from Chapter 2:

Figure 6.7
to obtain

where F represents all external forces on besides gravity, which is


written separately.

Question 6.8 This is a good place to ask: Why is Equation (6.54)


restricted to bodies in motion near the earth?

We n o w proceed to compute the various terms in Equation (6.54).


First w e note that

Next w e compute the acceleration of the site (O'). We utilize Equa­


tion (6.53) again, this time with the "moving point" being O' and the
origin in the moving frame being O:

Answer 6.8 It has been assumed that the strength of the gravitational field is constant,
but of course gravity decreases as C m o v e s farther and farther from the earth's surface.
Page 412

This time, however, note that v and a^, are zero; this follows from the
rel

fact that O' has no motion relative to the moving frame (the earth). Thus
the only term surviving is:

Therefore, from Equation (6.54),

Neglecting | r | with respect to | R | and expressing F in terms of its compo­


nents (F , F , F ), w e arrive at a set of differential equations governing the
x y z

motion of C:

(6.55)

We complete this brief section with an example illustrating the use of


Equations (6.55).

EXAMPLE 6.9

Due to the earth's rotation, the resultant force it exerts on a particle P at rest on its
surface is not quite directed toward its center of mass. Use Equations (6.55) to find
this deviation, assuming a spherical earth.
Solution
We have and all zero, so that the equations of morion of P (which
moves!) in the inertial frame are:

Calling we see (Figure E6.9a) that the earth must push on P at a


small angle ( ) with the "geometric vertical" in order for P to remain at rest in
the "moving frame" rigidly attached to the surface of the planet. It is the angle
not the angle that defines "local vertical"; this is because is the angle that a
plumb bob string makes with the axis X in the equatorial plane.
Figure E6.9b shows that (with )

Figure E6.9a
Page 413

We note that k, in comparison with mg, is quite small:

This means that we can use small angle approximations on the angle :
Figure E6.9b

so that the deviation of the local "plumb-line" vertical from the "geometric
vertical" is

Of course there is no deviation in direction at the poles (where ) or the


equator (where ). The maximum, at 45° latitude, is 0.0017 rad, or about
0.1°.

PROBLEMS • Section 6.6

6.42 If it were possible for a train to travel continuously


around the world on a meridional track as shown in Fig­
ure P6.42(a), one side of the track would wear out in time
due to the Coriolis acceleration. Explain which side will
wear out in each of the four numbered quadrants of the
circular path. Figure P6.42(b) shows how the train's
wheels rest on the track.
6.43 Explain in detail how the Coriolis acceleration is
related to the deflection of the air rushing toward a low- Figure P6.42(b)
pressure area, thereby forming a hurricane. (See Fig­
ure P6.43.) Do the problem for each hemisphere!

Equator

Figure P6.42(a) Figure P6.43


Page 414

6.44 A car travels south along a meridian at a certain * 6.46 A projectile is fired from a site at latitude with the
time; its speed relative to the earth is 60 mph, increasing at initial velocity components (at time t = 0) . De­
2
the rate of 2 ft/sec . (See Figure P6.44.) Find the acceler­ termine the maximum height reached by the projectile,
ation of the car in a frame having origin always at the neglecting air resistance and the terms in Equations
center of the earth and z axis along the polar axis of rota­ (6.55).
tion, but not rotating about the axis with the earth. * 6.47 Refer to Example 6.9. Show that if the component
equations of (6.54) are written in terms of axes (x', y', z')
(see Figure P6.47) instead of (x, y, z), they will have the
same form as do Equations (6.55) without their
terms, provided that

4000 mi replaces g; replaces and y' and z', respectively, re­


4v
place y and z.

Figure P6.44

6.45 If in the preceding problem the car is traveling from Figure P6.47
west to east at 45 °N latitude instead of along a meridian,
find the car's acceleration in the same frame. (See Fig­
ure P6.45.) * 6.48 Using equations from the preceding problem, find
the location at which a falling rock will strike the earth if
dropped from rest on the z' axis from a height H. Neglect
air resistance and assume the rock strikes the earth when
z' = 0.

45°

Figure P6.45

6.7 Velocity and Acceleration Equations for Two Points of the


Same Rigid Body
We next apply the concepts of position, velocity, acceleration, angular
velocity, and angular acceleration (developed in Chapter 1 and Sec­
tions 6.2 to 6.5) to the kinematics of a rigid body in general mo­
tion in a frame The equation relating the velocities in of two
points of a rigid body to its angular velocity is a special case of
Equation (6.44). We have seen in the text following that equation
Page 415

Figure 6.8 Points P and O' of a rigid body in general motion.

that if P joins O' as a fixed point of (see Figure 6.8), the relationship
between the velocities of these two points is given by the equation

(6.56)
From the text following Equation (6.52), w e also know the relationship
between the accelerations of these two points of :

(6.57)

We emphasize that Equations (6.56) and (6.57) follow from the general
equations (6.44) and (6.51) w h e n and if r is a constant in in that case,
O'P

Furthermore, the two points P and


O' in Equations (6.56) and (6.57) may be replaced by any pair of points
fixed to since being fixed to is the only restriction on either of them.
Speaking loosely, with Equations (6.56, 6.57) w e are interested in two
points on one body, whereas with Equations (6.44,6.51) w e were studying
one point in motion relative to two bodies. We n o w consider examples
involving the use of the two rigid-body equations (6.56) and (6.57).

EXAMPLE 6.10

Rework Examples 6.6 and 6.8 by treating point A as a point of body instead of
as a point moving with a known motion in
Solution
From Equation (6.56), and Figure E6.10, we have

Now recognizing that Q is also a point of we obtain

(as we obtained in Example 6.6)


Next we relate the accelerations of A and Q with Equation (6.57):
Axes (x, y, z) embedded in
Figure E6.10
Page 416

The first term on the right side, using Q and O' as points of body is

Thus

or

which we previously obtained in Example 6.8 by another approach.

In the next example w e will see that sometimes there is an indeter­


minate component of angular velocity.

EXAMPLE 6.11

Collars and in Figure E6.11a are attached at Q and C to rod by ball and
2

socket joints. At the instant shown, C is moving away from the origin at speed
2

Find the velocity of Q at the same instant. Can the angular velocity of

Solution
The velocity of C is determined by (see Figure E6.11b):
2

14 c m

where

8 cm
so
Figure E6.11a

Now we can calculate the velocities of C and C using Equation (6.56):


1 2

(1)
We note that the component of the angular velocity along the line C Ct 2

cannot be determined from the given information, because any value of it what­
6 cm soever will not affect Equation (1). However, dotting this equation with
8 cm

Figure E6.11b
Page 417

so that

and

EXAMPLE 6 . 1 2

The cone in Figures E6.12a,b rolls on the floor i n such a way that the center Q
of the base of the cone travels on a horizontal circle at constant velocity
Let denote an i n t e r m e d i a t e f r a m e ( " b e t w e e n " c o n e and the ground ) in
which and are fixed. T h e u n i t v e c t o r is a l w a y s d i r e c t e d a l o n g OQ, a n d
is normal to the p l a n e of a n d t h e c o n t a c t line, in a d i r e c t i o n parallel to
finally, Find the angular velocity of in

Figure E6.12a Figure E6.12b

Solution
We shall denote by simply . Since v = v = 0, then their difference,
A O

must also vanish. This requires to be parallel at all times to the


line of contact of with the ground. Thus

Next, using Equation (6.56),

Therefore

so that

Thus
Page 418

We wish to make some further remarks about the preceding exam­


ple. The addition theorem gives:
(1)
where is given by

Substituting and into the addition theorem equation (1) above,


w e find

Note the check on the direction since the only way can move
relative to is to rotate around OQ.
Consider finally two ways of depicting the components of
From Figure 6.9a, note that the vector sum of the two components of
is parallel to the contact line since

Figure 6.9a

Figure 6.9b illustrates the addition theorem. Note that the "direction
check" again results in being along the contact line, this time be­
cause

Figure 6.9b

EXAMPLE 6.13

In Example 6.12, find the angular acceleration of in


Solution
Differentiating ,

We had

and

Thus
Page 419

and we obtain

or

We see in the above example that acceleration equations need not be


used to compute if is known.
The final example in this section illustrates the workings of a compli­
cated three-dimensional gear train.

EXAMPLE 6.14

Very large alterations in speed along a given direction may be obtained by using
the gear arrangement shown in the diagram. Gears and all rotate about
the x axis in but has a more complicated motion:
1. It rolls on the fixed (to ) gear currently contacting it at P.
2. It rolls on (The contacting teeth are at A in Figure E6.14a.)
3. It rolls on (at B in the figure).
4. It turns with respect to about the line which is fixed in both and
Considering to be the driven gear, find the ratio of to

43 cm

30 cm

9 cm

(given)

Figure E6.14a
Page 420

Solution
The velocity of the contact points of and is, using body

(1)
in which we use just one subscript on when it is the angular velocity of a body
with respect to
Next we find another expression for v , this time by relating the velocity of
B

the tooth point of at B to that of the point of that contacts the reference frame
( is fixed to ) a t P .

(2)
To obtain r , we use Figure E6.14b and see that
PB

Therefore

36 cm (3)
45 cm

Also, by the addition theorem,


9 cm

Figure E6.14b (4)

in which we have used the fact that we know the directions (but not the magni­
tudes yet) of and
Substituting Equations (3) and (4) into (2) and substituting the result into
Equation (1) gives us

or, simplifying,

(5)
18 cm To get another equation in these variables, we shall use the point T, which
60 belongs to both and and is shown in Figure E6.14c along with some essential
45 cm geometry. First, as a point of we have
18 cm
27 cm
Ml
9 cm

Figure E6.14c (6)


Page 421

Next, as a point of

(7)
The cross product in Equation (7) is one-half the cross product in Equation (2).

Question 6.9 Why is this so?

Therefore

(8)
Equating the right sides of Equations (6) and (8) gives

or

(9)
Substituting Equation (9) into (5) leads to

(10)

and

(11)

Substituting Equations (10) and (11) into (4) gives us which we shall need in
the last step of the problem. The result is

(12)

We can now relate the velocities of the contacting points of bodies and at
point A; first, using points A and £ of we get

(13)

To get another expression for v , we relate the velocities of the two points A and P
A

on body
(14)
60° 18 cm
To obtain the position vector r , we use the geometry shown in Figure E6.14d.
PA

The distance x, needed in forming r , is equal to (27 — d)/sin 60°:


PA

60°
30 cm 27cm (15)
60° 9 cm

Figure E6.14d Answer 6.9 Because r PT = r /2.


PB
Page 422

Therefore

(16)
Substituting Equations (12) and (16) into (14), we get

(17)
Equating the two expressions for in Equations (13) and (17) gives
(18)
It is seen that the angular speed of gear is 36 times that of and in the
opposite direction.

Question 6.10 Give an argument why has to be turning in the


opposite direction from that of (Hint: Use the original figure and focus
your attention on points P and O of )

Does A n Instantaneous A x i s of Rotation Exist in General?

We recall that in Chapter 3 w e were able to s h o w that in plane motion a


rigid body except w h e n its angular velocity vanishes, always has a
point of zero velocity (the instantaneous center and hence a line of
points of zero velocity exists which w e may call the instantaneous axis of
rotation). We n o w show that in general (three-dimensional) motion, such
an axis does not always exist. We start with an arbitrary point P with
velocity v , and sketch its velocity along with the angular velocity vector
P

Plane
of in the reference frame (See Figure 6.10(a).)
Note that in Figure 6.10a there is a plane defined by the vectors v P

and drawn through P, unless the two vectors are parallel. If they are,
then the motion of in is like that of a screwdriver — the body turns
around a line that translates along its axis. The general case ( v not P

parallel to ) may also be reduced to a screwdriver motion as follows.


(a) First w e replace v by its components parallel
P and perpendicular
to (See Figure 6.10b.) Next w e consider a plane parallel to
and separated from by the distance d as shown in Figure 6.10(b). Point
Q is the projection of P into the plane and w e may write its velocity in

Answer 6.10 Each point of (extended) which lies on line has zero velocity. Thus
the velocity of B is its distance from times and the same is true for point A. The
former is seen to be coming out of the paper, and the latter going into it, both about
(b) Therefore as determined by the direction of v , is in the negative x direction, oppo­
A

Figure 6.10 site to


Page 423

of P by Equation (6.46):

Note from Figure 6.10b that


this is the unit vector
directed from P to Q.

The vector triple product is equal to

so that

Therefore if d is chosen equal to the line will be a "screwdriver


line" and the motion of will be as s h o w n in Figure 6.10(c).

Figure 6.10(c)

All points on have velocities along at the given instant, while those
off the line have the same velocity component parallel to in addition,
they rotate around it. This is the simplest reduction possible for the
motion of and it is clear that unless no points can have zero
velocity. Thus in three dimensions w e are no longer assured of having an
instantaneous axis as w e were in dealing with plane motion.
There are, however, special cases in three dimensions in which an
instantaneous axis exists; a good example is a cone rolling on a plane
(Figure 6.11). Note that in this case the entire line of contact is at each

Figure 6.11
Page 424

instant at rest on the plane. Since the angular velocity of the cone is
parallel to this line, all the contact points have which w e have
shown must be true if the instantaneous axis is to exist in three dimen­
sions.

PROBLEMS Section 6.7

6.49 A youngster finds an old wagon wheel and


pushes it around with the center C moving at constant
speed in a horizontal circle. (See Figure P6.49(a).) If the 6 In.
end O of the axle stays fixed while C returns to its starting 3 in, 1 in. 4 in.
point in T seconds, compute the angular velocity vector of
the wheel where is the ground frame (Fig­
ure P6.49(b)). Give the result in terms of b, and T.

Figure P6.50

6.51 In the preceding problem, find and


6.52 Find the angular acceleration of the wagon wheel
of Problem 6.49.
6.53 Using the data of Problems 6.25 and 6.26, find the
(a) velocity and acceleration in of the point Q at the end of
board (beneath the little girl) having position vector
Do this using Equations (6.56, 6.57) of this
section, then check your answers using Equations (6.44,
6.51).
6.54 The angular velocity of a rigid body in motion in
frame is rad/sec. If possible, locate (from
P) a point Q of with zero velocity when:

In both cases explain why Q exists or not in light of the


discussion about points of zero velocity at the end of the
(b) preceding section.
Figure P6.49
6.55 Disk in Figure P6.55 spins relative to the bent
shaft at constant angular speed rad/sec; rotates in
the reference frame at the constant rate rad/sec.
6.50 The bevel gears and in Figure P6.50 support (The directions are indicated in the figures.) Using the
the turning shaft whose angular velocity is given rigid-body equations (6.56 and 6.57), find and
by rad/sec, in which is parallel to the z for point Q on the periphery of the disk. Express the result
direction in both frames and (Gears and are seen in terms of components along the (x, y , z) axes, which are
to be part of ) Find and fixed in
Page 425

6.59 The bevel gear in Figure P6.59 is fixed to a refer­


ence frame in which the mating gear moves. The axis
OC of turns about the z axis at the constant rate
= 0.2 rad/sec, and the angle OQC is 30°. Find the angu­
lar velocity of in

Ball joint

Figure P6.S5 Figure P6.59

In the preceding problem, find the angular acceler­


ation of in for the same defined motion.
6.56 Rework the preceding problem, this time using the 6.61 A differential friction gear can be made with either
moving-frame concept of Section 6.5. Let bar be the bevel gears, as shown at the top of Figure P6.61, or fric­
frame in motion with respect to and let Q be the point tion disks, as shown at the bottom. In each case, body
moving relative to both and rolls on and and may rum without resistance on the
crank arm Find and the velocities of points
6.57 A wheel of radius r turns on an axle that rotates
A and B of
with angular velocity about a vertical axis (z) fixed
relative to ground (Figure P6.57). If the wheel rolls on the
horizontal plane and is constant, find:
a. The angular velocity and angular acceleration of
the wheel relative to the ground
b. The acceleration, relative to the ground, of the
point on the wheel in contact with the horizon­
tal plane.

Wheel

Axle

Figure P6.57

Tight
6.58 Rework Example 6.14, but this time suppose that fit
the radius of gear is 44 cm instead of 30 cm (and that
its center is at the same point of ). Explain why gears
and are now moving in the same direction. Figure P6.61
Page 426

6.62 The uniform, solid, right circular cone in Fig­


ure P6.62 rolls on the horizontal plane Let represent a
frame in which the vertical axis z and the cone's axis
are fixed. (Hence has a simple angular velocity in
about the vertical.) Show that the cone can roll so that
are constants and

Figure P6.65

Figure P6.62

17.5 m
6.63 The center C of the bevel gear in Figure P6.63
rotates in a horizontal circle at a constant speed of 40
m m / s (clockwise when viewed from above). The mating
gear is fixed to the reference frame shaft rigidly
7.5 m
attached to is connected to through a ball and socket 5m
joint at O. Find the angular velocity vector of in

Figure P6.69

6.66 In Example 6.14 find the radius of gear for which


30°
it will remain stationary in as and turn.
6.67 In Example 6.14 label the radius of gear as H and
call the radius of the 28-cm gear R. Show that the rela­
50 mm tionship between and is given by
( T h e c e n t e r of is fixed i n )

Figure P6.63 6.68 If the speed of point C is constant in Example 6.11,


2

find the acceleration of Q at the instant given.


6.69 Collars and in Figure P6.69 are attached at Q
and C to rod by ball and socket joints. Point C has a
2 2
3
motion along the x axis given by x = — 0.012t m. Find
2
6.64 Find for the gear in Problem 6.63.
the velocity of C as a function of time.
1

*6.65 Plate in Figure P6.65 has the following motion:


*6.70 Cone rolls on cone so that its axis of symmetry
1. Corner A moves on the x axis. (x) moves in a horizontal plane through O, turning about z
2. Comer B moves on the y axis with constant veloc­ at rate rad/sec. (See Figure P6.70.) Cone is rotating
about (—z) at rad/sec. Find, with respect to the frame
ity in./sec.
in which turns, the angular velocity and angular
3. Some point of the top edge of (point Q at the
acceleration of and the acceleration of point A. (Axes
instant shown) is always in contact with the z axis. (x, y, z) are fixed in frame which turns so that x is
Find the angular velocity vector of the plate when x A
always along the axis of symmetry of and z is always
= 3 in. vertical. Also, and are constants.)
Page 427

To drive shaft

Pin

60°

120°

Pin

(a)
Figure P6.70

30 ft
20 m p h

6 ft
(b)

Figure P6.73

angular velocity of In the process note that the differ­


Figure P6.71 ential allows the driven wheels to turn at different angu­
lar speeds.

6.71 The two shafts and are fixed to bevel gears It is assumed in the following three problems that not all
and as shown in Figure P6.71. (a) Prove that if the ve­ points of body have zero acceleration.
locities of each pair of contacting points are to match
along the line of contact of the gears, then points A , B, and 6.74 Show that for a rigid body in general motion,
C must coincide, (b) Let A , B, and C coincide and find the there is a point Q of zero acceleration if and
ratio of to is not parallel to Hint: Let P be an arbitrary point, and
let and
6.72 (a) In the preceding problem show that if noting that there is no loss in generality in these assump­
, the result is still true about A , B, and C coinciding tions. Set set
(b) Find for this case. and solve for x, y, and z.
6.73 Depicted in Figure P6.73(a) are the main features of 6.75 (a) Following up the previous problem, show that if
an automobile differential. The left and right axles, and and at a given instant, then at this instant
are keyed to the bevel gears and . Gear is fixed to there is a point Q of zero acceleration if and only if the
the case and the combination is free to turn in bearings accelerations of all points of B are perpendicular to (b)
around line Gear meshes with gear attached to the Investigate the case and
car's drive shaft. As the casing turns about the common
axis of and its pins bear against the other two bevel 6.76 Here is another follow-up on Problem 6.74: Show
gears within which are and . (Observe that these that at any instant when the two vectors and are
two gears do not turn about their axes at all on a straight parallel, there is a point of with zero acceleration if and
road.) only if the accelerations of all its points are perpendicular
to and
Suppose the car makes a 30-ft turn at a speed of 20
mph (Figure P6.73(b)). If the tire radius is 14 in., find the * 6.77 Find the angular acceleration of the plate of Prob­
angular velocities of and and use them to compute the lem 6.65 at the same instant of time.
Page 428

6.8 Describing the Orientation of a Rigid Body


In the case of plane motion of a rigid body if w e know the angular
velocity

as a function of time, w e may clearly integrate to find the orientation of


at any time t:

Thus in plane motion w e may completely specify the position of by


giving the xy coordinates of a point (usually the mass center C is chosen to
be the point) and the orientation angle

T h e Eulerian A n g l e s

A major difference between planar and general motion is that the angles
which yield a body's orientation in space in three-dimensional motion
are not the integrals by simple quadratures of the angular velocity com­
ponents of the body. In fact, finding (in general) the orientation of a body
in closed form, given and the orientation of at t = 0, is an
unsolved fundamental problem in rigid-body kinematics. We n o w intro­
duce the Eulerian angles in order to s h o w the difficulty of detennining a
body's orientation in space w h e n the motion is nonplanar.
We begin with the body oriented so that the body-fixed axes
{x, y, z) initially coincide respectively with axes (X, Y, Z) embedded in
the reference frame Let and be sets of unit vectors re­
spectively parallel to (x, y, z) and (X, Y, Z). Three successive rotations
about specific axes will n o w be described that will orient in (See
Figure 6.12(a))
In Figure 6.12(b) the first rotation is through the angle about the Z
axis. Let the n e w positions of (x, y, z) after this first rotation be denoted by
(x , y , z ) as shown; these positions are embedded in an intermediate
1 1 1

frame . Note that axes Z and z are identical and that


1 has the simple
angular velocity in Note also from Figure 6.12 that and
are unit vectors that are respectively and always parallel to x , y , z . Next
1 1 1

(Figure 6.12(c)) a rotation through the angle about axis J/J moves the
body axes into the coordinate directions (x , y ,z )
2 2 2 of a second interme­
diate frame having unit vectors . A final rotation, this
time of amount about z (Figure 6.12(d)), turns the body axes into their
2

final positions in indicated by (x, y,z).


It is clear that w e may use the addition theorem to express the angular
velocity of in as follows:

(6.58)
Page 429

(a) Showing all three rotations

(b) First Eulerian angle (c) Second Eulerian angle (d) Third Eulerian angle
rotation , rotation rotation ,
about Z about y, about z2

Figure 6.12 Eulerian angles.

in which w e again remark that is a unit vector in the jth coordinate


direction of frame
If the preceding expression for is to be functionally useful, its
three terms should all be expressed in the same frame — that is, in terms
of components associated with the directions of base vectors all fixed in
direction in a common frame. N o w in practice, as w e shall see, they are
sometimes expressed by components associated with body-fixed direc­
tions, at other times with space-fixed directions, and at still other
times with directions fixed in intermediate frames such as or . For
example, let us write in body components. This means, looking at
Equation (6.58), that w e must express and in terms of and .
Page 430

We see from Figure 6.12 that


(6.59)
and that

or
(6.60)

where and so forth. Substituting these expressions


into Equation (6.58) then gives in body components:
(6.61)

Alternatively, w e may express in terms of its components


in frame by writing and in terms of Again referring to
the figure, w e see that
(6.62)
and

or
(6.63)
so that, substituting into Equation (6.58), w e have written in
(6.64)
Finally, w e notice that expressing in the intermediate frames is
easier still. To write it in terms of its components in , w e note that

(6.65)
and
(6.66)

so that from Equation (6.58) w e get

(6.67)
All these expressions for appear different, but of course they all
represent the same vector written in different frames. The components
may vary from frame to frame, but the vector is the same. In the first
exercise at the end of this section, the reader will be asked to write in
yet another frame,
The angles are known as the Eulerian angles. They represent
one way of orientating a rigid body in space. Unfortunately the Eulerian
angles do not carry the same symbol from one book to the next;
worse still, the order and even the directions of the rotations vary from
writer to writer. Obviously, then, it is important to choose a set to work
with and then be consistent.
Page 431

reference frame A physical feel for the Eulerian angles may be gained by considering
(axes X, Y, Z) a gyroscope spinning in a Cardan suspension as s h o w n in Figure 6.13.
In this system the Eulerian angles may be used as follows to
pinpoint the orientation of the rotor B in space:

1. First Rotation: With respect to frame w e rotate the plane of the


outer gimbal G„ (frame in the earlier theory) about axis Z through angle
. Axis X is turned into x and axis Y into y ; frames (bodies) G and are
t 1 i

not s h o w n yet, but they move rigidly with G in this first rotation. (See
O

Figure 6.14.)
2. Second Rotation: Next w e turn the plane of the inner gimbal
about axis y , through angle thereby tilting G with respect to G .
2 i O

G = outer gimbal (axes x,, y z,)


0 u Axis z is thereby rotated into z and axis x into x . Body (not s h o w n in
1 2 1 2
G, = inner gimbal (axes x y z )
= rotor (axes x, y, z)
2) 2) 2
Figure 6.15) goes along for the ride.
Figure 6.13 3. Third Rotation: The third and last rotation turns the rotor about
axis z through angle . (See Figure 6.16.) This allows to spin relative to
2

G . Axis x is turned into x and axis y into y.


i 2 2

Through these three Eulerian angle rotations, the body can be


positioned in any desired orientation in space . We are n o w able to see
the difficulty of solving for the orientation of a rigid body in general
motion. If w e write then Equation (6.61) is
equivalent to

(6.68)

Solving for the rates of change of the Eulerian angles gives

(6.69)

Figure 6.14 First rotation. Figure 6.15 Second rotation. Figure 6.16 Third rotation.
Page 432

We see from these equations that even if w e knew the components as


functions of time in closed form,* it would still be a formidable task to
integrate Equations (6.69) analytically to obtain the Eulerian angles and
thus to know the body's orientation in space. This is usually not even
possible, and resort is made to computers that can numerically carry out
integrations with a step-by-step scheme such as Runge-Kutta.
Incidentally, the sin denominators in Equations (6.69) present seri­
ous obstacles in the dynamics of space vehicles; whenever is zero or a
multiple of the equations develop a singularity. Sophisticated pro­
gramming or, in some cases, completely different mathematical schemes
for orienting the body are required to overcome such difficulties.
We mention that use of the preceding set of Eulerian angles as de­
fined requires that w e maintain the order of rotation. To illustrate the
importance of rotation order, w e remark that if this book is rotated
through two rotations about the space axes Y and Z, in opposite
orders as suggested by Figure 6.17, it will end up in a different position.
We should point out, however, that there are ways of setting up the axes
and angles which make a body's final orientation independent of order.
For instance, just as in our Eulerian angle development, let Z be fixed in
let z be fixed in and let y be always perpendicular to both Z and z. But
n o w restrict Z and z to be nonparallel. (Let z, the axis of lie along X
initially, for example.) In this case, the angles as defined earlier
may be performed in any of the six possible orders and the body's
resulting orientation in will be the same each time! †

First Y ..then Z . . . gives this view

BUT! . . . first Z . then Y... . gives this!

Figure 6.17 Finite rotations.

* The equations governing these three components of angular velocity are the Euler
equations of rigid-body kinetics. They themselves are also nonlinear and unsolvable in
closed form in general three-dimensional motion except for a few special cases. We shall
be studying these equations in Chapter 7.
†Also, subsequent reorientations will likewise be order-independent; see "Successive
Finite Rotations," by T. R. Kane and D. A. Levinson, Journal of Applied Mechanics, Dec.
1978, Vol. 45, pp. 9 4 5 - 9 4 6 .
Page 433

A n alternative set of three rotations has become popular in the litera­


ture in recent years:

1. Rotate through about X.


2. Then rotate through about y . 1

3. Then rotate through about z . 2

This sequence results in angular velocity components in — that is, in


the body-axis system (x, y, z) — of

(6.70)

Although (6.70) is not made up of classic Eulerian angles, the result is an


equally valid set of relations between the components and the
angles of rotation. The order and axes of rotations of the in this
system are quite simple to remember.
There are alternatives to using the Eulerian angles or the angles in
Equation (6.70) to orient a body in space. One such alternative is to use
the quaternions of Hamilton, which are free of the singular points caused
by zero denominators at certain values of the Eulerian angles. (For this
reason, quaternions were used in the Skylab Orbital Assembly's attitude
control system.) Another approach to the orientation of a body in space is
to determine the direction cosines of a unit vector, fixed in direction in
space with respect to a set of axes fixed in the body Let this unit
vector (call it ) have direction cosines (p, q, r) with respect to axes (x, y, z)
fixed in Further, let be unit vectors, always respectively parallel
to (x, y, z). Then

and, differentiating in w e obtain

This vector equation has the following scalar component equations:

(6.71)

If the components are either prescribed or else found from kinetics


equations (to be studied in Chapter 7), then Equations (6.71) may be
solved for the direction cosines, thereby orienting in Equations (6.71)
are known as the Poisson equations. They are alternatives to equations
such as (6.68) and (6.70).
Page 434

PROBLEMS • Section 6.8

6.78 Using the Eulerian angles discussed in this 6.83 Chasle's theorem states: The most general dis­
section, express in (terms of its components in) the placement of a rigid body is equivalent to the translation
frame of some point A followed by a rotation about an axis
6.79 Show that the magnitudes of are all the same through A. Show that this result follows immediately
as expressed (a) in in Equation (6.61); (b) in in Equa­ from the previous problem.
tion (6.64); and (c) in in Equation (6.67). 6.84 The circular drum of radius R in Figure P6.84 is
6.80 Derive Equations (6.70). pivoted to a support at O, where O is a distance R/2 from
the center C of the drum. A weight (particle) hangs
6.81 Write in by using the successive rotations , from a cord wrapped around the drum. The drum is
, and that resulted in Equation (6.70) when expressed slowly rotated rad clockwise about O. Find the dis­
in placement of Hint: Use the result of the preceding
6.82 Euler's theorem for finite rotations is stated as fol­ problem, with C being the point A and with a
lows: The most general rotation of a rigid body with rotation following the translation. Add the displacements
respect to a point A is equivalent to a rotation about some of during each part.
axis through A. Prove the theorem. Hint: Let A be consid­ 6.85 In Figure P6.85, the sphere rolls on the plane,
ered fixed in the reference frame in which moves. (See and its angular velocity in the reference frame in which
Figure P6.82.) Let point P be at P prior to the rotation and
1
(x, y, z) are fixed, is given by Equation (6.64). Noting
at P afterward; assume the same for point Q (Q before,
2 1
that write the constraint equations (the
Q after). Bisect angle P,AP with a plane normal to the
2 2
"no slip" conditions) relating and to the Eulerian
plane of the angle. Do the same for Q , A Q and consider
2
angles and their derivatives.
tho intersection of the two planes.

Intersection
of planes

Drum

Figure P6.82 Figure P6.84 Figure P6.85

6.9 Rotation Matrices


Suppose that a vector Q is written in a frame and that its components
are the elements of a column matrix . It is possible to develop a set
of 3 X 3 matrices [T ], [T ], and [T ] each of which, w h e n postmultiplied
x y z

by , gives the components of Q in a n e w frame rotated about an


x, y, or z axis, respectively, of These matrices are handy work-savers.
For example, they reduce the work involved, in going from Equa­
tion (6.58) to (6.61) or (6.64), to a pair of simple matrix multiplications.
We shall develop [T ] and then state the results for [T ] and
2 r

[T ], which are derived similarly. Let


y in
which are a triad of unit vectors having fixed directions
Page 435

along axes (x, y, z) of frame Suppose further that is a frame


whose orientation may be obtained from that of by a rotation
through the angle about z. The rotated axes, which were aligned
with (x, y, z) prior to the rotation, will be denoted (x , y , z ) with
1 1 1

associated unit vectors


We note that in the "new" frame w e may write
where Q = Q and
Z1 Z since the rotation is about
this axis, common to both frames. To get Q and Q in terms of Q , Q ,
x1 y1 x y

and , w e use (see Figure 6.18):

(6.72)

and similarly,

(6.73)

Figure 6.18

Together with Q = Q , the Equations (6.72, 6.73) give the compo­


Z1 Z

nents of Q in the rotated frame in terms of its components Q , Q , Q ) x y z

back in N o w w e are ready to observe that if the matrix [T ] is defined as


z

then the same results for the components of Q in the rotated frame are
obtained from the matrix product
Page 436

The rotation matrices for rotations of about x, and about y, are


respectively given by [T ] and [T ]:
x y

The student may wish to verify one or both of these matrices, as w e


did for [T ]. Note the change in the "sign of sine" in the [T ] matrix.
z y

Moreover, if w e must turn about an axis through a negative angle, we


need only change the signs of both sine terms; this follows from the fact
that while We n o w consider ex­
amples of the use of the rotation matrices. We shall use some shorthand
common in the literature of kinematics: for for etc.

EXAMPLE 6.15

Use rotation matrices to obtain the components of in body coordinates,


given its representation (Equation 6.64) in the reference or space frame
Solution
We premultiply expressed in in matrix form, with rotation matrices of
about the 3-axis, then about the new 2-axis, and then about the new and final
3-axis:

components of

This gives the components


of

This yields the components


of

Finally, we obtain the components of in


Page 437

Comparing the elements of this matrix with the components in Equation (6.61),
we see that rotation matrices indeed furnish us with a rapid means of "convert­
ing" a vector from one frame to another. Note also that the bracket in the second
line above contains expressed in , previously derived as Equation (6.67).
The bracket in the third line gives the components of in

The next example illustrates a use that was made of rotation matrices
by one of the authors in the development of earth stations.

EXAMPLE 6.16

Using rotation matrices, compute the angles A (azimuth) and E (elevation)


through which an antenna must respectively turn about (a) the negative of local
vertical and then (b) the new, rotated position of the elevation axis in order to
sight a satellite in geosynchronous orbit (see Problems 2.64,65). Angles A and E
are called look angles, and an antenna that performs azimuth followed by eleva­
tion in this manner is said to have "el over az" positioning. The azimuth angle A is
a function of the local latitude and the relative west longitude of the satellite;
the elevation angle E depends additionally on R /R the ratio of the earth and
e

orbit radii. (See Figure E6.16a.)

Site P

Satellite
Equator

Geosynchronous orbit
Figure E6.16a

Solution
The frame has origin at the center of the earth as shown; the xy plane
contains the site P and its meridian. The coordinates of the satellite in this frame
are seen to be given by the position vector

First we rotate the frame through the latitude angle about z in order to line up
the new axis x with the local vertical at the site P. We call the resulting rotated
1
Page 438

frame and obtain the following for the new components of


and so forth):

Next we translate the axes to the site, as shown in Figure E6.16b. The only
component of that changes is x , and we see by inspection that
1

Note that we must subtract the earth radius R from x to get the proper x
e t 2

coordinate of the satellite.


The second rotation is the azimuth rotation about—x ; if we call the rotated
2

frame , the coordinates (x , y , z ) of S in this frame are given by


3 3 3

Figure E6.16b

Here we take an important step. We want the z component of r to be zero


3 PS

because we wish to rotate next in elevation about z and end up with the "bore-
3

sight" (axis) of the antenna aiming at the satellite. Thus angle A (see Fig­
ure E6.16c) is determined by setting the third element of the preceding matrix to
zero:

(1)
Figure E6.16c
Finally we rotate through angle E about the z axis:
3
Page 439

Now we come to the condition that will allow us to determine the value of
angle E (see Figure E6.16d): We wish the antenna to aim directly at the satellite.
Since the antenna boresight is now in the — y direction, we wish the elevation
3

rotation to stop when the x coordinate is zero:


3

(2)
Figure E6.16d

If then (2) becomes

in which the azimuth angle A is given by Equation (1), so that

(3)

There is a single circle in the sky in which geosynchronous satellites


can exist. This circle, which was examined in Problem 2.64, has rapidly
become very crowded, however. As of the summer of 1994, there were
nearly 500 satellites in geosynchronous orbit. Six years earlier, there
were just over 100, and in early 1982, only around 30.

PROBLEMS • Section 6.9

6.86 For the United States the eastern and western limits
of usable satellite positions in the geosynchronous arc
(see Problem 2.64) are about 70° and 143°W longitude,
respectively. Find the ranges in azimuth and elevation
that are required if the antenna in Example 6.16 is to
sweep from the eastern to the western limit for a site at: (a)
34°N latitude and 84°W longitude; (b) your home town.
(Select a city in the contiguous United States if you are
from another country.)
6.87 Use Equations (6.70) together with rotation matri­
ces to compute the angular velocity components in space-
fixed axes.
6.88 An antenna has three rotational degrees of free­
dom (see Figure P6.88):
1. Azimuth angle A about local vertical z, Figure P6.88
2. Elevation angle E about an axis originally parallel
to x,
3. Polarization angle P about the axis of symmetry of
the dish (originally parallel to y).
Use the addition theorem together with rotation matrices 6.89 In the preceding problem calculate in terms of
to calculate in terms of its components in its components in
Page 440

• 6 . 9 0 Plot the elevation angle E versus the satellite


angle (see Example 6.16) for the following values of
(on the same graph): and 80°.
What do the crossings of the axis of these curves
physically represent?
• 6.91 Calculate the look angles (see Example 6.16)forthe
case in which the elevation rotation is performed prior to
azimuth ("az over el" positioning).
6.92 It takes six orbital parameters to establish the loca­
tion of a planet with respect to a frame fixed in space. (See
Figure P6.92.) To find its orbital path, we first turn Vernal Ascending
through the angle ft in the ecliptic plane (the plane con­ equinox
NODE
taining the path of the sun as we see it from earth) to the
ascending node (the intersection of the ecliptic plane with
the planet's path when going north). Next we turn in
inclination through the angle i about x\ to obtain the tilt of
the planet's plane. (Thus earth's inclination is defined as Figure P6.92
zero.) Finally, the angle locates the perihelion of the
planet's orbit. This is the closest point of the orbit to the
sun's center S. Two other quantities give the orbit's shape, the space frame use rotation matrices to obtain the
and a sixth one locates the planet in its orbit with respect components of v in the frame located as shown
to the perihelion. If v = (v , v , v ) is a vector defined in
x y z in the orbital path at P, in terms of i, and

PROJECT PROBLEM • Chapter 6

6.93 After reading Example 6.3, construct a simple


model that illustrates the workings of a universal joint
(see Figure P6.93 for some examples).

Figure P6.93
Page 441

SUMMARY • Chapter 6

The key concept of this chapter is angular velocity. If and are two
rigid bodies (or frames of reference), the angular velocity of relative to
(or of in ), is the unique vector that connects the derivatives
relative to and of any vector, say Q, by

The angular velocity describes the rate at which the orientation of one
body changes relative to another. Among the important properties are:

and the addition theorem (a chain rule):

which can be extended by repetition to

and in fact to any number of bodies.


This rather abstract-seeming concept of angular velocity reduces to
the familiar form of Chapter 3 w h e n w e have "plane-motion" situations
in which a direction is fixed in both of two bodies. So if is the constant
vector (denotes a fixed direction) in both of the bodies and w e have

This equation, along with the addition theorem, gives us an important


tool to deal with systems of bodies that appear complex but where each
body moves relative to a neighbor by rotating about an axis fixed in the
neighbor.
Angular acceleration is the derivative of angular velocity, and the
derivative may be calculated in either of the two bodies (frames) in­
volved.

Angular accelerations cannot be added with the simplicity of an addition


theorem such as (6.27), but instead obey the formula:

Velocities of a point P relative to two frames and are related by

where 0' is a point fixed in Similarly for accelerations, w e have

By letting P be fixed in (as is O') w e can deduce from the two


previous equations a pair of expressions relating the velocities and then
the accelerations (relative to a frame ) of two points such as P and O',
Page 442

both fixed in :

These are the counterparts in three dimensions of Equations (3.8) and


(3.19).
Finally, w e have discussed a system that is often used to describe the
orientation of a body relative to a frame of reference — Euler angles.
Angular velocity in terms of rates of change of Euler angles has been
developed.

REVIEW QUESTIONS • Chapter 6


True or False?

1. The angular velocity of body in frame depends only upon the


changes in orientation of with respect to
2. The addition theorem for angular velocity applies equally well to
angular acceleration.
3. The formula relating the velocities of two points of a rigid body in
plane motion, applies to three-dimensional
problems provided that and the v's become three-
dimensional vectors.
4. The formula relating the accelerations of two points of a rigid body in
plane motion, applies to three-
dimensional problems provided that and the a's become
three-dimensional vectors.
5. The equation in plane motion extends to three similar linear
equations in general motion for determining the orientation angles.
6. For a point P to have a nonvanishing Coriolis acceleration, there
must be both a relative velocity of P with respect to the "moving
frame" and an angular velocity of the moving frame relative to the
reference frame.
7. If w e premultiply a vector {v} by a rotation matrix [T], the 3 X 1
vector w e get contains the "new" components of v in the rotated
frame.
8. The Eulerian angles are used to orient a body in three-dimensional
space.
9. The Eulerian angles are three rotations and about axes which
were originally distinct and orthogonal.
10. It is possible, for any moving point P, to choose a moving frame such
that the Coriolis acceleration of P vanishes identically.
11. The angular velocity vector is used to relate the derivatives of a vector
in two frames.
12. If one yoke of a misaligned universal (Hooke's) joint turns at constant
angular speed, so does the other.
443

13. In general motion of a rigid body as long as there is a point of


zero velocity of or -extended.
14. The order of rotations is important in orienting a body if the Eulerian
angles are used as defined in this chapter (in conjunction with the
Cardan suspension of the gyroscope).

ANSWERS: 1.T 2.F 3.T 4.F 5. F 6.T 7. T B.T 9. F 10. T 11. T 12. F 13. F
14. T
7 KINETICS OF A RIGID BODY IN
GENERAL MOTION

7.1 Introduction
7.2 Moment of Momentum (Angular Momentum) in Three
Dimensions
7.3 Transformations of Inertia Properties
Transformation of Inertia Properties at a Point
7.4 Principal Axes and Principal Moments of Inertia
Principal Axes at C
Calculation of Principal Moments of Inertia
Calculation of Principal Directions
Principal Axes at Any Point
Orthogonality of Principal Axes
Equal Moments of Inertia
Maximum and Minimum Moments of Inertia
7.5 The Moment Equation Governing Rotational Motion
The Euler Equations
Use of Non-Principal Axes
Use of an Intermediate Frame
7.6 Gyroscopes
Steady Precession
Torque-Free Motion
7.7 Impulse and Momentum
7.8 Work and Kinetic Energy
SUMMARY
REVIEW QUESTIONS

Page 444
Page 445

7.1 Introduction
I n C h a p t e r 2 w e f o u n d t h a t , r e l a t i v e t o a n i n e r t i a l f r a m e of r e f e r e n c e ,
m o t i o n of a b o d y is g o v e r n e d b y

(7.1)

and a m o m e n t equation

(7.2)

or

(7.3)

w i t h O b e i n g fixed in t h e i n e r t i a l f r a m e . T h e s e g e n e r a l e q u a t i o n s w e r e
s p e c i a l i z e d t o plane m o t i o n of a rigid b o d y in C h a p t e r 4 a n d will n o w
b e u s e d t o s t u d y t h e general m o t i o n of in three dimensions.
A s w e i n d i c a t e d i n C h a p t e r 2 , t h e first o f t h e t w o v e c t o r e q u a t i o n s
g i v e n a b o v e d e s c r i b e s t h e m a s s c e n t e r m o t i o n of any s y s t e m . * It is a p p l i ­
c a b l e , for e x a m p l e , t o rigid o r d e f o r m a b l e s o l i d s , s y s t e m s of s m a l l m a s s e s ,
liquids, a n d gases. For a b o d y in general (three-dimensional) motion,
E q u a t i o n (7.1) n o w p o s s e s s e s t h r e e n o n t r i v i a l s c a l a r c o m p o n e n t e q u a ­
t i o n s w h o s e s o l u t i o n s a l l o w u s t o l o c a t e t h e m a s s c e n t e r C.
W e n o t e that, as w a s the case w i t h p l a n e motion, the m a s s center can
m o v e i n d e p e n d e n t l y of t h e b o d y ' s c h a n g i n g o r i e n t a t i o n ( p r o v i d e d t h a t
t h e external forces d o n o t t h e m s e l v e s d e p e n d o n t h e b o d y ' s a n g u l a r
m o t i o n , w h i c h is f r e q u e n t l y t h e c a s e ) . W e s a w s u c h a n e x a m p l e i n S e c t i o n
6.6 w h e n w e e x a m i n e d t h e m o t i o n o f (a p a r t i c l e o r ) t h e m a s s c e n t e r of a
body near the rotating earth. W e emphasize that such a simple a n d
n a t u r a l e x t e n s i o n f r o m t w o t o t h r e e d i m e n s i o n s w i l l not o c c u r w i t h t h e
orientation (or a n g u l a r ) m o t i o n of a s w e shall s e e in Section 7.5. T h e
r e a s o n is t h a t i n E q u a t i o n (7.2) cannot b e w r i t t e n a s t h e s u m of
t h r e e t e r m s of t h e f o r m
I n t h e r e m a i n d e r of t h e c h a p t e r w e s h a l l first d e v e l o p t h e e x p r e s s i o n
for t h e m o m e n t of m o m e n t u m of a rigid b o d y in general motion. This
w i l l t h e n l e a d u s i n t o a s t u d y of t h e i n e r t i a p r o p e r t i e s of Having
p a r t i a l l y e x a m i n e d t h e c o n c e p t of i n e r t i a i n C h a p t e r 4 , w e s h a l l e x t e n d
this s t u d y to include transformations at a point as well as principal m o ­
m e n t s a n d a x e s of i n e r t i a . T h e n a n d o n l y t h e n s h a l l w e b e f u l l y p r e p a r e d
t o d e r i v e t h e E u l e r e q u a t i o n s t h a t g o v e r n t h e r o t a t i o n a l m o t i o n of a rigid
b o d y in g e n e r a l m o t i o n . W e shall also e x a m i n e , a s w e d i d for t h e p l a n e

* Excluding throughout, of course, relativistic effects occurring when velocities are not
small compared to the speed of light.
Page 446

m o t i o n i n C h a p t e r 5 , s o m e s p e c i a l i n t e g r a l s of t h e e q u a t i o n s of m o t i o n ,
w h i c h a r e k n o w n a s t h e p r i n c i p l e s of i m p u l s e a n d m o m e n t u m , a n g u l a r
impulse a n d angular m o m e n t u m , a n d w o r k a n d kinetic energy.

7.2 Moment of Momentum (Angular Momentum) in


Three Dimensions
W e s a w i n C h a p t e r 4 t h a t w h e n it is r e a s o n a b l e t o t r e a t a b o d y a s rigid,
t h e e q u a t i o n s of m o t i o n of a r e greatly simplified. T h e m a s s center
b e c o m e s fixed i n t h e b o d y , a n d t h e m o m e n t of m o m e n t u m is e x p r e s s i b l e
i n t e r m s of t h e a n g u l a r v e l o c i t y a n d t h e i n e r t i a p r o p e r t i e s of — h e n c e t h e
o t h e r n a m e of m o m e n t of m o m e n t u m : angular m o m e n t u m . W e s h a l l
p r o c e e d n o w t o s t u d y t h e a n g u l a r m o m e n t u m H of a b o u t a p o i n t P i n
P

general (three-dimensional) motion. W e shall see that t h e equations that


result are m u c h m o r e complicated t h a n their plane m o t i o n counterparts.
L e t u s b e g i n b y i n t r o d u c i n g a s y s t e m of r e c t a n g u l a r a x e s (x, y, z)
w h i c h h a v e t h e i r o r i g i n a t P . T h e a n g u l a r v e l o c i t y of i n r e f e r e n c e f r a m e
m a y t h e n b e e x p r e s s e d i n t e r m s of its c o m p o n e n t s a l o n g t h e s e a x e s b y

(7.4)

T h e l o c a t i o n , r e l a t i v e t o P , of a t y p i c a l p o i n t i n t h e b o d y is g i v e n b y

(7.5)

T h e m o m e n t of m o m e n t u m of r e l a t i v e t o P is n o w d e f i n e d ( s e e S e c ­
tions 2.6 a n d 4.3) t o b e
(7.6)

i n w h i c h v, t h e v e l o c i t y of t h e m a s s e l e m e n t dm, i s n o t t h e d e r i v a t i v e of r
b u t r a t h e r of t h e p o s i t i o n v e c t o r t o t h e e l e m e n t f r o m a p o i n t fixed i n t h e
reference frame as s h o w n in Figure 7.1.

•Path
of dm

Figure 7.1

Since is a rigid b o d y , w e k n o w f r o m E q u a t i o n ( 6 . 5 6 ) t h a t
a n d w e m a y substitute this expression into E q u a t i o n (7.6) to
obtain
Page 447

U s i n g t h e m a s s c e n t e r d e f i n i t i o n , t h e i n t e g r a l i n t h e first t e r m o n t h e
r i g h t - h a n d s i d e is mr PC Therefore

(7.7)

I n t h e c a s e s w h e r e e i t h e r (a) P is c h o s e n t o b e t h e m a s s c e n t e r C, o r ( b )
v = 0, o r (c)
P , t h e first t e r m o n t h e r i g h t s i d e of E q u a t i o n (7.7)
v a n i s h e s . For these cases,

(7.8)

Substituting a n d r f r o m E q u a t i o n s (7.4) a n d (7.5) a n d u s i n g t h e i d e n t i t y

(7.9)

w e obtain the result

(7.10)

Question 7.1 W h y may the c o m p o n e n t s b e b r o u g h t outside t h e


various integrals in Equation (7.10)?

O n c e w e recognize t h e inertia p r o p e r t i e s (see Sections 4.3 a n d 4.4),


t h i s a n g u l a r m o m e n t u m e x p r e s s i o n b e c o m e s , for t h e c a s e w h e n P is C ,

(7.11)

T h e f o r m of t h e e q u a t i o n is i d e n t i c a l if p o i n t P is n o t C, b u t r a t h e r e i t h e r
v = 0 or
P ; t h e o n l y d i f f e r e n c e is t h a t t h e i n e r t i a p r o p e r t i e s a r e
c a l c u l a t e d w i t h r e s p e c t t o a x e s a t P i n s t e a d of a t t h e m a s s c e n t e r :

(7.12)

B o t h of t h e s e f o r m s for t h e a n g u l a r m o m e n t u m v e c t o r ( E q u a t i o n s 7 . 1 1
a n d 7.12) will p r o v e i m p o r t a n t to u s in t h e sections to follow.

Answer 7,1 As we have seen in Chapter 6, depends only on how a set of unit vec­
tors, locked into change their directions in The angular velocity is a constant with
regard to integration at a particular instant over the body's volume.
Page 4 4 8

PROBLEMS • Section 7.2

7.1 Find the angular momentum vector H O of the 7.5 Depicted in Figure P7.5 is a grinder in a grinding
wagon wheel of Problem 6.49. mill that is composed of three main parts:
7.2 Find the angular momentum vector of the disk in 1. The vertical shaft which rotates at constant angu­
Problem 6.27 about (a) C and (b) O. lar speed
7.3 Find the angular momentum vector for the bent bar 2. The slanted shaft of length which is pinned to
of Example 4.16 about the mass center, if it turns about and rums with it
the z axis at angular speed 3. The grinder of radius r, turning in bearings at C
7.4 A thin homogeneous disk of mass M and radius r about and rolling on the inner surface of
rotates with constant angular speed about the shaft
As shaft gets up to speed body swings outward,
(Figure P7.4). This shaft is cantilevered from the vertical
and then the angle remains constant during operation.
shaft and rotates with constant angular speed about
Treat the grinder as a disk and find its angular momen­
the axis of Find the angular momentum of the disk
tum vector H in convenient coordinates. (A suggested set
c
about point Q, and s h o w the direction of the vector in a
is shown.)
sketch.
7.6 In the preceding problem note that point O is a
fixed point of all three bodies and extended. Com­
pute the angular momentum H of and verify that
O

H = H + r X L.
O C OC

Figure P7.4 Figure P7.5

7.3 Transformations of Inertia Properties


S o m e t i m e s w e n e e d t h e m o m e n t s a n d p r o d u c t s of i n e r t i a a t p o i n t s o t h e r
t h a n t h e m a s s c e n t e r C of a rigid b o d y These properties can be found
w i t h o u t further integration b y using the parallel-axis theorems, w h i c h
w e r e derived in C h a p t e r 4. These are restated below, w h e r e are
t h e c o o r d i n a t e s of t h e m a s s c e n t e r C r e l a t i v e t o a x e s a t P. F o r t h e m o m e n t s
of i n e r t i a a t P:

(7.13a)
(7.13b)
(7.13c)
Page 449

A n d for t h e p r o d u c t s of i n e r t i a a t P :

(7.14a)
(7.14b)
(7.14c)

EXAMPLE 7 . 1

As a review example, compute the inertia properties at the corner B of the uni­
form rectangular solid of mass m s h o w n in Figure E7.1.
Solution
For the moments of interna w e obtain

Note that the distance between x and x is B c In the same way,

For the products of inertia,


Figure E7.1

Transformation of Inertia Properties at a Point

We n o w consider a second a n d equally important transformation, which


w i l l d e m o n s t r a t e t h a t if w e k n o w t h e m o m e n t s a n d p r o d u c t s of i n e r t i a
a s s o c i a t e d w i t h a s e t of o r t h o g o n a l a x e s t h r o u g h a p o i n t P , w e c a n e a s i l y
c o m p u t e t h e m o m e n t s a n d p r o d u c t s of i n e r t i a a s s o c i a t e d w i t h a n y other
s e t of a x e s h a v i n g t h e s a m e o r i g i n . C o n s i d e r t w o s e t s of a x e s w i t h a
c o m m o n o r i g i n a t P . ( S e e F i g u r e 7.2.) L e t b e t h e d i r e c t i o n c o s i n e s of
x' r e l a t i v e t o x, y, a n d z, r e s p e c t i v e l y . T h e n t h e r e c t a n g u l a r c o o r d i n a t e x' of
a p o i n t Q i n t h e b o d y is r e l a t e d t o t h e r e c t a n g u l a r c o o r d i n a t e s x, y, z b y

1
unit vector along x axis

Figure 7.2 (7.15)


Page 450

W e s e e k a f o r m u l a for i n t e r m s of t h e i n e r t i a p r o p e r t i e s w r i t t e n
w i t h r e s p e c t t o t h e (x, y, z) a x e s . T h e d e f i n i t i o n of is

(7.16)

Since ( e a c h is t h e s q u a r e of t h e l e n g t h of
2
r , we may
PQ add a n d s u b t r a c t x' to p r o d u c e this quantity in Equa­
tion (7.16):

(7.17)

S u b s t i t u t i n g x' f r o m E q u a t i o n ( 7 . 1 5 ) i n t o ( 7 . 1 7 ) g i v e s

E x p a n d i n g t h e trinomial a n d r e a r r a n g i n g , w e get
(7.18)
Since the a r e t h e d i r e c t i o n c o s i n e s of t h e v e c t o r i n t h e d i r e c t i o n of t h e
x' a x i s , w e k n o w t h a t

U s i n g t h i s r e l a t i o n i n t h e first t h r e e t e r m s of t h e i n t e g r a n d i n E q u a ­
tion (7.18) gives

Rearranging, we have

(7.19)

R e c o g n i z i n g t h e six i n t e g r a l s i n E q u a t i o n ( 7 . 1 9 ) a s t h e i n e r t i a p r o p e r t i e s
a s s o c i a t e d w i t h t h e (x, y, z) d i r e c t i o n s a t P, w e a r r i v e a t o u r g o a l :

(7.20)

T h i s f o r m u l a a l l o w s u s t o c o m p u t e t h e m o m e n t of i n e r t i a of t h e m a s s of
a b o u t a n y l i n e t h r o u g h P if w e k n o w t h e p r o p e r t i e s a t P f o r a n y s e t of
o r t h o g o n a l a x e s . W e n o w i l l u s t r a t e its u s e w i t h a n e x a m p l e .

EXAMPLE 7 . 2

Compute the moment of inertia about the diagonal BA of the rectangular solid of
Example 7.1.
Solution
We define the axis x' to emanate from B, pointing toward A as in Figure E7.2. The
inertia properties at B were computed in the prior example. The direction cosines
Page 4 5 1

of x' are seen by inspection to be

in which

Substituting the and the inertia properties at B into Equation (7.20) gives

Figure E7.2

I n t h e p r e c e d i n g e x a m p l e w e o b s e r v e t h a t t h e l i n e BA a l s o p a s s e s
t h r o u g h t h e m a s s c e n t e r C. T h e m o m e n t of i n e r t i a a b o u t l i n e BA is of
course the s a m e n o matter w h i c h point o n the line o n e uses to m a k e the
c a l c u l a t i o n . (It w o u l d a c t u a l l y b e e a s i e r t o c o m p u t e a t C i n t h i s c a s e ,
because the a r e t h e s a m e w h i l e t h e p r o d u c t s of i n e r t i a v a n i s h ! )
A r e s u l t s i m i l a r t o E q u a t i o n ( 7 . 2 0 ) for p r o d u c t s of i n e r t i a w i l l n o w b e
d e r i v e d . L e t n , n , a n d n b e t h e d i r e c t i o n c o s i n e s of a x i s y ' . N o t e t h a t t h e
x y z

r e c t a n g u l a r c o o r d i n a t e y' m a y t h e n b e w r i t t e n , i n t h e s a m e w a y a s E q u a ­
tion (7.15), as follows:

(7.21)

By d e f i n i t i o n ,

T h e r e f o r e , e x p a n d i n g a n d r e c o g n i z i n g t h e p r o d u c t of i n e r t i a i n t e g r a l s ,

(7.22)

Since a n d ( n , n , n ) a r e c o m p o n e n t s of u n i t v e c t o r s a l o n g t h e
x y z

m u t u a l l y p e r p e n d i c u l a r a x e s x' a n d y', w e m a y d o t t h e s e v e c t o r s t o g e t h e r
and obtain

T h e i n t e g r a n d in E q u a t i o n (7.22) m a y therefore b e w r i t t e n as

(7.23)
Page 452

Substituting E q u a t i o n (7.23) into (7.22) t h e n yields t h e desired transfor­


m a t i o n e q u a t i o n f o r t h e p r o d u c t s of i n e r t i a :

(7.24)

EXAMPLE 7 . 3

In Examples 7.1 a n d 7.2 let t h e solid b e a cube (a = b = d) a n d let y' b e defined as


follows (see Figure E7.3):

1. y' is perpendicular to x'.


2. y' is the s a m e p l a n e as z a n d x'.

Find

Solution
From t h e equations in Example 7.2 w e h a v e
for t h e unit vector along w e n o w force t h e c o m p o n e n t s of — that
is, (n , n , n ) — to b e s u c h t h a t conditions 1 a n d 2 are satisfied:
x y z

Figure E7.3

(a vector perpendicular to the plane of x'and y')

These t w o equations give n = n a n d n = 2n . W e m u s t also ensure t h a t


y x z x is
a unit vector:

Substituting t h e c o m p o n e n t s of a n d t h e inertia properties at B (letting the


general point P in (7.24) b e B in this problem) into Equation (7.24) gives

W e n o t e t h a t i n E x a m p l e 7.3 t h e z e r o r e s u l t is n o t o b v i o u s a t t h i s
p o i n t i n o u r s t u d y . W h i l e it is t r u e t h a t , f o r t h i s c a s e of a = b = d, t h e x ' y '
Page 4 5 3

p l a n e is a p l a n e of s y m m e t r y , t h i s g u a r a n t e e s ( s e e S e c t i o n 4 . 4 ) t h a t
and are zero but not necessarily N o t e also that there are t w o
d i r e c t i o n s ( 1 8 0 ° a p a r t ) for y' t h a t b o t h satisfy c o n d i t i o n s 1 a n d 2 i n t h e
preceding example.

Question 7.2 W h e r e in the solution did w e choose o n e of these direc­


tions? (And does it matter?)

W e close this section b y n o t i n g t h a t E q u a t i o n s (7.20) a n d (7.24) are


t h e t r a n s f o r m a t i o n e q u a t i o n s satisfied b y a s y m m e t r i c s e c o n d - o r d e r t e n ­
sor; t h u s t h e inertia p r o p e r t i e s d o i n d e e d f o r m s u c h a tensor. W e also n o t e
f r o m t h e s e t w o e q u a t i o n s t h a t o n l y if t h e p r o d u c t s of i n e r t i a a r e d e f i n e d
w i t h t h e m i n u s s i g n (see E q u a t i o n s 4 . 2 ) d o w e g e t t h e c o r r e c t t e n s o r
transformation equations.

Answer 7.2 When we said that that is, took the positive square root, we
chose y' to be in the direction making an acute angle with x ; had we chosen B

we would have gotten the opposite direction for y'. And had I been nonzero, the signx'y'

of the answer would have been opposite also.

PROBLEMS • Section 7.3

7.7 T h e three h o m o g e n e o u s rods in Figure P7.7 are 7.9 In t h e preceding problem, find t h e m o m e n t of iner­
w e l d e d together at O to form a rigid b o d y . Find the m a s s tia about the line OP.
m o m e n t s a n d products of inertia at point Q w i t h respect 7.10 C o m p u t e t h e m o m e n t of inertia w i t h respect to line
to axes there that are parallel to x, y, a n d z.
AB for the bent bar in Figure P7.10. The bar lies in a plane
a n d h a s mass 4 m .

Figure P7.10

Figure P7.7

Figure P7.8

7.8 Find the mass m o m e n t s a n d products of inertia of


Figure P7.11
the b o d y in Figure P 7 . 8 , w i t h respect to a set of axes

t h r o u g h the point P at , respectively, parallel to


7.11 Find the product of inertia for the h o o p of
x, y, a n d z. Each of the t w o perpendicular r o d s of the " T " mass m a n d radius R in Figure P 7 . l l . The p l a n e of is
h a s m a s s m a n d length misaligned w i t h the xy p l a n e by angle .
Page 454

7.12 C o m p u t e t h e m o m e n t of inertia a b o u t line BA in 7.16 Find t h e inertia properties at O for t h e b o d y s h o w n


Example 7.2 b y using Equation (7.20) a t C instead of B. in Figure P7.16, w h i c h is c o m p o s e d of a rod a n d ring t h a t
S h o w that if a = b = d, t h e a n s w e r becomes equal to h a v e equal cross sections a n d densities. T h e r o d is per­
(which equals in this case). pendicular to t h e p l a n e of t h e ring.
7.13 T h e centroidal m o m e n t s of inertia for t h e solid el­
lipsoid in Figure P 7 . 1 3 are

Figure P7.16
Further, t h e mass of is pabc, w h e r e is the mass
density. Find t h e m o m e n t of inertia of t h e m a s s of about
the line m a k i n g equal angles w i t h x, y, a n d z.
7.14 S h o w that t h e s u m of a n y t w o of and
always exceeds t h e third.
7.15 Part of a special-purpose, dual-driven a n t e n n a sys­
t e m consists of an octagonal rotator as s h o w n in Fig­ Figure P7.18
ure P 7 . 1 5 . Each of t h e eight equal sections is a square steel
tube with t h e indicated dimensions a n d thickness in.
Find t h e m o m e n t of inertia of t h e rotator a b o u t t h e axis of * 7.17 Find for t h e p r o b l e m of Example 7.3 if b = d
rotation. (Consider each section to h a v e squared-off e n d s = a/2. T h e axes h a v e their origin at B just as in t h e exam­
at the average 18-in. length a n d ignore t h e small overlaps. ple.
3
Use a density of 15.2 s l u g / f t . ) * 7.18 For t h e rigid b o d y in Figure P7.18 let t h e inertia
properties at P b e k n o w n . Further, let there be
m e a s u r e d along t h e s a m e axes t h e quantities

Figure P7.13

where and are as defined in Section 7.3. S h o w


that Equation (7.20) t h e n implies

This is t h e equation of an ellipsoid centered at P. Devel­


oped b y C a u c h y in 1827, it is called t h e ellipsoid of inertia.
S h o w that t h e m o m e n t of inertia a b o u t a n y line x'
t h r o u g h P equals t h e reciprocal of t h e square of t h e dis­
tance from P to t h e point w h e r e X' intersects t h e ellipsoid.
4 in.
18 in.
Figure P7.15 * Asterisks identify the more difficult problems.
Page 455

7.19 I n t h e preceding problem, if t h e p r o d u c t s of inertia Round plates


vanish, t h e equation of t h e ellipsoid of inertia written in (mass m each)
terms of t h e resulting m o m e n t s of inertia is

2 2 2
S h o w that n o t all ellipsoids of t h e form AX + BY + cz
= 1 can b e ellipsoids of inertia. HINT: T h e s u m of a n y t w o
m o m e n t s of inertia m u s t always exceed t h e third, as
stated in Problem 7.14.
**7.20 Calculate t h e inertia properties at O of t h e three
circular fan blades connected b y light rods in Fig­
Figure P7.20
ure P7.20. T h e blades are tilted 30° w i t h respect to t h e
axes OC OC , a n d O C ; t h e s h a d e d halves of each are
v 2 3

b e h i n d t h e plane of the d r a w i n g a n d t h e u n s h a d e d halves


are in front of it.

7.4 Principal Axes and Principal Moments of Inertia


I n t h i s s e c t i o n w e d e m o n s t r a t e a p a r t i c u l a r l y u s e f u l w a y of d e s c r i b i n g t h e
i n e r t i a c h a r a c t e r i s t i c s of a r i g i d b o d y It h a p p e n s t h a t , a t a n y p o i n t P of
it is a l w a y s p o s s i b l e t o f i n d a s e t of r e c t a n g u l a r a x e s s o t h a t t h e p r o d u c t s of
inertia at P w i t h respect t o t h e s e axes all v a n i s h . T h e s e axes a r e called t h e
principal axes of inertia a t P , a n d t h e m o m e n t s of i n e r t i a w i t h r e s p e c t t o
t h e m a r e k n o w n a s t h e principal m o m e n t s of inertia f o r t h e p o i n t .
S p e c i f i c a l l y , a n axis x is a p r i n c i p a l a x i s a t P if w h e r e is a n y
axis t h r o u g h P t h a t is p e r p e n d i c u l a r t o x. ( S e e F i g u r e 7.3.) W e c a n s h o w
t h a t if , i n w h i c h (x, y, z) f o r m a t r i a d of r e c t a n g u l a r a x e s a t P ,
t h e n x is a p r i n c i p a l axis a t P . F o r t h e p r o o f , w e s h a l l s h o w t h a t
implies where is t h e a r b i t r a r y a x i s t h r o u g h P n o r m a l t o x.
U s i n g E q u a t i o n (7.24) w e m a y write

Figure 7.3
In this equation a r e t h e d i r e c t i o n c o s i n e s of x ( t h e first
subscript in ) w i t h r e s p e c t t o x, y, a n d z, r e s p e c t i v e l y , w h i l e (n , n , n )
x y z

a r e t h e d i r e c t i o n c o s i n e s of (the second subscript in ), a l s o w i t h


r e s p e c t t o x, y, a n d z. S i n c e w e h a v e (n , n , n ) = ( 0 , n , n ).
x y z y z

Substituting these a n d n's, w e get

But so that a n d w e s e e t h a t all it t a k e s t o m a k e a n


a x i s , s u c h a s x, a p r i n c i p a l a x i s a t a p o i n t is t o s h o w t h a t and
w h e r e (x, y, z) f o r m a n o r t h o g o n a l t r i a d a t P . W e s h a l l n e e d t h i s
r e s u l t i n w h a t is t o f o l l o w .
Page 4 5 6

Principal Axes at C

W e n o w p r o c e e d t o a c o m p u t a t i o n a l p r o c e d u r e for f i n d i n g t h e p r i n c i p a l
a x e s a n d m o m e n t s of i n e r t i a . W e s h a l l d o t h e d e r i v a t i o n w h e n P is C a n d
t h e n l a t e r e x p l a i n h o w it a p p l i e s e q u a l l y w e l l t o all p o i n t s of
F o r t h e m o m e n t l e t (x, y, z) b e c e n t e r e d a t C a n d a t s o m e i n s t a n t l e t x
be parallel to t h e angular velocity of in a reference frame
Then a n d E q u a t i o n ( 7 . 1 1 ) g i v e s , f o r t h e a n g u l a r m o m e n t u m of
a b o u t C, t h e s i m p l i f i e d e x p r e s s i o n

(7.25)

W e n o t e f r o m E q u a t i o n ( 7 . 2 5 ) t h a t H is p a r a l l e l t o
c if a n d o n l y if
— t h a t i s , if x is a p r i n c i p a l axis a t C. T h u s w e s e e , a s d i d
L e o n h a r d E u l e r h i m s e l f i n t h e m i d d l e of t h e e i g h t e e n t h c e n t u r y , t h a t t h e
p r i n c i p a l a x e s h a v e t h e p r o p e r t y t h a t w h e n t h e a n g u l a r velocity lies a l o n g
o n e of t h e m , s o w i l l t h e a n g u l a r m o m e n t u m . E u l e r w a s s e e k i n g a n axis
t h r o u g h C f o r w h i c h , w h e n w a s s e t s p i n n i n g a b o u t it, t h e m o t i o n w o u l d
c o n t i n u e a b o u t this axis w i t h o u t a n y n e e d for external m o m e n t s to m a i n ­
t a i n it. N o t e f u r t h e r t h a t w h e n C­­H is p a r a l l e l t o "), the propor­
t i o n a l i t y c o n s t a n t is n e c e s s a r i l y t h e m o m e n t of i n e r t i a I a b o u t t h e i r c o m ­
m o n axis.
N o w w e a r e r e a d y for t h e big s t e p . W e let be a
u n i t v e c t o r a n d w e s e e k t h e d i r e c t i o n of fi t h a t w i l l e n s u r e its b e i n g a
p r i n c i p a l a x i s . I n o t h e r w o r d s , w e w a n t t o f i n d t h e v a l u e s of t h e d i r e c t i o n
c o s i n e s of x(n , n , n ) s u c h t h a t if
y z then Writing in
c o m p o n e n t form gives

(7.26)

S u b s t i t u t i n g f r o m E q u a t i o n ( 7 . 1 1 ) for H , t h e v e c t o r r e l a t i o n
c

t h e n gives t h e following t h r e e scalar c o m p o n e n t equations:

(7.27)

If w e d i v i d e all t h r e e of E q u a t i o n s ( 7 . 2 7 ) b y a n d note that since


we have and then we may
rearrange t h e equations as follows:

(7.28)

W e n o w h a v e a s e t of t h r e e e q u a t i o n s t h a t a r e a l g e b r a i c , l i n e a r , a n d
h o m o g e n e o u s i n t h e t h r e e v a r i a b l e s n , n , n . S u c h a s y s t e m is k n o w n t o
x y z

h a v e a n o n t r i v i a l s o l u t i o n if a n d o n l y if t h e d e t e r m i n a n t of t h e coeffi-
PAGE 4 5 7

c i e n t s o f t h e v a r i a b l e s is z e r o . * I n t h i s c a s e , w e m a y d r o p t h e " n o n t r i v i a l "
a d j e c t i v e b e c a u s e t h e t r i v i a l s o l u t i o n ( n = n = n = 0) fails t o s a t i s f y t h e
x y z

side condition
(7.29)

w h i c h m u s t a l w a y s b e t r u e f o r t h e d i r e c t i o n c o s i n e s of a v e c t o r .

Calculation of Principal Moments of Inertia

S e t t i n g t h e d e t e r m i n a n t of t h e coefficients i n ( 7 . 2 8 ) e q u a l t o z e r o w i l l l e a d
first t o t h e s p e c i a l v a l u e s of J for w h i c h t h e t h r e e e q u a t i o n s h a v e a
s o l u t i o n . E a c h s p e c i a l v a l u e J is c a l l e d a n eigenvalue, or characteristic
value, a n d w i l l b e a p r i n c i p a l m o m e n t of i n e r t i a ; t h e c o r r e s p o n d i n g
( w i t h c o m p o n e n t s n , n , n ) is c a l l e d t h e eigenvector a s s o c i a t e d w i t h t h i s
x y z

e i g e n v a l u e I. T h e u n i t v e c t o r p o i n t s i n t h e d i r e c t i o n of a p r i n c i p a l axis of
i n e r t i a a t C. T h e d e t e r m i n a n t is e q u a t e d t o z e r o b e l o w :

(7.30)

If w e e x p a n d t h i s c h a r a c t e r i s t i c d e t e r m i n a n t , w e c l e a r l y g e t a c u b i c p o l y ­
n o m i a l i n I:
(7.31)

T h e a a r e o f c o u r s e f u n c t i o n s of t h e i n e r t i a p r o p e r t i e s . N o w w e k n o w
i

f r o m a l g e b r a t h a t if p o l y n o m i a l s w i t h r e a l coefficients h a v e a n y c o m p l e x
roots, they m u s t occur in conjugate pairs. T h u s the polynomial derived
a b o v e h a s a t t h i s p o i n t a t l e a s t o n e r e a l r o o t I (It is p o s i t i v e b y t h e
1

d e f i n i t i o n of t h e q u a n t i t y " m o m e n t of i n e r t i a " t h a t it r e p r e s e n t s . ) W e
n o w a r e g u a r a n t e e d a t l e a s t o n e p r i n c i p a l m o m e n t of i n e r t i a a n d c o r r e ­
s p o n d i n g p r i n c i p a l a x i s of i n e r t i a .
In order to s h o w that there are t w o others, w e next reorient our
o r t h o g o n a l t r i a d of r e f e r e n c e a x e s s o t h a t o n e of t h e m (x) c o i n c i d e s w i t h
t h e a l r e a d y identified p r i n c i p a l axis; this t h e n a l l o w s u s to write
and w h e r e y a n d z a r e n o w a n e w p a i r of axes n o r m a l to
o u r n e w (principal) x axis. E q u a t i o n s (7.28) n o w a p p e a r as

(7.32)

and the determinantal equation becomes:

(7.33)

* Cramer's rule clearly gives n = n = n = 0 as the only solution if the equations are
x y z

independent, in which case the determinant D of the coefficients is not zero. If D = 0,


then Cramer's rule yields the indeterminate form 0 / 0 for the n's and there is a chance
for other solutions, as the equations are then dependent.
Page 458

This time t h e resulting cubic b e c o m e s factorable. E x p a n d i n g t h e determi­


nant, w e have

(7.34)

T h e p r i n c i p a l m o m e n t s of i n e r t i a a t C a r e t h e r o o t s of E q u a t i o n ( 7 . 3 4 ) .
T h e first r o o t ( t h e o n e w e a l r e a d y k n e w ) is r e a f f i r m e d b y s e t t i n g t h e first
factor to zero:

T h e o t h e r s will b e s e e n to c o m e from e q u a t i n g t h e s e c o n d factor to zero:

(7.35)

T h i s is, of c o u r s e , a q u a d r a t i c e q u a t i o n i n I. R e c a l l i n g t h a t t h e t w o r o o t s t o

are

w e s e e t h a t w e s h a l l h a v e t w o ( m o r e ) r e a l r o o t s I a n d I if t h e d i s c r i m i ­
2 3

n a n t is p o s i t i v e o r z e r o :

(7.36)

T h e r e f o r e all t h r e e r o o t s of t h e c h a r a c t e r i s t i c c u b i c e q u a t i o n a r e r e a l ( a n d
p o s i t i v e ) , a n d s o w e a l w a y s h a v e t h r e e p r i n c i p a l m o m e n t s of i n e r t i a a t C,
e a c h w i t h its o w n c o r r e s p o n d i n g p r i n c i p a l axis.*

Calculation of Principal Directions

W e m e n t i o n a t t h i s p o i n t t h e p r o c e d u r e for o b t a i n i n g t h e p r i n c i p a l d i r e c ­
tion, g i v e n b y n ,n , x y a n d n , for each of t h e p r i n c i p a l m o m e n t s of i n e r t i a
z

(I 1 , o r 7 ) . E q u a t i o n s ( 7 . 2 8 ) , b e i n g d e p e n d e n t , m a y n o t b e s o l v e d for t h e
v 2 3

t h r e e c o m p o n e n t s of e a c h in themselves; however, together with the


identity a s o l u t i o n m a y b e f o u n d . T h e i d e a is t o s o l v e
for, s a y , n a n d n i n t e r m s of n f r o m t w o of E q u a t i o n s ( 7 . 2 8 ) ; t h e n w e
y 2 x

s u b s t i t u t e i n t o E q u a t i o n ( 7 . 2 9 ) a n d s o l v e for n . E i t h e r s i g n m a y b e u s e d i n
x

t a k i n g t h e final s q u a r e r o o t , b e c a u s e t h e r e a r e o b v i o u s l y t w o l e g i t i m a t e
s e t s of d i r e c t i o n c o s i n e s . T h e s e t w o s e t s a r e n e g a t i v e s of e a c h o t h e r , a n d
e a c h yields t h e correct p r i n c i p a l axis. In Figure 7.4, e i t h e r or — defines
a p r i n c i p a l a x i s t h r o u g h C. T h e p r i n c i p a l axis is a n u n d i r e c t e d l i n e .
I n t h e first of t w o e x a m p l e s t o f o l l o w , w e s h a l l a g a i n s e e ( a s i n t h e
p r e c e d i n g d i s c u s s i o n ) t h a t if a t l e a s t t w o of t h e p r o d u c t s of i n e r t i a v a n i s h ,

* This was proved in 1755 for the first time by Segner, a contemporary of Euler. Segner
also showed that the principal axes (for distinct principal moments of inertia) are ortho­
Figure 7.4 gonal.
Page 4 5 9

t h e n t h e cubic e q u a t i o n (7.31) b e c o m e s factorable. In this case, w e d o n o t


h a v e t o s o l v e it n u m e r i c a l l y .

EXAMPLE 7 . 4

The inertia properties of a right triangular plate (see Figure E7.4a) are

Figure E7.4a
Find t h e principal m o m e n t s of inertia of the plate. T h e n find their associated
principal axes w h e n B = H.
Solution
Equations (7.28), w h i c h lead to t h e principal m o m e n t s a n d axes of inertia, b e ­
come, for t h e plate,

(1)

(2)

(3)

2
The algebra is simplified by dividing by m H / 3 6 a n d defining

(4)

This gives, in terms of t h e n o n d i m e n s i o n a l parameters B a n d I*,

(5)
(6)
(7)
Therefore t h e determinantal equation becomes

(8)

Expanding across t h e third r o w (or d o w n t h e third column), w e get

(9)
Therefore o n e of the brackets m u s t v a n i s h a n d the roots come from

(10)
Page 460

and

(11)
Equation (10) gives, using Equations (4),

(12)

Equation (11) gives, b y t h e quadratic formula,

(13)

Thus

(14)

The t h r e e principal m o m e n t s of inertia of t h e plate are given b y Equations (12)


a n d (14). In t h e case w h e n t h e right triangle is isosceles (b = H), w e h a v e B = 1 so
that, from Equation (13),

or

and (15)

Also, from Equation (10),

C h a n g i n g back to dimensional inertias b y Equation (4), w e see t h a t for a right


isosceles triangular plate,

(16)

We shall n o w d e t e r m i n e t h e principal axis associated w i t h each of these principal


m o m e n t s of inertia. W e first substitute (with B = 1) in each of Equations
(5) to (7) a n d get

- n - n = 0
x y

-n z - n y = 0

n = 0
z (17)

T h e third of these equations says that t h e principal axis for is in t h e p l a n e of the


plate (xy); t h e other t w o equations b o t h give

n= x -n y (18)

Substituting this result into

(19)

gives

(20)
461

(21)

Thus, b y Equation (18),

(22)

so t h a t either

(23)

or

(24)

T h e lines defined b y these t w o sets of direction cosines are s h o w n in Fig­


u r e E7.4b a n d Figure E7.4c. It is seen that t h e two preceding results represent the
same line; t h e positive directions are opposite b u t u n i m p o r t a n t . T h e inertia value,
2
being t h e integral of r dm, is i n d e p e n d e n t of t h e directivity of t h e line.

Principal axis
for I,

(a) Line of Eq. (7.12) (b) Line of Eq. (7.13)


Figure E7.4h Figure E7.4c

For Equations (5) to (7) b e c o m e

n -n
x y = 0

—n + n = 0
x y

n = 0
z (25)

This time n = n w i t h n again equaling zero, so this principal axis m a k e s equal


x y 2

angles w i t h x a n d y . (See Figure E7.4d a n d Figure E7.4e.) N o t e from t h e p r e -


c c

Principal axis
for I 2

Figure E7.4d Figure E7.4e


Page 462

ceding diagrams t h a t t h e r e is m u c h m o r e inertia a b o u t the principal axis for J,


t h a n there is for I ; t h e m a s s is m o r e closely clustered a b o u t t h a n it is a b o u t
2

The third principal axis is found from Equations (5) to (7) w h e n , for B = 1,

(26)

These equations are

-2n z - n = 0
y

-nx - 2n = 0
y

[2(2) - 4]n = 0 z (27)

The first t w o of these equations h a v e the solution n = n = 0. The third leaves n


x y z

indeterminate. But from Equation (19) w e h a v e n = 1 or -1. T h u s t h e principal


z

axis for I is t h e line n o r m a l to the plate at C. This will in fact always b e true: W h e n
3

a b o d y is a plate (that is, flat w i t h negligible thickness c o m p a r e d to its other


dimensions), t h e m o m e n t of inertia a b o u t t h e axis n o r m a l to the plate at a n y point
is principal for that point. It is e v e n true t h a t it is the s u m of the other t w o a n d
therefore the largest.

Principal Axes at Any Point

W e n o w w i s h t o r e m a r k t h a t a s e t of p r i n c i p a l a x e s e x i s t s a t every p o i n t of
n o t j u s t a t t h e m a s s c e n t e r C. T o s h o w t h i s w e first r e c a l l E q u a t i o n (7.7),
w h i c h w i t h E q u a t i o n (7.12) g i v e s t h e a n g u l a r m o m e n t u m H a b o u t a n y
p

p o i n t P of b o d y

Whenever the cross-product term in H vanishes, the terms that remain P

a r e i d e n t i c a l t o t h o s e of E q u a t i o n (7.11) if P r e p l a c e s C. T h e r e f o r e , for
cases in w h i c h r X PC = 0, w e n e e d o n l y r e c a l l o u r a r g u m e n t s m a d e for
C a n d w e shall b e led, t h r o u g h a n identical procedure, to t h e principal
a x e s a n d m o m e n t s of i n e r t i a f o r any p o i n t P of
T h e r e f o r e w e o n l y n e e d t o i m a g i n e t h a t a t s o m e i n s t a n t o u r p o i n t P of
interest h a s either v = 0 or P In either case, r X v = 0 a n d H
PC P P

t h e n h a s t h e f o r m of E q u a t i o n (7.12). R e t r a c i n g o u r s t e p s f r o m t h a t p o i n t ,
w e a r r i v e a t t h e t h r e e a x e s ( t h r o u g h P t h i s time) f o r w h i c h a l l t h r e e
p r o d u c t s of i n e r t i a a r e z e r o ; t h e s e a r e t h e s a m e t h r e e a x e s t h r o u g h P for
which whenever is a l i g n e d w i t h o n e of t h e m a n d v e i t h e r
P

v a n i s h e s o r is p a r a l l e l t o r . O f c o u r s e , all r e f e r e n c e s t o t h e m o t i o n
PC

c o n d i t i o n s t h a t l e d t o t h e d e t e r m i n a n t a l e q u a t i o n a r e a g a i n l o s t (as t h e y
w e r e f o r C ) , s o t h a t t h e p r i n c i p a l m o m e n t s a n d a x e s of i n e r t i a d e p e n d
only o n the body's m a s s distribution.
Page 4 6 3

EXAMPLE 7 . 5

Find t h e principal m o m e n t s of inertia at O a n d the directions of their associated


principal axes for the b o d y s h o w n in Figure E7.5a. It is m a d e u p of three rigid,
identical slender rods w e l d e d together at right angles to form a single rigid b o d y
Solution
Using t h e m o m e n t s of inertia s h o w n for o n e r o d in Figure E7.5b, plus t h e paral­
lel-axis (transfer) theorem, the six inertia properties are calculated below. The
reader s h o u l d verify each of t h e entries.

Figure E7.5a

I is small
Figure E7.5b
Page 4 6 4

For this problem, then, Equations (7.28) m a y b e expressed as

(1)

in w h i c h w e h a v e multiplied t h e three equations b y w e m a y replace


b y a n d write t h e d e t e r m i n a n t as

Expanding gives t h e characteristic cubic equation:

If a c o m p u t e r or p r o g r a m m a b l e calculator is n o t available,* w e can a l w a y s solve a


cubic b y trial a n d error r a t h e r quickly. N o t i n g t h a t is 1342 at a n d is
negative at for example, o n a calculator w e m a y proceed a n d within a few
m i n u t e s obtain t h e root b e t w e e n these v a l u e s . †T h e p r o c e d u r e is as follows:

3
2
2 6 3.10 (still positive)
2.61 -0.93 (so it is > 2 6 and < 2 6 1 , closer to the latter)
2 608 -0.123 (back up slightly!)
2 607 0 2802 (so it is about two-thirds of the way from 2.607 to 2 608)
2.6076 0.03835 (so just a little farther)
2 6077 - 0 00195 (the root is close to this number!)
2 60769 0 00208 (should be halfway between the last one and this
one . . .)
2 607695 0 00006 (now double-check)
2 607696 - 0 00034

Next w e use synthetic division to obtain t h e r e d u c e d quadratic:

* See Appendix B for a numerical solution to this problem using the Newton-Raphson
method.
†We abandon our three-digit consistency in numerical analyses like this one in order to
illustrate the speed of convergence.
Page 4 6 5

Using t h e quadratic formula, w e obtain

The value of strongly h i n t s that 22 m i g h t b e a rational root. Synthetic division


s h o w s t h a t it is, a n d refined values from t h e r e d u c e d quadratic are t h e n

Since our dimensional principal m o m e n t s of inertia are, to six signif­


icant figures,

We next illustrate t h e c o m p u t a t i o n of t h e direction cosines, w h i c h locate for


us t h e principal axes of inertia. We find t h e m from Equations (1), for w h i c h
and are t h e only special values (eigenvalues) of for w h i c h these equations
h a v e a solution. First w e seek t h e principal axis associated w i t h
The first of Equations (1) becomes

7.392305n - 9n + 3n = 0
x y z

Solving for n in terms of n a n d n a n d substituting t h e result into t h e second of


z x y

Equations (1) gives

n = 0.732051n
y x

Thus

n = -0.267950n
z x

Substituting these expressions for n a n d n into


y z yields

T h u s t h e angles t h a t t h e principal axis of m i n i m u m m o m e n t of inertia m a k e s


w i t h x, y, a n d z are, respectively, 3 7 . 9 4 ° , 54.74°, a n d 102.20°. This axis h a s
to be t h e o n e to w h i c h , loosely speaking, t h e m a s s finds itself closest. Examina­
tion of Figure E7.5c at t h e left, together w i t h these angles, s h o w s that this m a k e s
sense.
Next w e m a y follow t h e s a m e p r o c e d u r e for t h e principal axis of m a x i m u m
m o m e n t of inertia . T h e results are, as the reader
m a y verify,

Figure E7.5c
Page 466

A n d for t h e intermediate m o m e n t of inertia the


principal axis is defined by

Orthogonality of Principal Axes


N o t e in the preceding example that
a n d that * These are very good checks o n the solution
s i n c e t h e p r i n c i p a l a x e s , w h e n t h e p r i n c i p a l m o m e n t s of i n e r t i a a r e
distinct are orthogonal. To p r o v e this in general,
let a n d l e t x lie a l o n g t h e a x i s of I T h e n f r o m t h e first of
1

E q u a t i o n s ( 7 . 3 2 ) , w h i c h is

(I -I)n
1 x = 0

w e s e e t h a t if I i s e i t h e r I o r I , t h e n n = 0 ; t h a t i s , t h e c o s i n e of t h e a n g l e
2 3 x

b e t w e e n x a n d t h e c o r r e s p o n d i n g principal axis h a s to b e zero. This


m e a n s t h a t x is p e r p e n d i c u l a r t o t h e o t h e r t w o p r i n c i p a l a x e s . I n t u r n ,
these t w o axes are n o r m a l to each other; this follows from reorienting the
a x e s o n c e m o r e s o t h a t x still lies a l o n g t h e axis of I b u t n o w y lies a l o n g
1

t h e axis of I . T h i s t i m e
2 is a l s o z e r o , s o t h e n e w s e c o n d e q u a t i o n
becomes

(I - I)n
2 y = 0

T h i s s h o w s t h a t for t h e v a l u e of n for t h e t h i r d p r i n c i p a l
y

a x i s v a n i s h e s a n d it is t h e n n o r m a l n o t o n l y t o x ( w h i c h it still i s , s i n c e w e
h a v e o n l y r o t a t e d it a b o u t x) b u t a l s o t o y. T h u s t h e t h r e e p r i n c i p a l a x e s
a r e o r t h o g o n a l if t h e p r i n c i p a l m o m e n t s of i n e r t i a a r e all d i f f e r e n t . W e
n o w t u r n o u r a t t e n t i o n t o w h a t h a p p e n s if t h e y a r e n o t .

Equal Moments of Inertia


A c o m m o n l y o c c u r r i n g c a s e is for t w o of t h e p r i n c i p a l m o m e n t s of i n e r t i a
at a p o i n t P to b e e q u a l to each o t h e r b u t different from t h e third. W h e n
t h i s h a p p e n s , w e c a n s h o w t h a t every l i n e t h r o u g h P i n t h e p l a n e o f t h e
t w o a x e s (call t h e m x a n d y) w i t h e q u a l m o m e n t s of i n e r t i a ( s a y I = I ) is
1 2

a p r i n c i p a l a x i s h a v i n g t h i s s a m e v a l u e for its m o m e n t of i n e r t i a .
T o d o t h i s , w e first s h o w t h a t if t h e n t h e axis a s s o c i a t e d
w i t h I (call it z) is p e r p e n d i c u l a r t o t h o s e (x, y) of I a n d I . T h e t h i r d
3 1 2

e q u a t i o n of ( 7 . 3 2 ) g i v e s

(I - I)n
3 z = 0

T h u s w h e n I is I o r I , t h e n n = 0. H e n c e t h e a n g l e b e t w e e n z a n d x ( a n d
1 2 z

* This could have been —equally correct!


Page 4 6 7

b e t w e e n z a n d y ) is a l s o 9 0 ° . It d o e s n o t f o l l o w i n l i k e m a n n e r f r o m t h e
equations, h o w e v e r , t h a t axes x a n d y are perpendicular. To h a n d l e this
c a s e , w e b e g i n b y s h o w i n g t h a t if t h e a x e s of t h e o t h e r t w o p r i n c i p a l
m o m e n t s of i n e r t i a (I = I ) a r e n o t p r e s u m e d t o b e o r t h o g o n a l ( s a y t h e y
1 2

p p
a r e a a n d x i n F i g u r e 7.5 w i t h I = I = I ), then all is w e l l b e c a u s e I is
qq 1 xx yy

also e q u a l t o I . 1

To p r o v e this, w e u s e E q u a t i o n (7.20):

x and z are principal!

Figure 7.5

T h u s w e see that

I =I
yy 1

N e x t w e let b e a u n i t v e c t o r i n t h e d i r e c t i o n of a n a r b i t r a r y a x i s x' i n
t h e p l a n e of t h e p e r p e n d i c u l a r a x e s x a n d y. T h e n u s i n g E q u a t i o n ( 7 . 2 0 )
again gives

A n d e q u a t i o n ( 7 . 2 4 ) y i e l d s ( w i t h y' p e r p e n d i c u l a r t o x' a n d l y i n g i n t h e
p l a n e of x', x, a n d y a s i n F i g u r e 7 . 6 :

P
N o t i n g t h a t z is p e r p e n d i c u l a r t o x' a n d t h a t I = 0 s i n c e z is p r i n c i p a l , w e
x'z

t h e n h a v e t h e r e s u l t t h a t x' is p r i n c i p a l w i t h t h e s a m e m o m e n t o f i n e r t i a a s
x a n d y, a n d t h i s i s w h a t w e w a n t e d t o p r o v e . N o t e t h e n , for a b o d y w i t h
a n a x i s of s y m m e t r y , f o r e x a m p l e , t h a t a n y a x i s w h i c h p a s s e s t h r o u g h a n d
is n o r m a l t o t h e s y m m e t r y axis of, is a l w a y s p r i n c i p a l ; t h i s is t r u e even if
the axis is not fixed in the body, w h i c h w i l l p r o v e u s e f u l t o u s l a t e r .
Figure 7.6
F i n a l l y , if all t h r e e p r i n c i p a l a x e s (x, y, z) t h r o u g h P h a v e t h e s a m e
c o r r e s p o n d i n g p r i n c i p a l m o m e n t of i n e r t i a I , t h e n every a x i s t h r o u g h P is
1

p r i n c i p a l w i t h p r i n c i p a l m o m e n t of i n e r t i a I . L e t b e t h e u n i t v e c t o r
1

a l o n g a n a r b i t r a r y a x i s x' t h r o u g h P i n t h i s c a s e o f t h r e e e q u a l p r i n c i p a l
m o m e n t s of i n e r t i a . A l s o l e t y' a n d z ' c o m p l e t e a n o r t h o g o n a l t r i a d w i t h
x', w i t h and b e i n g u n i t v e c t o r s i n t h e r e s p e c t i v e y' a n d z ' d i r e c t i o n s .
Thus , (m , m , m ), a n d (n , n , n ) a r e t h e r e s p e c t i v e s e t s of
x y z x y z

d i r e c t i o n c o s i n e s of x', y', a n d z ' w i t h r e s p e c t t o (x, y, z). E q u a t i o n ( 7 . 2 4 )


Page 468

t h e n gives

p
In the s a m e way, I x'z' = 0 since a s w e l l . T h u s t h e a r b i t r a r y a x i s x'
t h r o u g h P i s p r i n c i p a l , a n d E q u a t i o n ( 7 . 2 0 ) s h o w s t h a t its m o m e n t of
inertia is also I : 1

E x a m p l e s of t h e p r e c e d i n g r e s u l t s r e g a r d i n g t w o a n d t h r e e e q u a l p r i n c i ­
p a l m o m e n t s of i n e r t i a a r e g i v e n i n t h e t a b l e o n t h e n e x t p a g e .

Maximum and Minimum Moments of Inertia

A n i m p o r t a n t p r o p e r t y of p r i n c i p a l m o m e n t s of i n e r t i a is t h a t t h e l a r g e s t
a n d s m a l l e s t of t h e s e a r e t h e l a r g e s t a n d s m a l l e s t m o m e n t s of i n e r t i a
a s s o c i a t e d w i t h any a x i s t h r o u g h t h e p o i n t i n q u e s t i o n . T o s h o w t h a t t h i s
is t r u e , l e t b e t h e p r i n c i p a l m o m e n t s of i n e r t i a a t P a n d l e t t h e
c o r r e s p o n d i n g p r i n c i p a l a x e s b e x, y, a n d z. T h e m o m e n t of i n e r t i a a b o u t
s o m e o t h e r a x i s , x', is g i v e n b y E q u a t i o n ( 7 . 2 0 ) :

or

since

and

Thus s o t h a t no l i n e t h r o u g h P h a s a s m a l l e r m o m e n t of i n e r t i a
t h a n t h e s m a l l e s t principal m o m e n t of i n e r t i a . B y a s i m i l a r a r g u m e n t
which demonstrates that so that n o line t h r o u g h
P h a s a l a r g e r a s s o c i a t e d m o m e n t of i n e r t i a t h a n t h e l a r g e s t prin­
cipal m o m e n t of i n e r t i a . T h u s t h e l a r g e s t a n d s m a l l e s t m o m e n t s of i n e r t i a
a t a p o i n t P a r e f o u n d a m o n g t h e p r i n c i p a l m o m e n t s of i n e r t i a for P .
It m a y n o w b e s h o w n q u i t e e a s i l y t h a t t h e s m a l l e s t m o m e n t of i n e r t i a a t
t h e m a s s c e n t e r is t h e m i n i m u m I for any l i n e t h r o u g h any point of
or extended.
Page 4 6 9

Two Equal I's Three Equal I's


Solid
cylinder

Solid sphere:

in any direction through C.

All axes through C in the shaded plane have this same


C
inertia and are principal. Note that I = mR /2 and is
ZZ
2

generally not equal to / and l ; if, however,


xx yy then
every axis through C is principal with the same principal
m o m e n t of inertia!

Solid cube:

in any direction through C. Therefore if line is a diagonal o1


the cube, then lg = ms /6 even though it would be
2

formidable t o obtain this result bv intearation. Note that the


direction cosines of are
since it makes equal angles with x, y, and z. Thus Equa­
tion (7.20) gives

Question 7.3 Write a one-sentence proof of this statement by using


the preceding results together with the parallel-axis theorem.

Answer 7.3 The moment of inertia about any line through any point P other than C is
larger than the moment of inertia about the line through C parallel to by the transfer
2
term md , and thus the smallest I at C is the smallest of all.
Page 470

Problems • Section 7.4

7.21 Find the principal axes a n d associated principal 7.27 S h o w that if a line is a principal axis for t w o of its
m o m e n t s of inertia at O for t h e semicircular plate of mass points, t h e n it is a principal axis for t h e m a s s center.
M a n d radius R s h o w n in Figure P 7 . 2 1 . 7.21 S h o w that t h e three principal axes for a n y point
7.22 Find the principal axes a n d associated principal lying o n a principal axis for C are parallel to t h e principal
m o m e n t s of inertia for the p l a n a r wire s h o w n in Fig­ axes for C.
ure P7.22, at t h e m a s s center. 7.IS Find the principal m o m e n t s of inertia a n d their as­
7.23 Find the vector from O t o the m a s s center of t h e sociated axes at O for t h e thin plate (Figure P7.29) in
b e n t bar in Example 7.5. Observe t h a t it d o e s n o t lie along terms of its density a n d thickness t.
any of t h e three principal axes at O. P P P
• 7,31 Find t h e m o m e n t s of inertia I , I , a n d I for the
xx yy zz

7.24 Use the definitions of the m o m e n t s of inertia to b o d y depicted i n Example 4.16. T h e n find t h e principal
prove that if a b o d y lies essentially in t h e xy p l a n e (that is, m o m e n t s of inertia a n d corresponding principal axes at P.
it has very small dimensions normal t o it), t h e n • 7.31 In Problem 7.16 extend t h e p r o b l e m a n d find t h e
w h e r e P is any point in the plane, a n d t h a t z is a principal m o m e n t s of inertia, a n d their principal axes, at
principal axis at P . p o i n t O.
7.25 S h o w t h a t if an axis through the m a s s center C of • 7 . 3 2 Calculate t h e principal m o m e n t s of inertia at O, a n d
body is principal at C, t h e n it is a principal axis for every t h e direction cosines of their respective principal axes, for
point on that axis. Hint: Use t h e transfer theorem for b o d y in Figure P 7 . 3 2 . It is m a d e u p of three b e n t bars
products of inertia, together w i t h t h e orthogonality of w e l d e d together; all legs are either along, or parallel to,
C C
principal axes. (See Figure P7.25.) Transfer I a n d I to P !
YZ XZ
the coordinate axes.
7.26 S h o w that if a principal axis for a point (such as P in • 7.33 Find the principal m o m e n t s of inertia a n d related
the preceding problem) passes t h r o u g h C, t h e n it is also principal axes at t h e origin for t h e b o d y in Figure P 7 . 3 3 .
principal for C. (Same hint!)

Figure P7.21

Figure P7.29
Figure P7.32

Figure P7.22

Figure P7.25 Figure P7.33


Page 4 7 1

• 7.34 In Example 7.5 find t h e principal m o m e n t s a n d • 7.39 Four slender bars, each of m a s s m a n d length are
axes of inertia at t h e m a s s center C. w e l d e d together to form t h e b o d y s h o w n in Figure P7.39.
• 7.35 Find t h e principal m o m e n t s of inertia a n d their as­ Find: (a) t h e inertia properties at t h e m a s s center C; (b) t h e
sociated principal axes for t h e plate of Example 7.4 w h e n principal axes a n d principal m o m e n t s of inertia at C.
b = 2H. • 7.40 In Figure P7.40, t h e axis of symmetry, y , of t h e c

disk is parallel to y; t h e p l a n e of t h e disk is parallel to xz.


7.36 Find t h e principal m o m e n t s a n d axes of inertia if
the b o d y of Example 7.4 h a s d e p t h L instead of being a Find t h e principal m o m e n t s of inertia of D at t h e origin O,
thin plate. (See Figure P7.36.) Hint: T h e xy p l a n e is still a n d for t h e smallest one, d e t e r m i n e t h e angles t h a t its
C
one of symmetry, so I = I = 0 again! C associated principal axis forms w i t h x, y, a n d z.
xz yz

* 7.37 For the h o m o g e n e o u s rectangular solid s h o w n in •* 7.41 Figure P 7 . 4 1 s h o w s p a r t of a space station being
constructed in orbit. Find t h e principal axes a n d m o m e n t s
Figure P7.37, find t h e smallest of t h e three angles b e ­
of inertia at C . T h e m o d u l e s h a v e 33-ft diameters, b u t
t w e e n line AB a n d t h e principal axes of inertia at A. 3

d u e to t h e material within t h e y are n o t hollow. For t h e


* 7.38 At t h e origin, find t h e principal m o m e n t s of inertia p u r p o s e s of this problem, treat each as a uniform hollow
a n d associated principal axes for a b o d y consisting of shell w i t h a radius of gyration about its axis of 12 ft.
three square plates w e l d e d along their edges as s h o w n in
Figure P7.38. (The axis of t h e smallest value of I s h o u l d
be t h e o n e t h a t t h e m a s s lies closest to in an overall sense.
Make this r o u g h check o n your solution.) Mass = 3m,
side = a.

Figure P7.39

Figure P7.36

D (mass 100 kg)


Figure P7.40

Each of 3 modules; 16,000 lb length = 70 ft


;

Figure P7.37

Solar
panels:
2
6000 ft
and 8000 lb total
in x z plane
3 3

Figure P7.38 Figure P7.41


Page 4 7 2

7.5 The Moment Equation Governing Rotational Motion


The Euler Equations
In this section w e derive t h e t h r e e differential e q u a t i o n s g o v e r n i n g the
a n g u l a r m o t i o n of a r i g i d b o d y T h e i r s o l u t i o n , w h i c h is difficult t o
o b t a i n i n c l o s e d f o r m i n m o s t c a s e s , y i e l d s t h e t h r e e c o m p o n e n t s of t h e
a n g u l a r v e l o c i t y of in a n i n e r t i a l f r a m e We b e g i n w i t h E q u a t i o n (7.2),
n o t i n g t h a t t h e t i m e d e r i v a t i v e is t a k e n i n However, the angular m o ­
m e n t u m H was most conveniently expressed "in"* body
c in E q u a ­
tion (7.11). T h u s w e shall u s e E q u a t i o n (6.20) to m o v e t h e derivative in
(7.2) f r o m to :

(7.37)

W e n o w fix t h e a x e s (x, y, z) t o b o d y so that, relative to t h e inertia


p r o p e r t i e s a r e c o n s t a n t . U s i n g E q u a t i o n ( 7 . 1 1 ) , t h e first t e r m o n t h e right
s i d e of ( 7 . 3 7 ) is

(7.38)

w h e r e the unit vectors a n d a r e r e s p e c t i v e l y p a r a l l e l t o x, y, a n d z, a n d


t h e r e f o r e n o w a r e fixed i n d i r e c t i o n i n . T h e s e c o n d t e r m i n E q u a ­
tion ( 7 . 3 7 ) , a f t e r c o m p u t i n g t h e c r o s s p r o d u c t , is

(7.39)

T h e s u m of E q u a t i o n s ( 7 . 3 8 ) a n d ( 7 . 3 9 ) y i e l d s t h e r i g h t s i d e of E q u a ­
t i o n ( 7 . 3 7 ) , w h i c h i n t u r n e q u a l s t h e m o m e n t a b o u t t h e m a s s c e n t e r C of
all t h e e x t e r n a l f o r c e s a n d c o u p l e s a c t i n g o n
It is c l e a r t h a t t h i s e q u a t i o n is e x t r e m e l y l e n g t h y a n d c o m p l i c a t e d . If
w e s e l e c t t h e b o d y - f i x e d a x e s (x, y, z) t o b e t h e principal a x e s t h r o u g h C,
h o w e v e r , t h e n all p r o d u c t - o f - i n e r t i a t e r m s v a n i s h a n d w e o b t a i n

and

so that, substituting i n t o E q u a t i o n (7.37) a n d e q u a t i n g t h e respective

* "Expressing a vector in a frame" simply means the vector is expressed in terms of unit
vectors fixed in that frame.
Page 4 7 3

coefficients of and we obtain the Euler equations:

(7.40)

W e n o t e that the Euler equations are nonlinear in t h e components and


that the plane-motion equation does n o t extend simply to
general motion.
It is v e r y i m p o r t a n t t o r e a l i z e t h a t , if a b o d y h a s a p i v o t ( p e r m a n e n t l y
fixed p o i n t ) , e q u a t i o n s a n a l o g o u s t o t h e p r e c e d i n g p e r t a i n . A n d i n f a c t
t h e s e a r e m e r e l y w h a t is o b t a i n e d b y s u b s t i t u t i n g O ( t h e p i v o t ) f o r C i n all
e q u a t i o n s f r o m (7.37) t h r o u g h (7.40).
W e c a n u s e E q u a t i o n s (7.40) to m a k e a n i m p o r t a n t o b s e r v a t i o n a b o u t
t h e s p e c i a l c a s e of " t o r q u e - f r e e " m o t i o n , m e a n i n g . S u p p o s e at
an instant w h e r e x, y, a n d z a r e p r i n c i p a l a x e s .
Then, with E q u a t i o n s ( 7 . 4 0 ) tell u s t h a t
t h a t is, if t h e b o d y w e r e i n i t i a l l y t o b e s p u n a b o u t a p r i n c i p a l axis it w o u l d
c o n t i n u e t o s p i n a b o u t t h a t axis a n d a t c o n s t a n t r a t e . C o n v e r s e l y , if w e
s e e k c o n d i t i o n s for w h i c h w e find f r o m E q u a ­
t i o n s ( 7 . 4 0 ) t h a t t w o of and m u s t v a n i s h . T h u s t h e s p i n will
p e r s i s t i n t h e a b s e n c e of e x t e r n a l m o m e n t if a n d o n l y if t h e axis of i n i t i a l
s p i n is a p r i n c i p a l a x i s . T h i s i n v e s t i g a t i o n is w h a t l e d E u l e r t o d i s c o v e r t h e
principal-axis concept in 1750.

Question 7.4 C a n a similar conclusion b e d r a w n for spinning a b o u t


a n axis t h r o u g h a pivot?

Use of Non-Principal Axes

S o m e t i m e s o t h e r f o r m s of m o m e n t e q u a t i o n s a r e m o r e a d v a n t a g e o u s t o
a p p l y i n p a r t i c u l a r p r o b l e m s t h a n E q u a t i o n s ( 7 . 4 0 ) . Firstly, w e m a y f i n d
it c o n v e n i e n t t o u s e r e f e r e n c e a x e s t h a t a r e b o d y - f i x e d b u t n o t p r i n c i p a l .
O f t e n it is l e s s t r o u b l e t o s i m p l y d e a l w i t h n o n z e r o p r o d u c t s of i n e r t i a a n d
a x e s t h a t a r e c o n v e n i e n t t o t h e b o d y (or its a n g u l a r v e l o c i t y ) t h a n t o
c o m p u t e principal directions a n d associated inertia properties. The com­
p o n e n t equations are formed b y combining Equations (7.37)-(7.39).
W h e n s p e c i a l i z e d t o (x, y) p l a n e m o t i o n , w e r e c o v e r t h e f o l l o w i n g e q u a ­
tions d e v e l o p e d in C h a p t e r 4:

(7.41)

Answer 7.4 Yes, provided there is n o net m o m e n t about the pivot.


Page 4 7 4

Use of an Intermediate Frame

S e c o n d l y , w e o f t e n f i n d it c o n v e n i e n t t o e x p r e s s e x t e r n a l m o m e n t s
a n d / o r a n g u l a r m o m e n t u m i n t e r m s of c o m p o n e n t s a s s o c i a t e d w i t h
d i r e c t i o n s fixed n e i t h e r i n t h e b o d y n o r i n t h e i n e r t i a l f r a m e . T h a t is, w e
m a y choose to involve a n intermediate frame, say and use

(7.42)

or, w h e n t h e r e is a p i v o t O ,

(7.43)

T h i s a p p r o a c h is p a r t i c u l a r l y u s e f u l w h e n , u s u a l l y b e c a u s e of s y m m e ­
t r i e s , m o m e n t s a n d p r o d u c t s of i n e r t i a of t h e b o d y r e m a i n c o n s t a n t r e l a ­
t i v e t o a x e s fixed i n t h e i n t e r m e d i a t e f r a m e .
W e c l o s e t h i s s e c t i o n w i t h five e x a m p l e s , t h e first e m p l o y i n g E u l e r ' s
e q u a t i o n s t o s t u d y t o r q u e - f r e e m o t i o n w h e n t h e i n i t i a l s p i n is n o t a b o u t a
principal axis. I n t h e s e c o n d w e u s e body-fixed axes t h a t a r e n o t principal
i n a p r a c t i c a l p r o b l e m of a s a t e l l i t e d i s h a n t e n n a . T h e final t h r e e e x a m p l e s
i l l u s t r a t e t h e u s e of i n t e r m e d i a t e f r a m e s of r e f e r e n c e .

EXAMPLE 7 . 6

A satellite* is m o v i n g t h r o u g h d e e p space far from t h e influence of atmospheric


d r a g a n d gravity. (See Figure E7.6.) If t h e z axis is o n e of s y m m e t r y a n d if at some
instant called t = 0 w e h a v e along the body-fixed mass-center
axes, find . A s s u m e t h e satellite to b e a rigid b o d y .
Solution
C C C
The Euler equations, if I xx = I yy = I and I zz = J, are

(1)
Figure E7.6
(2)
(3)
in w h i c h the m o m e n t c o m p o n e n t s are zero in the absence of external forces a n d
couples. Equation (3) gives

(4)
so t h a t Equations (1) a n d (2) b e c o m e linear a n d are:

(5)

(6)
Differentiating Equation (6) a n d solving for w e get

(7)

* Such as the Voyager spacecraft, which left our solar system in 1983.
Page 4 7 5

This expression m a y b e substituted into (5) to yield a n equation free of :

or

in w h i c h . The solution to this equation is harmonic:

Since at t = 0, w e see t h a t Finally, Equation (6) gives

or

T h e initial condition for gives us

so that t h e other t w o c o m p o n e n t s (besides of are

EXAMPLE 7 . 7

For reasons of interference with other bodies, an a n t e n n a w a s recently designed


a n d built w i t h a n offset axis as s h o w n in Figure E7.7a a n d Figure E7.7b. The
a n t e n n a is c o m p o s e d of a 12-ft, 1200-lb parabolic reflector« a (ourrteroeteht W,
a reflector s u p p o r t structure S, a n d a positioner. The positioner consists of (1) a
pedestal P t h a t is fixed to the (inertial) reference frame; (2) a n azimuth bearing at

Counterweight (x, y, z) are


fixed in S

Figure E7.7a Back View Figure E7.7b Side View


Page 4 7 6

O a n d ring gear b y m e a n s of w h i c h t h e a r m A is m a d e to rotate about t h e vertical;


a n d (3) a n elevation torque m o t o r at E t h a t rotates t h e s u p p o r t structure S with
respect to t h e a r m A. Rotations of t h e reflector consist of an a z i m u t h angle
a b o u t Z, a n d an elevation angle a b o u t x.

a. Find t h e weight W of t h e counterweight W.


b. Find t h e inertia properties at point O of t h e (assumed rigid) b o d y
c o m p o s e d of R + W+S.
c. Write t h e equations of rotational m o t i o n of
d. D e t e r m i n e t h e n e t m o m e n t s t h a t m u s t be exerted about axes t h r o u g h
O for t h e position s h o w n if first (in units of r a d i a n s a n d seconds)
and then

Solution
Let us m o d e l t h e a n t e n n a as follows. T h e reflector is treated as a thin disk, t h e
counterweight as a point mass, a n d t h e s u p p o r t structure as being rigid b u t light.
Axes (X, Y, Z) are fixed in t h e inertial frame w h i l e (x, y, z) are attached to .
In p a r t (a) t h e p u r p o s e of t h e counterweight is to place t h e m a s s center of o n
its elevation axis. T h u s
1200(4) = W(2)

W = 2400 lb
For part (b) w e generate t h e inertia properties of a b o u t t h e point O. (See the
table o n t h e next page.)
Therefore t h e inertia matrix m a y be written as

2
slug-ft
(1)
For part (c), o n e w a y to proceed is to c o m p u t e t h e principal axes a n d m o ­
m e n t s of inertia from this matrix a n d t h e n use the Euler equations (7.40). This
w o u l d b e a very u n w i s e a p p r o a c h in this case, h o w e v e r . N o t only is it tedious to
locate t h e principal axes, b u t following this w e w o u l d h a v e to break u p the
angular velocity into its c o m p o n e n t s along these directions; w e w o u l d t h e n ob­
tain not-so-useful m o m e n t c o m p o n e n t s a b o u t axes s k e w e d with respect to t h e
rotation axes. It is m u c h simpler to use t h e second of Euler's laws:*

T h e only prices w e will h a v e to p a y are (1) to retain a n d deal with t h e n o n z e r o


O
product of inertia I a n d (2) to m o v e t h e derivative from frame to . T h e angular
xy

m o m e n t u m of a b o u t O is, for our problem,

(2)

• All that is required of O for this equation to be valid is that it be a point of the inertial
frame; in what follows, however, it also needs to be, and is, a pivot of
Page 4 7 7

Reflector + Counterweight = Total

O O
* The y axis through O is an axis of symmetry of R; hence I = 0 = I . And the third
xy yz
O
product of inertia of R, I , vanishes because zy is a plane of symmetry for body R.
xz

T h e angular velocity of in frame is found by the addition t h e o r e m . The


reflector a n d its supporting structure rotate in elevation with a simple angular
velocity with respect to t h e h o u s i n g A; likewise, A rotates in a z i m u t h with a
simple angular velocity with respect to t h e pedestal (which is rigidly fixed to
t h e reference frame ). Therefore

(3)
Page 4 7 8

Substituting t h e c o m p o n e n t s into Equation (2) gives t h e angular m o m e n t u m of


i n , expressed in (terms of its c o m p o n e n t s in)

(4)

Next w e use Equation (6.20) to differentiate H 0 (note t h a t point O is fixed in


a n d . ):

(5)

Taking t h e derivative a n d performing t h e cross product, w e find t h a t t h e three


c o m p o n e n t equations are as follows. (Note that t h e inertia properties d o not
c h a n g e in )

(6a)

(6b)

(6c)

As a n indication of t h e increased difficulty of three-dimensional d y n a m i c s p r o b ­


lems, n o t e t h a t all four of t h e n o n v a n i s h i n g inertia properties contribute to each
component of t h e external m o m e n t acting o n at O!
In t h e indicated position, . Therefore

(7a)

(7b)

(7c)

In part (d), for t h e case specified, while Also,


while . This case physically corresponds to t h e a n t e n n a , at 45 ° elevation,
swinging a r o u n d t h e vertical at 30 ° / s e c a n d s u d d e n l y sensing a n object traveling
t o w a r d zenith; t h e controls activate a m o t o r w h o s e torque produces a n angular
acceleration t h a t will s e n d t h e a n t e n n a u p w a r d in elevation. T h e angular acceler­
ations are large because t h e n e e d is to get t h e r e quickly.
Substituting these values of and into Equations (7), along w i t h
t h e inertia values, gives o u r answer:
Page 4 7 9

(8)
These are t h e m o m e n t s exerted b y P o n t o A, excluding t h a t required to balance
the d e a d weight of t h e a n t e n n a . In t h e inertial reference frame, t h e m o m e n t s are

(9)
In the opposite case w h e n t h e a n t e n n a is tracking in elevation, say
r a d / s e c , a n d receives a s u d d e n c o m m a n d resulting in " at
, t h e m o m e n t c o m p o n e n t s b e c o m e (here

(10)

O n c e again w e see t h e considerable effect of the product of inertia term.


T h e negatives of the X, Y, Z c o m p o n e n t s respectively b e n d , b e n d , a n d twist
the pedestal a n d are considerations in its design; far larger a n d m o r e i m p o r t a n t
m o m e n t s , h o w e v e r , arise from the w i n d blowing against the " d i s h " a n d from
gravity. T h e r e are also forces exerted on A at O d u e to gravity a n d the m a s s center
acceleration.

EXAMPLE 7 . 8

A symmetric wheel D spins at angular speed a b o u t its axis, which is a line fixed
in b o d y , as well as in D. (See Figure E7.8a.) The m o m e n t s of inertia of D at C are
Figure E7.8a C C C
I = J and I = I = I. Body h a s negligible m a s s a n d rotates at angular speed
yy xx zz
Page 480

about t h e x axis at O. If a m o m e n t M is applied to parallel to t h e x axis, find,


0 c

at t h e instant s h o w n , t h e rates of c h a n g e of and a n d t h e force a n d m o m e n t


exerted o n D at C b y t h e pin. Neglect friction.
Solution
Let u s d e n o t e t h e inertial reference frame (in w h i c h moves) b y T h e addition
t h e o r e m for angular velocity t h e n gives, for t h e w h e e l ,

w h e r e t h e axes (x , y , z ) are fixed in a n d their associated unit vectors


c c c

are fixed in direction in N o t e that w e m a y use t h e equation

for t h e angular m o m e n t u m of D because even t h o u g h (x , y , z ) are n o t fixed in


c c c

b o d y D, t h e y are nonetheless p e r m a n e n t l y principal. Therefore

T h e second l a w of Euler t h e n yields

Figure E7.8b

w h e r e for convenience w e differentiate H in frame since t h e vector h a s b e e n


c

written in terms of its c o m p o n e n t s there. Continuing,

From Figure E7.8b, a free-body d i a g r a m of D, w e get t h e c o m p o n e n t s of


so t h a t

T h u s w e see t h a t

(1)
(2)

(3)

In addition, for t h e disk yields

so t h a t
(4)

(5)
(6)
T u r n i n g n o w to Figure E7.8c, a free-body d i a g r a m of t h e light b o d y we
have so that
(7)
C o m b i n i n g Equations (1), (5), a n d (7),

(8)
Figure E7.8c
Page 4 8 1

and

(9)

and

(10)

The results (4), (6), (8), (9), a n d (10) are recognizable as w h a t w o u l d h a v e b e e n


obtained h a d t h e disk b e e n frozen in its bearings, in w h i c h case a n d D w o u l d
constitute a single rigid b o d y in plane motion.
Equation (3), h o w e v e r , does n o t follow intuitively from the s t u d y of p l a n e
motion. The term is sometimes called a gyroscopic m o m e n t , a n d t h e e q u a ­
tion says t h a t a m o m e n t of this m a g n i t u d e m u s t act o n D a b o u t z if t h e given
c

m o t i o n is to occur. N o t e t h a t in this case t h e b o d y D is not allowed to t u r n about zc

as it spins (about y ) . If it were, say b y m e a n s of a bearing b e t w e e n C a n d O, t h e n


c

the m o m e n t c o m p o n e n t M w o u l d b e c o m e zero, a n d a third


PZ component
(about z ) w o u l d appear. N o t e also from Figure E7.8c t h a t the gyroscopic m o ­
c

m e n t twists the shaft of This, for example, is a consideration in t h e retraction of


the wheels of some airplanes.

EXAMPLE 7 . 9

The thin disk D t u r n s o n the light arm b y w a y of a s m o o t h bearing t h a t keeps


the axes of D a n d aligned. The a r m is h i n g e d to a shaft driven at constant
angular speed by a motor. The system is set u p so t h a t the a r m is horizontal
w h e n the disk contacts t h e g r o u n d as s h o w n in Figure E7.9a. A s s u m i n g t h e disk,
of m a s s m, to roll o n t h e g r o u n d , find t h e forces exerted by the g r o u n d o n the disk,
the forces a n d m o m e n t s exerted b y the a r m on t h e disk, a n d the forces a n d
m o m e n t s exerted b y t h e shaft on the arm.
Solution
We shall begin the analysis b y applying to the disk the equations of motion:

Figure E7.9a
Page 482

and

Because C is o n t h e axis of t h e a r m w h i c h t u r n s at constant rate w e k n o w that

w h e r e w e are using t h e axes a n d unit vectors s h o w n in Figure E7.9a a n d these are


fixed in the arm To obtain a n expression for H w e n e e d to d e t e r m i n e
c So,
using t h e addition t h e o r e m ,

w h e r e t h e only m o t i o n D can h a v e in is to t u r n (in simple angular velocity)


a b o u t their c o m m o n axis.
Because t h e disk rolls, its point E in contact w i t h t h e g r o u n d h a s zero veloc­
ity; using Equation (6.56), w e obtain

or

and

Therefore, since w e a s s u m e t h e disk to b e relatively thin so t h a t to good approxi­


mation

a n d since we have

N o t e that since is constant,

a n d so
Page 483

N o w using Figure E7.9b, t h e free-body d i a g r a m of t h e disk, t h e equation

yields

(1)
(2)

(3)

a n d t h e equation

likewise yields
Figure E7.9b
(4)

(5)

(6)
Using Figure E7.9c, t h e free-body d i a g r a m of t h e massless bar, w e h a v e

(7)

Figure E7.9c

We shall delay using and w h i c h will ultimately


yield R , R ,R ,T a n d T , a n d focus o n Equations ( l ) - ( 7 ) from w h i c h w e obtain
x y z y z
Page 484

which is as far as w e can go because this problem is actually "dynamically"


indeterminate. However, if w e assume friction to be small (just enough to pro­
duce rolling as this system is slowly brought up to speed) so that , then w e
have M ,B , B and N uniquely determined. With that condition w e may n o w use
x y z

the remaining "equilibrium" equations for the light rod to obtain

Let us make a couple of observations: first, note that T = 0 means that the motor
y

doesn't have to supply any driving torque in order to maintain the constant
— remember w e have smooth bearings; and secondly note that if is large, so
is N:

and that extra part of N, over and above the weight mg, is often said to be due to
gyroscopic action.

EXAMPLE 7 . 1 0

It is possible for the homogeneous cone of base radius R in the Figure E7.10a to
roll steadily around on a flat horizontal table in such a way that its uncon­
strained vertex O remains fixed and the center point Q of its base travels on a
horizontal circle at constant speed. Let this motion be begun by forces which are
then released. Assume that there is sufficient friction between the cone and the
table to prevent slipping. Besides having enough friction, there is yet another
special condition that must be satisfied in order that the motion occur. Find the
friction and normal force resultants, their lines of action, and the special condi­
tion. Refer to Example 6.12 for some related kinematics.

Figure E7.10a

Solution
Since the mass center C moves on a horizontal circle at the constant speed (see
Figure E7.10b), the friction force / is given by:
Page 4 8 5

or

Figure E7.10b

since . T h e n o r m a l force N is simply mg since a = 0. There is n o


cz

friction force c o m p o n e n t n o r m a l to t h e p l a n e of t h e paper, since a also vanishes.


cy

T h e directed line of action of f is as s h o w n above, along AO. For t h e line of action


of N, w e u s e t h e m o m e n t equation of motion; n o t e t h a t is our inertial frame:

w h e r e frame contains t h e Z axis a n d t h e axis of t h e cone. N o w t h e angular


m o m e n t u m H is, since
o are in principal directions at t h e fixed p o i n t O:

In t h e preceding equation, t h e -components are those of t h e b o d y , in . From


t h e previous examples, and, substituting for J, a n d I,

Noting t h a t , w e find

a n d from t h e free-body diagram, using N = mg,


Page 4 8 6

or

w h i c h gives t h e line of action of t h e n o r m a l force resultant.


The "special c o n d i t i o n " m e n t i o n e d i n t h e p r o b l e m s t a t e m e n t is t h a t the
n o r m a l force m u s t intersect t h e p l a n e at a point of physical contact w i t h the cone,
i.e.:

Thus

w h i c h w h e n simplified is

Holding constant, w e see that w h a t h a p p e n s if v is too large (or g or R too


Q

small) is t h a t the n o r m a l force n e e d s to act b e y o n d the point A. Since t h a t cannot


physically h a p p e n , t h e specified m o t i o n will n o t occur b u t will give w a y to
another, t h a t of tipping outward.

PROBLEMS • Section 7.5

7.42 The r o d is rigidly attached to shaft S w h i c h is free


to turn in the t w o bearings as indicated in Figure P7.42.
The y a n d y' axes point into the p a g e at C. S h o w t h a t
the m o m e n t w i t h respect to C t h a t m u s t b e supplied
by the bearings to shaft S to sustain t h e m o t i o n m u s t
h a v e the c o m p o n e n t s :

D o this in t w o w a y s : (1) Use the Euler equations (7.40)


with the principal axes (x, y, z); (2) u s e t h e Equa­ Figure P7.42
tions (7.41) w i t h the axes (x', y', z') in t h e figure.
7.43 Find the reactions exerted b y t h e bearings o n the
shaft to w h i c h a 30-kg thin plate p is w e l d e d . (See Fig­
u r e P7.43.) T h e assembly is turning at t h e constant angu­
lar speed of 30 r a d / s . Work the p r o b l e m b y using

that is, b y expressing H using principal directions in p


c

(which omits t h e n e e d for c o m p u t i n g t h e n o n z e r o product


of inertia I ). xz
Figure P7.43
Page 4 8 7

Figure P7.45

7.44 Rework a n d check t h e results of t h e preceding n a m i c bearing reactions at a n d B caused b y t h e mis­


2

p r o b l e m b y using Equations (7.24) to calculate I ; t h e n


xz
alignment, in terms of , by: (a) using principal axes; (b)
use Equations (7.41) to obtain t h e bearing reactions. w i t h o u t using principal axes (i.e., calculate a n d utilize t h e
p r o d u c t s of inertia).
7.45 T h e equilateral triangular plate w a s to be
m o u n t e d o n t o t h e rotating shaft as s h o w n in Figure P7.45 7.46 A slender r o d A a n d a small ball each w e i g h 0.3 lb.
in t h e solid figure. T h e installation resulted in t h e mis­ (See Figure P7.46.) T h e bodies rotate about t h e vertical
alignment angle (see t h e d a s h e d position). T h e plate h a s along w i t h t h e slender shaft S a n d are s u p p o r t e d b y a
m a s s 2 slugs, a n d t h e shaft is light. Determine t h e d y ­ s m o o t h step or thrust bearing at D a n d b y t h e cord C. Find
t h e tension in the cord if t h e angular speed of the system
is 20 r a d / s e c .
7.47 T h e disk (mass m, radius r) in Figure P7.47 is rigidly
attached to t h e shaft, a n d t h e assembly is s p u n u p to
angular speed a b o u t t h e z axis. Determine t h e bearing
c

reactions at A a n d B in terms of m, r, g, a n d L.
7.48 A bicycle w h e e l weighing 5 lb a n d h a v i n g a 14-in.
radius is misaligned b y 1° w i t h t h e vertical. T h e t o p of t h e
w h e e l tilts t o w a r d t h e right w i t h t h e bike m o v i n g for­
w a r d . If t h e bike is driven along a straight p a t h at 15 m p h ,
find t h e m o m e n t exerted o n t h e w h e e l . Use t h e re­
sult of P r o b l e m 7.11, neglecting t h e spokes a n d h u b .

Figure P7.46 7.49 In t h e preceding problem, s u p p o s e instead t h a t t h e


bicycle is in a 50-ft-radius t u r n to t h e (a) right a n d (b) left.
Determine t h e n e w values of t h e m o m e n t exerted o n t h e
misaligned w h e e l . Neglect t h e lean angle.

Figure P7.47
Page 488

observed b y S. C h a n d l e r in 1 8 9 1 . T h e difference is attrib­


7.50 Writing Euler's l a w s as and results in w h a t is k n o w n as t h e re­
u t e d to t h e nonrigidity of t h e earth. A l t h o u g h e n e r g y dis­
versed effective force (— m a ) a n d t h e inertia torque
c
sipation s h o u l d d a m p o u t this " w o b b l e , " in fact it d o e s
If these quantities are a d d e d to the free-body diagram, t h e
not. The ongoing cause of t h e w o b b l e is a n u n s o l v e d
object m a y b e treated as t h o u g h it w e r e in equilibrium.
p r o b l e m in g e o d y n a m i c s at this time. See Science News, 24
For a car at t h e instant of o v e r t u r n i n g while traveling at October 1981.)
speed v on a curve of r a d i u s R a n d b a n k angle , s u c h a
d i a g r a m w o u l d a p p e a r as s h o w n in Figure P7.50. 7.56 A screwdriver-like m o t i o n b e t w e e n t h e p l a n a r case
a n d general (three-dimensional) m o t i o n is defined as fol­
lows . All points of t h e b o d y h a v e , at a n y time, identical z
c o m p o n e n t s of velocity in a reference frame F. T h e unit
vector of this c o m p o n e n t is constant in b o t h a n d F,
t h o u g h can vary w i t h time. T h u s t h e a n g u l a r velocity
vector is still expressible as Derive a m o m e n t
equation for t h a t is valid for this motion.
7.57 The disk in Figure P 7 . 5 7 is s p i n n i n g a b o u t t h e light
axle, a n d t h e axle is precessing at t h e constant rate of
r a d / s . If t h e axle is o b s e r v e d to r e m a i n horizontal, find
Figure P7.50 t h e m a g n i t u d e a n d direction of t h e spin of t h e disk.

Calculate t h e m o m e n t a b o u t t h e p o i n t Q a n d , n o t i n g
that for this free-body diagram, c o m p a r e t h e
effects of t h e reversed effective force (or inertia force)
— ma w i t h that of t h e inertia t o r q u e —
c o n t h e over­
turning tendency. W h a t factors m a k e t h e car m o r e likely
to turn over?
7.51 S u p p o s e that t h e c o m p o n e n t s of t h e m a s s center
velocity v are written in b o d y instead of in a n inertial
c

frame U s e t h e p r o p e r t y (6.20) of to derive the


scalar e q u a t i o n s of m o t i o n of t h e m a s s center from Euler's
first law: Figure P7.57
7.52 S h o w that if a rigid b o d y u n d e r g o i n g torque-free
m o t i o n in a n inertial frame h a s t h r e e equal principal 7.58 A single-engine aircraft h a s a four-bladed propeller
m o m e n t s of inertia at its m a s s center, t h e n its a n g u l a r weighing 128 lb w i t h a radius of gyration a b o u t its center
velocity is constant in of m a s s of 3 ft. It rotates counterclockwise at 2000 r p m
w h e n v i e w e d from t h e rear. Find t h e gyroscopic m o m e n t
7.53 In Example 7.8 define t h e (x, y, z) axes at O as prin­ on t h e propeller shaft w h e n t h e p l a n e is at t h e b o t t o m of a
cipal for w i t h associated respective m o m e n t s of inertia vertical loop of 2000-ft r a d i u s w i t h a speed of 500 m p h . In
a n d . Rework t h e p r o b l e m w i t h o u t a s s u m i n g t h a t w h i c h direction will t h e tail of t h e p l a n e t e n d to m o v e
h a s negligible mass. T h e t w o sets of axes are respectively because of this m o m e n t ?
parallel prior to t h e application of M . O

7.59 T h e blades of a fan t u r n at 1750 r p m , a n d t h e fan


7.54 S h o w t h a t if t h e solution of Example 7.6 is
oscillates a b o u t t h e vertical axis z (Figure P7.59(a)) at the
projected into t h e xy p l a n e , t h e tip of t h e projection vector rate of o n e cycle every 10 sec. A s s u m i n g t h a t t h e fan
travels o n a circle of r a d i u s at t h e frequency travels at t h e constant a n g u l a r velocity of 0.2 r a d / s e c
except w h e n it is reversing direction (Figure P7.59(b)),
7.55 A result of t h e e a r t h ' s bulge is t h a t I/J = 0.997. find t h e m o m e n t exerted by t h e b a s e o n t h e a r m section A
Use this result to c o m p u t e t h e period of o n e revolution of at t h e -cycle p o i n t d u e to gyroscopic action. For t h e cal­
the earth's a n g u l a r velocity vector ( N o r t h Pole!) a b o u t its culation (only!) consider the blades (Figure P7.59(c)) to be
axis of s y m m e t r y . (The a n s w e r , o b t a i n e d b y Euler in 4-in.-diameter circular a l u m i n u m plates all in t h e s a m e
3
1752, is a b o u t 4 m o n t h s less t h a n t h e actual p e r i o d first plane and in. thick. Use a density of 0.1 l b / i n . .
Page 489

7.64 A disk D rolls a r o u n d in a circle w i t h its p l a n e verti­


cal a n d its center traveling at constant s p e e d V . Find t h e
C

tension in the string, a n d t h e friction force exerted o n D


by t h e floor. (See Figure P7.64.)
7.65 There is a relationship a m o n g V , g, r, R a n d such
C

that t h e disk can roll a r o u n d in a circle as s h o w n in Fig­


ures P7.65(a) a n d (b), w i t h V a n d r e m a i n i n g constant.
C

Find this relationship.

(a)
(c)

(b)
Figure P7.59

Figure P7.61

Figure P7.60
Figure P7.64
7.60 Disk D in Figure P7.60 turns in bearings at C at
angular rate a b o u t t h e light r o d R, a n d b o t h precess
about axis z at angular rate
a as s h o w n . S h o w w i t h a
free-body diagram h o w it is possible for t h e m a s s center C
to remain in a horizontal plane. T h e n find t h e reactions
exerted o n t o R b y t h e socket at O. Is there a n y difference
in t h e solution if D a n d R are rigidly connected?

7.61 In t h e grinding mill of Problem 7.5, s u p p o s e that


the wall is absent. (See Figure P7.61.) Find, for a given
(constant angular speed of S ) , t h e angle that t h e axis of
the grinder D will m a k e w i t h t h e vertical. Observe that
with t h e wall present a n d fixed, larger s p e e d s t h a n this (a)
will allow t h e grinder to work. In particular, s h o w that
the following set of p a r a m e t e r s is satisfactory: r = 2.5 ft,
rad/sec, and . Neglect t h e m a s s
of b o d y in comparison w i t h t h e h e a v y grinding disk D.
7.62 Find t h e grinding force N p r o d u c e d at t h e wall of
the grinding mill of P r o b l e m s 7.5 a n d 7.61 for t h e given
parameters.
7.63 Find t h e m a g n i t u d e a n d direction of t h e force
a n d / o r couple exerted o n disk D b y t h e shaft S in P r o b ­ (b)
lem 7.4. Figure P7.65
Page 490

7.66 Obtain the results of Example 7.9 by using the b. If the ship o n a straight course in rough seas
Euler equations (7.42). Hint: This time the axes are body- pitches sinusoidally at amplitude with a
fixed, and the part of changes direction in D; there­ 6-s period, what are the maximum bearing re­
fore the components and have derivatives that actions then?
were formerly zero in The component of is A heavy disk D of mass m and radius r spins at the
angular rate with respect to the rigid, but
light, bent bar (See Figure P7.68.) Body turns at rate
where is the angle of roll as s h o w n in Figure P7.66. about a vertical axis through O, a point of
Differentiate this expression, substitute and both and then take
the inertial frame G. Finally,
Find the force and go to Equations (7.42) and
couple

stitute your results.


7.67 A ship's turbine has a mass of 2500 kg and a radius
of gyration about its axis (y in Figure P7.67) of 0.45 m .
c

It is mounted on bearings as indicated and turns at


5000 rpm clockwise w h e n viewed from the stern (rear)
of the boat.

a. If the ship is in a steady turn to the right of


radius 500 m and is traveling at 15 knots, what
are the reactions exerted on the shaft by the
bearings? (1 knot = 1.15 m p h = 1.85 k m / h r )

Figure P7.68

Figure P7.66

(a)

(b)
Figure P7.67 Figure P7.69
Page 491

that m u s t b e acting o n at O to p r o d u c e a m o t i o n of t h e HKIU, length


system for w h i c h and are constants. Both sets of mass m,
radius R
axes in t h e figure are fixed in a n d n o t e t h a t (X , Y , Z )
C C C

are always principal axes for D at C e v e n t h o u g h t h e y are


not fixed in D.

7.69 C o m p u t e t h e m o m e n t M applied to the shaft S in


Figures P7.69(a) a n d (b), o n t h e preceding page, as a
function of t h e angle if and are constants.
7.70 A bike rider enters a turn of radius R at a constant
speed of v . (See Figure P7.70.) O t h e r quantities are d e ­
c

fined below:
Figure P7.71
r=radiusof w h e e l
d=distanceb e t w e e n axle a n d C 7.72 In P r o b l e m 6.24, if and are constants,
I , I = principal m o m e n t s of inertia of entire bike
1 2
find t h e resultant m o m e n t exerted o n D at its m a s s center,
plus rider w i t h respect to and w h e n is pointing straight u p . T h e b o d y D is symmetri­
directions t h r o u g h C cal, w i t h centroidal principal m o m e n t s of inertia J along
a n d I n o r m a l to
i = m o m e n t of inertia, w i t h respect to
direction, of o n e w h e e l a b o u t its axis • 7.73 T h e b o d y in Figure P 7 . 7 3 is a n ellipsoid of revo­
of s y m m e t r y lution w i t h m a s s = 1 slug, a n d semiminor a n d semimajor
axis lengths a a n d 2a, w i t h a = 1 ft. T h u s
m = total m a s s

= angle s h o w n

Solve for t h e resultant force and moment in


and
terms of these quantities. C o m p a r e t h e effects of t h e
D'Alembert force (— m a ) a n d t h e inertia torque
c in
righting t h e bike w h e n is small. Neglect t h e p r o d u c t s of
inertia.
T h e s h a d e d light frame A is driven a r o u n d the fixed post
p , w i t h angular velocity b y a m o t o r torque T applied
2

at P. A n o t h e r m o t o r (neither is s h o w n ) b e t w e e n A a n d
applies a torque which causes t h e b o d y to spin in the
frame. The axes a n d unit vectors s h o w n are fixed in A.
During a n interval of motion, rad/sec and
= 2t r a d / s e c . Find all forces a n d couples applied onto A
at P w h e n t = 1 sec. T h e distance from P to t h e x axis is
2 ft.

Figure P7.70

• 7.71 Disk D in Figure P7.71 turns in bearings a r o u n d


r o d R as it rolls o n t h e g r o u n d . Find all t h e forces acting o n
D, a n d contrast t h e solution w i t h t h a t of Example 7.9.
The rod is free to slide along the vertical post about w h i c h
it turns at constant rate. Figure P7.73
Page 492

made to turn relative to arm about the axis of and


is made to turn with respect to the ground (inertial frame)
about the vertical through point O. At the given instant,
the angular velocity of D in expressed in terms of unit
vectors fixed in is
In addition, at this instant
w e are given and
C 2 C
For the disk, m = 10 slugs, I = 1.4 slug-ft , and I = 0.7
xx yy

2
slug-ft . Find all forces and couples that are acting on

• 7 . 7 5 In the preceding problem, use and their


derivatives to compute the right-hand sides of the Euler
equations (7.40). Explain w h y these results are not the
components of the moments of external forces acting on
D at C.
Figure P7.74
• 7 . 7 6 In the preceding two problems, at the given in­
stant find and for i = 1, 2, and 3, where is the
7.74 By means of appropriately mounted control motors angle of rotation of D with respect to is the angle
(not shown), the disk D in Figure P7.74 is made to turn of rotation of with respect to , and is the angle of
about its axis of symmetry relative to the arm is rotation of with respect to

7.6 Gyroscopes
W e n o w return our attention to the gyroscope w h o s e orientation w a s
e x a m i n e d i n S e c t i o n 6 . 8 . First w e s h a l l d e r i v e t h e e q u a t i o n s of r o t a t i o n a l
m o t i o n of s u c h a g y r o s c o p e G.* W e b e g i n b y e x p r e s s i n g its a n g u l a r v e l o c ­
ity i n t h e f r a m e F ; s e e F i g u r e s 7.7, 7.8 a n d 7.9, r e p e a t e d f r o m
2

Section 6.8.

(7.44)

Figure 7.7 First rotation. Figure 7.8 Second rotation. Figure 7.9 Third rotation.

* We are taking G to be the rotor and are considering it as heavy with respect to the
inner and outer gimbals, w h o s e mass w e then neglect. We also assume Gtobe symmet­
ric about its axis. Of course, a gyroscope does not have to possess any gimbals; the earth
is a massive gyro, as will be seen in an example to follow.
Page 493

The axes (x ,y ,z ) of the inner gimbal are not fixed inGbecause of its
2 2 2

spin — but they are nonetheless permanently principal. This important


fact allows u s to write the angular momentum of the gyroscope in the
frame f as2

(7.45)

Euler's second law, together with Property (6.20) of the angular v e ­


locity vector, then gives the equations of motion of the gyroscope as
follows:

(7.46)

In the preceding calculations w e have used the addition theorem to


observe the following:

(7.47)

so that is the same vector as in Equation (7.44) if is omitted. The


equations of motion of G are therefore

(7.48)

where is the component of about the spin axis of symmetry of the


gyroscope. Note that it is made u p of part of the precession speed as well
as all of the spin.
The gyroscope equations are seen to be nonlinear, including not only
products of the angles' derivatives but also trigonometric functions of
them. Their general solution is an unsolved problem; however, there are
two special solutions that are quite worthy of study. The first of these is
steady precession; the second is torque-free motion. We shall have a look
at each in turn.

Steady Precession

Steady precession is defined by the nutation angle the precession


speed and the spin speed each being constant throughout the mo­
tion. Let us call these constants and and substitute them into
Equation (7.48) to obtain:

(7.49)
Page 494

W e s e e t h a t o n l y a m o m e n t a b o u t t h e y axis is n e e d e d t o s u s t a i n s t e a d y
2

precession. Also means that is a c o n s t a n t . *


In t h e case in w h i c h , w e h a v e t h e precession a n d spin axes
o r t h o g o n a l ; t h e s i t u a t i o n is s h o w n i n F i g u r e 7 . 1 0 . If t h e g y r o is s p i n n i n g
a n d t h e t o r q u e is a p p l i e d , t h e r e w i l l s i m u l t a n e o u s l y o c c u r a p r e c e s s i o n
t h a t t e n d s t o t u r n t h e s p i n v e c t o r t o w a r d t h e t o r q u e v e c t o r . T h i s is s o m e ­
t i m e s c a l l e d t h e l a w of g y r o s c o p i c p r e c e s s i o n . T h e t o r q u e i n t h i s c a s e ,
f r o m E q u a t i o n ( 7 . 4 9 ) , is

(7.50)

a n d it is s e e n t o b e t h e p r o d u c t of t h e s p i n m o m e n t u m and the
precessional angular speed
W e t u r n n o w t o a n i l l u s t r a t i o n of t h e l a w of g y r o s c o p i c p r e c e s s i o n .
Figure 7.10 W e h a v e j u s t s e e n t h a t w h e n a f r e e l y s p i n n i n g b o d y is t o r q u e d a b o u t a n
axis n o r m a l t o t h e s p i n a x i s , it p r e c e s s e s a b o u t a t h i r d axis t h a t f o r m s a n
o r t h o g o n a l t r i a d w i t h t h e s p i n a n d t o r q u e v e c t o r s . T h e d i r e c t i o n of t h e
p r e c e s s i o n is s u c h t h a t it t u r n s t h e s p i n v e c t o r t o w a r d t h e t o r q u e v e c t o r .
T h i s l a w of g y r o s c o p i c p r e c e s s i o n is r e s p o n s i b l e f o r t h e l u n i s o l a r p r e c e s ­
s i o n of t h e e q u i n o x e s .
W h a t is t h e l u n i s o l a r p r e c e s s i o n ? B e c a u s e of t h e b i l l i o n s of y e a r s of
g r a v i t a t i o n a l p u l l f r o m t h e s u n a n d m o o n , t h e e a r t h is s l i g h t l y b u l g e d
i n s t e a d of r o u n d . It is i n f a c t a b o u t 2 7 m i l e s s h o r t e r a c r o s s t h e p o l e s t h a n it
is a c r o s s t h e e q u a t o r . T h i s b u l g e , p l u s t h e fact t h a t its a x i s is t i l t e d to
t h e ecliptic, c a u s e s t h e s u n ( a n d m o o n ) to t o r q u e t h e e a r t h i n a d d i t i o n to
the gravity pull, as s h o w n in Figure 7.11.

Daily spin

Earth
Sun

Sun pulls
harder on
shaded half
(because it is closer), giving a
torque (T) as well as a force (F)
Figure 7.11

W e s e e , t h e n , t h a t w e l i v e o n t h e s u r f a c e of a s p i n n i n g g y r o s c o p e t h a t
is c o n s t a n t l y b e i n g a c t e d o n b y a n e x t e r n a l t o r q u e , a n d a p r e c e s s i o n is
t h u s o n g o i n g . T h i s p r e c e s s i o n , s h o w n i n F i g u r e 7 . 1 2 , t u r n s t h e s p i n axis
of t h e e a r t h o u t of t h e p l a n e of t h e p a p e r t o w a r d t h e t o r q u e v e c t o r . T h i s
m o t i o n r e s u l t s i n c o u n t e r c l o c k w i s e m o v e m e n t , o n t h e c e l e s t i a l s p h e r e , of
t h e c e l e s t i a l p o l e t o w h i c h t h e e a r t h ' s r o t a t i o n a x i s is d i r e c t e d . ( T h i s p o i n t

* N o t e that z and z are the same axis.


2

The spin axis of the earth is aligned with its angular velocity vector The point
Now 13,000 years
where the vector, placed at C, cuts !he surface of the earth is the real meaning of the
from now
North Pole. The North Pole wanders about the geometric pole (on the symmetry axis)
Figure 7.12 as time passes; it has remained within a few feet of it in this centurv.
Page 495

is c u r r e n t l y c l o s e t o P o l a r i s , t h e N o r t h S t a r . ) T h e p e r i o d of t h i s r o t a t i o n is
a b o u t 2 6 , 0 0 0 y e a r s , a n d it i s i n t e r e s t i n g t h a t t h e m o o n ' s effect is 2 . 2 t i m e s
t h a t of t h e s u n ' s b e c a u s e it is m u c h c l o s e r .

Torque-Free Motion

W e n o w t a k e u p t h e o t h e r e x a m p l e of a s o l u t i o n t o t h e g y r o s c o p e e q u a ­
t i o n s : t h e c a s e of torque-free m o t i o n . " T o r q u e - f r e e " m e a n s t h a t
v a n i s h e s , s o t h a t H is a c o n s t a n t . T h i s f o l l o w s f r o m E u l e r ' s s e c o n d l a w :
c

W e s h a l l c o n v e n i e n t l y l e t t h e d i r e c t i o n of t h e Z a x i s ( s e e F i g u r e 7 . 1 3 ) ,
w h i c h is a r b i t r a r y , c o i n c i d e w i t h t h e c o n s t a n t d i r e c t i o n of H . T h e n Zc

b e c o m e s t h e p r e c e s s i o n a x i s of t h e m o t i o n , a n d t h e ( x , y , z ) a x e s a p p e a r
2 2 2

a s s h o w n i n F i g u r e 7 . 1 3 . it is s e e n t h a t s i n c e

(7.51)

a n d since H is s e e n a l w a y s t o lie i n t h e x z
C 2 2 plane, then must vanish:

(7.52)

Note that lies i n t h e x z p l a n e a l o n g w i t h H , s i n c e i t s y c o m p o n e n t ,


2 2 c 2

vanishes. Furthermore, w e see that


Figure 7.13

(7.53)

But also, a s w e c a n s e e f r o m Figure 7.13,

(7.54)

T h e r e f o r e , e q u a t i n g t h e first of E q u a t i o n s ( 7 . 5 3 ) a n d ( 7 . 5 4 ) f o r , we
obtain

(Since H , c

a n d I are constants) (7.55)

(7.56)

a n d w e see that

(a c o n s t a n t ) (7.57)

S i m i l a r l y , e q u a t i n g t h e t w o p r e c e d i n g v a l u e s of gives

(7.58)

(7.59)

• Note again that the (x , y , z ) axes are permanently principal, even though x and y
2 2 2 2 2

are not body-fixed, and this lets us write H in terms of the C along these axes.
Page 496

so that

(7.60)

T h e r e f o r e all c o n d i t i o n s a r e s a t i s f i e d f o r t h e t o r q u e - f r e e b o d y t o b e i n a
s t a t e of s t e a d y p r e c e s s i o n a b o u t t h e z a x i s fixed i n !
Dividing E q u a t i o n (7.55) b y (7.58) leads to

(7.61)

a n d Figure 7.14 s h o w s t h a t

(7.62)

where is t h e a n g l e b e t w e e n z a n d
2 . Therefore

Figure 7.14

a n d w e see that t h e a n s w e r to w h e t h e r is l a r g e r o r s m a l l e r t h a n
d e p e n d s o n t h e r a t i o of J t o I. If J < I, a s i n t h e e l o n g a t e d s h a p e
in Figure 7.14, t h e n a n d t h e a n g u l a r v e l o c i t y v e c t o r lies i n s i d e of
H a n d z , making a constant angle with each. T w o cones m a y be imag­
C 2

i n e d — o n e fixed t o t h e b o d y , t h e o t h e r i n s p a c e . T h e b o d y c o n e is
s e e n t o roll o n t h e fixed s p a c e c o n e a s its s p i n a n d p r e c e s s i o n v e c t o r i -
ally a d d t o t h e v e c t o r , w h i c h c h a n g e s only in direction.
T h i s p r e c e s s i o n ( F i g u r e 7 . 1 5 ) is c a l l e d direct b e c a u s e and have
the same counterclockwise sense w h e n observed from the vector out­
s i d e t h e c o n e s . If, h o w e v e r , J > I, t h e n and lies outside t h e a n g l e

Space cone

(negative!)

Space
cone Body cone
Body cone

Figure 7.15 Figure 7.16


Page 4 9 7

ZCz. T h i s s i t u a t i o n is h a r d e r t o d e p i c t , b u t j u s t a s i m p o r t a n t . T h i s t i m e t h e
b o d y c o n e r o l l s a r o u n d t h e o u t s i d e of t h e e n c l o s e d , fixed s p a c e c o n e
(Figure 7.16), a n d t h e t w o r o t a t i o n s and h a v e opposite senses. This
p r e c e s s i o n is c a l l e d retrograde.
A final n o t e o n t h e t h e o r y of t h e t o r q u e - f r e e b o d y : If t h e c o n s t a n t
v a l u e of is e i t h e r 0 o r 9 0 ° , t h e r e is no p r e c e s s i o n a n d t h e g y r o s c o p e is
s i m p l y i n a s t a t e of p u r e r o t a t i o n , p l a n a r m o t i o n :

Here z = 2, so that and If the Here w e have and . Thus:


body simply spins about its axis. In this case, the rates and
cannot be distinguished

If , the body spins about a transverse axis without


precessing

Question 7.5 W h v can w e not use Equations (7.57) and


(7 60) to get and in this case?

It r e q u i r e s a g r e a t m a n y t e r m s t o d e s c r i b e t h e c o m p l e x m o t i o n of t h e
e a r t h . W e h a v e s e e n o n e e x a m p l e of t h i s i n t h e l u n i s o l a r p r e c e s s i o n
caused b y the gravity torque exerted o n the earth b y the sun a n d m o o n .
T h i s m o t i o n is a n a l o g o u s t o a d i f f e r e n t i a l e q u a t i o n ' s p a r t i c u l a r s o l u t i o n ,
w h i c h it h a s w h e n e v e r t h e e q u a t i o n h a s a n o n z e r o r i g h t - h a n d s i d e . T h e
c o m p l e m e n t a r y , o r h o m o g e n e o u s , s o l u t i o n is a n a l o g o u s t o t h e t o r q u e -
free p a r t of t h e s o l u t i o n t o t h e e a r t h ' s r o t a t i o n a l m o t i o n . T h i s p a r t , c a l l e d
t h e free p r e c e s s i o n of t h e e a r t h , is i n fact r e t r o g r a d e . B o t h t h e s p a c e a n d
b o d y cones are very thin as the , H , a n d z a x e s a r e all q u i t e c l o s e
C

t o g e t h e r ; e a c h lies a b o u t off t h e n o r m a l t o t h e e c l i p t i c p l a n e .

Answer 7.5 In deriving (7.57), if w e h a v e divided both sides of an equation by


zero. This result is then u s e d in getting in (7.60).
Page 498

PROBLEMS • Section 7.6

7.77 Find the angular acceleration in of the gyroscope 7.81 In the preceding problem, suppose that is given to
for the case of steady precession. be 400 r a d / s in the same direction as given in the figure
and that the cone's height H is not given. Find the value of
7.78 The spinning top (Figure P7.78) is another example
H for which this steady precession will occur.
of a gyroscope. Show that if the top's peg is not moving
across the floor, the condition for steady precession is 7.82 Using the fact that the sum of any two moments of
given by inertia at a point is always larger than the third (Problem
7.14), s h o w that for a torque-free axisymmetric body un­
dergoing retrograde precession, and that the z
axis of the 7.79
body isAalways outside the
top steadily space cone.
precesses about the fixed direction Z
7.83 The graph in Figure P7.83a depicts the stability ofthe top as a cone of
at 60 rpm. (See Figure P 7 . 7 9 . ) Treating
radius 1.2 in. and height 2.0 in., find the rate of spin of symmetrical satellites spinning about the axis z normal to
c

the top about its axis of symmetry. the orbital plane. The abscissa is the ratio of to the
7.80 Cone C in Figure P7.80 has radius 0.2 m and height moment of inertiaI ,about any lateral axis (they are all the
1

0.5 m. It is processing about the vertical axis through the same for what is called a "symmetrical" satellite—it need
ball joint, in the direction shown, at the rate of not be physically symmetric about z ) . The ordinate is the
c

If the angle is observed to be 20° and ratio of the spin speed (about z ) in the orbit to the
c

unchanging, what must be the rate of spin of the cone? orbital angular speed .

a. For a satellite equivalent to four solid cylinders


each of mass m, radius R, and height 3R, find
and I . The distance from C to any cylinder's
1

center is 2R, and the connecting cross is light.


The cylinders' axes are normal to the orbital
plane. (See Figure 7.83b.)

Moment of
inertia =

Figure P7.78

Stable

Unstable

C (5 kg; radius 0 2 m,
height 0.5 m)

Unstable
Figure P7.79

Stable

Figure P7.80 Figure P7.83a


Page 499

b. Determine w h e t h e r t h e station is stable for t h e is also zero. N o n e of t h e particles of t h e t o p n o t o n


following cases: t h e Z axis are in equilibrium, h o w e v e r , because t h e y all
C

i. T h e station's orientation is fixed in inertial h a v e (inward) accelerations . A b o d y is in equilibrium


space. if a n d only if all its particles are in equilibrium, so the
ii. T h e station travels a r o u n d t h e earth as t h e sleeping t o p c a n n o t b e in equilibrium. Explain this state­
m o o n does. m e n t in light of and , which were the
iii. T h e station h a s twice t h e angular velocity of equilibrium equations for a b o d y in statics. Hint: If O is a
t h e orbiting frame. fixed point of rigid b o d y in a n inertial frame then
iw. T h e s a m e as (iii), b u t t h e spin is opposite in
direction to t h e orbital angular speed.

Therefore s h o w that just because and


n e e d n o t b e zero. Use t h e t o p as a counterexample
a n d explain w h y t h e first t w o terms o n t h e right side of
t h e preceding equation vanish. T h u s are
Figure P7.83b necessary b u t n o t sufficient conditions for equilibrium of a
rigid body.
7.85 In t h e sleeping top counterexample of P r o b l e m
7.84, t h e terms and b o t h vanish i n d e p e n ­
T h e following five p r o b l e m s are a d v a n c e d looks at statics dently. S h o w t h a t there are m o r e complicated counterex­
of rigid bodies that d e p e n d o n o u r s t u d y of dynamics. amples in w h i c h is n o t constant in direction in a n d
a n d in w h i c h t h e t w o terms add to zero. Hint:
7.84 It is possible for a spinning t o p to " s l e e p , " m e a n i n g
. W h a t is for t h e torque-free
that its axis remains vertical a n d its p e g stationary as it
spins o n a floor. (See Figure P7.84.) In t h e absence of a
friction couple a b o u t t h e axis of t h e top, n o t e t h a t t h e spin 7.86 S h o w that if at all times, t h e n so is . Is
speed is constant a n d that t h e equations of m o t i o n t h e converse true?
t h e n reduce to and . It t h u s follows t h a t
7.87 If a frame is m o v i n g relative to a n inertial
frame , it can be s h o w n that is also a n inertial frame
if a n d only if at all times and t h e acceleration
in of at least o n e point of is zero at all times. Use this
t h e o r e m to s h o w that if a rigid b o d y is in equilibrium
in a n inertial frame then is itself a n inertial frame.
Is t h e converse true?

7.88 S h o w that a rigid b o d y is in equilibrium in an


inertial f r a m e . if a n d only if (a) at least o n e point of is
fixed in a n d (b) at all times. W h a t is t h e mini­
m u m n u m b e r of constraints o n that will satisfy (a) a n d
(b)? Describe o n e set of physical constraints t h a t will as­
Figure P7.84 sure equilibrium.

7.7 Impulse and Momentum


A s w e d i d i n C h a p t e r 5 f o r t h e c a s e of p l a n e m o t i o n , w e c o u l d a p p l y t h e
p r i n c i p l e s of i m p u l s e a n d m o m e n t u m a n d t h o s e of a n g u l a r i m p u l s e a n d
a n g u l a r m o m e n t u m t o t h e t h r e e - d i m e n s i o n a l m o t i o n of a r i g i d b o d y As
Page 500

w e s a w in Section 5.3, h o w e v e r , these applications are really n o t h i n g


m o r e t h a n t i m e i n t e g r a t i o n s of t h e e q u a t i o n s of m o t i o n .
T h e r e is o n e t y p e of p r o b l e m , h o w e v e r , i n w h i c h t h e s e t w o p r i n c i p l e s
f u r n i s h u s w i t h a m e a n s of s o l u t i o n — p r o b l e m s i n v o l v i n g i m p a c t . S o m e
t h r e e - d i m e n s i o n a l a s p e c t s a r e sufficiently d i f f e r e n t f r o m t h e p l a n a r c a s e
t o w a r r a n t a n e x a m p l e . B u t first w e u s e t h e i n t e g r a l s of t h e E u l e r l a w s t o
derive the n e e d e d relations:

(7.63)

(7.64)

A n a l t e r n a t i v e t o t h e r o t a t i o n a l e q u a t i o n ( 7 . 6 4 ) is t o i n t e g r a t e t h e e q u a l l y
general equation

(7.65)

w h e r e O is n o w a fixed p o i n t of t h e i n e r t i a l f r a m e

(7.66)

To u s e either E q u a t i o n (7.64) or (7.66) in a n i m p a c t situation, w e u s e


E q u a t i o n ( 7 . 1 1 ) for t h e b o d y ' s a n g u l a r m o m e n t u m before the deformation
starts ( a t t ) a n d t h e n a g a i n after it ends (at t ). T h e f o l l o w i n g e x a m p l e
t t

illustrates t h e procedure.

EXAMPLE 7 . 1 1

The b e n t b a r , of Example 4.16 is d r o p p e d from a height H a n d strikes a rigid,


s m o o t h surface o n o n e e n d of as s h o w n in Figure E 7 . l l . If the coefficient of
restitution is e, find the angular velocity of as well as t h e velocity of C, just after
the collision.
Solution
Using t h e y c o m p o n e n t equation of (7.63) yields

(1)
in w h i c h t h e impulse of t h e gravity force is neglected as small in comparison w i t h
the impulsive u p w a r d force exerted by the surface over the short time interval
Next w e write the c o m p o n e n t equations of (7.64); w e first n e e d the inertia
properties of t h e body, w h i c h can be c o m p u t e d to be

(2)
Figure E7.11
Page 5 0 1

We t h e n obtain, from Equation (7.64),

(3)

in w h i c h t h e initial angular velocity c o m p o n e n t s vanish a n d t h e desired final


c o m p o n e n t s are T h e c o m p o n e n t equations of (3) are

(4)

(5)

(6)

At this point w e h a v e four equations in t h e five u n k n o w n s


a n d t h e impulse . W e get a fifth equation from t h e definition of t h e coeffi­
cient of restitution together w i t h t h e y c o m p o n e n t of t h e rigid-body velocity
relationship b e t w e e n P a n d C:

(7)

and

(8)
w h i c h h a s t h e y - c o m p o n e n t equation

(9)
Using Equation (7), w e obtain

(10)

T h e solution to t h e five equations ( 1 , 4, 5, 6, 10) is

(11)

Returning to Equation (8), w e find t h a t t h e x a n d z c o m p o n e n t s of v , vanish:


c

(12)

T h e results in Equations (12) are obvious, since if t h e r e is n o friction at t h e point of


Page 502

contact t h e r e can b e n o impulsive forces in t h e horizontal p l a n e to c h a n g e the


m o m e n t u m (from zero) in t h e x or z directions.
It is seen that t h e single n o n z e r o p r o d u c t of inertia causes a coupling
between and (see Equations (4) a n d (6)), w h i c h prevents from v a n ­
ishing—even t h o u g h t h e only m o m e n t c o m p o n e n t w i t h respect to C is about
the x axis!

W e s h a l l n o w s e e w i t h a n o t h e r e x a m p l e t h e a d v a n t a g e s of E q u a t i o n
(7.66), w h i c h m a y b e u s e d to eliminate u n d e s i r e d forces f r o m m o m e n t
e q u a t i o n s , j u s t a s w a s d o n e i n o u r s t u d y of s t a t i c s .

EXAMPLE 7 . 1 2

Rework the preceding example by using Equation (7.66) instead of the combina­
tion of Equations (7.63) a n d (7.64). Find t h e value of after impact.
Solution
Equation (7.66) allows u s to eliminate t h e impulse by summing moments
about the point (P') of impact:

We still h a v e to u s e t h e coefficient of restitution a n d relate v a n d v exactly as


P c

before; m a k i n g this substitution for , leads to the following three scalar c o m p o ­


n e n t equations:

These equations, of course, h a v e the s a m e solution as in the preced­


ing example.

PROBLEMS • Section 7.7

7.89 Bend a coat h a n g e r or pipe cleaner into the s h a p e of locity direction following impact agrees w i t h t h e results of
the b e n t bar of Example 7.11. E>rop it o n t o t h e edge of a the example.
table as in t h e example a n d observe t h a t t h e angular v e ­
Page 503

• 7 . 9 0 T h e equilateral triangular d i n n e r bell in Fig­ • 7.94 A diver D leaves a diving b o a r d in a straight, s y m ­


u r e P7.90 is struck w i t h a horizontal force in t h e y direc­ metric position w i t h angular velocity a n d a n g u l a r m o ­
tion that imparts a n impulse to t h e bell. Find the m e n t u m vectors each in t h e x direction as indicated in
angular velocity of t h e bell immediately after t h e b l o w is Figure P7.94a. Since is zero, there will b e n o change
struck. Is t h e a n s w e r t h e s a m e if t h e bell is a n equilateral in t h e angular m o m e n t u m H e in t h e inertial frame (the
triangular plate of t h e s a m e mass? W h y or w h y not? s w i m m i n g pool) as long as t h e diver is in t h e air. T h e r e ­
• 7 . 9 1 In t h e preceding problem, s u p p o s e t h e h a m m e r is fore, as long as h e r e m a i n s in t h e straight position, his
replaced b y a bullet of m a s s m a n d s p e e d v that r e b o u n d s constant angular m o m e n t u m is expressed b y
b

straight back w i t h a coefficient of restitution e = 0 . 1 . De­


termine t h e resulting angular velocity of the bell. (1)
7.92 Repeat P r o b l e m 7.90, b u t this time s u p p o s e t h e bell
h a n g s from a string instead of from a ball a n d socket joint. (2)
7.93 T h e b e n t b a r of Figure P7.93 h a s t h e inertia p r o p ­
erties listed b e l o w . It is i n m o t i o n i n a n inertial frame (3)
a n d at a certain instant h a s angular velocity
. Use t h e angular impulse a n d angular w h e r e w e a s s u m e t h e b o d y to b e sufficiently internally
m o m e n t u m principle to a n s w e r t h e following question: symmetric so t h a t t h e p r o d u c t s of inertia all vanish. N o w
Is it possible to strike at point Q w i t h a n impulse s u p p o s e t h e diver instantaneously m o v e s his a r m s as
that reduces to zero s h o w n in Figure P 7 . 9 4 b to initiate a twist. Following t h e
after t h e impulse? If so, find t h e c o m p o n e n t s of t h e m a n e u v e r , h e m a y again b e treated as a rigid b o d y a n d w e
impulse in terms of m, and . If not, s h o w w h y m a y u s e t h e s a m e body-fixed axes as before. (Note that
not. t h e m a s s center c h a n g e s very little.)
a. From Figure P7.94c a r g u e t h a t t h e indicated
changes in the products of inertia occur. (Only
the s h a d e d a r m s contribute to t h e p r o d u c t s of
C C
inertia.) A r g u e also t h a t I is smaller t h a n I
yz xz

a n d also less t h a n , O b s e r v e t h a t all three

Figure P7.90 Figure P7.94a

Figure P7.93 Figure P7.94b


Page 5 0 4

C
C I <0
C
I >0yz
I >0xz
xy

Figure P7.94c

b. After t h e r a p i d twist m a n e u v e r , b u t before t h e


p r o d u c t s of inertia are small c o m p a r e d w i t h t h e
C C diver begins to twist, his axes are still instan­
three m o m e n t s of inertia a n d that I < I yy xx

C C taneously aligned w i t h those of t h e frame


< I , with I , b e i n g very m u c h smaller t h a n t h e
zz yy

Use Equations (1) to (3), w i t h t h e right sides


other t w o m o m e n t s of inertia. N o t e that
a n d t h e n o w n o n z e r o p r o d u c t s of
(x, y, z) are n o longer principal, b u t this d o e s
inertia, t o s h o w that:
n o t m a t t e r since w e are n o t m a k i n g u s e of prin­
cipal axes h e r e . i. There will b e a small (compared t o t h e
original ) angular velocity d e v e l o p e d
As t h e diver's b o d y begins to twist a n d
a b o u t t h e — Z direction (negative
C ).
turn, t h e right sides of Equations (1) to (3) will
ii. There will b e a n angular velocity of twist
change a n d none of t h e quantities o n t h e left
d e v e l o p e d a b o u t Y (positive ).
will r e m a i n zero. But t h e right sides will consti­ C

tute t h e c o m p o n e n t s in the body frame D of t h e iii. There will b e a n increase in t h e somer­


vector H , w h i c h will still vectorially a d d to saulting angular velocity c o m p o n e n t
c

w h e r e is t h e original direction in of H c In arguing statements (i) to (iii), a s s u m e n o t h i n g a b o u t t h e


after t h e diver leaves t h e diving b o a r d (to t h e following t h e m a n e u v e r except that is still in t h e
right in t h e first sketch). s a m e direction as before.

7.8 Work and Kinetic Energy


A s p e c i a l i n t e g r a l of t h e e q u a t i o n s of m o t i o n of a r i g i d b o d y yields a
r e l a t i o n s h i p b e t w e e n t h e w o r k of t h e e x t e r n a l f o r c e s ( a n d / o r c o u p l e s )
a n d t h e c h a n g e i n t h e k i n e t i c e n e r g y of To d e v e l o p this relation, w e
m u s t first e x p l o r e e x p r e s s i o n s f o r t h e k i n e t i c e n e r g y of t h e r i g i d b o d y .
Kinetic e n e r g y i s u s u a l l y d e n o t e d b y t h e l e t t e r T a n d i s d e f i n e d b y ( s e e
Section 5.2)

(7.67)

i n w h i c h v i s t h e d e r i v a t i v e of t h e p o s i t i o n v e c t o r f r o m O (fixed p o i n t i n
the inertial frame i n F i g u r e 7 . 1 7 ) t o t h e d i f f e r e n t i a l m a s s e l e m e n t dm. I n
this section all t i m e derivatives, velocities, a n d a n g u l a r velocities a r e
taken in u n l e s s o t h e r w i s e specified.
Since i s a r i g i d b o d y , w e m a y r e l a t e v t o t h e v e l o c i t y v of t h e m a s s
c

c e n t e r C of :
Figure 7.17 (7.68)
Page 505

in w h i c h is a n d r is t h e p o s i t i o n v e c t o r f r o m C t o dm a s s h o w n i n
Figure 7.17. Substituting E q u a t i o n (7.68) into (7.67), w e get

(7.69)

where v and c do not vary over the body's volume a n d can thus be
t a k e n o u t s i d e t h e i n t e g r a l s . T h e i n t e g r a l i n t h e l a s t t e r m i s z e r o b y v i r t u e of
t h e d e f i n i t i o n of t h e m a s s c e n t e r :

(7.70)

T h e i n t e g r a l i n t h e first t e r m o n t h e r i g h t s i d e of E q u a t i o n ( 7 . 6 9 ) is of
c o u r s e t h e m a s s m of T h e i n t e g r a n d of t h e r e m a i n i n g t e r m m a y b e
simplified b y t h e vector identity:*

(7.71)

Therefore E q u a t i o n (7.69) b e c o m e s

(7.72)

A s w e h a v e a l r e a d y s e e n i n S e c t i o n 7.2, t h e i n t e g r a l i n E q u a t i o n
( 7 . 7 2 ) is t h e a n g u l a r m o m e n t u m ( m o m e n t of m o m e n t u m ) of t h e b o d y
w i t h r e s p e c t t o C, a n d t h u s w e c a n w r i t e

(7.73)

It is s e e n t h a t t h e k i n e t i c e n e r g y c a n b e r e p r e s e n t e d a s t h e s u m of t w o
terms:

1. A part t h a t t h e b o d y p o s s e s s e s if its m a s s c e n t e r is
in motion.
2. A part t h a t is d u e t o t h e d i f f e r e n c e b e t w e e n t h e v e l o c i ­
t i e s of t h e p o i n t s of a n d t h e v e l o c i t y of i t s m a s s c e n t e r .

The term c a n b e i n t e r p r e t e d q u i t e s i m p l y if a t a n i n s t a n t w e let


t h a t is, if w e a l i g n t h e r e f e r e n c e axis x w i t h t h e a n g u l a r v e l o c i t y
vector at t h a t i n s t a n t . In this case, u s i n g E q u a t i o n s (7.11), w e o b t a i n

(7.74)

* Which is nothing more than interchanging the dot and cross of the scalar triple prod­
uct where E is the vector C X D.
Page 506

so that

(7.75)

T h i s m e a n s t h a t t h e " r o t a t i o n a l p a r t " of T is instantaneously of t h e s a m e


f o r m a s it w a s f o r t h e p l a n e c a s e i n C h a p t e r 4 . T h e d i f f e r e n c e , of c o u r s e , is
t h a t t h e d i r e c t i o n of t h e a n g u l a r v e l o c i t y v e c t o r c h a n g e s in t h e general
(tluee-dimensional) case.
Suppose the body h a s a p o i n t P w i t h z e r o v e l o c i t y . ( T h i s is n o t
a l w a y s t h e c a s e i n g e n e r a l m o t i o n a s w e h a v e a l r e a d y s e e n i n C h a p t e r 6.)
T h e n if v i n E q u a t i o n ( 7 . 6 7 ) is r e p l a c e d b y , where
r ' e x t e n d s f r o m P t o t h e m a s s e l e m e n t dm, w e o b t a i n

(7.76)

T h e i d e n t i c a l s t e p s t h a t p r o d u c e d t h e s e c o n d t e r m of E q u a t i o n ( 7 . 7 3 )
f r o m t h e m i d d l e t e r m of ( 7 . 6 9 ) t h e n g i v e

(7.77)

a n d t h e t w o t e r m s of E q u a t i o n ( 7 . 7 3 ) h a v e c o l l a p s e d i n t o o n e if H is
e x p r e s s e d r e l a t i v e t o a p o i n t of z e r o v e l o c i t y i n s t e a d of C.
I n S e c t i o n s 2 . 4 a n d 5.2 w e d e m o n s t r a t e d o n e w o r k a n d k i n e t i c e n ­
e r g y p r i n c i p l e t h a t r e m a i n s t r u e for t h e g e n e r a l c a s e . T h i s r e s u l t c a m e
from integrating

(7.78)

A s e c o n d principle will n o w b e d e d u c e d from t h e m o m e n t equation*

(7.79)

b u t first w e n e e d t o p r o v e t h e n o n - o b v i o u s r e s u l t t h a t :

T o d o t h i s , w e first r e c a l l t h a t

(7.80)

If is t h e d e r i v a t i v e of H t a k e n i n t h e b o d y
c , then the derivative
relative to t h e inertial frame can b e written

(7.81)

* Derivatives such as are taken in the inertial frame in this section unless the letter
appears to the left of the dot, in which case the derivative is taken in the body.
Page 507

Dotting w i t h b o t h s i d e s of E q u a t i o n ( 7 . 8 1 ) s h o w s t h a t

(7.82)

a n d s i n c e r is c o n s t a n t i n t i m e r e l a t i v e t o b o d y w e c a n differentiate
E q u a t i o n (7.80) there a n d o b t a i n

(7.83)

I n E q u a t i o n ( 7 . 8 3 ) w e h a v e u s e d t h e p r o p e r t y of t h a t its d e r i v a t i v e s i n
and a r e t h e s a m e ; t h a t is,

S u b s t i t u t i n g E q u a t i o n (7.83) i n t o (7.82) t h e n gives

Hence

(7.84)

W e are n o w in a position to observe that

(7.85)

Integrating E q u a t i o n (7.85), w e h a v e

(7.86)

N o t e t h a t t h e right s i d e s of E q u a t i o n s ( 7 . 7 8 ) a n d ( 7 . 8 6 ) e a c h r e p r e s e n t s
the change, occurring in the time interval , of p a r t of t h e kinetic
e n e r g y of t h e b o d y . T h e left s i d e s of t h e s e e q u a t i o n s a r e u s u a l l y c a l l e d a
f o r m of work.
While the relationships b e t w e e n w o r k a n d kinetic energy that h a v e
been developed are important, another relationship that combines t h e m
is o f t e n m o r e u s e f u l . W e c a n d i f f e r e n t i a t e E q u a t i o n ( 7 . 7 3 ) a n d g e t

Using Euler's l a w s a n d E q u a t i o n (7.84), this m a y b e p u t into t h e f o r m

(7.87)
Page 508

If w e n o w l e t F ! , F 2 , . . represent t h e external forces acting o n t h e b o d y ,


a n d C , C , . . . r e p r e s e n t t h e m o m e n t s of t h e e x t e r n a l c o u p l e s , t h e n
1 2

(7.88a)

(7.88b)

w h e r e P , P , . . . a r e t h e p o i n t s of w h e r e F , F , . . . a r e r e s p e c t i v e l y
t 2 1 2

applied a n d where r = r , r = r , a n d s o forth, a s s h o w n i n Fig­


1 CP1 2 CP2

u r e 7 . 1 8 . Recall from statics t h a t a c o u p l e h a s t h e s a m e m o m e n t a b o u t


a n y point in space, so that t h e C 's are simply a d d e d into t h e m o m e n t
i

equation (7.88b).
S u b s t i m t i n g E q u a t i o n (7.88) into (7.87), w e o b t a i n

Figure 7.18

(7.89)
However,

so that

(7.90)

W e note that is t h e v e l o c i t y v of p o i n t P ,
1 l t h e p o i n t of
a p p l i c a t i o n of F . T h e r e f o r e
1

(7.91)

E q u a t i o n ( 7 . 9 1 ) l e a d s u s t o d e f i n e t h e p o w e r , o r rate of work, a s f o l l o w s :

(7.92)

Therefore

Integrating E q u a t i o n (7.91), w e get

(7.93)

T h e i n t e g r a l o n t h e left s i d e of E q u a t i o n ( 7 . 9 3 ) is c a l l e d t h e w o r k d o n e o n
b e t w e e n t a n d t b y t h e external forces a n d couples. H e n c e
1 2

(7.94)

T h a t is, t h e w o r k d o n e o n e q u a l s i t s c h a n g e i n k i n e t i c e n e r g y . It is
left a s a n e x e r c i s e f o r t h e r e a d e r t o s h o w t h a t E q u a t i o n ( 7 . 9 4 ) is i n fact
t h e s u m of t h e t w o " s u b e q u a t i o n s " ( 7 . 7 8 ) a n d ( 7 . 8 6 ) .
Page 509

EXAMPLE 7 . 1 3

Find t h e w o r k d o n e on the b e n t bar of Example 7.11 b y a m o t o r that brings it u p to


speed from rest. (See Figure E7.13.)
Solution
The mass center C does n o t m o v e , so that Equation (7.73) gives, in this case,

(1)

Since h a s only a component, w e m a y substitute Equation (7.11) into (1)


Figure E7.13 a n d get

(2)

C
W e n o t e t h a t e v e n t h o u g h I is n o t zero, it h a s n o effect o n t h e kinetic energy of
xz

since it is multiplied by w h i c h is forced to vanish by the bearings aligned


with z.
T h u s the w o r k d o n e b y the motor o n is given simply by Equation (7.94):

(3)

when from Example 7.11. The motor w o u l d , of course, h a v e to


do additional w o r k besides that given by (3) to overcome its o w n a r m a t u r e
inertia, bearing a n d belt friction, a n d air resistance.

W e n o w consider a n e x a m p l e in three dimensions in w h i c h the


p r o d u c t s of i n e r t i a d o p l a y a r o l e i n t h e k i n e t i c e n e r g y c a l c u l a t i o n .

EXAMPLE 7 . 1 4

Find t h e kinetic energy lost b y the b e n t bar of Example 7.10 w h e n it strikes the
table t o p as s h o w n in Figure E7.14a.
Solution
During the impact w i t h the table top, t h e bodies d o n o t b e h a v e rigidly. The
kinetic energy lost b y b a r is transformed into noise, heat, vibration, a n d b o t h
elastic a n d p e r m a n e n t deformation. In Example 7.11 w e found v a n d just c

before a n d after impact; w e n o w u s e these vectors to find t h e kinetic energy lost


by Just after impact w e h a v e

Figure E7.14a
Page 510

The term can b e written just after impact, using Equation (7.11), as
follows. (Note t h a t a n d t w o of the products of inertia are zero here.)

Using this result a n d Equations (11) a n d (12) from Example 7.14, w e obtain

which, after simplification, equals

The initial kinetic energy (just prior to t h e collision) w a s

Thus the change in kinetic energy of t h e b e n t bar is given by

W e see that if e = 1 (purely elastic collision), n o loss in kinetic energy occurs a n d


h e n c e n o w o r k is d o n e in changing T. T h e energy lost is seen in Figure E7.14b to
vary quadratically, w i t h a m a x i m u m percentage loss ( w h e n e = 0) of

in this case. Because t h e point of striking is the e n d of the bar, 83.9 percent of the
kinetic energy is retained. If t h e m a s s center of t h e b a r w e r e t h e p o i n t t h a t struck
Figure E7.14b the table, h o w e v e r , all t h e kinetic energy w o u l d h a v e b e e n lost if e = 0.

PROBLEMS • Section7.8

7.95 Find the kinetic energy of disk in P r o b l e m 6.27.


7.96 The center of m a s s C of a gyroscope G is fixed. where are the Eulerian angles a n d A, A, C are the
S h o w that the kinetic energy of G is principal m o m e n t s of inertia of G at C.
Page 511

7.97 Find the kinetic energy of the w a g o n wheel in • 7.100 The rigid body in Figure P7.100 consists of a disk D
Problem 6.49 and use it to deduce the work done by the and rod R, welded together perpendicularly as s h o w n in
boy in getting it up to its final speed from rest. the figure. If the body is spun up to angular speed
about the z axis, h o w much work was done o n it (exclud­
7.98 A disk D of mass 10 kg and radius 25 cm is welded
ing the overcoming of frictional resistance)?
at a 45° angle to a vertical shaft S. (See Figure P7.98.) The
shaft is then spun up from rest to a constant angular speed

a. H o w much work is done in bringing the assem­


bly up to speed?
b. Find the force and couple system acting on the
plate at C after it is turning at the constant
speed Figure P7.100

* 7.101 Figure P7.101 shows a thin homogeneous triangu­


lar plate of mass m, base a, and height 2a. It is welded to a
light axle that can turn freely in bearings at A and B.
Given:

Figure P7.98
a. If the plate is turning at constant angular speed
find the torque that must be applied to the
axle, and find the dynamic bearing reactions.
b. Find the principal axes at A and the principal
moments of inertia there. Draw the axes o n a
sketch.
c. If possible, give the radius of a hole that, w h e n
7.99 A thin rectangular plate (Figure P7.99) is brought drilled at C, will eliminate the bearing reactions,
up from rest to speed about a horizontal axis Y. Give the answer in terms of m and pt (density
times thickness) of the plate.
a. Find the work that is done.
d. Find the work done in bringing the plate up to
b. If two concentrated masses of m/2 each are
speed from rest.
added on the x axis, one on each side of the
c

mass center, find their distances d from the


mass center that will eliminate the bearing reac­
tions.

Bearing

Figure P7.99 Figure P7.101


Page 512

• 7.102 A thin equilateral triangular plate P of side s is • 7.105 A ring is w e l d e d to a r o d at a point A as s h o w n in


w e l d e d to t h e vertical shaft at A in Figure P7.102. The Figure P7.105. T h e cross sections a n d densities of t h e rod
shaft is b r o u g h t u p to speed from rest by a motor. a n d ring are the s a m e . The c o m b i n e d b o d y is released
w i t h a gentle n u d g e w i t h e n d B of the r o d connected to the
a. H o w m u c h w o r k is d o n e in bringing the system
s m o o t h p l a n e b y a ball joint a n d w i t h point A at its highest
u p to speed?
point as s h o w n . At the instant w h e n A reaches its lowest
b. Find the force a n d couple system acting o n the point, find the relationship b e t w e e n the horizontal a n d
plate at A after it is t u r n i n g at t h e speed and
vertical angular velocity c o m p o n e n t s of the body.
the motor is t u r n e d off.
7.106 If in the preceding p r o b l e m t h e p l a n e is r o u g h
e n o u g h to p r e v e n t slipping, find the m a g n i t u d e of the
angular velocity w h e n A reaches the floor.

Figure P7.102

7.103 T w o concentrated masses m = 10 kg a n d m = 20


1 2

kg are connected by a 15-kg slender rod m of length 3


Figure P7.103
1.5 m . As s h o w n in Figure P 7 . 1 0 3 , are unit vectors
fixed in direction in t h e inertial frame a n d are
parallel to principal axes fixed at C in t h e c o m b i n e d body.
At t w o times t a n d t , the velocities of C a n d t h e angular
1 2

velocities of the combined b o d y are

Find t h e total w o r k d o n e o n t h e system b e t w e e n t a n d t .


1 2

7.104 Find t h e kinetic energy of t h e grinder in Problem


7.62. Is this equal to the w o r k d o n e b y a m o t o r on S w h i c h
brings the system u p to speed? (Neglect t h e masses of S
and ) Figure P7.105

COMPUTER PROBLEM • Chapter 7

• 7 . 1 0 7 Use a c o m p u t e r to generate data for a plot of maxi­


m u m values of versus in Example 7.10, for
Page 513

SUMMARY • Chapter 7
I n t h i s c h a p t e r w e h a v e d e v e l o p e d e x p r e s s i o n s f o r a n g u l a r m o m e n t u m of
a rigid b o d y in general t h r e e - d i m e n s i o n a l m o t i o n . W i t h respect to t h e
m a s s c e n t e r , it is

A n d if P is t h e l o c a t i o n of a p o i n t of t h e b o d y w i t h z e r o v e l o c i t y ,

T r a n s f o r m a t i o n p r o p e r t i e s of m o m e n t s a n d p r o d u c t s of i n e r t i a i n ­
clude the parallel-axis t h e o r e m s

t o g e t h e r w i t h f o r m u l a s for o b t a i n i n g t h e m o m e n t s a n d p r o d u c t s of i n e r ­
tia a s s o c i a t e d w i t h a x e s t h r o u g h a p o i n t w h e n t h o s e p r o p e r t i e s a r e k n o w n
for o t h e r a x e s t h r o u g h t h e p o i n t :

In these t w o equations a n d n , n , n are r e s p e c t i v e l y t h e d i r e c t i o n


x y 2 ,

c o s i n e s of x' a n d y' r e l a t i v e t o a x e s x, y, a n d z.
P r i n c i p a l a x e s of i n e r t i a a r e v e r y i m p o r t a n t a n d h a v e t h e k e y p r o p ­
erty t h a t w e r e a b o d y to r o t a t e a b o u t a principal axis at a p o i n t P , t h e n t h e
a n g u l a r m o m e n t u m w i t h respect t o P w o u l d b e in t h e s a m e direction as
the a n g u l a r velocity, or

w h e r e J is t h e m o m e n t of i n e r t i a a b o u t t h e p r i n c i p a l a x i s , a n d is c a l l e d a
p r i n c i p a l m o m e n t of i n e r t i a .
A l l p r o d u c t s of i n e r t i a a s s o c i a t e d w i t h a p r i n c i p a l axis v a n i s h , a n d a t
a n y point there are three mutually perpendicular principal axes. The
l a r g e s t a n d s m a l l e s t of t h e p r i n c i p a l m o m e n t s of i n e r t i a a r e t h e l a r g e s t
a n d s m a l l e s t of all t h e m o m e n t s of i n e r t i a a b o u t a x e s t h r o u g h t h e p o i n t .
Page 514

S o m e i m p o r t a n t special cases are:

1. If P lies i n a p l a n e of s y m m e t r y of t h e b o d y , t h e n t h e axis t h r o u g h P
a n d p e r p e n d i c u l a r t o t h e p l a n e is a p r i n c i p a l a x i s .
2. If P lies o n a n a x i s of s y m m e t r y of t h e b o d y , t h e n t h a t a x i s a n d e v e r y
l i n e t h r o u g h P a n d p e r p e n d i c u l a r t o it is a p r i n c i p a l a x i s . F u r t h e r ­
m o r e , t h e m o m e n t s of i n e r t i a a b o u t t h e s e t r a n s v e r s e a x e s t h r o u g h a
g i v e n p o i n t a r e all t h e s a m e .
3. If P is a p o i n t of s p h e r i c a l s y m m e t r y , e.g., t h e c e n t e r of a u n i f o r m
s p h e r e , t h e n e v e r y l i n e t h r o u g h P is a p r i n c i p a l axis a n d a l l of t h e
c o r r e s p o n d i n g p r i n c i p a l m o m e n t s of i n e r t i a a r e e q u a l .

T h e m o s t c o n v e n i e n t f o r m of E u l e r ' s s e c o n d l a w , to use in a
p a r t i c u l a r p r o b l e m is o f t e n d e p e n d e n t o n t h e p r o b l e m . W h e n b o d y - f i x e d
principal axes a r e u s e d for reference, t h e n w e h a v e w h a t a r e u s u a l l y
referred to as t h e Euler equations:

H o w e v e r , it is v e r y o f t e n m o r e c o n v e n i e n t t o e x p r e s s t h e a n g u l a r m o ­
m e n t u m i n t e r m s of i t s c o m p o n e n t s p a r a l l e l t o r e f e r e n c e a x e s a s s o c i a t e d
w i t h s o m e i n t e r m e d i a t e f r a m e of r e f e r e n c e , s a y f, w h i c h is n e i t h e r t h e
b o d y itself n o r t h e i n e r t i a l f r a m e so that

J u s t a s i n t h e c a s e of p l a n e m o t i o n ( C h a p t e r 5 ) , t h e w o r k of e x t e r n a l
f o r c e s e q u a l s t h e c h a n g e i n k i n e t i c e n e r g y f o r rigid b o d i e s i n g e n e r a l
m o t i o n . I n t h r e e - d i m e n s i o n a l m o t i o n t h e k i n e t i c e n e r g y , T, c a n b e w r i t t e n
in g e n e r a l as

T h e s e c o n d t e r m m a y b e c o m p a c t l y w r i t t e n as

w h e r e I is t h e m o m e n t of i n e r t i a a b o u t t h e a x i s , t h r o u g h C, t h a t is i n s t a n ­
taneously aligned with

REVIEW QUESTIONS • Chapter 7

True or False?
1. P r o d u c t s of i n e r t i a a s s o c i a t e d w i t h p r i n c i p a l a x e s a l w a y s v a n i s h , b u t
only at t h e mass center.
2. If t h e p r i n c i p a l m o m e n t s of i n e r t i a a t a p o i n t a r e d i s t i n c t , t h e n t h e
p r i n c i p a l a x e s of i n e r t i a a s s o c i a t e d w i t h t h e m a r e o r t h o g o n a l .
Page 515

3. T h e m a x i m u m m o m e n t of i n e r t i a a b o u t a n y l i n e t h r o u g h p o i n t P of
rigid b o d y is t h e l a r g e s t p r i n c i p a l m o m e n t of i n e r t i a a t P .
4. G e n e r a l m o t i o n is a m u c h m o r e difficult s u b j e c t t h a n p l a n e m o t i o n . A
m a j o r r e a s o n f o r t h i s i s t h a t n e i t h e r t h e k i n e m a t i c s n o r k i n e t i c s dif­
f e r e n t i a l e q u a t i o n s g o v e r n i n g t h e o r i e n t a t i o n m o t i o n of t h e b o d y a r e
linear.
5. If w e s o l v e t h e E u l e r e q u a t i o n s ( 7 . 4 0 ) , w e i m m e d i a t e l y k n o w t h e
o r i e n t a t i o n of t h e rigid b o d y i n s p a c e .
6. T h e s u n a n d t h e m o o n e x e r t g r a v i t y t o r q u e s o n t h e e a r t h , a n d t h e y
c a u s e t h e a x i s of o u r p l a n e t t o p r e c e s s .
7. If a t a c e r t a i n i n s t a n t t h e m o m e n t of i n e r t i a of t h e m a s s of b o d y
a b o u t a n axis t h r o u g h C p a r a l l e l t o t h e a n g u l a r v e l o c i t y v e c t o r is I,
t h e n t h e k i n e t i c e n e r g y of at t h a t i n s t a n t is
8. T h e e a r t h ' s l u n i s o l a r p r e c e s s i o n is t h e r e s u l t of both t h e b u l g e a t t h e
e q u a t o r and t h e tilt of t h e a x i s .
9. T h e k i n e t i c e n e r g y l o s t d u r i n g a c o l l i s i o n of t w o b o d i e s d o e s n o t
d e p e n d o n t h e a n g u l a r v e l o c i t i e s of t h e b o d i e s p r i o r t o i m p a c t .
10. T h e w o r k - e n e r g y a n d i m p u l s e - m o m e n t u m p r i n c i p l e s a r e g e n e r a l
i n t e g r a l s of t h e e q u a t i o n s of m o t i o n for a rigid b o d y .
11. S o m e t i m e s it is b e t t e r t o u s e t h e p r o d u c t s of i n e r t i a i n
t h a n t o t a k e t h e t i m e t o c o m p u t e p r i n c i p a l m o m e n t s a n d a x e s of
inertia so as to b e able to utilize Euler's e q u a t i o n s (7.40).
12. I n s t e a d y p r e c e s s i o n w i t h t h e n u t a t i o n a n g l e e q u a l i n g 9 0 ° , t h e s p i n
vector always precesses a w a y from the torque vector.

Aimran: 1. F 2. T 3. T 4. T 5. F 6. T 7. T 8. T 9. F 10. T 11. T 12. F


8 SPECIAL TOPICS

8.1 Introduction
8.2 Introduction to Vibrations
Free Vibration
Damped Vibration
Forced Vibration
8.3 Euler's Laws for a Control Volume
8.4 Central Force Motion
R E V I E W QUESTIONS

Page 5 1 6
Page 5 1 7

8.1 Introduction

In this chapter, we examine three subjects which are of considerable


practical importance in Dynamics. In the first of these special topics, we
introduce the reader to the subject of vibrations, limiting the presentation
to a single degree of freedom (in which the oscillatory motion can be
described by just one coordinate).
The second special topic deals with problems in which mass is con­
tinuously leaving and / or entering a region of space known as a control
volume. A rocket is a good example: as the fuel is burned and the com­
bustion products are ejected from a control volume enveloping the
rocket, its momentum changes and it is propelled through the atmo­
sphere. Euler's laws still apply, though the resulting equations are a bit
more complicated than they were for the "constant mass" particles and
bodies of earlier chapters.
The final topic in the chapter is central force motion, the most com­
mon example of which is that of orbits—such as the motions of planets
around the sun, and of the moon and of man-made satellites around the
earth.
The topics of Sections 8.2, 3, and 4 all stand alone, and can be read
and understood after the reader has mastered Chapters 1 and 2, except
for some of the problems in Section 8.2 in which the moment equation for
rigid bodies in plane motion from Chapter 4 is also needed.

8.2 Introduction to Vibrations

Vibration is a term used to describe oscillatory motions of a body or


system of bodies. These motions may be caused by isolated disturbances
as when the wheel of an automobile strikes a bump or by fluctuating
forces as in the case of the fuselage panels in an airplane vibrating in
response to engine noise. Similarly, the oscillatory ground motions re­
sulting from an earthquake cause vibrations of buildings. In each of these
cases the undesirable motion may cause discomfort to occupants; more­
over, the oscillating stresses induced within the body may lead to a
fatigue failure of the structure, vehicle, or machine.
Free Vibration
For perhaps the simplest example of a mechanical oscillator consider the
Figure 8.1
rigid block and linear spring shown in Figure 8.1. The block is constrained
to translate vertically; thus a single parameter (scalar) is sufficient to
establish position and hence the system is called a single-degree-of-
freedom system. We choose z to be the parameter and let z = 0 corre­
spond to the configuration in which the spring is neither stretched nor
compressed.
Using a free-body diagram of the block in an arbitrary position (Fig­
ure 8.2), Euler's first law yields
Figure 8.2 mz = mg — kz
Page 518

or

mz + kz = mg (8.1)

which is a second-order linear differential equation with constant coeffi­


cients describing the motion of the block. The fact that the differential
equation is nonhomogeneous (the right-hand side is not zero) is a conse­
quence of our choice of datum for the displacement parameter z. For if we
make the substitution y = z — m g / k , the governing equation (8.1) be­
comes

my + ky = 0 (8.2)

which is a homogeneous differential equation. It is not a coincidence that


this occurs when the displacement variable is chosen so that it vanishes
when the block is in the equilibrium configuration—that is, when the
spring is compressed mg / k.
Motion described by an equation such as (8.2) is called a free vibra­
tion since there is no external force (external, that is, to the spring-mass
system) stimulating it.
Rewriting Equation (8.2), we obtain

or, defining w „
n

(8.3)

which has as its general solution


(8.4)
or

(8.5)
where

Whether expressed in the form of (8.4) or (8.5), y is called a simple


harmonic function of time, w is called the natural circular frequency, C
n

is called the amplitude of the displacement y, and (p is said to be the


phase angle by which y leads the reference function, sin co„t. The simple
harmonic function is periodic and its period is r„ = 2n / w . Another
n

=
quantity called frequency is f = 1 / T W / 2n, which gives the num­
n n n

ber of cycles in a unit of time. When the unit of time is the second, the unit
for f is the hertz (Hz); 1 Hz is 1 cycle per second.
n

The constants A and B in (8.4), or equivalently C and (p in (8.5), are


determined from initial conditions of position and velocity. Thus if
Page 519

and

then

and

Now let us investigate what might seem an entirely different situa­


tion—that of a rigid body constrained to rotate about a fixed horizontal
axis (through O as in Figure 8.3). Since the only kinematic freedom the
body has is that of rotation, a single angle is sufficient to describe a
configuration of the body. Let the angle be 9 as shown, where we note
that when the mass center C is located directly below the pivot O.
Neglecting any friction at the axis of rotation, the free-body diagram
appropriate to an arbitrary instant during the motion is shown in Figure
8.4. Summing moments about the axis of rotation, we get
Figure 8.3
(8.6)

where I is the mass moment of inertia about the axis of rotation. Equa­
0

tion (8.6) is a nonlinear differential equation because sin 9 is a nonlinear


function of 9, but if we restrict our attention to sufficiently small angles so
that sin Equation (8.6) becomes

(8.7)

That is, is a simple harmonic function:


Figure 8.4

where now

The two preceding examples have an important feature in common:


Motion near the equilibrium configuration is governed by a homoge­
neous, second-order, linear differential equation with constant coeffi­
cients, and in each case the motion is simple harmonic. A point of differ­
ence is that in the block-spring case the gravitational field plays no role
other than establishing the equilibrium configuration; in particular the
natural frequency does not depend on the strength (g) of the field. In the
second case where the body is basically behaving as a pendulum, the
gravitational field provides the "restoring action" and the natural fre­
quency is proportional to
Page 520

EXAMPLE 8 . 1

Find the natural frequency of small oscillations about the equilibrium position of
a uniform ball (sphere) rolling on a cylindrical surface.
Solution
Let m be the mass of the ball, let R be the radius of the path of its center, and let
be the polar coordinate angle locating the center as shown in Figure E8.1a. Thus

and the angular acceleration of the ball is a = — because of the no-slip


condition. We shall now use a and a in the equations of motion:
c

(1)

Figure E 8 . 1 a Figure E8.1b

Hence, from the free-body diagram shown in Figure E8.lb, the and compo­
nent equations of (1) are:

(2)
and

(3)

Also, from summing moments about C, we have

or

(4)

Eliminating the friction force F between Equations (2) and (4), we obtain the
differential equation

For small so that sin


Page 521

from which we see that

or

(w = 0.845
n

Damped Vibration
The simple harmonic motion in our examples of free vibration has a
feature that conflicts with our experience in the real world; that is, the
motion calculated persists forever unabated. Intuition would suggest
decaying oscillations and finally the body coming to rest. Of course the
problem here is that we have not incorporated any mechanism for energy
dissipation in the analytical model. To do that, we shall return to the
simple block-spring system and introduce a new element: a viscous
damper (Figure 8.5). The rate of extension of this element is proportional
to the force applied, through a damping constant c, so that the force is c
times the rate of extension.
Referring to the free-body diagram in Figure 8.5 and letting y = 0
designate the e q u i l i i u m position as before, we have
Damper

or
(8.8)
Figure 8.5
The appearance of the cy term in (8.8) has a profound effect on the
solution to the differential equation and hence on the motion being
described. Solutions to (8.8) may be found from
y = Ae n
(8.9)
where A is an arbitrary constant and r is a characteristic parameter.
Substituting (8.9) into (8.8), we obtain
2 rt
(mr + cr + k)Ae = 0 (8.10)
which is satisfied nontrivially (i.e., for A = 0) with
2
mr + cr + k = 0 (8.11)
This characteristic equation has two roots given by

(8.12)
2
Except for the case in which (c / 2m) = k/m, the roots are distinct; if we
call them r and r , then the general solution to (8.8) is
x 2
Page 522

2
In the exceptional case (c / 2m) = k / m, there is only the one re­
peated root r = — c / 2m, but direct substitution will verify that there is a
-(c/2m)t
solution to (8.8) of the form te so that the general solution in that
case is
(8.13)

With initial conditions

and

we find that
A = yo
1

and

Since

the solution is

(8.14)
Displacements given by (8.14) are plotted in Figure 8.6 for several
representative sets of initial conditions (positive y„ but positive and nega­
tive v ). Two features of the motion are apparent:
0

1. y 0 (the equilibrium position) as t


2. The motion is not oscillatory; the equilibrium position is "overshot"
at most once and only then when the initial speed is sufficiently large
and in the direction opposite to that of the initial displacement.
In the case we have just studied, the damping is called critical
damping, because it separates two quite different mathematical solu-

Figure 8.6 Motion of a critically damped system.


Pag523

tions: For greater damping the roots of the characteristic equation (8.11)
are both real and negative, and for small damping the roots are complex
conjugates. If we let the critical damping be denoted by then we have
seen that

(8.15)

Now let us consider the case for which c > c^; the mechanical
system is then said to be overdamped or the damping is said to be
supercritical. In this case the roots given by (8.12) are both real and
2
negative since (c / 2m) > k/m; if we call these roots — a and — a , with
x 2

a > fli > 0, then the general solution to the differential equation of
2

motion is
(8.16)
The motion described here is in no way qualitatively different from that
for the case of critical damping, which we have just discussed. For a given
set of initial conditions, Equation (8.16) yields a slower approach to y = 0
than does (8.13). That is, the overdamped motion is more "sluggish" than
the critically damped motion as we would anticipate because of the
greater damping.
Finally we consider the case in which the system is said to be under-
damped or subcritically damped; that is, c < c^. The roots given by
(8.12) are the complex conjugates

where i = It is possible to express the general solution to the gov­


erning differential equation as
y = (8.17)
where co = d A typical displacement history corre­
sponding to (8.17) is shown in Figure 8.7. We note that, just as in the
preceding cases, y 0 as f ; however, here the motion is oscillatory.
We see that the simple harmonic motion obtained for the model without

Displacement curve

"Envelope"

Figure 8.7 Motion of an underdamped system.


Page 524

damping is given by (8.17) with c = 0. Moreover, we see that with light


damping (small c) the analytical model that does not include damping
adequately describes the motion during the first several oscillations. It is
this case—subcritical damping—that is of greatest practical importance
in studies of vibration.

EXAMPLE 8 . 2
Find the damping constant c that gives critical damping of therigidbar executing
motions near the equilibrium position shown in Figure E8.2a.
Solution
We are going to restrict our attention to small angles 9, and thus we may ignore
any tilting of the damper or the spring. However, it may help us develop the
equation of motion in an orderly way if we assume that the upper ends of the
Figure E 8 . 2 a
spring and damper slide along so that each remains vertical as the bar rotates
through the angle 6. Without further restriction, if we sum moments about A with

(1)

b where is the spring stretch at equilibrium. Thus for small O (that is, sinO
1) we linearize Equation (1) and obtain
Figure E 8 . 2 b

Of course, is the equilibrium configuration so that

The linear governing differential equation is then

and for critical damping we get, associating the coefficients of with those of

or

Any c less than this critical value will result in oscillations of decreasing ampli­
tude.

Forced Vibration
Fluctuating external forces may have destructive effects on mechanical
systems; this is perhaps the primary motivation for studying mechanical
vibration. It is common for the external loading to be a periodic function
of time, in which case the loading may be expressed as a series of simple
Page 525

harmonic functions (Fourier series). Consequently it is instructive to con­


k
sider the case in which the loading is simple-harmonic. For the mass-
m
Psin spring-damper system shown in Figure 8.8, the differential equation of
motion is
Figure 8.9 mx + cx + kx = P sin cot (8.18)

The general solution is composed of two parts: a particular solution


(anything that satisfies the differential equation) and what is called the
complementary solution (the general solution to the homogeneous differ­
ential equation). A particular solution of the form x = X sin(cof — <p) may
be found. If we substitute this expression in (8.18) we obtain

or

or

For this to be satisfied at every instant of time,


(8.19)
and

(8.20)
From (8.20) we get

(8.21)

so that

and

Substituting these expressions for sin and cos into (8.19), we obtain

so that

(8.22)
Page 526

We may now write the complete solution to the differential equation


(8.18):
(8.23)
where x is the complementary solution and is one of the three cases
c

enumerated in the preceding section. That is, the form of x depends on


c

whether the system is overdamped, critically damped, or underdamped.


However, in each of these cases the negative exponent causes the func­
tion to approach zero as time becomes large. Thus for large time x tends
c

to zero and x(t) tends to the particular solution. For this reason the
simple-harmonic particular solution is called the steady-state displace­
ment, since it represents the long-term behavior of the system.
We note that the steady-state motion is a simple harmonic function
having amplitude X and lagging the excitation (force) function by the
phase angle We may put these in a convenient form by dividing
numerator and denominator of (8.21) and (8.22) by k, so that

tan (8.24)

and

(8.25)

Investigating the dimensionless quantity ceo / k, we find

But we know that 2mco„ = c ^ , , the critical damping, so if we let be the


damping ratio ( c / c ^ ) ,

(8.26)

and

tan (8.27)

and

(8.28)

The phase angle and the dimensionless displacement amplitude


kX/P are plotted against the frequency ratio w/w in Figures 8.9 and
n

8.10, respectively, for various values of the damping ratio We see that,
with small damping, large amplitudes of displacement occur when the
excitation frequency to is near the natural frequency w . This phenome­
n

non is called resonance, and the desire to avoid it has led to the develop­
ment of methods for estimating natural frequencies of mechanical sys-
Page 527

Figure 8.9

Figure 8.10

terns. Note that the steady-state response curves are insensitive to the
damping for sufficiently small damping (say < 0.1) provided that we
are not in the near vicinity of w / w„ = 1. This is an important observa­
tion because often in engineering practice we have reason to believe that
the damping is small but we do not have accurate quantitative informa­
tion about it.
We close this section by discussing the usual source of a simple
harmonic external loading—an imbalance in a piece of rotating ma­
chinery. Let the machine be made up of two parts. The first, of mass m , is x

a rigid body constrained to rotate about an axis fixed in the second body
(mass m ), which translates relative to the inertial frame of reference. Let
2

the mass center of the rotating body lie off the axis of rotation a distance e
and let the body rotate at constant angular speed to. (See Figure 8.11(a).)
Referring to the free-body diagram in Figure 8.11(b), we have

If we denote the total mass of the machine by m, then m = m + m and


1 2

Figure 8.11
Page 528

Thus the amplitude of the apparent "external" sinusoidal loading is


m ew
1 2and its frequency is the angular speed of the rotating element.

EXAMPLE 8 . 3

A piece of machinery weighing 200 lb has a rotating element with imbalance (m e


1

and an operating speed of 1200 rpm. There are four springs, each of stiffness
1500 lb/in., supporting the machine whose frame is constrained to translate
vertically. The damping ratio is = 0.3. Find the steady-state displacement of the
frame.

Solution
The effective spring stiffness is

k = 4(1500) = 6000 lb/in.

so that

The effective external force amplitude is

From Equation (8.28) we get

The phase angle is given by

tan

so that = 205 rad (117°).


Page 529

EXAMPLE 8 . 4

The machine of Example 8.3 (weight = 200 lb, imbalance = 5 Ib-in., operating
speed = 1200 rpm) is to be supported by springs with negligible damping. If the
machine were bolted directly to the floor, the amplitude of force transmitted to
the floor would be
2
(m e)W = 206 lb
1

What should the stiffness of the support system be so that the amplitude of the
force transmitted to the floor is less than 20 lb?
Solution
The force exerted on theflooris transmitted through the supporting springs and
is of amplitude kX, where X is the amplitude of displacement of the machine.
From Equation (8.28) we have

or with negligible damping

Thus for

2 2 2 2
it is clear that 1 — w /w is negative. Note that only w h e n w / w > 2 is
n n

Therefore we inquire into the condition for which

or

or

since w= 126 rad/sec. But

Thus to satisfy the given conditions the support stiffness must be less than
725 lb/in.
If the only springs available give a greater stiffness, the problem may be
solved by increasing the mass; particularly we might mount the machine on a
Page 530

block of material, say concrete, and then support the machine and block by
springs. For example, if the only springs available were those of Example 8.3

for which the weight is


(4.29)(386) = 1660 lb
Therefore we need a slab or block weighing
1660 - 200 = 1460 lb

EXAMPLE 8 . 5
Find the steady-state displacement x(t) of the mass in Figure E8.5 if y(t) = 0.1
2
cos 120t inch, where f is in seconds, m = 0.01 lb-sec /in., k = 100 lb/in., and
c = 2 lb-sec/in. In particular: (a) What is the amplitude of x(t)? (b) What is the
angle by which x(t) leads or lags y(t)l
Solution
The differential equation of motion of the mass is seen to be
Figure E 8 . 5

or

where Y = 0.1 in. and co = 120 rad/sec. Using Equation (8.18), we see thatkYis

or

The phase angle is

= 76.9° or 1.34 rad (lagging)


Thus the steady-state motion is
= 0.0406 cos(120t- 1.34) in.
Page 531

PROBLEMS • Section 8.2

8.1 Find the frequency of small vibrations of the round 8.6 A uniform cylinder of mass m and radius R is float­
wheel C as it rolls back and forth on the cylindrical surface ing in water. (See Figure P8.6.) The cylinder has a spring
in Figure P8.1. The radius of gyration of C with respect to of modulus k attached to its top center point. If the specific
the axis through C normal to the plane of thefigureis k . c
weight of the water is y,findthe frequency of the vertical
Verify the result of Example 8.1 with your answer. bobbing motion of the cylinder. Hint: The upward
(buoyant) force on the bottom of the cylinder equals
B . 2 - 8 . 4 Find the equations of motion and periods of vi­
the weight of water displaced at any time (Archimedes'
bration of the systems shown in Figure P8.2 to P8.4. In
each case, neglect the mass of the rigid bar to which the principle).
ball (particle) is attached.
8.5 The cylinder in Figure P8.5 is in equilibrium in the
position shown. For no slipping, find the natural fre­
quency of free vibration about this equilibrium position.

Figure P8.6

Figure P8.1

8.7 It is possible to determine experimentally the mo­


ments of inertia of large objects, such as the rocket shown
in Figure P8.7. If the rocket is turned through a slight
angle about z and released, for example, it oscillates with
c

Figure P 8 . 2 a period of 2.8 sec. Find the radius of gyration k . 2c

Figure P8.3

Figure P8.7

Figure P 8 . 4

8.8 In the preceding problem, when the rocket is.


caused to swing with small angles about axisttas shown,
the period is observed to be 8 sec. Find from this informa­
tion the value of k .
Xc

Figure P8.5 8.9 Prove statements (1) and (2) on page 522.
Page 532

8.10 Find the frequency of small amplitude oscillations 8.14 A sack of cement of mass m is to be dropped on the
of the uniform half-cylinder near the equilibrium position center of a simply supported beam as shown in Figure
shown in Figure P8.10. Assume that the cylinder rolls on P8.14. Assume that the mass of the beam may be ne­
the horizontal plane. glected, so that it may be treated as a simple linear spring
8.11 A particle of mass m is attached to a light, taut of stiffness k. Estimate the maximum deflection at the
center of the beam.
string. The string is under tension, T, sufficiently large
that the string is, for all practical purposes, straight when • 8 . 1 5 A particle Pof mass m moves on a rough, horizontal
the system is in equilibrium as shown in Figure P8.ll. rail with friction coefficient m. (See Figure P8.15.) It is
Find the natural frequency of small transverse oscillations attached to a fixed point on the rail by a linear spring of
of the particle. modulus k. The initial stretch of the spring is 7 figm / k.
• 8.12 The masses in Figure P8.12 are connected by an Describe the subsequent motion if it is known that the
inextensible string. Find the frequency of small oscilla­ particle starts from rest. Show that the mass stops for
tions if mass m is lowered slightly and released. good when Hint: The differential equa­
tion doesn't have quite the form found in the text; also,
* The solid homogeneous cylinder in Figure P 8 . 1 3 every time the particle reverses direction, so does the fric­
weighs 200 lb and rolls on the horizontal plane. When the tion force—thus the equation needs rewriting with each
cylinder is at rest, the springs are each stretched 2 ft. The stop.
modulus of each spring is 15 lb/ft. The mass center C is
given an initial velocity of ft / sec to the right. A spring with modulus 120 lb/in. supports a
200-lb block. (See Figure P8.16.) The block is fastened to
a. How far to the right will C go? the spring. A 400-lb downward force is applied to the top
b. How long will it take to get there? of the block at t = 0 when the block is at rest. Find the
c. How long will it take to go halfway to the ex­ maximum deflection of the spring in the ensuing time.
treme position? • 8 . 1 7 A block weighing 1 lb is dropped from height
H = 0.1 in. (See Figure P8.17.) If k = 2.5 lb/in., find the
time interval for which the ends of the springs are in
contact with the ground.
Mass m

Figure P8.10

Figure P8.14
Figure P 8 . 1 1

Figure P8.15

1
g = 386 in./sec
Figure P 8 . 1 6
Figure P8.12

Figure P8.13 Figure P8.17


533

8.18 Assume that the slender rigid bar Ein Figure P8.18 8.23 If k= 100 lb/in. and the mass of the uniform,
2
undergoes only small angles of rotation. Find the angle of slender, rigid bar in Figure P8.23 is 0.03 lb-sec /in., what
rotation 6{t) if the bar is in equilibrium prior to f = 0, at damping modulus c results in critical damping? Compare
which time the constant force P begins to traverse the bar with the c from Problem 8.22. For this damping, find (t)
at constant speed v. if the bar is turned through a small angle and then
8.19. Refer to the preceding problem: (a) Find the work released from rest. If the dashpot were removed, what
done by P in traversing the bar B; (b) show that this work would be the period of free vibration?
equals the change in mechanical energy (which is the 8.24 A cannon weighing 1200 lb shoots a 100-lb can-
kinetic energy of B plus the potential energy stored in the nonball at a velocity of 600 ft/sec. (See Figure P8.24.) It
spring). then immediately comes into contact with a spring of
' 8.20 The turntable in Figure P8.20 rotates in a horizontal stiffness 149 lb / ft and a dashpot that is set up to critically
damp the system. Assuming that there is no friction be­
plane at a constant angular speed to. The particle P
tween the wheels and the plane, find the displacement
(mass = m) moves in the frictionless slot and is attached
toward the wall after sec has elapsed.
to the spring (modulus k, free length as shown.
a. Derive the differential equation describing the
motion y(t) of the particle relative to the slot.
b. What is the extension of the spring such that P
does not accelerate relative to the slot?
c. Suppose the motion is initiated with the spring
unstretched and the particle at rest relative to
the slot. Find the ensuing motion y(f).
Find the value of c to give critical damping of the
pendulum in Figure P8.21. Neglect the mass of the rigid Figure P8.21
bar to which the particle of mass m is attached.
8.22 If k = 100 lb/in. and the mass of the uniform,
2
slender, rigid bar in Figure P8.22 is 0.03 lb-sec / in., what
damping constant c results in critical damping?

Figure P 8 . 2 2

Figure P 8 . 1 8

Figure P 8 . 2 3

Figure P 8 . 2 0 Figure P 8 . 2 4
534

8.25 Consider free oscillations of a subcritically damped


oscillator. Do local maxima in the response occur periodi­
cally?
In Figure P8.26, find the steady-state displacement
x(t) if y(t) = 0.1 sin lOOf inch, where Ms in seconds,
2
m = 0.01 lb-sec /in., k = 100 lb/in., and c = 2 lb-sec/
in. In particular:
a. What is the amplitude of x(t)7
Figure P 8 . 2 9
b. What is the angle by which x(t) leads or lags y(t)l

Figure P 8 . 3 0

• 8.30 In Figure P8.30 find the response x (t) for the initial
t

Figure P 8 . 2 6 conditions x (0) = x (0) = 0 if


1 1

k = 100 lb/in.
2
m = 0.01 lb-sec /in.
8.27 In Figure P8.27 find the steady-state displacement
x(t) if y(t) = 0.2 sin 90f inch, where f is in seconds, c = 1.0 lb-sec/in.
2
m = 0.01 lb-sec /in., = 501b/in.,andc = 1 lb-sec/in. X = 0.05 in.
2

In particular:
w = 100 rad/sec
a. What is the amplitude of x{t)l
b. What is the phase angle by which x(t) leads or • 8.31 Repeat the preceding problem if (a) c = 2.0 lb-
lags y(0? sec/in.; (b) c = 0.5 lb-sec/in.
Optical equipment is mounted on a table whose
four legs are pneumatic springs. If the table and equip­
ment together weigh 700 lb, what should be the stiffness
of each spring so that the amplitude of steady, simple-
harmonic, vertical displacement of the table will not be
greater than 5 percent of a corresponding motion of the
Figure P 8 . 2 7
floor? The forcing frequency is 30 rad/sec. Neglect
damping in your calculations.

8.28 The cart in Figure P 8 . 2 8 is at rest prior to t = 0, at • 8.33 The block of mass m in Figure P8.33 is mounted
which time the right end of the spring is given the motion through springs k and damper c on a vibrating floor. De­
y = vt, where v is a constant. Find x(t). rive an expression for the steady-state acceleration of the
block (whose motion is vertical translation). Show that
the amplitude of the acceleration is less than that of the
floor, regardless of the value of c, provided that co
where co„ is the frequency of free undamped
vibrations of the block. Show further that if
then the smaller the damping the better the isolation.
Figure P 8 . 2 8

B.29 The block in Figure P8.29 is at rest in equilibrium


y = Y sin wt
prior to the application of the constant force P = 50 lb at
2
f = 0. If k = 100 lb/in., m = 0.01 lb-sec /in., and the
system is critically damped,findx(t). Figure P 8 . 3 3
535

8.3 Euler's Laws for a Control Volume

Euler's laws describe the relationship between external forces and the
motion of any body whether it be a solid, liquid, or gas. Sometimes,
however, it is desirable to focus attention on some region of space (con­
trol volume) through which material may flow rather than on the fixed
collection of particles that constitute a body. Examples of this sort are
abundant in the field of fluid mechanics and include the important prob­
lem of describing and analyzing rocket-powered flight. Our purpose in
this section is to discuss the forms taken by Euler's laws when the focus of
attention is the control volume rather than the body.
We take as self-evident what might be called the "law of accumula­
tion, production, and transport" — that is, the rate of accumulation of
something within a region of space is equal to the rate of its production
within the region plus the rate at which it is transported into the region.*
Thus, for example, the rate of accumulation of peaches in Georgia equals
the rate of production of peaches in the state plus the net rate at which
they are shipped in. This idea can be applied in mechanics whenever we
are dealing with a quantity whose measure for a body is the sum of the
measures for the particles making up the body. Thus we can apply this
principle to things such as mass, momentum, moment of momentum,
and kinetic energy.
Suppose that at an instant a closed region V (control volume) con­
tains material (particles) making up body B. Let m denote the mass of
e

body and m denote the mass associated with V (that is, the mass of
v

whatever particles happen to be in V at some time). Instantaneously


m = m , but because some of the material of is flowing out of V and
v e

some other material is flowing in, m # m . In fact by the accumulation


v B

principle stated above


(net rate of mass flow into V) (8.29)
since clearly m represents the rate of buildup (accumulation) of mass in
v

V and since m , the rate of change of mass of the material instantaneously


B

within V, represents the production term. Of course a body, being a


specific collection of particles, has constant mass; thus m = 0 and (8.29)
B

becomes m = (rate of mass flow into V), which is often called the conti­
v

nuity equation.
For momentum L, the statement corresponding to Equation (8.29) is
(net rate of flow of momentum into V) (8.30)
But Euler's first l a w applies to a b o d y (such as ) so that where
EF is t h e resultant o f t h e external forces on ? — or, in other words, the
resultant o f t h e external forces acting o n t h e material i n s t a n t a n e o u s in V. T

The mathematical statement of this is known as the Reynolds Transport Theorem.


Page 536

which is the control-volume form of Euler's first law. The momentum


flow rate on the right of (8.31) is calculated by summing up (or integrat­
ing) the momentum flow rates across infinitesimal elements of the
boundary of V, where the momentum flow rate per unit of boundary area
is the product of the mass flow rate per unit of area and the instantaneous
velocity of the material as it crosses the boundary.
A similar derivation produces a control-volume form of Euler's
second law, for which the result is
(net rate of flow into V of moment
of momentum with respect to O)
where O is a point fixed in the inertial frame of reference.
It is important to realize that nothing in our derivations here has
restricted the control volume except that it be a closed region in space. It
may be moving relative to the frame of reference in almost any imagin­
able way, and it may be changing in shape or volume with time. We
conclude this section with examples of two of the most common applica­
tions of Equation (8.31).

EXAMPLE 8 . 6
A fluid undergoes steady flow in a pipeline and encounters a bend at which the
cross-sectional area of pipe changes from A to A . At inlet 1 the density is p and
1 2 1

the velocity (approximately uniform over the cross section) is v .At outlet 2 the
t

density is p . Find the resultant force exerted on the pipe bend by thefluid.(See
2

Figure E8.6a.)

Figure E 8 . 6 a

Solution
Let the velocity of flow at the outlet be given by u (cos 6i + sin 6)). Then for
2

steady flow the rate of mass flow at the inlet section is the same as that at the
outlet section:

so that

Control
volume V

Let the control volume be the region bounded by the inner surface of the
Figure E 8 . 6 b pipe bend and the inlet and outlet cross sections. (See Figure 8.6b.) A conse-
Page 537

quence of the condition of steady flow is that within the control volume the
distributions of velocity and density are independent of time. Thus the total
momentum associated with V is a constant and

But

(net rate of momentum flow into V)

Therefore
(net rate of momentum flow out of V)

And if P is the force exerted on the fluid by the bend, then

where p and p are the inlet and outlet fluid pressures respectively. Therefore
1 2

and the force exerted on the bend by the fluid is — P with

EXAMPLE 8 . 7
To illustrate how the control volume form of Euler's first law is used to describe
the motion of a rocket vehicle, consider such a vehicle climbing in a vertical
rectilinear flight. Let v) be the velocity of the vehicle from which combustion
products are being expelled at velocity — v j relative to the rocket. Further let
e

M(t) be the mass at time t of the vehicle and its contents, let p be the rate of
mass flow of the ejected gases, and let p be the gas pressure at the nozzle exit of
cross section A.
Solution
Force D in the free-body diagram (Figure E8.7), representing the drag or resist­
ance to motion, is the resultant of (1) all the shear stresses acting on the surface of

Figure E 8 . 7 Free-body diagram of rocket


Page 538

terms have been separated so that we may point out that the force pA remains

At this point we must approximate L by Mvj, this is an approximation because


v

or

But of course

so that

which is of the form of force = mass X acceleration, where one of the "forces" is
the "thrust" pv .e

This term may be neglected if exhaust gases have expanded to atmospheric pressure or
nearly so.

PROBLEMS • Section 8.3

8.34 Let dm /dt and dm /dt be the respective rates at


t 0 8.36 A child aims a garden hose at the back of a friend.
which mass enters and leaves a system. Show that Equa­ (See Figure P8.36.) If the watertion (8.31)weight
(specific may be62.4
expressed
lb/ in terms of these r
3
ft ) stream has a diameter of {in. and a speed of 50 ft / sec,
estimate the force exerted on the "target" if: (a) he is
stationary; (b) he is running away from the stream at a
speed of 10 ft/ sec. Assume the flow in contact with the
where mv = L and it is assumed that all the incoming
v

particles have a common velocity v,- (in an inertia] frame)


and that all the exiting particles have a common velocity

8.35 Liquid of specific weight w flows out of a hole in the side of a tank in a jet of cross section A. If t
the jet is v, determine the force exerted on the tank by the
supporting structure that holds the tank at rest. Note that
the pressure in the jet will be atmospheric pressure.
Figure P8.36
Page 539

boy's back to be vertical relative to him; that is, neglect (with torque M applied to the drum on the right) at con­
any splashback. stant speed v . Find the power that the motor must de­
B

8.37 A steady jet of liquid is directed against a smooth liver, neglecting friction in the shaft bearings and assum­
rigid surface and the jet splits as shown in Figure P8.37. ing the belt does not slip. Hint: Use the control volume
Assume that each fluid particle moves in a plane parallel indicated by the dashed lines to compute the difference in
to that of the figure and ignore gravity. Ignoring gravity belt tensions, neglecting any sag of the belt due to the
and friction, it can be shown that the particle speed after weight of the rocks.
the split is still v as depicted. Estimate the fraction of the 8.41 Sand is being dumped on aflatcarof mass M at the
flow rate occurring in each of the upper and lower constant mass flow rate of q. (See Figure P8.41.) The car is
branches. Hint: Use the fact that no external force tangent being pulled by a constant force P, and friction is negligi­
to the surface acts on the liquid. ble. The car was at rest at t = 0. Determine the car's accel­
eration as a function of P, M, q, and t.

Figure P 8 . 3 7

Figure P8.41

8.38 Air flows into the intake of a jet engine at mass flow
rate a (slug/sec or kg/s). If v is the speed of the airplane
flying through still air and u is the speed of engine exhaust
relative to the plane, derive an expression for the force
(thrust) of the flowing fluid on the engine. Neglect the
fact that the rate of exhaust is slightly greater than q be­ 8.42 The pressure in a 90° bend of a water pipe is 2 psi
cause of the addition of fuel in the engine. (gauge). The inside diameter of the pipe is 6 in. and water
8.39 Revise the analysis of the preceding problem to ac­
count for the mass of fuel injected into the engine. Let/be
the mass flow rate of the fuel, and assume that the fuel is
injected with no velocity relative to the engine housing. 8.43 The reducing section in Figure P8.43 connects a 36-
in. inside-diameter pipe to a 24-in. inside-diameter pipe.
8.40 In a quarry, rocks slide onto a conveyor belt at the
Water enters the reducer at 10 ft /sec and 5 psi (gauge)
constant mass flow rate k, and at speed v , relative to the
re
and leaves at 2 psi. Find the force exerted on the reducer
ground. (See Figure P8.40.) The belt is driven by a motor
by the steadily flowing water.

10 ft/sec

Figure P 8 . 4 3

Figure P8.40
Page 5 4 0

8.44 If the plane of Figure P8.44 is horizontal, find the speed relative to the car of 2500 ft / sec. The bullets origi­
force and moment at O that will allow the body B to nally comprised 2 percent of an initial total mass of m 0

remain in equilibrium when the open stream of water = 20 slugs. If the system starts from rest at t = 0, find:
impinges steadily on it as shown. The stream's velocity is (a) the maximum speed of Bonnie and Clyde; 60 ft / (b)
sec, how
and its constant area i
3
weight is 62.4 lb/ft . long it takes to attain this speed.

1 ft
0.5 ft
30
Figure P 8 . 4 6
1 ft

Figure P 8 . 4 4

8.45 A coal truck weighs 5 tons when empty. Itispushed


under a loading chute by a constant force of 500 lb. The
chute, inclined at 60° as shown in Figure P8.45, delivers
100 lb of coal per second to the truck at a velocity of 30
ft / sec. When the truck contains 10 tons of coal, its velocity is 10 ft/sec to the right, (a) What is its acceleration

Figure P 8 . 4 7

8.47 A black box with an initial mass of m (of which 10 0

percent is box and 90 percent is fuel) is released from rest


on the inclined plane in Figure P8.47. The coefficient of
friction is fi between the box and the plane.
60° a. Show that with tan the box will begin to
slide down the plane.
b. Assume now that tan and that a mecha­
500 lb nism in the box is able to sense its velocity and
eject particles rearward (up the plane) at a con­
stant mass flow rate of k , and at a relative ve­
0

Figure P 8 . 4 5 locity always equal to the negative of the veloc­


ity of the box. Find the velocity of the box at
the time t when the last of the fuel leaves.
f
8.46 Bonnie and Clyde are making a getaway in a cart
with negligible friction beneath its wheels. (See Fig­ c. Show that the box is going 5.5 times faster at t
ure P8.46.) Clyde is killing two birds with one stone by = t than it would have gone if no fuel had
f

using his machine gun to propel the car as well as to ward been ejected.
off pursuers. Hefires500 rounds (shots) per minute with 8.48 Santa Claus weighs 450 lb and drops down a 20-ft
each bullet weighing 1 oz and exiting the muzzle with a chimney (Figure P8.48). He gains mass in the form of
Page 541

ashes and soot at the rate of 3 slugs / sec from a very dirty which the pressure force pA and the drag D are negligible,
chimney. the initial mass of the rocket is m , and the gravitational
0

acceleration g, the rate p, and the relative velocity v are all


a. Find Santa's velocity as a function of time. (Ne­ t

glect friction.) constants. Initially, the rocket is at rest.


b. Calculate the velocity v and the time t at
b b
8.52 Repeat the preceding problem, but this time in­
which he would hit bottom without adding clude a drag force of —to,where k is a positive constant.
mass and then compare v with his "ashes and
b 8.53 (a) Extending Problem 8.51, find the height of the
soot velocity" at the same t . b rocket as a function of rime, (b) If the fraction of m which
0

is fuel is /, find the rocket's "burnout" velocity and posi­


tion when all the fuel is spent.
8.54 Spherical raindrops produced by condensation are
precipitated form a cloud when their radius is a. They fall
freely from rest, and their radii increase by accretion of
moisture at a uniform rate k. Find the velocity of a rain­
drop at time f, and show that the distance fallen in that
time is

' 8.55 A chain of length L weighing y per unit length


begins to fall through a hole in a ceiling. (See Fig­
ure P8.55.) Referring to the hint in Problem 8.50:
Figure P8.48
a. Find v(x) if v = 0 when x and t are zero.
b. Show that the acceleration of the end of the
falling chain is the constant g/ 3.
8.49 A small rocket is fired vertically upward. Air resist c. Show that when the last link has left the ceil­
ance is neglected. Show that for the rocket to have con ing, the chain has lost more potential energy
stant acceleration upward, its mass m must vary with time than it has gained in kinetic energy, the differ­
t according to the equation ence being yL /6. Give the reason for this loss.
2

where a is the acceleration of the rocket and u is the


velocity of the escaping gas relative to the rocket.
B.50 The end of a chain of length L and weight per unit constant
length w, which is piled on a platform, is lifted vertically
by a variable force P so that it has a constant velocity v.
(See Figure P8.50.) FindP as a function of x. Hint: Choose
a control-volume boundary so that material crosses the
boundary (with negligible velocity) just before it is acted
on by the moving material already in the control volume. Figure PB.50
That is, there is no force transmitted across the boundary
of the control volume. The solution will be an approxima­
tion to reality because of assuming arbitrarily small indi­
vidual links; but the more links having the common ve­
locity of the fully engaged links within the volume, the
better the approximation will be.
8.51 Solve the final equation of Example 8.7 for the ve­
locity if j of the rocket as a function of time in the case in Figure P8.55
Page 542

8.56 The machine gun in Figure P8.56 has mass M ex­ in the gravitational attraction, (a) show that the greatest
clusive of its bullets, which have mass M ' in total. The upward speed is attained when the mass of the rocket is
bullets arefiredat the mass rate of Kg "slugs" per second, reduced to M, and determine this speed, (b) Show also
with velocity u relative to the ground. If the coefficient of
0 that the rocket rises to a height
friction between the gun's frame and the ground is fi, find
the velocity of the gun at the instant the last bullet is fired.
8.60 With the same notation and conditions as in Prob
lem 8.34, show that Equation (8.32^may be written as

where r, and i are position vectors for the mass centers of


0

the incoming and exiting particles.


Coefficient of friction
• A pinwheel of radius a, which can turn freely about
a horizontal axis, is initially of mass M and moment of
Figure P 8 . 5 6 inertia I about its center. A charge is spread along the rim
and ignited at time t = 0. While the charge is burning, the
rim of the wheel loses mass at a constant rate m mass
x

units per second, and at the rim a mass m of gas is taken


2

" 8 . 5 7 A particle of mass m, initially at rest, is projected up per second from the atmosphere, which is at rest. The
with velocity v at an angle a to the horizontal and moves total mass m, + m is discharged per second tangentially
2
0

under gravity. (See Figure P8.57.) During its flight, it from therim,with velocity v relative to the rim. Prove that
gains mass at the uniform rate k. If air resistance is ne­ if 8 is the angle through which the wheel has turned after
glected, show that its equation of motion is t sec, then

and that the equation of its path is


where

• 8.62 A wheel of radius a starts from rest and fires out


matter at a uniform rate from all points on the rim (Fig­
ure P8.62). The matter leaves tangentially with relative
speed v and at such a rate that the mass decreases at the
rim by m mass units per second. Show that the angle 8
turned through by the wheel is given by

Figure P 8 . 5 7
in which I is the initial moment of inertia of the wheel
0

about its axis.

•8.58 If in the preceding problem the air offers a resist­


ance —ci, determine the equation of the path.
• From a rocket that is free to move vertically up­
ward, matter is ejected downward with a constant rela­
tive velocity gT at a constant rate 2M/T. Initially the
rocket is at rest and has mass 2M, half of which is avail­
able for ejection. Neglecting air resistance and variations Figure P 8 . 6 2
Page 543

8.4 Central Force Motion

In Chapter 2 we defined a central force acting on a particle pas one which


(1) always passes through a certain point O fixed in the inertial reference
frame J and (2) depends only on the distance r between O and p. (See
Figure 8.12.) In this section we are going to treat the central force in more
detail. We shall go as far as we can without specializing F(r) — that is,
without saying how F depends on r. In the second part of the section we
shall study the most important of central forces: gravitational attraction.
If the central force F is the only force acting on the particle, then
F = ma; and since the central force always passes through point O, r X F
is identically zero. These two facts allow us to write:

Figure 8.12
or, since r= v,

Therefore for a particle acted on only by a central force,


= constant vector in (8.33)
Dotting this equation with r, we find, since r X v is perpendicular to r,

and we see that r is always perpendicular to a vector that is constant in J;


therefore Amoves in a plane in Using polar coordinates to then de­
scribe the motion of pm this plane, the governing equations are:
(8.34)
and

(8.35)

From Equation (8.35) we see immediately that


constant (8.36)
where h is the magnitude of the constant vector h of Equation (8.33)
Q o

because, expressing r and v in polar coordinates, we find

constant
so that
constant (8.37)
Equation (8.37) is a statement of the conservation of the moment of
momentum, or angular momentum of P; the constant h is the magnitude
0

of the angular momentum H of p divided by its mass m. Thus we shall


o

call h the angular momentum (magnitude) per unit mass.


0
Page 544

We can use the previous pair of results to show that the second of
Kepler's three laws of planetary motion is in fact valid for any central
force. This law states that the radius vector from the sun to a planet
sweeps out equal areas in equal time intervals. From Figure 8.13 the
incremental planar area A swept out by p between 8 (at t) and 6 + A6
(at i + t) is approximately given by the area of the triangle OBB'*
(see Figure 8.14):

base height
sin
ura 8.13
Dividing by the time increment t and taking the limit as we
have

or
Figure 8.14
2
(a constant that is r 8/ 2) (8.38)

Thus the rate of sweeping out area is a constant. This is why a satellite
or a planet in elliptical orbit (Figure 8.15) has to travel faster when it is
near the perigee than the apogee—the same area must be swept out in the
same period of time.' We emphasize again that this result is valid for all
central force trajectories, not just elliptical orbits and not just if the central
force is gravity.

Perigee P*
A pogce

Figure B,15

Next we focus our attention on the most important central force:


gravitational attraction. If G is the universal gravitation constant and M
and m are the masses of what we are considering to be the attracting and
attracted bodies,* then the central force acting on m for this case is

* " A p p r o x i m a t e l y ' ' b e c a u s e t h e a r e a b e t w e e n a r c a n d c h o r d i s o u t s i d e t h e triangle.


W e u s e perigee a n d apogee in a g e n e r a l s e n s e ; t e c h n i c a l l y t h e s e w o r d s refer t o t h e n e a r ­
est a n d f a r t h e s t p o i n t s , respectively, f o r t h e m o o n a n d artificial satellites. F o r t h e orbits
of p l a n e t s , t h e p r o p e r t e r m s a r e perihelion a n d aphelion.
Actually, of course, b o t h a r e attracting a n d b o t h a r e attracted — e a c h t o t h e other! T h e
2 1 3 J
c o n s t a n t G M is, f o r t h e s u n , 4 . 6 8 X 1 0 f f / s e c .
Page 545

(8.39)

and Equation (8.34) becomes

(8.40)

2
Canceling m and inserting h for r 8 gives
a

(8.41)

Multiplying Equation (8.41) by r will allow us to integrate it:


(8.42)
Integrating, we get

(8.43)

2
If we multiply Equation (8.43) by m and replace h by r O, we see that
Q

(8.44)

and the left side of Equation (8.44) is seen to be the total energy of p,
kinetic plus potential. Thus we shall replace C by E, the energy of p-pex
t

unit mass, and obtain


(8.45)
This equation will be helpful to us later. But now we are interested in
studying the trajectory of particle p— that is, in finding r as a function of
By the chain rule,

and since from Equation

(8.46)

We need the second derivative of r in Equation (8.41), so we apply the


chain rule once more:

(8.47)
Page 5 4 6

Substituting into Equation (8.41), we get

or

(8.48)

The following simple change of variables will make the solution to this
differential equation immediately recognizable:

(8.49)

Substituting Equation (8.49) into (8.48) along with

(8.50)

gives

(8.51)

or

(8.52)

The solution to Equation (8.52), from elementary differential equations,


consists of a homogeneous (or complementary) part plus a particular
part:

(8.53)

Switching variables back from u to r by Equation (8.49), we obtain

(8.54)

This solution for r(6) is the equation of a conic; it can be put into a
more recognizable form after a brief review of conic sections. For every
point P on a conic, the ratio of the distances from P to a fixed point (O: the
focus) and to a fixed line (l: the directrix) is a constant called the eccen­
tricity of the conic:

(8.55)

Therefore, in terms of the parameters in Figure 8.16,

(8.56)
Figure 8 . 1 6
Page 547

or, solving for r,

(8.57)

The conic specified by Equation (8.57) is a:


Hyperbola if | e \ > 1
Parabola if e = 1
Ellipse if - 1 < e < 1 (8.58)
The ellipse becomes a circle if e = 0. It is an ellipse with perigee (closest
point t o O ) a t i if 0 < e < 1 and an ellipse with apogee (farthest
point from O) at if — 1 < e < 0; this latter type is called a subcircu-
lar ellipse.
Returning to our solution (8.54) for , it is customary to select one
of the constants and B j so that, as suggested by Figure 8.16, dr/d
= 0 when . This condition easily gives B = 0, as the reader may
1

wish to demonstrate using calculus. The result simply means that we are
measuring 8 from the perigee of the conic. At this point we should
compare Equations (8.57) and (8.54) with B = 0: t

(8.59)

and

(8.60)

By direct comparison of these two expressions for r, we see that

It is more customary, however, to express the constant A (as well as the


1

eccentricity) in terms of the energy E of the orbit. To do this, Equa­


tions (8.45) and (8.46) give

(8.61)

At the point r where 8 = 0 and dr /dd = 0, we see that


P

(8.62)

Thus not all of h , r , and E are independent. We shall eliminate r .


0 P P

Multiplying Equation (8.62) by r , we get


(8.63)
Solving via the quadratic formula, we have

(8.64)
Page 5 4 8

in which we use the plus sign since we need the smaller root for closed
conies (£ < 0). The positive sign also ensures a positive r for open conies
P

(E > 0).
Returning to our solution (8.59), when 6 = 0 then

(8.65)

Equating the two expressions for r>, Equations (8.64) and (8.65), we can
solve for A . t

We see by comparing Equations (8.59) and (8.60) that the eccentric­


ity e of our conic will be (h} /GM)A .
) Equating the right sides of Equa­
1

tions (8.64) and (8.65) and solving for this quantity, we get

(8.66)

Therefore

(8.67)

which expresses r as a function of 9, the constant GM, the energy E, and


the angular momentum per unit mass h . Note that by again comparing
Q

Equations (8.59) and (8.60) we can obtain the distance a between the
focus O and the directrix /:

(8.68)

Thefirstof Kepler's three laws of planetary motion states that the


planets travel in elliptical orbits with the sun at one focus.* These ellipses
are very nearly circular for most of the planets; the eccentricity of earth is
e = 0.017. To obtain the third of Kepler's laws, we return once more to
our equations and obtain for elliptic orbits, from (8.67), the distance r
when 8 = 90°:

(8.69)

This distance, the semilatus rectum, may be used to express the distance r A

between the focus O and apogee A*, and the distance r between O and P

the perigee P*. (See Figure 8.17.) At apogee, 9 — n and Equations (8.59),
(8.60), and (8.69) give

(8.70)

Kepler's laws, based on his astronomical observations and set forth in 1609 and 1619,
were studied by Newton before the Englishman published the Principia, which con­
Figure 8.17 tained his own laws of motion.
Page 549

and at perigee = 0),

(8.71)

The semimajor axis length of the ellipse is

(8.72)

and the seinirninor axis length is, from analytic geometry,

(8.73)

An ellipse has area

or

(8.74)

With these results in hand, we shall now prove Kepler's third law.
Since dA / dt is constant,

(8.75)

where we take A = 0 when t = 0, say at the perigee. Over one orbit we


have, with T being the orbit period,

area of ellipse

(8.76)

Since (from Equations (8.69) and (8.72))


(8.77)

we obtain the following from Equation (8.76):

(8.78)

so that

or

(8.79)
Page 550

Equation (8.79) states the third of Kepler's laws: The squares of the
planets' orbital periods are proportional to the cubes of the semimajor
axes of their orbits.

EXAMPLE 8 . 8

Calculate the semimajor axis length of an earth satellite with a period of 90 min.
Solution
We can solve this problem using Kepler's third law. The weight of a particle
(mass m) on the earth's (mass M) surface is both mg and GMm thus

and we see that the product of the unwieldy constants G and M is

Therefore

which is about 170 mi above the earth's surface.

We shall present one more example on elliptical orbits under gravity,


but first we need two equations relating the velocities v and v at any two
1 2

points P and P with radii r and r on the orbit. The first of these comes
1 2 1 2

from Equation (8.37), which states that = constant. From Fig­


ure 8.18, since the velocity v is always tangent to the path, we see that if
is the angle between r and v, then in cylindrical coordinates

v sin = (transverse component of v) =


so that
constant
or, for two points P and P on the orbital path,
1 2

Figure 8 . 1 8 (8.80)
Note that at apogee and perigee, = 9 0 ° . Thus letting P and P be these
1 2

two points, we get from Equation (8.80)


(8.81)
and the two velocities are inversely proportional to the radii, with v being
faster at perigee as we have already seen from Kepler's second law.
The other equation relating v and v comes from the potential for
1 2

gravity, which from Equation (2.27) and Example 8.8 is


Page 551

Using conservation of energy between P and P , 1 2

(8.82)

If we let point P represent the perigee P*, as suggested in Figure 8.18,


2

then Equation (8.80) becomes

(8.83)

= n
where sin 2 s i 90° = 1. Now if r v and (p^ are initial (launch)
1 # l t

values of r, v, and then we may consider these as given quantities.


Substituting Equation (8.83) into (8.82), we can obtain an equation for
the perigee radius

Multiplying through by and rearranging, we get

(8.84)

We see that Equation (8.84) is simply a quadratic equation in the ratio


and that [2GM / is a nondimensional parameter of the
orbit. We now illustrate the use of this important equation in an example.

EXAMPLE 8 . 9
A satellite is put into an orbit with the following launch parameters: H
= 1000 mi, i>! = 17,000 mph, and 0 = 100° (see Figure E8.9). Find the apogee
and perigee radii of the resulting orbit.
Solution
3 2
We need GM in mi /hr ; therefore

Figure E 8 . 9

The parameter 2GM in Equation (8.84) is therefore


Page 552

Equation (8.84) becomes

The quadratic formula gives

= 0.911 and 1.46


Therefore
0.911(4960) = 4520 mi
The other root corresponds to the apogee. (Since the starting condition of sin
= 1 is the same for apogee and perigee, both answers are produced by the
quadratic formula!)
1.46(4960) = 7240 mi
The altitudes are
Perigee height = 4520 - 3960 = 560 mi
Apogee height = 7240 - 3960 = 3280 mi
To pin down the orbit in space, we need to know the angle to the perigee point
from the launch point and also the orbit's eccentricity. Problems 8.81 and 8.82
will be concerned with finding these two quantities given initial values of r,v, and

PROBLEMS • Section 8.4

8.63 Show that a satellite in orbit has a period T given by sages have been recorded since 240 B.C.!) What is the
approximate semimajor axis length of Halley's comet?
T= (Use 76 years as the period.)
8.66 Find the minimum period of a satellite in circular
8.64 Show that, for a body in elliptical orbit (Fig­ orbit about the earth. Upon what assumption is your an­
ure P8.64), b swer based?
8.67 Repeat the preceding problem if the satellite orbits
the moon. Assume

&moon earth


r
moon 0.27r earth

8.68 Show that for circular orbits around an attracting:


Figure P 8 . 6 4 2 6
body of mass M, rv = GM. Then use the 93 X 10 mi
average orbital radius of earth, and the fact that its orbit is
8-65 Halley's comet's latest return to Earth was in 1986. nearly circular, to find the constant GM, for the sun as
The comet orbits the sun in an elongated ellipse every 74 attracting center (heliocentric system).
to 79 years; the period varies due to perturbations in its 8.69 The first artificial satellite to orbit the earth was the
orbit caused by the four largest (Jovian) planets. (Its pas­ Russians' Sputnik I. Following insertion into orbit it had a
Page 553

period of 96.2 min. Find the semimajor axis length. If the


initial eccentricity was 0.0517, find the maximum and
minimum distances from earth following its injection into
orbit.
8.70 Show that if a satellite is in a circular orbit at radius r 200 mi
around a planet of mass M, the velocity to which it must
increase to escape the planet's gravitational attraction is
given by

2800 mi
Figure P8.77
Find Vnayc if "to which" is replaced by "by which."
8.71 Show that if the launch velocity in Example 8.9 is
15,000 mi/hr, the satellite will fail to orbit the earth.
8.72 Using Equation (8.54), show that B, = 0 follows
from the condition dr/d0=O when 0=0.
8.73 Prove that Equation (8.66) follows from (8.64) and
(8.65).
8.74 Find the form of the central force F(r) for which all
circular orbits of a particle about an attracting center O
have the same angular momentum (and the same rate of
sweeping out area).

8.75 Show that, in terms of the radiusrp.and speedvp. at perigee, the energy and eccentricity of the o
Figure P8.78

will send the satellite to position A, at radius R (<R )< 2 1

and 180° away. Then find the second negative velocity incre­
ment, this time applied at A, that will put the satellite in a
circular orbit of radius R . Hint: Use Problem 8.76.
2

B.79 A satellite has = 8000 mi and tp. = 5000 mi. If


8.76 Use Equations (8.81) and (8.82) to show that the it was launched with a velocity of 15,000 mi/hr, what
velocities at apogee and perigee, in terms of the known was its launch radius? What was the angle between r
radii r . and rp., are:
A and v at launch? Hint: Use Problem 8.76.
8.80 What is the period of the satellite in the preceding
problem?

and •8.81 Show that if the launch parameters, T ,V , and ,


1 1

are known, then the angle from perigee to the launch


point of a satellite in orbit is given by

tan C3
• 8.77 A rocket is in a 200-mi-high circular parking orbit GM GM
above a planet. What velocity boost at point P will result
• B.82 Show that if the launch parameters r,, v , and t
in the new, elliptical orbit shown in Figure P8.77? Hint-
are known, then the eccentricity of the resulting conic is:
Use the results of Problem 8.75.
• 8.78 A satellite is in a circular orbit of radius R . (See
1

Figure P8.78.) Find the (negative) velocity increment that


Page 554

* 8.83 A large meteorite approaches the earth. (See Fig­ 8.84 Classify the various orbits according to values of
ure P8.83.) Measurements indicate that at a given time it the dimensionless parameter GM/(r i>o) for a satellite
0

has a speed of 8000 mph at a radius of 100,000 mi. Will it launched with the conditions of Figure P8.84.
orbit the earth? If so, what is the period? If not, what is the •8.85 Find the kinetic energy increase needed to move a
maximum velocity v that would have resulted in an orbit? satellite from radius R to nR(n > 1) in circular orbits. Hint:
Hint: Use the result of the preceding problem.
Use Problem 8.76.
*8.86 A particle of mass m moves in the xy plane under
the influence of an attractive central force that is propor­
tional to its distance from the origin (F(r) = kr). It has the
same initial conditions as Problem 8.84. Find the largest
and smallest values of r in the ensuing motion.
8.87 A satellite has apogee and perigee points 1000 and
180 mi, respectively, above the earth's surface. Compute
the satellite's period.
8.88 In the preceding problem find the speeds of the
satellite at perigee and at apogee. Hint: Use Problem 8.76.
Figure P 8 . 8 3 Figure P 8 . 8 4

REVIEW QUESTIONS • Chapter 8


True or False?
1. Frequencies of vibration of periodic motion are associated only with
small amplitudes.
2. Natural frequencies of vibration are associated only with transla-
tional motion.
3. Natural frequencies of vibration of bodies in a gravitational field do
not have to depend on "g."
4. Free vibrations will always decrease in time due to "real world
damping."
5. There are three types of damped vibrations, and the subcritical case
has the greatest practical importance.
6. In forced vibration, the steady-state part of the response dies out due
to damping inherent in the physical system.
7. In general, the rate of accumulation of momentum within a region of
space equals the rate at which it is transported into that region.

In using the control volume form of Euler's Laws, the control vol­
ume:
B. has to be fixed in the inertial frame of reference;
9. may change in shape or volume with time;
10. has to be a closed region of space.
11. In a one-dimensional control-volume problem, Euler's first law be­
comes, in general,
Page 555

12. The law of accumulation, production, and transport applies only to


scalar quantities.
13. A central force depends only on the distance r between the attracted
and ataacting particles.
14. There are central forces besides gravity.
15. In every central force problem the angular momentum about the
attracting point O is a constant.
16. All three of Kepler's laws apply to motions of a particle under the
influence of any type of central force.
17. All central force problems result in paths which are conies.
18. In a gravitational central force problem, the type of conic is deter­
mined by the eccentricity.

Answers
1.F 2. F 3.T 4.T 5. T 6. F 7. F 8. F 9. T 10. T 11. F 12. F 13. T 14. T 15. T
16. F 17. F 18. T


• APPENDICES CONTENTS

Appendix A UNITS
Appendix B EXAMPLES OF NUMERICAL ANALYSIS/THE NEWTON-
RAPHSON METHOD
Appendix C MOMENTS OF INERTIA OF MASSES
Appendix D ANSWERS TO ODD-NUMBERED PROBLEMS
Appendix E ADDITIONAL MODEL-BASED PROBLEMS FOR
ENGINEERING MECHANICS

Page 557
A UNITS

The numerical value assigned to a physical entity expresses the relationship of


that entity to certain standards of measurement called units. There is currently an
international set of standards called the International System (SI) of Units, a
descendant of the meter-kilogram-second (mks) metric system. In the SI system
the unit of time is the second (s), the unit of length is the meter (m), and the unit
of mass is the kilogram (kg). These independent (or basic) units are defined by
physical entities or phenomena. The second is defined by the period of a radia­
tion occurring in atomic physics; the meter is defined by the wavelength of a
different radiation; the kilogram is defined to be the mass of a certain body of
material stored in France. Any other SI units we shall need are derived from these
three basic units. For instance, the unit of force, the newton (N), is a derived
quantity in the SI system, as we shall see.
Until recendy almost all engineers in the United States used the system
(sometimes called British gravitational or U.S.) in which the basic units are the
second (sec) for time, the foot (ft) for length,* and the pound (lb) for force. The
pound is the weight, at a standard gravitational condition (location), of a certain
body of material stored in the United States. In this system the unit of mass, the
slug, is a derived quantity. It is a source of some confusion that sometimes there is
used a unit of mass called the pound (the mass whose weight is one pound of
force at standard gravitational conditions); also, particularly in Europe, the term
kilogram is also sometimes used for a unit of force.f Grocery shoppers in the
United States are exposed to this confusion by the fact that packages are marked
by weight (or is it mass?) both in pounds and in kilograms. Throughout this text,
without exception, the pound is a unit of force and the kilogram is a unit of mass.
The United States is currently in the painful process of gradual changeover
to the metric system of units after more than 200 years of attachment to the U.S.
system. The new engineers who begin practicing their profession in the 1990s
will doubtless encounter both systems, and thus it is crucial to master both
(including thinking in terms of the units of either) and to be able to convert from
one to the other. The units mentioned here are summarized in Table A. 1 for the SI
and the U.S. systems.

* Sometimes, particularly in the field of mechanical vibrations, the inch is used as the
2
unit of length; in that case the unit of mass is 1 lb-sec /in., which equals 12 slugs.
| A kilogram was a force unit in one of two mks systems, compounding the misunder­
Page 558 standing.
Page 559

Table A.1

SI (Standard International
Quantity or "Metric") Unit U.S. Unit

force newton (N) pound (lb)


mass kilogram (kg) slug
length meter (m) foot (ft)
time second (s) second (sec)

We now examine how the newton of force is derived in SI units and the slug
of mass is derived in U.S. units. Let the dimensions of the four basic dimensional
quantities be labeled as F (force), M (mass), L (length), and T (time). From the first
law of motion (discussed in detail in Chapter 2), F = ma, we observe that the four
basic units are always related as follows:

F=

This means, of course, that we may select three of the units as basic and derive the
fourth. Two ways in which this has been done are the gravitational and the
absolute systems. The former describes the U.S. system; the latter describes SI.
(See Table A.2.) Therefore, in U.S. units the mass of an object weighing W lb is
W / 3 2 . 2 slugs. Similarly, in SI units the weight of an object having a mass of M kg
is 9.81M newtons.

Table A.2

Gravitational System Absolute System

The basic units are force, length, and The basic units are mass, length,
time, and mass is derived: and time, and force is derived:

This system has traditionally been This system has traditionally been
more popular with engineers more popular with physicists.
As an example, in the U.S. As an example, in the SI
system of units the pound, foot, and (metric) system of units the
second are basic Thus the mass kilogram, meter, and second are
unit, the slug, is derived: basic. Thus the force unit, the
newton, is derived:

This is s u m m e d up by: A slug is the


quantity of mass that will be This is s u m m e d up by: A newton is
2
accelerated at 1 f t / s e c w h e n acted the amount of force that will
upon by a force of 1 lb accelerate a mass of 1 kg at 1 m / s 2

In the SI system the unit of moment of force is the newton • meter (N • m);
in the U.S. system it is the pound-foot (lb-ft). Work and energy have this same
dimension; the U.S. unit is the ft-lb whereas the SI unit is the joule (J), which
equals 1 N • m. In the SI system the unit of power is called the watt (W) and
equals one joule per second ( J / s ) ; in the U.S. system it is the ft-lb/sec. The unit of
Page 560

2
pressure or stress in the SI system is called the pascal (Pa) and equals 1 N / m ; in
2
the U.S. system it is the l b / f t , although often the inch is used as the unit of length
2
so that the unit of pressure is the l b / i n . (or psi). In both systems the unit of
frequency is called the hertz (Hz), which is one cycle per second. Other units of
interest in dynamics include those in Table A.3.

Table A3

Quantity SI Unit U.S. Unit

velocity m/s ft/sec


angular velocity rad/s rad/sec
2
acceleration m/s 2
ft/sec
2 2
angular acceleration rad/s rad/sec
2
mass m o m e n t of inertia kg . m 2
slug-ft
momentum kg • m / s slug-ft/sec
moment of m o m e n t u m kg • m / s 2
slug-ft /sec2

impulse N • s ( = kg • m/s) lb-sec


2
angular impulse N • m • s ( = kg • m / s ) Ib-ft-sec
3
mass density kg/m slug/ft 3

3 3
specific weight N/m lb/ft

Moreover, in the SI system there are standard prefixes to indicate multiplica­


tion by powers of 10. For example, kilo (k) is used to indicate multiplication by
1000, or 1 0 ; thus 5 kilonewtons, written 5 kN, stands for 5 X 1 0 N. Other
3 3

prefixes that commonly appear in engineering are shown in Table A.4. We reem-
phasize that for the foreseeable future American engineers will find it desirable to
know both the U.S. and SI systems well; for that reason we have used both sets of
units in examples and problems throughout this book.

Table A.4

tera T 10 12
centi c -2
10
9
giga G 10 milli m 1C -3

mega M 10 6
micro µ 1C -6

kilo k 10 3
nano n -9
10
hecto h 10 2
pico P
-12
10
-15
deka da 10 1
femto f 10
deci d 10- 1
atto a 10-8

We turn now to the question of unit conversion. The conversion of units is


quickly and efficiently accomplished by multiplying by equivalent fractions until
the desired units are achieved. Suppose we wish to know how many newton-
meters (N m) of torque are equivalent to 1 lb-ft. Since we know there to be
3.281 ft per meter and 4.448 N per pound,

1 lb-ft = 1 = 1.356 N • m

Note that if the undesired unit (such as lb in this example) does not cancel, the
conversion fraction is upside-down!
For a second example, let us find how many slugs of mass there are in a
kilogram:
Page 561

lkg = = 0.06852 slug

Inversely, 1 slug = 14.59 kg. A set of conversion factors to use in going back and
forth between SI and U.S. units is given in Table A.5.*

Table A.5

Reciprocal (to Get


To Convert From To Multiply By from SI to U.S. Units)

Length, area, volume


foot (ft) meter (m) 0.30480 3.2808
inch (in.) m 0.025400 39.370
statute mile (mi) m 1609.3 6.2137 X 1 0 - 4

2 2
f o o t (ft ) meter (m ) 2 2
0 092903 10.764
2 2
inch (in. ) m 2
6.4516 X 10 -4
1550.0
3 3
f o o t (ft ) meter (m ) 3 3
0.028317 35.315
3 3
inch (in. ) m 3
1 6387 X 10 -5
61024
Velocity
feet/second (ft/sec) meter/second (m/s) 0.30480 3.2808
f e e t / m i n u t e (ft/min) m/s 0.0050800 196.85
knot (nautical mi/hr) m/s 0.51444 1 9438
mile/hour (mi/hr) m/s 0.44704 2.2369
mile/hour (mi/hr) kilometer/hour (km/h) 1.6093 0 62137
Acceleration
2 2
f e e t / s e c o n d (ft/sec ) m e t e r / s e c o n d (m/s ) 2 2
0.30480 3.2808
2 2
i n c h / s e c o n d (in./sec ) m/s 2
0.025400 39.370
Mass
pound-mass (Ibm) kilogram (kg) 0.45359 2.20462
2
slug (lb-sec /ft) kg 14.594 0.068522
Force
pound (lb) or
pound-force (Ibf) newton (N) 4.4482 0.22481
Density
3 3
pound-mass/inch (lbm/in. ) kg/m 3
2 7680 X 1 0 4
3.6127 X 10 -5

3 3
pound-mass/foot (lbm/ft ) kg/m 3
16.018 0.062428
3 3
s l u g / f o o t (slug/ft ) kg/m 3
515.38 0.0019403
Energy, work, or moment of force
foot-pound or pound-foot joule (J) 1.3558 0.73757
(ft-lb) (lb-ft) or newton • meter (N • m)
Power
foot-pound/minute (ft-lb/min) watt (W) 0.022597 44.254
horsepower (hp) (550 ft-lb/sec) W 745.70 0 0013410
Stress, pressure
2 2
p o u n d / i n c h ( l b / i n . or psi) 2
N / m (or Pa) 6894.8 1.4504 X 10-4

2 2
p o u n d / f o o t (lb/ft ) 2
N / m (or Pa) 47.880 0.020886
Mass moment of inertia
2 2 2
slug-foot (slug-ft or lb-ft-sec ) kg • m 2
1.3558 0.73756

• Rounded to the five digits cited. Note, for example, that 1 ft = 0.30480 m, so that

(Number of feet) X = number of meters


Page 562

Table A.5 Continued

Reciprocal (to Get


To Convert From To Multiply By from SI to U.S. Units)

Momentum (or linear momentum)


slug-foot/second (slug-ft/sec) kg • m / s 4 4482 0.22481
Impulse (or linear impulse)
pound-second (lb-sec) N • s (or kg • m/s) 4 4482 0 22481
Moment of momentum (or angular momentum)
2 2 2
slug-foot /second (slug-ft /sec) kg • m / s 1 3558 0 73756
Angular impulse
pound-foot-second (Ib-ft-sec) 2
N • m • s (or kg • m / s ) 1 3558 0 73756

Note that the units for time (s or sec), angular velocity (rad/s or 1/s), and
2 2
angular acceleration ( r a d / s or 1/s ) are the same for the two systems. To five
2
digits, the acceleration of gravity at sea level is 32.174 f t / s in the U.S. system
2
and 9.8067 m / s in SI units.
We wish to remind the reader of the care that must be exercised in numerical
calculations involving different units. For example, if two lengths are to be
summed in which one length is 2 ft and the other is 6 in., the simple sum of these
measures, 2 + 6 = 8, does not of course provide a measure of the desired length.
It is also true that we may not add or equate the numerical measures of different
types of entities; thus it makes no sense to attempt to add a mass to a length.
These are said to have different dimensions. A dimension is the name assigned to
the kind of measurement standard involved as contrasted with the choice of a
particular measurement standard (unit). In science and engineering we attempt
to develop equations expressing the relationships among various physical enti­
ties in a physical phenomenon. We express these equations in symbolic form so
that they are valid regardless of the choice of a system of units, but nonetheless
they must be dimensionality consistent. In the following equation, for example, we
may check that the units on the left and right sides agree; r is a radial distance, P is
a force, and dots denote time derivatives:
2
P — mg cos 6 = m(r — r0 )

Dimensions of are

P F

mg cos d

mr

2
-mr8

Therefore the units of (every term in) the equation are those of force. If such a
check is made prior to the substitution of numerical values, much time can be
saved if an error has been made.
Page 563

PROBLEMS • Appendix A

A.1 Find the units of the universal gravitational con­ A.6 Is the following equation dimensionally correct?
stant G, defined by
(v = velocity;
a = acceleration)

A. 7 The equation for the distance r from the center of


5

in (a) the SI system and (b) the U.S. system. the earth to the geosynchronous satellite orbit is
A.2 Find the weight in pounds of 1 kg of mass. (i = angular speed of earth;
A. 3 Find the weight in newtons of 1 slug of mass. = earth radius)
A.4 One pound-mass (lbm) is the mass of a substance a. Show that the equation is dimensionally correct.
that is acted on by 1 lb of gravitational force at sea level. b. Use the equation to find the ratio of the orbit
Find the relationship between (a) 1 lbm and 1 slug; (b) radius to earth radius.
1 lbm and 1 kg.
A.8 The universal gravitational constant is G = 6.67
A. 5 The momentum of a body is the product of its mass
X 10 N • m / k g . Express G in units of lb-f^/slug .
-1 2 2 2

m and the velocity v of its mass center. A child throws an


c

8-oz ball into the air with an initial speed of 20 mph. Find
the magnitude of the momentum of the ball in (a) slug-
ft/sec; (b) kg • m / s .
B B EXAMPLES OF NUMERICAL ANALYSIS / THE NEWTON-RAPHSON METH

There are a few places in this book where equations arise whose solutions are not
easily found by elementary algebra; they are either polynomials of degree higher
than 2 or else transcendental equations. In this appendix we explain in brief the
fundamental idea behind the Newton-Raphson numerical method for solving
such equations. We shall first do this while applying the method to the solution
for one of the roots of a cubic polynomial equation that occurs in Chapter 7.
To solve the cubic equation of Example 7.5,

we could, alternatively, use the Newton-Raphson algorithm. This procedure


finds a root of the equation = 0 (it need not be a polynomial equation,
however) by using the slope of the curve. The algorithm, found in more detail in
any book on numerical analysis, works as follows. If is an initial estimate of a
root , then a better approximation is

Figures B. 1 and B.2 indicate what is happening. The quantity causes


a backup in the approximation — in our case from the initial value of 3 to the
improved estimate ,:
1342

Actual root
= 3 - 0.408602150
w h i c h w e seek
= 2.591397850
where

-152 so that/'(3) = —372. Repeating the algorithm, we get

Figure B.1

= 2.591397850 + 0.016270894
Page 564 = 2.607668744
Appendix B / Examples of Numerical Analysis/The Newton-Raphson Method Page 565

This, distance is

Actual root
which
we seek

Old Figure B.2


estimate
Root

And one more time:

Figure B.3

= 2.607668744 + 0.000026410
= 2.607695154

This algorithm is easily programmed on a computer. After doing this, the results
(with the same initial guess = 3) are:

=3
Old = 2.591397850
Root estimate
= 2.607668744
= 2.607695154
= 2.607695156
= 2.6076951531
Figure B.4 = 2.607695153 convergence!
= 2.607695153J
which is in agreement with the results in Example 7.5.
Incidentally, note from Figures B.3 to B.5 that adding to form the
new estimate works equally well for the three other sign combinations of f andf'.
Note also that if the estimate is too far from the root, such as P in Figure B.4, the
procedure might not converge; the tangent at Q in this case would send us far
from the desired root.
Rool We next consider the equation from Problem 5.140 when M = 4m:

(B.l)

Old estimate with the derivative of / being

Figure B.5
Page 566

functions or There is but one root of Equation (B. 1) for > 0, as can be seen from Figure B.6,
which shows the two functions making up . To find this root, we can use
Newton-Raphson as previously described. Figure B. 7 suggests that might serve
as a good first guess at the root. A Newton-Raphson program shows that it is, and
Root yields the answer below very quickly:

3.141592654

Figure B.6 2.094395103


1.913222955
1.895671752

1.895494285

1.895494267'

1.895494267 convergence!

1.895494267-
Figure B.7
The last example in this appendix will be to solve the equation

from Example 2.6. We write this equation as

with

f(q) The rough plot in Figure B.8 shows a few points which indicate that is fairly
close to the root. Here are the results of a program, which uses the Newton-
Raphson method as in the first two examples, to narrow down on the root quickly
0.707 and accurately:

1.570796327
0.121-
1.683007224

1.679300543
1.679296821'
1.679296821 convergence!
-0.879
-1 1.679296821-
Figure B.8
c C MOMENTS OF INERTIA OF
MASSES (SEE ALSO SECTION 4 . 3 )

Mass Center Coordinates M o m e n t s of Inertia


Object and Volume V About Indicated Axes

slender rod 10, 0, 0)


V = At
(A = area of cross section)

slender
circular rod

bent
slender rod

Page 567
Page 568 APPENDIX C / MOMENTS OF INERTIA OF MASSES

Mass Center Coordinates M o m e n t s of Inertia


Object and Volume V About Indicated Axes

rectangular (0, 0, 0)
solid
V = abc

hollow (0, 0, 0)
cylinder 2 2
V = TT(IR - i )H

Four Special (0, 0 0)


;

Cases 2
V = irR H
1. If r = 0:
solid
cylinder

[0, 0, 0)
V = 2-rrRtH

(0, 0, 0)
2 2
V = r{R - r )H
Page 569

Mass Center Coordinates M o m e n t s of Inertia


Object and V o l u m e V About Indicated Axes

thin right
triangular
plate

thin elliptical
plate

thin
paraboloidal
plate

thin circular
sector
plate
Page 570

Mass Center Coordinates M o m e n t s of Inertia


Object and Volume V About Indicated Axes

2. If a = 2<rr:
circular
plate

thin circular
segment
plate

rectangular
tetrahedron

hollow sphere

solid ellipsoid
Page 571

Mass Center Coordinates M o m e n t s of Inertia


Object and Volume V About Indicated Axes

solid
spherical cap

paraboloid of
revolution

ei iptic
paraboloid

solid cone

solid right
rectangular
prism
Page 572

Mass Center Coordinates M o m e n t s of Inertia


Object and V o l u m e V About Indicated Axes

solid toroid

frustum
of cone
D D ANSWERS TO
ODD-NUMBERED PROBLEMS

In the solutions to problems in Chapters 1 - 5 , unless identified otherwise below,


and are unit vectors in the respective directions , and out of the page. In
Chapters 6 - 8 , the unit vectors are respectively parallel to axes defined in the
problems.

C H A P T E R 1

a,
z
30.8
5

2 11.3
1.17 Answer given in problem. I
2 5 2 5 D

1.33 15 sec to return (21 seconds total elapsed time)

a,
5 10 15 20 20 ,

- 125 -10
-1125

Page 573
Page 574 APPENDIX D / ANSWERS TO ODD-NUMBERED PROBLEMS

2
1.129 6.5f + C , where C is a constant of integration

1.97 For x within the intervals (290, 1200) ft and


(1800, 2700) ft

1.155 Answers given in problem.

C H A P T E R 2

2.1 Answer eiven in problem.

measuring r from pulley to

bumper, is the velocity of the shingles; it is also the


component of along the rope.

2.47 Answer given in problem. 2.49 0.788 sec


2.51 (a) 13.1 m (b) smaller because now resists
the motion of _ 2.53 0.032 lb 2.55 136 N

2.59 Answer given in problem.


2.61 (a) a (vertical) component of the string tension;
1.125 Answer given in problem.
1.127 Answer given in problem.
Page 575

(c) another component of the string tension, this one m 2.173 11.5 ft upward 2.175 rf/4 upward
the direction. 2.177 Answer given in problem.
2.63 170 miles 2.65 Answer given in problem. 2.179 Answer given in problem.
2.67 3.13 rad/sec 2.69 p = a / b ; 52.0 mph, so yes.
2

2.71 0.5, at =0
C H A P T E R 3

2.79 Answer given in problem. 3.1 a,c,d,e

2.97 Answer given in problem.


2 . 9 9 Answer given in problem. (Set N = 0 to find the
leaving point.) 2.101 76,300 l b / f t
2.103 20.7 ft/sec down the plane.
2.105 (a) 12 lb (b) 2.84 ft/sec 2.107 72 l b / f t 3.27 The plots can be constructed from the answer to

3.31 In each case, is at the intersection of the radial


line OA and the normal to the slot at B;

2.129 Distance between them is ( — 0.02) m, where


= unstretched length. They are 0.22 m closer
together. Final spring force = 1 lb (compressive).
2.131 1.41 m i l e s / s e c 2.133 7.45 sec
2.135 36.2 — f t / s e c 2.137 7.86 sec; 491 f t / s e c

2.147 Answer given in problem. (If e > 1, there would


be an energy gain!) 2.149 0.446

2.165 Answers given in problem.

2.169 The derivation of Equation (2.36) nowhere


requires that the point be the mass center.
2.171 Answer given in problem.
Page 576

3.141 Answer given in problem.

3.153 Answer given in problem.

3.157 Answer given in problem.

C H A P T E R 4

3.131 Let x and y respectively be directed down and


toward the plane, with origin at the center of the disk.
Then the point has (x, y) = (4.80, 3.60) ft.

4.33 Answer given in problem.

2
3.137 21.6 m / s (It is the highest point of p.)
3.139 (a) Answer given in problem,
(b) Curve is concave downward.
Page 577

4.41 From the comet, 4.95 (a) Answer given in problem.

4.55 Answer given in problem

4.61 Answer given in problem; only (b) starts without approximation.


4.63 2 g / 3 ; 5 g / 7 ; g / 2
4.65 (a) Wally, Sally, Carolyn, Harry;

4.133 Answers given in problem.

4.143 On the section to the left of the cut,

10.4
8,27

4.149 Answer given in problem.


2.07 3
4.93 (a) (a)
4.157 For rolling on fixed surface, is normal to the
surface, hence toward geometric center of round body;
thus since geometric center is mass center.
10.4

2.07 3
(b) lb]

13.8

9.51 2.85

1.48 3

to
Page 578

5 . 5 9 It starts out to the right, the spring goes slack, and


then it leaves on the right. (It would need one more
foot of plane to stay on.)
up on left bearing and down on right bearing.

down on left and up on

right, onto shaft and turning with it.


4 . 1 9 1 By parallel-axis theorem, since
C is on z-axis. Thus using the theorem again, = 0.
Same arguments for

5 . 7 3 Answer given in problem.


C H A P T E R 5

5 . 4 3 8 in.; the two points are the intersections of the


perimeter of (in the starting position) with a circle of
radius 12 in. and center at (in the final position! 5 . 1 2 1 Answer given in problem. 5.123

final motion is given by (a).


Page 579

6.43

5.133 Answer given in problem.

2
5.137 0.545 m from left end; 0.562 kg-m ;
2
0.0957 ke-m ; 0.657 m from left end is a "force" that will change the
particles' velocity directions relative to the earth so as
to produce ccw rotation in the northern hemisphere.
The effect is opposite in the southern hemisphere.
6.47 Answer given in problem.

CHAPTER 6
6.1 Answer given in problem.

6.15 To the components in 6.13, respectively,


6.63 1.36 rad/s, directed from O through the line of
and the
contact between C and 2>.
cross-product is not generally zero this time.
6.67 Answer given in problem.
Also, so right side is (6H - 260)<a«

6.71 (a) Answer given in problem;

6.75 (a) Answer given in problem.


(b) Answer same if is replaced by at.

6.41 Answer given in problem.


Page 580

6.83 Let point A be displaced from its original to its


final position. Then, using Euler's Theorem, all other
points of the body may be placed in their final
positions via a single rotation about an axis through A.

with in any direction normal to

working with six digits


6 . 9 3 Be sure the cross is ngid and planar! and rounding at the end

C H A P T E R 7

7.1 With along the axle from O through the wheel


center, and out of the page,

7.5 With and parallel to x and y of the figure,


Note: There is a precision problem here because
(1) and are so much larger than
and (2) and are nearly equal.

7.19 If , the ellipsoid


at B: same magnitude but
cannot be an ellipsoid of inertia, for then it would
represent a body having one moment of inertia sum
of other two, a physical impossibility.

the direction is different.

so that z is a principal axis for every point on that axis

7.55 332 days

7.63 With x along S from Q and y upward,


PAGE 5 8 1

7.95 815m ft-lb, where m is the mass in slugs

7.69

(c) no hole is physically possible;


7.103 358 1

7.107 Some check values:


7.71 With from O toward C, and j in the direction

of v , F on
c by the bearing at C,

from gravity, and mgk from the ground;


C H A P T E R 8
and C on 3 by bearing =

The difference is that in this problem the normal


force from the ground is just the weight.
7.73
7.75 RHS of x-eqn = 5.6 (the same);
RHS of y-eqn = 5.6 7.7; RHS of z-eqn = 2.8 1.4;
Axes used for Euler Equations must be body-fixed, 8.9 Answer given in problem.
not just permanently principal.
8.13 (a) 0.278 ft (b) 0.876 sec (c) 0.291 sec

B.I5 (1) moves to left with x

(2) moves to right with x


(i) is unstable and the other three are stable.
7.85 For a torque-free body in general motion in an (3) moves to left with x
inertial frame with not parallel to ,
stops for good to left of unstretched position.
we have
Time in each interval is and total distance
and the two terms add to zero.
traveled
7.87 If is in equilibrium in all its points are
stationary there; thus for all these points,
and also Hence is an inertial frame.
But if is an inertial frame, it can at most translate
at constant velocity with respect to another
inertial frame . Thus it need not be stationary in
i.e., need not be in equilibrium in
even though none of its points accelerates in
7.89 Answer agrees with Example 7.11.

8.25 Yes, they do. (Start with


7.93 No. Two different results are obtained for and investigate when
Page 582

(c) Mechanical energy is lost (to heat, deformation,


vibration, etc.) as the links suddenly join the falling
part of the chain. 8.57 Answers given in problem.
8.59 (a)0.193gT. (b) Answer given in problem.
8.61 Answer given in problem.
8.63 Answer given in problem. (Use Kepler's laws!)

8.67 107 min ("No air resistance" needn't be assumed


this time!) 8.69 4320 mi; 583 mi and 137 mi
the radicand is , and for these co's,
the radicand is smaller for smaller values of c. 8.73 Answer given in problem. (Isolate the radical and
square both sides.) 8.75 Answer given in problem.
where W is the weight of tank plus fluid.

8.47 (a, c) Answer given m problem;


A P P E N D I X A

8.49 Answer given in problem.


valid until fuel
gone
8.53
INDEX

Absolute system, 559 Balance, static, 292


Acceleration, 6, 8 Balancing a rotating body, 295
angular, 163-164, 398-399 Bearing reactions, 291-294
Coriolis, 207, 405-406 Binormal vector, unit, 51
in different frames, 207-208 Body, 56
equations of, 414-415 cone, 496
in moving frames of reference, 401, mass center of, 62-64
405-406 rigid. See Rigid body
radial and transverse components of, unbalanced, rotation of, 291 —
32 293
tangential and normal components of, Body extended, 130
43
Action-reaction principle, 64
Addition theorem for angular velocities, Cardan suspension, 431
385-386,428 Cartesian coordinates, 24, 381
Amplitude, 518 Center
Angle of curvature, 49
Eulerian, 428-433 of mass, 58-60, 62,219
friction, 66 of percussion, 362
nutation, 493 of zero velocity, 149-151
phase, 518 Central force, 89-90
Angular acceleration, 163-164 Central force motion, 543
Angular acceleration vector, 398-399 Central impact, 107
Angular impulse, 344 Chasle's theorem, 434
Angular momentum, 118, 227-228 Circular frequency, 518
conservation of, 350 Circular orbit, 547
relative, 124 Coefficient of friction, 65
in three dimensions, 446 Coefficient of restitution, 106-108, 355,
Angular speed, 136, 184 361
Angular velocity, 134-135,136, Cones, body and space, 496
380-381, 383, 432 Conic, 546
addition theorem and, 385-386 Conservation
differentiation of, 387 of angular momentum, 350
properties of, 384-388 of energy, 92
simple, 386-388 of mechanical energy, 92, 330-331
Apogee, 544 of moment of momentum, 120-121
Ardength, 43 of momentum, 102-103, 350
Axes of inertia Conservative forces, 91-92, 329-
non-principal, 473 331
principal, 455, 456, 462, 466 Constant force, 89
Axis of rotation, 277-278, 422 Continuity equation, 535
Page 583
Page 584

Control volume for fixed-axis rotation, 2 7 7 - 2 7 8


Euler's laws for, 5 3 5 - 5 3 6 Escape velocity, 85, 553
Coordinates Euler equations, 432, 4 7 2 - 4 7 3
Cartesian, 24, 381 Eulerian angles, 4 2 8 - 4 3 3
cylindrical, 31 Euler's first law, 5 6 - 5 8 , 62
polar, 31 control-volume form of, 536
spherical, 43 momentum forms of, 1 0 1 - 1 0 2
Coriolis acceleration, 207, 4 0 5 - 4 0 6 Euler's second law, 1 1 7 - 1 1 8 , 495
Coulomb law of friction, 249 control-volume form of, 5 3 6
Couple, 309, 311 momentum forms of, 1 1 9 - 1 2 0
Cramer's rule, 457 External forces and couples, 5 8
Critical damping, 522
Curvature, 4 5 , 149 Fixed-axis rotation, 2 7 7 - 2 7 8
center of, 49 Fluid flow, 535
radius of, 45 Focus of conic, 546
Curvilinear translation, 148 Foot, 558
Cylindrical coordinates, 31 Force, 355
conservative, 9 1 - 9 2 , 3 2 9 - 3 3 1
Damped vibration, 521 nonconservative, 331
Damping work and, 8 9 - 9 0 , 309, 311, 313
critical, 522 Forced vibration, 5 2 4 - 5 2 5
subcritical, 523 Frame
supercritical, 523 earth as moving, 4 1 0 - 4 1 1
Deformation, 6 0 , 1 1 6 intermediate, 4 7 4
Derivative, 399 of reference, 3, 7, 401, 4 0 5 - 4 0 6
of a vector, 3 acceleration and, 2 0 7 - 2 0 8 , 4 0 1 ,
in different frames, 1 9 8 - 2 0 0 405-406
Derivative formula, 384 inertial, 57
Differential equations velocity and, 1 9 8 - 2 0 0 , 401
complementary solution, 5 2 6 Free vibration, 5 1 7 - 5 1 8
energy integral, 6 8 Free-body diagram, 6 2 - 6 4
particular solution, 526 Frequency, 518
of plane motion, 246 Friction, 249
Direct impact, 107
Direct precession, 4 9 6 Gears, 170, 1 8 3 - 1 8 4
Displacement, 522 Geosynchronous orbit, 81, 439
steady-state, 526 Gravitation, 57
Drag, 74, 85 Gravitational system, 559
Gravity, 9 1 , 351
Earth Gyration, 2 3 9 - 2 4 0
as a moving frame, 4 1 0 - 4 1 1 Gyroscopes, 4 9 2 - 4 9 5
gravity and, 91 Gyroscopic moment, 481
lunisolar precession of, 494, 497 Gyroscopic precession, 494
motion near the surface, 410
orbit around sun, 440 Harmonic motion, simple, 518
satellites, geosynchronous, 81, 439 Helix, 35
Eccentricity, 5 4 6 Hertz, 560
Eigenvalue, 457 Hooke's joint, 391
Eigenvector, 4 5 7
Ellipsoid of inertia, 454 Impact, 103, 3 5 5 - 3 5 6
Energy central, 107
conservation of, 92 direct, 107
kinetic. See Kinetic energy energy loss, 113, 356
mechanical, 3 3 0 - 3 3 1 restitution coefficient for, 1 0 6 - 1 0 8
potential, 3 2 9 - 3 3 1 Impulse, 1 0 2 - 1 0 3 , 344, 4 9 9 - 5 0 0
Energy integral, 6 8 Impulsive force, 3 5 5
Equations of motion, 2 4 9 - 2 5 0 , 2 7 2 - 2 7 3 , Inertia
274 ellipsoid of, 455
Page 5 8 5

moments of, 229, 235-236, 455, 457, maximum and minimum, 468
458, 466 of momentum, 118-119,120-121,
maximum and minimum, 468 227-228
principal axes of, 455, 456, 462, 466 conservation of, 120-121
products of, 240-241 in three dimensions, 446
torque (moment), 488 work of, 313
Inertia properties, 228-229 Moment equations of motion, 249-250,
transformations of, 448-449 272-273, 274
at a point, 449 forfixed-axisrotation, 277-278
Inertial frame of reference, 57 governing rotational motion, 472-473
Instantaneous axis of rotation, 149-151, Momentum, 343,499-500
422 angular, 118, 227-228, 350, 446
Instantaneous center of zero velocity, conservation of, 102-103, 350
149-151 and Euler's second law, 119-120
Intermediate frame, 474 linear, 101
Intrinsic components of velocity and moment of, 118-119,120-121,
acceleration, 43 227-228, 446
net rate offlowof, 535
Jerk, 52 of a particle, 101-102
Joule, 88, 359 Motion, 176, 305-306
central force, 543
Kane, T. R„ 383, 432 equations of, 249-250, 272-273,
Kepler's laws of planetary motion, 544, 274, 277-278
548, 550 of mass centers, 58-60, 62
Kilogram, 558 of particles, 62
Kinematics, 2, 7 planetary, Kepler's laws of, 544,548,
of a point, 8 550
Kinetic energy, 504 rectilinear, 6
alternative form of, 308 rotational 492
mass center, 87, 306 simple harmonic, 518
of a particle, 88 torque-free, 473,495, 497
rate of change of, 309 translation^, 147
ofrigidbody, 305-306 Moving a derivative, 399
rotational, 308, 506 Moving frames of reference, 401,405-406
translauonal, 308,506
work and, 87-89, 97 Natural circular frequency, 518
principle of, and, 305-306, 316, Net rate offlowof momentum, 535
327-328 Newton, 558
Kinetics, 2 Newton, I., 56
Newton frame of reference, 57
Levinson, D. A., 432 Newton-Raphson method, 564
Linear impulse, 102-103, 344, 499-500 Newton's laws, 56-58
Linear momentum. See Momentum Nonconservative force, 331
Linear spring, 90 Nonimpulsive force, 355
Local vertical, 411 Non-principal axes, 473
Lunisolar precession, 494, 497 Normal component of acceleration, 46
Normal, principal unit, 46
Mass center, 58-60, 62, 219, 246-247 Normal and tangential components of
Matrices, 434-435 velocity and acceleration, 43
Maximum moments of inertia, 468 Nutation angle, 493
Mechanical action, 56
Mechanical energy, 92, 330-331 Orbit
Meter, 558 apogee of, 544
Minimum moments of inertia, 468 eccentricity of, 546
Moment perigee of, 544
of inertia, 229, 235-236, 455, 457, period of, 549
458, 466 Orientation of arigidbody, 132, 428
of masses, 567-572 Orthogonal components, 4
Page 586

Orthogonality of principal axes, 466 parallel-axis theorem for, 241


Overdamped system, 523 Projectile motion, 65

Parallel-axis theorem Quantity of matter, 56


for moments of inertia, 235-236
for products of inertia, 241 Radial components of velocity and
Particle, 2,136 acceleration, 32
momentum of, 101-102 Radius
motion of, 62 of curvature, 45
work and kinetic energy for, 87-89, of gyration, 239-240
97 Rate
Particular solution, 526 of change of kinetic energy, 309
Path, 6 of work of a couple, 311, 508
Percussion, 362 of work of a force, 81, 311,508
Perigee, 544 Rectangular Cartesian coordinates, 24
Period, natural, of vibration, 518 Rectilinear motion, 8
orbit, 549 Rectilinear translation, 148
Phase angle, 518 Reference, frame of. See Frame of reference
Pivot, 152-153, 277-278, 473 Reference plane, 131-133
Plane, reference, 131-133 Relative
Plane motion, 130 acceleration, 207, 405
equations of angular momentum, 124
impulse and momentum for rigid velocity, 198, 402
body in, 343-345 Resonance, 526
motion of, 246-247 Restitution, coefficient of, 106-108, 355,
kinetic energy of rigid body in, 361
305-306 Resultant force, 57, 248
Point, 8 Resultant moment, 248
Poisson equations, 433 Retrograde precession, 497
Polar coordinates, 31, 249 Reversed effective forces, 488
Position vector, 6 Reynolds transport theorem, 535
Potential energy, 92, 329-331 Rigid body(ies), 3,130,198
of a central force, 90, 544 angular acceleration of, 163-164
of a constant force, 90, 330 kinetic energy of, 305-306
of a linear spring, 90, 330 orientation of, 132, 428
Pound, 558 points on
Power, 309, 311, 508 acceleration of, 163-164
Precession, 91, 493-495, 496, 497 velocities of, 134-135
direct, 496 principle of impulse and momentum
gyroscopic, 494 for, 343-345
lunisolar, 494, 497 in translation, 219-220
retrograde, 497 translational motion of, 147, 348
Primitive, 56 velocity and acceleration equations
Principal axes of inertia, 294, 455, 456, for, 414-415
462, 466 work and kinetic energy principles
Principal directions, 458-459 for, 311-312
Principal moments of inertia, 455, 457, Rigid extension of the body, 130
458, 466 Robot, 397
Principal unit normal vector, 46 Rocket, 537
Principle of Rolling, 170-172, 176
action and reaction, 64 Rotation
impulse and momentum, 343-345, of axes (transformation of inertia
500 properties), 448-449
Principle of axis of, 277-278
work and kinetic energy, 305-306, instantaneous, 149-151, 422
316, 327-328 of unbalanced bodies, 291-293
Products of inertia, 240-241 Rotation matrices, 434-435
Page 587

Rotational motion, 492 Unit, 558-562


Unit conversion, 560-562
Satellite, 81, 439, 474, 498 Unit tangent, 44
Second, 558 Universal gravitation, 57
Serret-Frenet formulas, 52 Universal joint, 391
SI units, 559
Simple angular velocity, 386-388 Vector, 472
Simple harmonic motion, 518 angular acceleration, 398-399
Single-degree-of-freedom system, 517 angular velocity, 384-385
Sliding friction, 66 derivatives of, 3,384
Slug, 558 position, 6
Space cone, 496 principal unit normal, 46
Speed, 6 Velocity, 6,107
angular, 136 angular. See Angular velocity
Spring, linear in different frames, 198-200, 401
modulus, 90 of points in a rigid body, 134-135
work done by, 90, 314-315 radial and transverse components of,
Steady-state displacement, 526 32
Subcritical damping, 523 tangential and normal components of,
Supercritical damping, 523 43
zero, 149-151,176, 422-423
Tangent vector, unit, 44 Velocity equations for rigid bodies,
Tangential and normal components of 414-415
velocity and acceleration, 43 Vibration, 517
Thrust, rocket, 537 damped, 521
Torque-free motion, 473, 495, 497 forced, 524-525
Total energy, 92 free, 517-518
Trajectory, ballistic, 64 v-t diagram, 11
Transfer theorem. See Parallel-axis
theorem Wave propagation, 116
Transformation of inertia properties, 449 Work, 309, 311, 313
Translation, 147-149 of a couple, 311
rigid bodies in, 219-220 of a force, 89-90, 309, 311, 313
Translational motion, 147, 348 of gravity, 91
Transverse components of velocity and and kinetic energy, 87-89,97,
acceleration, 32 305-306, 316, 327-328,504
Truesdell, C, 57 rate of, 508
of a spring, 90
Unbalanced bodies, 291-293
Underdamped system, 523 Zero velocity, 149-151,176, 422-423
Page 589

Rationale for the New Tear-Out Section

This new set of tear-out problems is designed to assist the student in summarizing and synthesizing
the more important equations and procedures in a first course in Dynamics. The goal of each problem is
stated at its outset.

These problems lie somewhere between the book's examples and homework problems. They are
like homework problems (and unlike examples) in that they ask the student to do the work; but they
are like the examples (and unlike homework problems) in that they lead the way.

Thus we view these problems as a learning step between (a) classroom lectures/textbook examples
(in which the student is passive) and (b) homework problems (in which he/she is active). Should the
professor wish to use them, the tear-out feature and "fill-in-the-blanks" format make submission and
grading relatively easy.
Page 591

GOAL 1:
To demonstrate understanding of both
rectilinear motion and motion on a circle.

Point P is projected at from point A along the circle shown,


with initial velocity and with . One sec­
ond later, point Q is projected from point B along BC with initial
speed and with . Determine so that P and Q
will reach point C simultaneously.

1. Integrate twice and determine the constants using initial conditions on the motion of P:

2. Substitute for 0 and determine the time at which P arrives at C:

3. Now, integrate twice and determine the constants using initial conditions on the motion
of Q; use a new time t for Q:
2
Page 592

4. Substitute t-1 for t in ( 3 ) :


2

5, Finally, substitute the value of XQ at point C, and of f from (2), to determine the required value
of :

Note that at any speed above this value, Q will arrive too soon; at any speed below it, Q will get there
too late.
PAGE 5 9 3

GOAL 2: To demonstrate an understanding of the use of cylindrical


coordinates to determine the velocity and acceleration of a point.

Point P travels on the spiral path around the surface of a cone as shown.
Points on the path of P obey the relationship Find and
when , if the angular rate is a constant.

1. Use the sketch below to write r as a function of z, and then incorporate to write r as
a function of :

2. Use Eq. (1.31) to obtain as a function of and the radius R and height H of the cone:
Page 5 9 4

3. Using your answer to (2), give the reason why the speed of P cannot be constant in
this problem:

4. Use Eq. (1.37) to obtain ap as a function of the same quantities:


Page 595

G O A L 3: To demonstrate understanding of tangential and normal


components of velocity and acceleration of a particle.

A particle P starts at rest at point A, and accelerates uniformly


for 10 seconds, when it reaches B. Find the tangential and
normal components of the velocity and acceleration of P just
before and just after P reaches B. If now at B the particle begins
to decelerate uniformly and reaches C 5 seconds later, find the
acceleration components of P when it reaches C.

1. From A to B, write an expression for in terms of the unknown acceleration magnitude of P:

2. Integrate to obtain expressions for and s:

3. Substitute s = 1000 m at t = 10 sec to obtain the acceleration magnitude of P between A and B, and
then the speed and velocity of P at B (noting that the velocity is always tangent to the path):
Page 596

4. The velocity in (3) is the same just before, when, and just after P reaches B. However, the
acceleration is not. Use (3) to express just before P reaches B:

5. Now find just after P passes B:

6. Now, between B and C, express as a new constant, with S2 measured from B:

7. Integrate twice, starting time over at B and noting that

8. Substitute at sec and obtain of P between B and C, and then the speed
of P at C:

9. Using (8) find a at point C:


P
Page 597

GOAL 4: To demonstrate ability to use rectangular Cartesian coordinates to compute


Vp and a , and to correlate the results with tangential and normal components.
P

Y
A particle P moves on the indicated circular path in a counterclock­
2 wise direction after starting from the origin of coordinates shown.
The speed of P is = constant. As a function of x, find the velocity
and acceleration of P by respectively differentiating and in
rectangular Cartesian coordinates. Check the results using tangential
and normal components for the three dotted points.

(0,0)
l 2 X

1. Write in terms of x, by solving for y from the given equation of the circle:

2. Differentiate to obtain in terms of x and :

3. Use to express in terms of and x:

Note from your answer that and cannot both be constant.


Page 5 9 8

4. Eliminate from (2) using (3), and thereby obtain as a function of x (and ):

5. Check by verifying that

6. Now differentiate from (4) and obtain a in terms of x (and


P ):

7. Check the answers for and at (0, 0) using tangential and normal components:
Page 599

8. Repeat (7) at the point (2, 2):

9. Repeat (7) at the point (1, ):


Page 601

GOAL 5: To demonstrate understanding of kinematics


and kinetics of rectilinear motion.

The small boxes and (each of mass m) are


connected by an inextensible cord that passes over
a small, smooth peg. Man is running at constant
speed to reach before reaches man ,
who is pulling down on a second cord with the
constant force Kmg. The friction coefficient
between and its floor is and both boxes start
from rest in the positions shown. Find the smallest
value of man speed such that he will reach
in time.

1. Complete the sketches below, with arrowheads and names of forces, to form fbd's of and

2. Noting the X and y directions in the figure, write theX-equationof motion for
1 and the y-equa-
tion for :

3. Eliminate the cord tension by combining your two equations:


Page 602

4. You should now have one equation in two unknowns, and . Write a kinematic equation
expressing constancy of the rope length, and differentiate it twice to obtain in terms of :

5. Substitute your result into ( 3 ) and integrate twice to obtain y(t) as a function of t (and the
constant K):

6. Set y = H to get the time t when


2 reaches man M : 2

7. When reaches , how far will have moved?

8. Integrate =V 0 to obtain x as a function of time:


2
603

9.. Compute the time t for M to reach


1 1

10. Set the time and answer the original question:

11. Show that your answer makes sense, namely that (a) the smaller is K, (b) the smaller is g, (c) the
smaller is H, and (d) the larger is then the smaller will be the required value of V .
0

12. Note that in (11) the answers for (a) and (d) are obvious. But (b) and (c) would seem to have
cancelling effects on and . Explain why they do not.
Page 605

G O A L 6: To demonstrate understanding of E q . ( 3 . 8 ) .

The block moves to the left


with a prescribed motion
mm. It is pinned
at A to rod , which is itself pinned to the center of gear at B. A cord
wrapped around a slot near the periphery of is fixed to block at Q.
Find at the given instant, at which t = 3 s. (The teeth on the gear and
wall are not shown.)

1. First, show that = 6ti mm/s, so that at t = 3, =18 mm/s:

2. Next, use Eq. (3.8) to relate (from (1)) to and on the rod. In your equation, be sure to
incorporate the constraint on the motion of B:

3. Solve and obtain V = 24 mm/s and


B
Page 606

4. Now, again using Eq. (3.8), relate (from (3)) to and on the gear, where T is the
"tooth point" in contact with the rack (a rack is a "straight line gear," in this case forming
the vertical fixed wall):

5. Solve and obtain

6. Next, use Eq. (3.8) for a third time to relate and to , where D is the point on the gear
directly above Q and to the left of B:

7. Solve and obtain

8. Finally, state why

Note the rigid adherence to the three rules accompanying Eq. (3.10) each and every time Eq. (3.8)
is used.
Page 607

G O A L 7: To demonstrate understanding of
the instantaneous center of zero velocity.

Repeat the preceding tear-out problem, using the concept of instantaneous center instead of Eq. (3.8).

1. Show with a sketch that — the instantaneous center of zero velocity of — is the point directly
below A and to the right of B:

2. Using , show first that = 0.2O rad/s and then that

3. State why T is

4. Using and from above, show that = 0.6O rad/s:

5. Using and from above, find

And as in the previous example, . Note that in using the instantaneous center, we
avoid the use of vectors and are usually finding one unknown at a time. Arrows (such as ) are
more helpful in this approach than unit vectors, and you must assign the proper directions to velocities
and angular velocities as you work through the problems.
Page 609

GOAL 8: To demonstrate understanding of Eq. (3.19)


and the rolling result Eq. (3.26).

In tear-out problem (6), find the acceleration of point Q.

1. Show that

2. Next, use Eq. (3.19) on to relate and

;
3. Solve and obtain and :

Note that if you tried to use to obtain and , you would obtain incorrect answers.
The instantaneous center of zero velocity is just what it says; it does not work for accelerations

4. Next, use Eq. (3.19) again, this time on to relate and (above) to
Page 6 1 0

At this point, note that the above vector equation, which comprises two scalar component equations,
contains three unknowns: and the x and y components of . But the gear is considered to be
rolling on the wall, so:

5. Use Eq. (3.26) to determine that

6. Solve the equation from (4) on the previous page and obtain

7. Use Eq. (3.19) a third time to relate and from above with

8. Solve and obtain

9. Obtain from Explain why


Page 6 1 1

GOAL 9: To demonstrate proficiency with Eq. (3.51).

The cylinder C rolls on the plane


to the right, with its center C at co
the instant shown, determine the
angular velocity of rod in and
the velocity, relative to of the
point P of O in contact with point
P'of

1. Write Eq. (3.51), relating to

2. Compute using Equation (3.8) to relate the velocities of P and C on C:

3. Compute (in terms of ) by using Equation (3.8) again, this time to relate the velocities
of P' and O on

4. Express in terms of an unknown magnitude and a known direction (unit vector):


Page 6 1 2

5. Substitute the results of Steps (2), (3), and (4) into (1):

6. Collect the separate and coefficients and solve the two resulting equations for and :

7. Using the results of (6), write the required vectors and :

8. In this problem, the directions of all three vectors in Step (1) — and — are known
at the outset. Complete the sketch below by drawing arrows representing the three vectors; note
that the second and third must add up vectorially to the first!
Page 613

GOAL 10: To demonstrate understanding of Eq. (3.55).

Bars and are symmetrically placed, pinned


at their lower ends to a floor at 0 and 0 . Two
1 2

collars C and C are connected together by means


1 2

of a ball and socket at P, and they slide on and resp

1. Relate to using Equation (3.55); let the point V be the point of


1 coincident with P at
the given instant:

2. In your equation you will need . Write Equation (3.51), relating to

3. Note that by the symmetry, P must move vertically (in ). Note further that is an unknown
magnitude multiplied by a unit vector along . Incorporate these results into your Step (2)
equation and solve for
Page 614

4. We remark that it is not necessary to use symmetry as above; but without it, one must also relate
to and end up with 4 equations in 4 unknowns. What would be these four unknowns?

5. In the Equation in Step ( 1 ) , use the same two notes given in Step (3), this time for accelerations
instead of velocities, to reduce the vector equation to two scalar equations in the required
unknowns. Then solve for them.
Page 615

GOAL 11: To demonstrate use of the transfer (parallel axis)


theorem for mass moments of inertia.

Use the results of Appendix C to compute for the body-


consisting of a pair of identical homogeneous solid cones
and C as shown. Then compute where X is an axis parallel
X 2

to x through the point P at (x, y, z) = (3R, R, H).

1. The equation in the appendix is the moment of inertia for one cone about an
axis such as x in the figure. We cannot transfer this result from
b a s e to x. because point O is not
the mass center of the lower cone C . First we must obtain
2 for C . Use the transfer theorem to
2

do this:

2. Now transfer your result from C to C, and double it to account for the other cone, symmetrically
2

placed:
616

3. Since C is the mass center of the combined body comprising C and C , transfer
1 2 to P and
obtain :
Page 617

GOAL 12: To demonstrate understanding of Eq. (4.15)


and of the steps listed on pp. 249-250.

Cylinder O rolls on the board A


cord wrapped around a slot near the
periphery of O passes around a light
pulley and is attached to the weight
at A. Each of the three bodies has ma

1. Complete the free-body-diagrams below by adding arrows and names of forces:

Note that because the wheels beneath are light,


we can neglect their masses. Thus in a free-
body of one of them:

we see that

But also I ' = 0 if mass is negligible, so that


C

Thus there is no friction force exerted on the bottom of by the wheels.


Page 618

2. Using the sign convention fill in the two equations below for O:

3. Now fill in this equation for

4. And this equation for

5. Now list the unknowns in your four equations thus far. There should be six — four kinematic
quantities (accelerations, angular accelerations) and two forces:

6. State why none of: the y-equation for O and the x-equation for and the moment equation for
and need be written.

7. As a result of (5), we need two more equations. Turn to kinematics and on O, relate and
In doing so, it is crucial to note that and are acceleration components relative to tne ground
(inertial frame); show that , and number this equation :
Page 619

8. Next, relate to the acceleration component of T, the point at the top of C:

9. And now relate to

10. Combine the results of (8, 9) to show that ; call this equation

11. Now we have six equations in six unknowns. Solve them and obtain

12. State why this result is dimensionally correct and also why it is plausible:
Page 621

GOAL 1 3 : To demonstrate proficiency with the work


and kinetic energy equation ( 5 . 1 5 ) .

Cylinder
(mass The cylinder O is pinned smoothly to the ground at C
2m) Ring , positioned between C and the fixed circular track
rolls on both these surfaces. The spring is attached to
spring has unstretched length 3r/2, and the system is
released from rest in the position shown. In terms of m,
r, and g, find the spring modulus k that will result in
coming to rest at its lowest (dashed) position P . If k is
2

half that value, find the angular velocity of O when


passes through the dashed position.

1. Complete the work done by gravity between P and P :


t 2

2. Calculate the work done by the spring between P and P :


1 2

3. From here, note that W > 0 for there to be K.E. remaining at P2, and from this
determine the value of k for which will come to rest at P : 2
Page 622

4. Write the kinetic energy of C:

5. And the kinetic energy of

6. Relate, now, the angular speeds of C and using Eq. (3.8):

7. Use (4-6) to express the total K.E. in terms of

8. Now, with k = half the value found in (3), use to find in position P :
2
Page 623

GOAL 14: To demonstrate an understanding of the impulse/


momentum principles for a rigid body in plane motion.

The two rods and disk are all rigidly attached, and they rotate along with
the solid ball in a vertical plane as indicated. Just after the position
shown, point T3 hits the obstruction and suddenly stops without rebound.
The 1.5 lb solid ball is flung outward by the impact. Find the impulse

imparted to the rod at B by the obstruction. Neglect the effect of the

1. Complete the fbd of the system comprising all the above bodies:

2. Write the angular impulse, about the fixed point O, imparted to the above system during
the impact:
Page 624

3. Write the angular momentum of the ball before and after impact:

4. Express for the body comprising rods plus disk:

5. Use the principle of angular impulse and momentum to find

6. To find the other impulse, use linear impulse and momentum; first write the total linear impulse
on the system:
Page 625

7. Next write the initial linear momentum:

8. And the final linear momentum:

9. Using the principle of linear impulse and momentum, find

10. Now, check by showing that use the equation for the
separate bodies:
I
Page 627

GOAL 15: To demonstrate understanding of important properties of


the angular velocity vector in general motion of a rigid body.

Shown is a gyroscope consisting of:


(a) Frame C which turns in at rate about
which is fixed in and
are (b) Frame which turns in C at rate about
fixed in which is fixed in C and
frame 6 (c) Body a which turns in at rate ro about u3

which is fixed in and a.

It is given that and are constants.


Note that is shown aligned with the direction
of at t = 0, but will not generally be parallel to

Write in frame O, and find

1. Use the addition theorem to express in O, i.e., in terms of the unit vectors fixed in
direction in O; note that the angle in the small sketch above is

2. Find by "moving the derivative" to frame C where the unit vectors are fixed:
Page 628

3. As a check, write differentiate the first term in directly, and "move the
derivative" in the second and third terms to
Page 629

GOAL 1 6 : Tocombine kinematics and kinetics in solving


a problem involving general motion of a rigid body.

Use Eq. (2.45) to investigate the motion of a uniform sphere of mass m and radius R rolling under
the influence of gravity on a fixed horizontal plane In particular, show that the mass center C of
must travel on a straight line at constant speed (i.e., that it has zero acceleration). Thus C cannot move
on a curved path without slipping regardless of the amount of friction available.

1. Complete the fbd of by adding arrowheads and naming the four forces that act on The unit
vectors are fixed in with being vertical:

2. Use Eq (7.11) to express . Note that all axes through C are principal and that the moment of
inertia for each such axis is : Note also that will serve as the inertial frame for this problem:

3. Now use Eq. (2.45), taking moments about the base point B of. Note that we choose this form,
of the moment equation because for the external forces acting on
Page 630

4. Note that, using (2), since are constant in Also, note that an
expression tor is needed. Write using Eq. (6.56) and the rolling condition that is always
zero in this problem smce is rolling on the fixed plane:

5. Now differentiate your equation from (4) to obtain in terms of

6. Substitute and from (4,5) into (3), and conclude that and thus that
is constant:

7. Using = constant in (4), conclude finally that is also constant, which means that C must move
in a straight line at constant speed:
Page 631

The previous result is fascinating because in fact the center of a rolling sphere can travel on a curve if

You might also like